Sie sind auf Seite 1von 123

LAW ON NATURAL RESOURCES and ENVIRONMENTAL LAW Cases and Special Laws |1

G.R. No. 135385 December 6, 2000 RESOLUTION PER CURIAM: Petitioners Isagani Cruz and Cesar Europa brought this suit for prohibition and mandamus as citizens and taxpayers, assailing the constitutionality of certain provisions of Republic Act No. 8371 (R.A. 8371), otherwise known as the Indigenous Peoples Rights Act of 1997 (IPRA), and its Implementing Rules and Regulations (Implementing Rules). In its resolution of September 29, 1998, the Court required respondents to comment. In compliance, respondents Chairperson and Commissioners of the National Commission on Indigenous Peoples (NCIP), the government agency created under the IPRA to implement its provisions, filed on October 13, 1998 their Comment to the Petition, in which they defend the constitutionality of the IPRA and pray that the petition be dismissed for lack of merit. On October 19, 1998, respondents Secretary of the Department of Environment and Natural Resources (DENR) and Secretary of the Department of Budget and Management (DBM) filed through the Solicitor General a consolidated Comment. The Solicitor General is of the view that the IPRA is partly unconstitutional on the ground that it grants ownership over natural resources to indigenous peoples and prays that the petition be granted in part. On November 10, 1998, a group of intervenors, composed of Sen. Juan Flavier, one of the authors of the IPRA, Mr. Ponciano Bennagen, a member of the 1986 Constitutional Commission, and the leaders and members of 112 groups of indigenous peoples (Flavier, et. al), filed their Motion for Leave to Intervene. They join the NCIP in defending the constitutionality of IPRA and praying for the dismissal of the petition. On March 22, 1999, the Commission on Human Rights (CHR) likewise filed a Motion to Intervene and/or to Appear as Amicus Curiae. The CHR asserts that IPRA is an expression of the principle of parens patriae and that the State has the responsibility to protect and guarantee the rights of those who are at a serious disadvantage like indigenous peoples. For this reason it prays that the petition be dismissed. On March 23, 1999, another group, composed of the Ikalahan Indigenous People and the Haribon Foundation for the Conservation of Natural Resources, Inc. (Haribon, et al.), filed a motion to Intervene with attached Comment-in-Intervention. They agree with the NCIP and Flavier, et al. that IPRA is consistent with the Constitution and pray that the petition for prohibition and mandamus be dismissed. The motions for intervention of the aforesaid groups and organizations were granted. Oral arguments were heard on April 13, 1999. Thereafter, the parties and intervenors filed their respective memoranda in which they reiterate the arguments adduced in their earlier pleadings and during the hearing.
1

ISAGANI CRUZ and CESAR EUROPA, petitioners, vs. SECRETARY OF ENVIRONMENT AND NATURAL RESOURCES, SECRETARY OF BUDGET AND MANAGEMENT and CHAIRMAN and COMMISSIONERS OF THE NATIONAL COMMISSION ON INDIGENOUS PEOPLES, respondents. HON. JUAN M .FLAVIER, HON. PONCIANO BENNAGEN, BAYANI ASCARRAGA, EDTAMI MANSAYANGAN, BASILIO WANDAG, EVELYN DUNUAN, YAOM TUGAS, ALFREMO CARPIANO, LIBERATO A. GABIN, MATERNIDAD M. COLAS, NARCISA M. DALUPINES, BAI KIRAM-CONNIE SATURNO, BAE MLOMOBEATRIZ T. ABASALA, DATU BALITUNGTUNG-ANTONIO D. LUMANDONG, DATU MANTUMUKAW TEOFISTO SABASALES, DATU EDUAARDO BANDA, DATU JOEL UNAD, DATU RAMON BAYAAN, TIMUAY JOSE ANOY, TIMUAY MACARIO D. SALACAO, TIMUAY EDWIN B. ENDING, DATU SAHAMPONG MALANAW VI, DATU BEN PENDAO CABIGON, BAI NANAPNAY-LIZA SAWAY, BAY INAY DAYA-MELINDA S. REYMUNDO, BAI TINANGHAGA HELINITA T. PANGAN, DATU MAKAPUKAW ADOLINO L. SAWAY, DATU MAUDAYAW-CRISPEN SAWAY, VICKY MAKAY, LOURDES D. AMOS, GILBERT P. HOGGANG, TERESA GASPAR, MANUEL S. ONALAN, MIA GRACE L. GIRON, ROSEMARIE G. PE, BENITO CARINO, JOSEPH JUDE CARANTES, LYNETTE CARANTES-VIVAL, LANGLEY SEGUNDO, SATUR S. BUGNAY, CARLING DOMULOT, ANDRES MENDIOGRIN, LEOPOLDO ABUGAN, VIRGILIO CAYETANO, CONCHITA G. DESCAGA, LEVY ESTEVES, ODETTE G. ESTEVEZ, RODOLFO C. AGUILAR, MAURO VALONES, PEPE H. ATONG, OFELIA T. DAVI, PERFECTO B. GUINOSAO, WALTER N. TIMOL, MANUEL T. SELEN, OSCAR DALUNHAY, RICO O. SULATAN, RAFFY MALINDA, ALFREDO ABILLANOS, JESSIE ANDILAB, MIRLANDO H. MANGKULINTAS, SAMIE SATURNO, ROMEO A. LINDAHAY, ROEL S. MANSANG-CAGAN, PAQUITO S. LIESES, FILIPE G. SAWAY, HERMINIA S. SAWAY, JULIUS S. SAWAY, LEONARDA SAWAY, JIMMY UGYUB, SALVADOR TIONGSON, VENANCIO APANG, MADION MALID, SUKIM MALID, NENENG MALID, MANGKATADONG AUGUSTO DIANO, JOSEPHINE M. ALBESO, MORENO MALID, MARIO MANGCAL, FELAY DIAMILING, SALOME P. SARZA, FELIPE P. BAGON, SAMMY SALNUNGAN, ANTONIO D. EMBA, NORMA MAPANSAGONOS, ROMEO SALIGA, SR., JERSON P. GERADA, RENATO T. BAGON, JR., SARING MASALONG, SOLEDAD M. GERARDA, ELIZABETH L. MENDI, MORANTE S. TIWAN, DANILO M. MALUDAO, MINORS MARICEL MALID, represented by her father CORNELIO MALID, MARCELINO M. LADRA, represented by her father MONICO D. LADRA, JENNYLYN MALID, represented by her father TONY MALID, ARIEL M. EVANGELISTA, represented by her mother LINAY BALBUENA, EDWARD M. EMUY, SR., SUSAN BOLANIO, OND, PULA BATO B'LAAN TRIBAL FARMER'S ASSOCIATION, INTER-PEOPLE'S EXCHANGE, INC. and GREEN FORUM-WESTERN VISAYAS, intervenors. COMMISSION ON HUMAN RIGHTS, intervenor. IKALAHAN INDIGENOUS PEOPLE and HARIBON FOUNDATION FOR THE CONSERVATION OF NATURAL RESOURCES, INC., intervenor.

LAW ON NATURAL RESOURCES and ENVIRONMENTAL LAW Cases and Special Laws |2
Petitioners assail the constitutionality of the following provisions of the IPRA and its Implementing Rules on the ground that they amount to an unlawful deprivation of the States ownership over lands of the public domain as well as minerals and other natural resources therein, in violation of the regalian doctrine embodied in Section 2, Article XII of the Constitution: "(1) Section 3(a) which defines the extent and coverage of ancestral domains, and Section 3(b) which, in turn, defines ancestral lands; "(2) Section 5, in relation to section 3(a), which provides that ancestral domains including inalienable public lands, bodies of water, mineral and other resources found within ancestral domains are private but community property of the indigenous peoples; "(3) Section 6 in relation to section 3(a) and 3(b) which defines the composition of ancestral domains and ancestral lands; "(4) Section 7 which recognizes and enumerates the rights of the indigenous peoples over the ancestral domains; (5) Section 8 which recognizes and enumerates the rights of the indigenous peoples over the ancestral lands; "(6) Section 57 which provides for priority rights of the indigenous peoples in the harvesting, extraction, development or exploration of minerals and other natural resources within the areas claimed to be their ancestral domains, and the right to enter into agreements with nonindigenous peoples for the development and utilization of natural resources therein for a period not exceeding 25 years, renewable for not more than 25 years; and "(7) Section 58 which gives the indigenous peoples the responsibility to maintain, develop, protect and conserve the ancestral domains and portions thereof which are found to be necessary for critical 2 watersheds, mangroves, wildlife sanctuaries, wilderness, protected areas, forest cover or reforestation." Petitioners also content that, by providing for an all-encompassing definition of "ancestral domains" and "ancestral lands" which might even include private lands found within said areas, Sections 3(a) and 3(b) 3 violate the rights of private landowners. In addition, petitioners question the provisions of the IPRA defining the powers and jurisdiction of the NCIP and making customary law applicable to the settlement of disputes involving ancestral domains and 4 ancestral lands on the ground that these provisions violate the due process clause of the Constitution. These provisions are: "(1) sections 51 to 53 and 59 which detail the process of delineation and recognition of ancestral domains and which vest on the NCIP the sole authority to delineate ancestral domains and ancestral lands; "(2) Section 52[i] which provides that upon certification by the NCIP that a particular area is an ancestral domain and upon notification to the following officials, namely, the Secretary of Environment and Natural Resources, Secretary of Interior and Local Governments, Secretary of Justice and Commissioner of the National Development Corporation, the jurisdiction of said officials over said area terminates; "(3) Section 63 which provides the customary law, traditions and practices of indigenous peoples shall be applied first with respect to property rights, claims of ownership, hereditary succession and settlement of land disputes, and that any doubt or ambiguity in the interpretation thereof shall be resolved in favor of the indigenous peoples; "(4) Section 65 which states that customary laws and practices shall be used to resolve disputes involving indigenous peoples; and "(5) Section 66 which vests on the NCIP the jurisdiction over all claims and disputes involving 5 rights of the indigenous peoples." Finally, petitioners assail the validity of Rule VII, Part II, Section 1 of the NCIP Administrative Order No. 1, series of 1998, which provides that "the administrative relationship of the NCIP to the Office of the President is characterized as a lateral but autonomous relationship for purposes of policy and program coordination." They contend that said Rule infringes upon the Presidents power of control over executive 6 departments under Section 17, Article VII of the Constitution. Petitioners pray for the following: "(1) A declaration that Sections 3, 5, 6, 7, 8, 52[I], 57, 58, 59, 63, 65 and 66 and other related provisions of R.A. 8371 are unconstitutional and invalid; "(2) The issuance of a writ of prohibition directing the Chairperson and Commissioners of the NCIP to cease and desist from implementing the assailed provisions of R.A. 8371 and its Implementing Rules; "(3) The issuance of a writ of prohibition directing the Secretary of the Department of Environment and Natural Resources to cease and desist from implementing Department of Environment and Natural Resources Circular No. 2, series of 1998;

LAW ON NATURAL RESOURCES and ENVIRONMENTAL LAW Cases and Special Laws |3
"(4) The issuance of a writ of prohibition directing the Secretary of Budget and Management to cease and desist from disbursing public funds for the implementation of the assailed provisions of R.A. 8371; and "(5) The issuance of a writ of mandamus commanding the Secretary of Environment and Natural Resources to comply with his duty of carrying out the States constitutional mandate to co ntrol and supervise the exploration, development, utilization and conservation of Philippine natural 7 resources." After due deliberation on the petition, the members of the Court voted as follows: Seven (7) voted to dismiss the petition. Justice Kapunan filed an opinion, which the Chief Justice and Justices Bellosillo, Quisumbing, and Santiago join, sustaining the validity of the challenged provisions of R.A. 8371. Justice Puno also filed a separate opinion sustaining all challenged provisions of the law with the exception of Section 1, Part II, Rule III of NCIP Administrative Order No. 1, series of 1998, the Rules and Regulations Implementing the IPRA, and Section 57 of the IPRA which he contends should be interpreted as dealing with the large-scale exploitation of natural resources and should be read in conjunction with Section 2, Article XII of the 1987 Constitution. On the other hand, Justice Mendoza voted to dismiss the petition solely on the ground that it does not raise a justiciable controversy and petitioners do not have standing to question the constitutionality of R.A. 8371. Seven (7) other members of the Court voted to grant the petition. Justice Panganiban filed a separate opinion expressing the view that Sections 3 (a)(b), 5, 6, 7 (a)(b), 8, and related provisions of R.A. 8371 are unconstitutional. He reserves judgment on the constitutionality of Sections 58, 59, 65, and 66 of the law, which he believes must await the filing of specific cases by those whose rights may have been violated by the IPRA. Justice Vitug also filed a separate opinion expressing the view that Sections 3(a), 7, and 57 of R.A. 8371 are unconstitutional. Justices Melo, Pardo, Buena, Gonzaga-Reyes, and De Leon join in the separate opinions of Justices Panganiban and Vitug. As the votes were equally divided (7 to 7) and the necessary majority was not obtained, the case was redeliberated upon. However, after redeliberation, the voting remained the same. Accordingly, pursuant to Rule 56, Section 7 of the Rules of Civil Procedure, the petition is DISMISSED. Attached hereto and made integral parts thereof are the separate opinions of Justices Puno, Vitug, Kapunan, Mendoza, and Panganiban. SO ORDERED. Davide, Jr., C.J., Bellosillo, Melo, Quisumbing, Pardo, Buena, Gonzaga-Reyes, Ynares-Santiago, and De Leon, Jr., JJ., concur. Puno, Vitug, Kapunan, Mendoza and Panganiban JJ., see separate opinion Footnotes
1

Rollo, p. 114. Petition, Rollo, pp. 16-23. Id. at 23-25.

Section 1, Article III of the Constitution states: "No person shall be deprived of life, liberty or property without due process of law, nor shall any person be denied the equal protection of the laws."
5

Rollo, pp. 25-27. Id. at 27-28. Transcript of Stenographic Notes of the hearing held on April 13, 1999, pp. 5-6.

SEPARATE OPINION PUNO, J.: PRECIS A classic essay on the utility of history was written in 1874 by Friedrich Nietzsche entitled "On the Uses 1 2 and Disadvantages of History for Life." Expounding on Nietzsche's essay, Judge Richard Posner wrote: "Law is the most historically oriented, or if you like the most backward-looking, the most 'pastdependent,' of the professions. It venerates tradition, precedent, pedigree, ritual, custom, ancient practices, ancient texts, archaic terminology, maturity, wisdom, seniority, gerontocracy, and interpretation conceived of as a method of recovering history. It is suspicious of innovation, discontinuities, 'paradigm shifts,' and the energy and brashness of youth. These ingrained attitudes are obstacles to anyone who wants to re-orient law in a more pragmatic direction. But, by the same token, pragmatic jurisprudence must come to terms with history." When Congress enacted the Indigenous Peoples Rights Act (IPRA), it introduced radical concepts into the Philippine legal system which appear to collide with settled constitutional and jural precepts on state ownership of land and other natural resources. The sense and subtleties of this law cannot be appreciated without considering its distinct sociology and the labyrinths of its history. This Opinion attempts to interpret IPRA by discovering its soul shrouded by the mist of our history. After all, the IPRA was enacted

LAW ON NATURAL RESOURCES and ENVIRONMENTAL LAW Cases and Special Laws |4
by Congress not only to fulfill the constitutional mandate of protecting the indigenous cultural communities' right to their ancestral land but more importantly, to correct a grave historical injustice to our indigenous people. This Opinion discusses the following: I. The Development of the Regalian Doctrine in the Philippine Legal System. 1. The indigenous concept of ownership and customary law A. The Laws of the Indies B. Valenton v. Murciano C. The Public Land Acts and the Torrens System D. The Philippine Constitutions II. The Indigenous Peoples Rights Act (IPRA). A. Indigenous Peoples 1. Indigenous Peoples: Their History 2. Their Concept of Land III. The IPRA is a Novel Piece of Legislation. A. Legislative History IV. The Provisions of the IPRA Do Not Contravene the Constitution. A. Ancestral domains and ancestral lands are the private property of indigenous peoples and do not constitute part of the land of the public domain. 1. The right to ancestral domains and ancestral lands: how acquired 2. The concept of native title (a) Cario v. Insular Government (b) Indian Title to land C. Sections 7 (a), 7 (b) and 57 of the IPRA do not violate the Regalian Doctrine enshrined in Section 2, Article XII of the 1987 Constitution. 1. The rights of ICCs/IPs over their ancestral domains and lands 2. The right of ICCs/IPs to develop lands and natural resources within the ancestral domains does not deprive the State of ownership over the natural resources, control and supervision in their development and exploitation. (a) Section 1, Part II, Rule III of the Implementing Rules goes beyond the parameters of Section 7(a) of the law on ownership of ancestral domains and is ultra vires. (b) The small-scale utilization of natural resources in Section 7 (b) of the IPRA is allowed under Paragraph 3, Section 2, Article XII of the 1987 Consitution. (c) The large-scale utilization of natural resources in Section 57 of the IPRA may be harmonized with Paragraphs 1 and 4, Section 2, Article XII of the 1987 Constitution. V. The IPRA is a Recognition of Our Active Participation in the International Indigenous Movement. DISCUSSION I. THE DEVELOPMENT OF THE REGALIAN DOCTRINE IN THE PHILIPPINE LEGAL SYSTEM. A. The Laws of the Indies The capacity of the State to own or acquire property is the state's power of dominium. This was the foundation for the early Spanish decrees embracing the feudal theory of jura regalia. The "Regalian Doctrine" or jura regaliais a Western legal concept that was first introduced by the Spaniards into the
3

(c) Why the Cario doctrine is unique 3. The option of securing a torrens title to the ancestral land B. The right of ownership and possession by the ICCs/IPs to their ancestral domains is a limited form of ownership and does not include the right to alienate the same.

LAW ON NATURAL RESOURCES and ENVIRONMENTAL LAW Cases and Special Laws |5
country through the Laws of the Indies and the Royal Cedulas. The Laws of the Indies, i.e., more specifically, Law 14, Title 12, Book 4 of the Novisima Recopilacion de Leyes de las Indias , set the policy of the Spanish Crown with respect to the Philippine Islands in the following manner: "We, having acquired full sovereignty over the Indies, and all lands, territories, and possessions not heretofore ceded away by our royal predecessors, or by us, or in our name, still pertaining to the royal crown and patrimony, it is our will that all lands which are held without proper and true deeds of grant be restored to us as they belong to us, in order that after reserving before all what to us or to our viceroys, audiencias, and governors may seem necessary for public squares, ways, pastures, and commons in those places which are peopled, taking into consideration not only their present condition, but also their future and their probable increase, and after distributing to the natives what may be necessary for tillage and pasturage, confirming them in what they now have and giving them more if necessary, all the rest of said lands may remain free and unencumbered for us to dispose of as we may wish. We therefore order and command that all viceroys and presidents of pretorial courts designate at such time as shall to them seem most expedient, a suitable period within which all possessors of tracts, farms, plantations, and estates shall exhibit to them and to the court officers appointed by them for this purpose, their title deeds thereto. And those who are in possession by virtue of proper deeds and receipts, or by virtue of just prescriptive right shall be protected, and all the rest shall be restored to us to 4 be disposed of at our will." The Philippines passed to Spain by virtue of "discovery" and conquest. Consequently, all lands became the exclusive patrimony and dominion of the Spanish Crown. The Spanish Government took charge of distributing the lands by issuing royal grants and concessions to Spaniards, both military and 5 civilian. Private land titles could only be acquired from the government either by purchase or by the 6 various modes of land grant from the Crown. The Laws of the Indies were followed by the Ley Hipotecaria, or the Mortgage Law of 1893. The Spanish Mortgage Law provided for the systematic registration of titles and deeds as well as possessory claims. The law sought to register and tax lands pursuant to the Royal Decree of 1880. The Royal Decree of 1894, or the "Maura Law," was partly an amendment of the Mortgage Law as well as the Laws of the Indies, as 8 already amended by previous orders and decrees. This was the last Spanish land law promulgated in the Philippines. It required the "adjustment" or registration of all agricultural lands, otherwise the lands shall revert to the state.kkkkkk Four years later, by the Treaty of Paris of December 10, 1898, Spain ceded to the government of the United States all rights, interests and claims over the national territory of the Philippine Islands. In 1903, the United States colonial government, through the Philippine Commission, passed Act No. 926, the first Public Land Act. B. Valenton v. Murciano
7

In 1904, under the American regime, this Court decided the case of Valenton v. Murciano.

Valenton resolved the question of which is the better basis for ownership of land: long-time occupation or paper title. Plaintiffs had entered into peaceful occupation of the subject land in 1860. Defendant's predecessor-in-interest, on the other hand, purchased the land from the provincial treasurer of Tarlac in 1892. The lower court ruled against the plaintiffs on the ground that they had lost all rights to the land by not objecting to the administrative sale. Plaintiffs appealed the judgment, asserting that their 30-year adverse possession, as an extraordinary period of prescription in the Partidas and the Civil Code, had given them title to the land as against everyone, including the State; and that the State, not owning the land, could not validly transmit it. The Court, speaking through Justice Willard, decided the case on the basis of "those special laws which 10 from earliest time have regulated the disposition of the public lands in the colonies." The question posed by the Court was: "Did these special laws recognize any right of prescription as against the State as to these lands; and if so, to what extent was it recognized?" Prior to 1880, the Court said, there were no laws specifically providing for the disposition of land in the Philippines. However, it was understood that in the absence of any special law to govern a specific colony, the Laws of the Indies would be followed. Indeed, in the Royal Order of July 5, 1862, it was decreed that until regulations on the subject could be prepared, the authorities of the Philippine Islands should follow 11 strictly the Laws of the Indies, theOrdenanza of the Intendentes of 1786, and the Royal Cedula of 1754. Quoting the preamble of Law 14, Title 12, Book 4 of the Recopilacion de Leyes de las Indias, the court interpreted it as follows: "In the preamble of this law there is, as is seen, a distinct statement that all those lands belong to the Crown which have not been granted by Philip, or in his name, or by the kings who preceded him. This statement excludes the idea that there might be lands not so granted, that did not belong to the king. It excludes the idea that the king was not still the owner of all ungranted lands , because some private person had been in the adverse occupation of them. By the mandatory part of the law all the occupants of the public lands are required to produce before the authorities named, and within a time to be fixed by them, their title papers. And those who h P; ad good title or showed prescription were to be protected in their holdings. It is apparent that it was not the intention of the law that mere possession for a length of time should make the possessors the 12 owners of the land possessed by them without any action on the part of the authorities." The preamble stated that all those lands which had not been granted by Philip, or in his name, or by the 13 kings who preceded him, belonged to the Crown. For those lands granted by the king, the decree provided for a system of assignment of such lands. It also ordered that all possessors of agricultural land 14 should exhibit their title deed, otherwise, the land would be restored to the Crown.

LAW ON NATURAL RESOURCES and ENVIRONMENTAL LAW Cases and Special Laws |6
The Royal Cedula of October 15, 1754 reinforced the Recopilacion when it ordered the Crown's principal subdelegate to issue a general order directing the publication of the Crown's instructions: "x x x to the end that any and all persons who, since the year 1700, and up to the date of the promulgation and publication of said order, shall have occupied royal lands, whether or not x x x cultivated or tenanted, may x x x appear and exhibit to said subdelegates the titles and patents by virtue of which said lands are occupied. x x x. Said subdelegates will at the same time warn the parties interested that in case of their failure to present their title deeds within the term designated, without a just and valid reason therefor, they will be deprived of and evicted from their lands, and they will be 15 granted to others." On June 25, 1880, the Crown adopted regulations for the adjustment of lands "wrongfully occupied" by private individuals in the Philippine Islands. Valenton construed these regulations together with contemporaneous legislative and executive interpretations of the law, and concluded that plaintiffs' case fared no better under the 1880 decree and other laws which followed it, than it did under the earlier ones. Thus as a general doctrine, the Court stated: "While the State has always recognized the right of the occupant to a deed if he proves a possession for a sufficient length of time, yet it has always insisted that he must make that proof before the proper administrative officers, and obtain from them his deed, and until he did that the State remained the 16 absolute owner." In conclusion, the Court ruled: "We hold that from 1860 to 1892 there was no law in force in these Islands by which the plaintiffs could obtain the ownership of these lands by prescription, without any action by 17 the State." Valenton had no rights other than those which accrued to mere possession. Murciano, on the other hand, was deemed to be the owner of the land by virtue of the grant by the provincial secretary. In effect, Valenton upheld the Spanish concept of state ownership of public land. As a fitting observation, the Court added that "[t]he policy pursued by the Spanish Government from earliest times, requiring settlers on the public lands to obtain title deeds therefor from the State, has 18 been continued by the American Government in Act No. 926. " C. The Public Land Acts and the Torrens System Act No. 926, the first Public Land Act, was passed in pursuance of the provisions of the the Philippine Bill of 1902. The law governed the disposition of lands of the public domain. It prescribed rules and regulations for the homesteading, selling, and leasing of portions of the public domain of the Philippine Islands, and prescribed the terms and conditions to enable persons to perfect their titles to public lands in the Islands. It also provided for the "issuance of patents to certain native settlers upon public lands," for the establishment of town sites and sale of lots therein, for the completion of imperfect titles, and for the cancellation or confirmation of Spanish concessions and grants in the Islands." In short, the Public Land Act operated on the assumption that title to public lands in the Philippine Islands remained in the 19 government; and that the government's title to public land sprung from the Treaty of Paris and other 20 subsequent treaties between Spain and the United States. The term "public land" referred to all lands of the public domain whose title still remained in the government and are thrown open to private 21 appropriation and settlement, and excluded the patrimonial property of the government and the friar 22 lands. Act No. 926 was superseded in 1919 by Act 2874, the second Public Land Act. This new law was passed under the Jones Law. It was more comprehensive in scope but limited the exploitation of agricultural lands to Filipinos and Americans and citizens of other countries which gave Filipinos the same 23 privileges. After the passage of the 1935 Constitution, Act 2874 was amended in 1936 by Commonwealth Act No. 141. Commonwealth Act No. 141 remains the present Public Land Law and it is essentially the same as Act 2874. The main difference between the two relates to the transitory provisions on the rights of American citizens and corporations during the Commonwealth period at par 24 with Filipino citizens and corporations. Grants of public land were brought under the operation of the Torrens system under Act 496, or the Land Registration Law of 1903. Enacted by the Philippine Commission, Act 496 placed all public and private lands in the Philippines under the Torrens system. The law is said to be almost a verbatim copy of 25 the Massachussetts Land Registration Act of 1898, which, in turn, followed the principles and procedure of the Torrens system of registration formulated by Sir Robert Torrens who patterned it after the Merchant Shipping Acts in South Australia. The Torrens system requires that the government issue an official certificate of title attesting to the fact that the person named is the owner of the property described therein, subject to such liens and encumbrances as thereon noted or the law warrants or 26 reserves. The certificate of title is indefeasible and imprescriptible and all claims to the parcel of land are 27 quieted upon issuance of said certificate. This system highly facilitates land conveyance and negotiation. D. The Philippine Constitutions The Regalian doctrine was enshrined in the 1935 Constitution. One of the fixed and dominating objectives of the 1935 Constitutional Convention was the nationalization and conservation of the natural resources 28 of the country. There was an overwhelming sentiment in the Convention in favor of the principle of 29 state ownership of natural resources and the adoption of the Regalian doctrine. State ownership of natural resources was seen as a necessary starting point to secure recognition of the state's power to 30 control their disposition, exploitation, development, or utilization. The delegates to the Constitutional Convention very well knew that the concept of State ownership of land and natural resources was introduced by the Spaniards, however, they were not certain whether it was continued and applied by the Americans. To remove all doubts, the Convention approved the provision in the Constitution affirming the 31 Regalian doctrine. Thus, the 1935 Constitution, in Section 1 of Article XIII on "Conservation and Utilization of Natural Resources," reads as follows:

LAW ON NATURAL RESOURCES and ENVIRONMENTAL LAW Cases and Special Laws |7
"Sec. 1. All agricultural, timber, and mineral lands of the public domain, waters, minerals, coal, petroleum, and other mineral oils, all forces of potential energy, and other natural resources of the Philippines belong to the State, and their disposition, exploitation, development, or utilization shall be limited to citizens of the Philippines, or to corporations or associations at least sixty per centum of the capital of which is owned by such citizens, subject to any existing right, grant, lease, or concession at the time of the inauguration of the Government established under this Constitution. Natural resources, with the exception of public agricultural land, shall not be alienated, and no license, concession, or lease for the exploitation, development, or utilization of any of the natural resources shall be granted for a period exceeding twenty-five years, except as to water rights for irrigation, water supply, fisheries, or industrial uses other than the development of water power, in which cases beneficial use may be the measure and the limit of the grant." The 1973 Constitution reiterated the Regalian doctrine in Section 8, Article XIV on the "National Economy and the Patrimony of the Nation," to wit: "Sec. 8. All lands of the public domain, waters, minerals, coal, petroleum and other mineral oils, all forces of potential energy, fisheries, wildlife, and other natural resources of the Philippines belong to the State. With the exception of agricultural, industrial or commercial, residential, and resettlement lands of the public domain, natural resources shall not be alienated, and no license, concession, or lease for the exploration, development, exploitation, or utilization of any of the natural resources shall be granted for a period exceeding twenty-five years, renewable for not more than twenty-five years,except as to water rights for irrigation, water supply, fisheries, or industrial uses other than the development of water power, in which cases beneficial use may be the measure and the limit of the grant." The 1987 Constitution reaffirmed the Regalian doctrine in Section 2 of Article XII on "National Economy and Patrimony," to wit: "Sec. 2. All lands of the public domain, waters, minerals, coal, petroleum, and other mineral oils, all forces of potential energy, fisheries, forests or timber, wildlife, flora and fauna, and other natural resources are owned by the State. With the exception of agricultural lands, all other natural resources shall not be alienated. The exploration, development and utilization of natural resources shall be under the full control and supervision of the State. The State may directly undertake such activities or it may enter into co-production, joint venture, or production-sharing agreements with Filipino citizens, or corporations or associations at least sixty per centum of whose capital is owned by such citizens. Such agreements may be for a period not exceeding twenty-five years, renewable for not more than twentyfive years, and under such terms and conditions as may be provided by law. In cases of water rights for irrigation, water supply, fisheries, or industrial uses other than the development of water power, beneficial use may be the measure and limit of the grant. x x x." Simply stated, all lands of the public domain as well as all natural resources enumerated therein, whether on public or private land, belong to the State. It is this concept of State ownership that petitioners claim is being violated by the IPRA. II. THE INDIGENOUS PEOPLES RIGHTS ACT. Republic Act No. 8371 is entitled "An Act to Recognize, Protect and Promote the Rights of Indigenous Cultural Communities/ Indigenous Peoples, Creating a National Commission on Indigenous Peoples, Establishing Implementing Mechanisms, Appropriating Funds Therefor, and for Other Purposes." It is simply known as "The Indigenous Peoples Rights Act of 1997" or the IPRA. The IPRA recognizes the existence of the indigenous cultural communities or indigenous peoples (ICCs/IPs) as a distinct sector in Philippine society. It grants these people the ownership and possession of their ancestral domains and ancestral lands, and defines the extent of these lands and domains. The ownership given is the indigenous concept of ownership under customary law which traces its origin to native title. Other rights are also granted the ICCs/IPs, and these are: - the right to develop lands and natural resources; - the right to stay in the territories; - the right in case of displacement; - the right to safe and clean air and water; - the right to claim parts of reservations; - the right to resolve conflict;
32

- the right to ancestral lands which include a. the right to transfer land/property to/among members of the same ICCs/IPs, subject to customary laws and traditions of the community concerned; b. the right to redemption for a period not exceeding 15 years from date of transfer, if the transfer is to a non-member of the ICC/IP and is tainted by vitiated consent of the 33 ICC/IP, or if the transfer is for an unconscionable consideration.

LAW ON NATURAL RESOURCES and ENVIRONMENTAL LAW Cases and Special Laws |8
Within their ancestral domains and ancestral lands, the ICCs/IPs are given the right to self-governance and 34 35 empowerment, social justice and human rights, the right to preserve and protect their culture, traditions, institutions and community intellectual rights, and the right to develop their own sciences and 36 technologies. To carry out the policies of the Act, the law created the National Commission on Indigenous Peoples (NCIP). The NCIP is an independent agency under the Office of the President and is composed of seven (7) Commissioners belonging to ICCs/IPs from each of the ethnographic areas- Region I and the Cordilleras; Region II; the rest of Luzon; Island groups including Mindoro, Palawan, Romblon, Panay and the rest of the 37 Visayas; Northern and Western Mindanao; Southern and Eastern Mindanao; and Central Mindanao. The NCIP took over the functions of the Office for Northern Cultural Communities and the Office for Southern Cultural Communities created by former President Corazon Aquino which were merged under a 38 revitalized structure. Disputes involving ICCs/IPs are to be resolved under customary laws and practices. When still 39 unresolved, the matter may be brought to the NCIP, which is granted quasi-judicial powers. The NCIP's decisions may be appealed to the Court of Appeals by a petition for review. Any person who violates any of the provisions of the Act such as, but not limited to, unauthorized and/or unlawful intrusion upon ancestral lands and domains shall be punished in accordance with customary laws or imprisoned from 9 months to 12 years and/or fined from P100,000.00 to P500,000.00 and obliged 40 to pay damages. A. Indigenous Peoples The IPRA is a law dealing with a specific group of people, i.e., the Indigenous Cultural Communities (ICCs) or the Indigenous Peoples (IPs). The term "ICCs" is used in the 1987 Constitution while that of "IPs" is the 41 contemporary international language in the International Labor Organization (ILO) Convention 169 and 42 the United Nations (UN) Draft Declaration on the Rights of Indigenous Peoples. ICCs/IPs are defined by the IPRA as: "Sec. 3 [h]. Indigenous Cultural Communities/ Indigenous Peoples- refer to a group of people or homogeneous societies identified by self-ascription and ascription by others, who have continuously lived as organized community on communally bounded and defined territory, and who have, under claims of ownership since time immemorial, occupied, possessed and utilized such territories, sharing common bonds of language, customs, traditions and other distinctive cultural traits, or who have, through resistance to political, social and cultural inroads of colonization, non-indigenous religions and cultures, became historically differentiated from the majority of Filipinos. ICCs/IPs shall likewise include peoples who are regarded as indigenous on account of their descent from the populations which inhabited the country, at the time of conquest or colonization, or at the time of inroads of non-indigenous religions and cultures, or the establishment of present state boundaries, who retain some or all of their own social, economic, cultural and political institutions, but who may have been displaced from their traditional domains or who may have resettled outside their ancestral domains." Indigenous Cultural Communities or Indigenous Peoples refer to a group of people or homogeneous societies who have continuously lived as an organized community on communally bounded and defined territory. These groups of people have actually occupied, possessed and utilized their territories under claim of ownership since time immemorial. They share common bonds of language, customs, traditions and other distinctive cultural traits, or, they, by their resistance to political, social and cultural inroads of colonization, non-indigenous religions and cultures, became historically differentiated from the Filipino majority. ICCs/IPs also include descendants of ICCs/IPs who inhabited the country at the time of conquest or colonization, who retain some or all of their own social, economic, cultural and political institutions but who may have been displaced from their traditional territories or who may have resettled outside their ancestral domains. 1. Indigenous Peoples: Their History Presently, Philippine indigenous peoples inhabit the interiors and mountains of Luzon, Mindanao, Mindoro, Negros, Samar, Leyte, and the Palawan and Sulu group of islands. They are composed of 110 tribes and are as follows: 1. In the Cordillera Autonomous Region- Kankaney, Ibaloi, Bontoc, Tinggian or Itneg, Ifugao, Kalinga, Yapayao, Aeta or Agta or Pugot, and Bago of Ilocos Norte and Pangasinan; Ibanag of Isabela, Cagayan; Ilongot of Quirino and Nueva Vizcaya; Gaddang of Quirino, Nueva Vizcaya, Itawis of Cagayan; Ivatan of Batanes, Aeta of Cagayan, Quirino and Isabela. 2. In Region III- Aetas. 3. In Region IV- Dumagats of Aurora, Rizal; Remontado of Aurora, Rizal, Quezon; Alangan or Mangyan, Batangan, Buid or Buhid, Hanunuo and Iraya of Oriental and Occidental Mindoro; Tadyawan of Occidental Mindoro; Cuyonon, Palawanon, Tagbanua and Tao't bato of Palawan. 4. In Region V- Aeta of Camarines Norte and Camarines Sur; Aeta-Abiyan, Isarog, and Kabihug of Camarines Norte; Agta, and Mayon of Camarines Sur; Itom of Albay, Cimaron of Sorsogon; and the Pullon of Masbate and Camarines Sur. 5. In Region VI- Ati of Negros Occidental, Iloilo and Antique, Capiz; the Magahat of Negros Occidental; the Corolano and Sulod. 6. In Region VII- Magahat of Negros Oriental and Eskaya of Bohol.

LAW ON NATURAL RESOURCES and ENVIRONMENTAL LAW Cases and Special Laws |9
7. In Region IX- the Badjao numbering about 192,000 in Tawi-Tawi, Zamboanga del Sur; the Kalibugan of Basilan, the Samal, Subanon and Yakat. 8. Region X- Numbering 1.6 million in Region X alone, the IPs are: the Banwaon, Bukidnon, Matigsalog, Talaanding of Bukidnon; the Camiguin of Camiguin Island; the Higa-unon of Agusan del Norte, Agusan del Sur, Bukidnon and Misamis Occidental; the Tigwahanon of Agusan del Sur, Misamis Oriental and and Misamis Occidental, the Manobo of the Agusan provinces, and the Umayamnon of Agusan and Bukidnon. 9. In Region XI- There are about 1,774,065 IPs in Region XI. They are tribes of the Dibabaon, Mansaka of Davao del Norte; B'laan, Kalagan, Langilad, T'boli and Talaingod of Davao del Sur; Mamamanua of Surigao del Sur; Mandaya of the Surigao provinces and Davao Oriental; Manobo Blit of South Cotabato; the Mangguangon of Davao and South Cotabato; Matigsalog of Davao del Norte and Del Sur; Tagakaolo, Tasaday and Ubo of South Cotabato; and Bagobo of Davao del sur and South Cotabato. 10. In Region XII- Ilianen, Tiruray, Maguindanao, Maranao, Tausug, Yakan/Samal, and Iranon.
43

death. Their rituals were based on beliefs in a ranking deity whom they called Bathalang Maykapal, and a host of other deities, in the environmental spirits and in soul spirits. The early Filipinos adored the sun, the moon, the animals and birds, for they seemed to consider the objects of Nature as something to be respected. They venerated almost any object that was close to their daily life, indicating the importance 51 of the relationship between man and the object of nature. The unit of government was the "barangay," a term that derived its meaning from the Malay word 52 "balangay," meaning, a boat, which transported them to these shores. The barangay was basically a family-based community and consisted of thirty to one hundred families. Each barangay was different and ruled by a chieftain called a "dato." It was the chieftain's duty to rule and govern his subjects and promote their welfare and interests. A chieftain had wide powers for he exercised all the functions of government. 53 He was the executive, legislator and judge and was the supreme commander in time of war. Laws were either customary or written. Customary laws were handed down orally from generation to generation and constituted the bulk of the laws of the barangay. They were preserved in songs and 54 chants and in the memory of the elder persons in the community. The written laws were those that the 55 chieftain and his elders promulgated from time to time as the necessity arose. The oldest known written body of laws was the Maragtas Code by Datu Sumakwel at about 1250 A.D. Other old codes are the 56 Muslim Code of Luwaran and the Principal Code of Sulu. Whether customary or written, the laws dealt with various subjects, such as inheritance, divorce, usury, loans, partnership, crime and punishment, property rights, family relations and adoption. Whenever disputes arose, these were decided peacefully through a court composed by the chieftain as "judge" and the barangay elders as "jury." Conflicts arising between subjects of different barangays were resolved by arbitration in which a board composed of 57 elders from neutral barangays acted as arbiters. Baranganic society had a distinguishing feature: the absence of private property in land. The chiefs merely administered the lands in the name of the barangay. The social order was an extension of the family with chiefs embodying the higher unity of the community. Each individual, therefore, participated in the community ownership of the soil and the instruments of production as a member of the 58 barangay. This ancient communalism was practiced in accordance with the concept of mutual sharing of resources so that no individual, regardless of status, was without sustenance. Ownership of land was non-existent or unimportant and the right of usufruct was what regulated the development of 59 lands. Marine resources and fishing grounds were likewise free to all. Coastal communities depended for their economic welfare on the kind of fishing sharing concept similar to those in land 60 communities. Recognized leaders, such as the chieftains and elders, by virtue of their positions of importance, enjoyed some economic privileges and benefits. But their rights, related to either land and sea, were subject to their responsibility to protect the communities from danger and to provide them 61 with the leadership and means of survival. Sometime in the 13th century, Islam was introduced to the archipelago in Maguindanao. The Sultanate of Sulu was established and claimed jurisdiction over territorial areas represented today by Tawi-tawi, Sulu, Palawan, Basilan and Zamboanga. Four ethnic groups were within this jurisdiction: Sama, Tausug,

How these indigenous peoples came to live in the Philippines goes back to as early as 25,000 to 30,000 B.C. Before the time of Western contact, the Philippine archipelago was peopled largely by the Negritos, 44 Indonesians and Malays. The strains from these groups eventually gave rise to common cultural features which became the dominant influence in ethnic reformulation in the archipelago. Influences from the Chinese and Indian civilizations in the third or fourth millenium B.C. augmented these ethnic strains. Chinese economic and socio-cultural influences came by way of Chinese porcelain, silk and traders. Indian 45 influence found their way into the religious-cultural aspect of pre-colonial society. The ancient Filipinos settled beside bodies of water. Hunting and food gathering became supplementary 46 activities as reliance on them was reduced by fishing and the cultivation of the soil. From the hinterland, coastal, and riverine communities, our ancestors evolved an essentially homogeneous culture, a basically common way of life where nature was a primary factor. Community life throughout the archipelago was influenced by, and responded to, common ecology. The generally benign tropical climate and the largely 47 uniform flora and fauna favored similarities, not differences. Life was essentially subsistence but not 48 harsh. The early Filipinos had a culture that was basically Malayan in structure and form. They had languages that traced their origin to the Austronesian parent-stock and used them not only as media of daily 49 communication but also as vehicles for the expression of their literary moods. They fashioned concepts 50 and beliefs about the world that they could not see, but which they sensed to be part of their lives. They had their own religion and religious beliefs. They believed in the immortality of the soul and life after

L A W O N N A T U R A L R E S O U R C E S a n d E N V I R O N M E N T A L L A W C a s e s a n d S p e c i a l L a w s | 10
Yakan and Subanon. The Sultanate of Maguindanao spread out from Cotabato toward Maranao 63 territory, now Lanao del Norte and Lanao del Sur. The Muslim societies evolved an Asiatic form of feudalism where land was still held in common but was private in use. This is clearly indicated in the Muslim Code of Luwaran. The Code contains a provision on the lease of cultivated lands. It, however, has no provision for the acquisition, transfer, cession or sale of 64 land. The societies encountered by Magellan and Legaspi therefore were primitive economies where most production was geared to the use of the producers and to the fulfillment of kinship obligations. They were 65 not economies geared to exchange and profit. Moreover, the family basis of barangay membership as well as of leadership and governance worked to splinter the population of the islands into numerous small 66 and separate communities. When the Spaniards settled permanently in the Philippines in 1565, they found the Filipinos living in barangay settlements scattered along water routes and river banks. One of the first tasks imposed on 67 the missionaries and the encomenderos was to collect all scattered Filipinos together in a reduccion. As early as 1551, the Spanish government assumed an unvarying solicitous attitude towards the 68 natives. The Spaniards regarded it a sacred "duty to conscience and humanity to civilize these less fortunate people living in the obscurity of ignorance" and to accord them the "moral and material 69 advantages" of community life and the "protection and vigilance afforded them by the same laws." The Spanish missionaries were ordered to establish pueblos where the church and convent would be constructed. All the new Christian converts were required to construct their houses around the church 70 and the unbaptized were invited to do the same. With the reduccion, the Spaniards attempted to "tame" the reluctant Filipinos through Christian indoctrination using the convento/casa real/plaza complex as focal point. The reduccion, to the Spaniards, was a "civilizing" device to make the Filipinos law-abiding citizens of the Spanish Crown, and in the long run, to make them ultimately adopt 71 Hispanic culture and civilization. All lands lost by the old barangays in the process of pueblo organization as well as all lands not assigned to them and the pueblos, were now declared to be crown lands or realengas, belonging to the Spanish 72 king. It was from the realengas that land grants were made to non-Filipinos. The abrogation of the Filipinos' ancestral rights in land and the introduction of the concept of public 73 domain were the most immediate fundamental results of Spanish colonial theory and law. The concept that the Spanish king was the owner of everything of value in the Indies or colonies was 74 imposed on the natives, and the natives were stripped of their ancestral rights to land. Increasing their foothold in the Philippines, the Spanish colonialists, civil and religious, classified the Filipinos according to their religious practices and beliefs, and divided them into three types . First were
62

the Indios, the Christianized Filipinos, who generally came from the lowland populations. Second, were 75 the Moros or the Muslim communities, and third, were the infieles or the indigenous communities. The Indio was a product of the advent of Spanish culture. This class was favored by the Spaniards and was allowed certain status although below the Spaniards. The Moros and infieles were regarded as the lowest 76 classes. The Moros and infieles resisted Spanish rule and Christianity. The Moros were driven from Manila and the Visayas to Mindanao; while the infieles, to the hinterlands. The Spaniards did not pursue them into the deep interior. The upland societies were naturally outside the immediate concern of Spanish interest, and the cliffs and forests of the hinterlands were difficult and inaccessible, allowing the infieles, in effect, 77 relative security. Thus, the infieles, which were peripheral to colonial administration, were not only able to preserve their own culture but also thwarted the Christianization process, separating themselves from 78 the newly evolved Christian community. Their own political, economic and social systems were kept constantly alive and vibrant. The pro-Christian or pro-Indio attitude of colonialism brought about a generally mutual feeling of suspicion, fear, and hostility between the Christians on the one hand and the non-Christians on the other. Colonialism tended to divide and rule an otherwise culturally and historically related populace through a 79 colonial system that exploited both the virtues and vices of the Filipinos. President McKinley, in his instructions to the Philippine Commission of April 7, 1900, addressed the existence of the infieles: "In dealing with the uncivilized tribes of the Islands, the Commission should adopt the same course followed by Congress in permitting the tribes of our North American Indians to maintain their tribal organization and government, and under which many of those tribes are now living in peace and contentment, surrounded by civilization to which they are unable or unwilling to conform. Such tribal government should, however, be subjected to wise and firm regulation; and, without undue or petty interference, constant and active effort should be exercised to prevent barbarous practices and introduce 80 civilized customs." Placed in an alternative of either letting the natives alone or guiding them in the path of civilization, the American government chose "to adopt the latter measure as one more in accord with humanity and with 81 the national conscience." The Americans classified the Filipinos into two: the Christian Filipinos and the non-Christian Filipinos. The term "non-Christian" referred not to religious belief, but to a geographical area, and more directly, "to natives of the Philippine Islands of a low grade of civilization, usually living in tribal relationship apart 82 from settled communities."

L A W O N N A T U R A L R E S O U R C E S a n d E N V I R O N M E N T A L L A W C a s e s a n d S p e c i a l L a w s | 11
Like the Spaniards, the Americans pursued a policy of assimilation . In 1903, they passed Act No. 253 creating the Bureau of Non-Christian Tribes (BNCT). Under the Department of the Interior, the BNCT's primary task was to conduct ethnographic research among unhispanized Filipinos, including those in Muslim Mindanao, with a "special view to determining the most practicable means for bringing about their advancement in civilization and prosperity." The BNCT was modeled after the bureau dealing with American Indians. The agency took a keen anthropological interest in Philippine cultural minorities and 83 produced a wealth of valuable materials about them. The 1935 Constitution did not carry any policy on the non-Christian Filipinos. The raging issue then was the conservation of the national patrimony for the Filipinos. In 1957, the Philippine Congress passed R.A. No. 1888, an "Act to effectuate in a more rapid and complete manner the economic, social, moral and political advancement of the non-Christian Filipinos or national cultural minorities and to render real, complete, and permanent the integration of all said national cultural minorities into the body politic, creating the Commission on National Integration charged with said functions." The law called for a policy of integration of indigenous peoples into the Philippine 84 mainstream and for this purpose created theCommission on National Integration (CNI). The CNI was given, more or less, the same task as the BNCT during the American regime. The post-independence policy of integration was like the colonial policy of assimilation understood in the context of a guardian85 ward relationship. The policy of assimilation and integration did not yield the desired result. Like the Spaniards and Americans, government attempts at integration met with fierce resistance. Since World War II, a tidal wave of Christian settlers from the lowlands of Luzon and the Visayas swamped the highlands and wide 86 open spaces in Mindanao. Knowledge by the settlers of the Public Land Acts and the Torrens system resulted in the titling of several ancestral lands in the settlers' names. With government initiative and participation, this titling displaced several indigenous peoples from their lands. Worse, these peoples were also displaced by projects undertaken by the national government in the name of national 87 development. It was in the 1973 Constitution that the State adopted the following provision: "The State shall consider the customs, traditions, beliefs, and interests of national cultural communities in 88 the formulation and implementation of State policies." For the first time in Philippine history, the "non-Christian tribes" or the "cultural minorities" were addressed by the highest law of the Republic, and they were referred to as "cultural communities."More importantly this time, their "uncivilized" culture was given some recognition and their "customs, traditions, beliefs and interests" were to be considered by the State in the formulation and implementation of State policies. President Marcos abolished the CNI and transferred its functions to the Presidential Adviser on National Minorities (PANAMIN). The PANAMIN was tasked to integrate the ethnic groups that sought full integration into the larger community, and at the same time "protect the 89 rights of those who wish to preserve their original lifeways beside the larger community." In short, while still adopting the integration policy, the decree recognized the right of tribal Filipinos to preserve their 90 way of life. In 1974, President Marcos promulgated P.D. No. 410, otherwise known as the Ancestral Lands Decree. The decree provided for the issuance of land occupancy certificates to members of the national 91 cultural communities who were given up to 1984 to register their claims. In 1979, the Commission on the Settlement of Land Problems was created under E.O. No. 561 which provided a mechanism for the 92 expeditious resolution of land problems involving small settlers, landowners, and tribal Filipinos. Despite the promulgation of these laws, from 1974 to the early 1980's, some 100,000 Kalingas and Bontoks of the Cordillera region were displaced by the Chico River dam project of the National Power Corporation (NPC). The Manobos of Bukidnon saw their land bulldozed by the Bukidnon Sugar Industries Company (BUSCO). In Agusan del Sur, the National Development Company was authorized by law in 1979 to take approximately 40,550 hectares of land that later became the NDC-Guthrie plantation in Agusan 93 del Sur. Most of the land was possessed by the Agusan natives. Timber concessions, water projects, plantations, mining, and cattle ranching and other projects of the national government led not only to the eviction of the indigenous peoples from their land but also to the reduction and destruction of their 94 natural environment. The Aquino government signified a total shift from the policy of integration to one of preservation.Invoking her powers under the Freedom Constitution, President Aquino created the Office of Muslim Affairs, Office for Northern Cultural Communities and the Office for Southern Cultural 95 Communities all under the Office of the President. The 1987 Constitution carries at least six (6) provisions which insure the right of tribal Filipinos to 96 preserve their way of life. This Constitution goes further than the 1973 Constitution by expressly guaranteeing the rights of tribal Filipinos to their ancestral domains and ancestral lands. By recognizing their right to their ancestral lands and domains, the State has effectively upheld their right to live in a culture distinctly their own. 2. Their Concept of Land Indigenous peoples share distinctive traits that set them apart from the Filipino mainstream. They are non-Christians. They live in less accessible, marginal, mostly upland areas. They have a system of selfgovernment not dependent upon the laws of the central administration of the Republic of the Philippines. They follow ways of life and customs that are perceived as different from those of the rest of the 97 population. The kind of response the indigenous peoples chose to deal with colonial threat worked well to their advantage by making it difficult for Western concepts and religion to erode their customs and traditions. The "infieles societies" which had become peripheral to colonial administration, represented,

L A W O N N A T U R A L R E S O U R C E S a n d E N V I R O N M E N T A L L A W C a s e s a n d S p e c i a l L a w s | 12
from a cultural perspective, a much older base of archipelagic culture. The political systems were still structured on the patriarchal and kinship oriented arrangement of power and authority. The economic activities were governed by the concepts of an ancient communalism and mutual help. The social structure which emphasized division of labor and distinction of functions, not status, was maintained. The cultural styles and forms of life portraying the varieties of social courtesies and ecological adjustments 98 were kept constantly vibrant. Land is the central element of the indigenous peoples' existence. There is no traditional concept of permanent, individual, land ownership. Among the Igorots, ownership of land more accurately applies to the tribal right to use the land or to territorial control. The people are the secondary owners or stewards of the land and that if a member of the tribe ceases to work, he loses his claim of ownership, and the land reverts to the beings of the spirit world who are its true and primary owners. Under the concept of "trusteeship," the right to possess the land does not only belong to the present generation but the future 99 ones as well. Customary law on land rests on the traditional belief that no one owns the land except the gods and 100 spirits, and that those who work the land are its mere stewards. Customary law has a strong preference for communal ownership, which could either be ownership by a group of individuals or 101 families who are related by blood or by marriage, or ownership by residents of the same locality who may not be related by blood or marriage. The system of communal ownership under customary laws draws its meaning from the subsistence and highly collectivized mode of economic production. The Kalingas, for instance, who are engaged in team occupation like hunting, foraging for forest products, and swidden farming found it natural that forest areas, swidden farms, orchards, pasture and burial grounds 102 should be communally-owned. For the Kalingas, everybody has a common right to a common economic base. Thus, as a rule, rights and obligations to the land are shared in common. Although highly bent on communal ownership, customary law on land also sanctions individual ownership. The residential lots and terrace rice farms are governed by a limited system of individual ownership. It is limited because while the individual owner has the right to use and dispose of the property, he does not possess all the rights of an exclusive and full owner as defined under our Civil 103 Code. Under Kalinga customary law, the alienation of individually-owned land is strongly discouraged except in marriage and succession and except to meet sudden financial needs due to sickness, death in 104 the family, or loss of crops. Moreover, and to be alienated should first be offered to a clan-member 105 before any village-member can purchase it, and in no case may land be sold to a non-member of the ili. Land titles do not exist in the indigenous peoples' economic and social system. The concept of individual land ownership under the civil law is alien to them. Inherently colonial in origin, our national land laws and governmental policies frown upon indigenous claims to ancestral lands. Communal 106 ownership is looked upon as inferior, if not inexistent. III. THE IPRA IS A NOVEL PIECE OF LEGISLATION. A. The Legislative History of the IPRA It was to address the centuries-old neglect of the Philippine indigenous peoples that the Tenth Congress of the Philippines, by their joint efforts, passed and approved R.A. No. 8371, the Indigenous Peoples Rights Act (IPRA) of 1997. The law was a consolidation of two Bills- Senate Bill No. 1728 and House Bill No. 9125. Principally sponsored by Senator Juan M. Flavier, Senate Bill No. 1728 was a consolidation of four proposed measures referred to the Committees on Cultural Communities, Environment and Natural Resources, Ways and Means, as well as Finance. It adopted almost en toto the comprehensive version of Senate Bill Nos. 1476 and 1486 which was a result of six regional consultations and one national 108 consultation with indigenous peoples nationwide. At the Second Regular Session of the Tenth Congress, Senator Flavier, in his sponsorship speech, gave a background on the situation of indigenous peoples in the Philippines, to wit: "The Indigenous Cultural Communities, including the Bangsa Moro, have long suffered from the dominance and neglect of government controlled by the majority. Massive migration of their Christian brothers to their homeland shrunk their territory and many of the tribal Filipinos were pushed to the hinterlands. Resisting the intrusion, dispossessed of their ancestral land and with the massive exploitation of their natural resources by the elite among the migrant population, they became marginalized. And the government has been an indispensable party to this insidious conspiracy against the Indigenous Cultural Communities (ICCs). It organized and supported the resettlement of people to their ancestral land, which was massive during the Commonwealth and early years of the Philippine Republic. Pursuant to the Regalian Doctrine first introduced to our system by Spain through the Royal Decree of 13 February 1894 or the Maura Law, the government passed laws to legitimize the wholesale landgrabbing and provide for 109 easy titling or grant of lands to migrant homesteaders within the traditional areas of the ICCs." Senator Flavier further declared: "The IPs are the offsprings and heirs of the peoples who have first inhabited and cared for the land long before any central government was established. Their ancestors had territories over which they ruled themselves and related with other tribes. These territories- the land- include people, their dwelling, the mountains, the water, the air, plants, forest and the animals. This is their environment in its totality. Their existence as indigenous peoples is manifested in their own lives through political, economic, socio-cultural and spiritual practices. The IPs culture is the living and irrefutable proof to this. Their survival depends on securing or acquiring land rights; asserting their rights to it; and depending on 110 it. Otherwise, IPs shall cease to exist as distinct peoples." To recognize the rights of the indigenous peoples effectively, Senator Flavier proposed a bill based on two postulates: (1) the concept of native title; and (2) the principle of parens patriae.
107

L A W O N N A T U R A L R E S O U R C E S a n d E N V I R O N M E N T A L L A W C a s e s a n d S p e c i a l L a w s | 13
According to Senator Flavier, "[w]hile our legal tradition subscribes to the Regalian Doctrine reinstated in Section 2, Article XII of the 1987 Constitution," our "decisional laws" and jurisprudence passed by the State have "made exception to the doctrine." This exception was first laid down in the case of Cario v. Insular Governmentwhere: "x x x the court has recognized long occupancy of land by an indigenous member of the cultural communities as one of private ownership, which, in legal concept, is termed "native title." This ruling has 111 not been overturned. In fact, it was affirmed in subsequent cases." Following Cario, the State passed Act No. 926, Act No. 2874, C.A. No. 141, P.D. 705, P.D. 410, P.D. 1529, R.A. 6734 (the Organic Act for the Autonomous Region of Muslim Mindanao). These laws, explicitly or implicitly, and liberally or restrictively, recognized "native title" or "private right" and the existence of ancestral lands and domains. Despite the passage of these laws, however, Senator Flavier continued: "x x x the executive department of government since the American occupation has not implemented the policy. In fact, it was more honored in its breach than in its observance, its wanton disregard shown during the period unto the Commonwealth and the early years of the Philippine Republic when government organized and supported massive resettlement of the people to the land of the ICCs." Senate Bill No. 1728 seeks to genuinely recognize the IPs right to own and possess their ancestral land. The bill was prepared also under the principle of parens patriae inherent in the supreme power of the State and deeply embedded in Philippine legal tradition. This principle mandates that persons suffering from serious disadvantage or handicap, which places them in a position of actual inequality in their relation or transaction with others, are entitled to the protection of the State. Senate Bill No. 1728 was passed on Third Reading by twenty-one (21) Senators voting in favor and none 112 against, with no abstention. House Bill No. 9125 was sponsored by Rep. Zapata, Chairman of the Committee on Cultural Communities. It was originally authored and subsequently presented and defended on the floor by Rep. Gregorio 113 Andolana of North Cotabato. Rep. Andolana's sponsorhip speech reads as follows: "This Representation, as early as in the 8th Congress, filed a bill of similar implications that would promote, recognize the rights of indigenous cultural communities within the framework of national unity and development. Apart from this, Mr. Speaker, is our obligation, the government's obligation to assure and ascertain that these rights shall be well-preserved and the cultural traditions as well as the indigenous laws that remained long before this Republic was established shall be preserved and promoted. There is a need, Mr. Speaker, to look into these matters seriously and early approval of the substitute bill shall bring into reality the aspirations, the hope and the dreams of more than 12 million Filipinos that they be considered 114 in the mainstream of the Philippine society as we fashion for the year 2000." Rep. Andolana stressed that H.B. No. 9125 is based on the policy of preservation as mandated in the Constitution. He also emphasized that the rights of IPs to their land was enunciated in Cario v. Insular Government which recognized the fact that they had vested rights prior to the establishment of the 115 Spanish and American regimes. After exhaustive interpellation, House Bill No. 9125, and its corresponding amendments, was approved on Second Reading with no objections. IV. THE PROVISIONS OF THE IPRA DO NOT CONTRAVENE THE CONSTITUTION. A. Ancestral Domains and Ancestral Lands are the Private Property of Indigenous Peoples and Do Not Constitute Part of the Land of the Public Domain. The IPRA grants to ICCs/IPs a distinct kind of ownership over ancestral domains and ancestral lands.Ancestral lands are not the same as ancestral domains. These are defined in Section 3 [a] and [b] of the Indigenous Peoples Right Act, viz: "Sec. 3 a) Ancestral Domains. - Subject to Section 56 hereof, refer to all areas generally belonging to ICCs/IPs comprising lands, inland waters, coastal areas, and natural resources therein, held under a claim of ownership, occupied or possessed by ICCs/IPs by themselves or through their ancestors, communally or individually since time immemorial, continuously to the present except when interrupted by war, force majeure or displacement by force, deceit, stealth or as a consequence of government projects or any other voluntary dealings entered into by government and private individuals/corporations, and which are necessary to ensure their economic, social and cultural welfare. It shall include ancestral lands, forests, pasture, residential, agricultural, and other lands individually owned whether alienable and disposable or otherwise, hunting grounds, burial grounds, worship areas, bodies of water, mineral and other natural resources, and lands which may no longer be exclusively occupied by ICCs/IPs but from which they traditionally had access to for their subsistence and traditional activities, particularly the home ranges of ICCs/IPs who are still nomadic and/or shifting cultivators; b) Ancestral Lands.- Subject to Section 56 hereof, refers to land occupied, possessed and utilized by individuals, families and clans who are members of the ICCs/IPs since time immemorial, by themselves or through their predecessors-in-interest, under claims of individual or traditional group ownership, continuously, to the present except when interrupted by war, force majeure or displacement by force, deceit, stealth, or as a consequence of government projects and other voluntary dealings entered into by government and private individuals/corporations, including, but not limited to, residential lots, rice terraces or paddies, private forests, swidden farms and tree lots."

L A W O N N A T U R A L R E S O U R C E S a n d E N V I R O N M E N T A L L A W C a s e s a n d S p e c i a l L a w s | 14
Ancestral domains are all areas belonging to ICCs/IPs held under a claim of ownership, occupied or possessed by ICCs/IPs by themselves or through their ancestors, communally or individually since time immemorial, continuously until the present, except when interrupted by war, force majeure or displacement by force, deceit, stealth or as a consequence of government projects or any other voluntary dealings with government and/or private individuals or corporations. Ancestral domains comprise lands, inland waters, coastal areas, and natural resources therein and includes ancestral lands, forests, pasture, residential, agricultural, and other lands individually owned whether alienable or not, hunting grounds, burial grounds, worship areas, bodies of water, mineral and other natural resources. They also include lands which may no longer be exclusively occupied by ICCs/IPs but from which they traditionally had access to for their subsistence and traditional activities, particularly the home ranges of ICCs/IPs who 116 are still nomadic and/or shifting cultivators. Ancestral lands are lands held by the ICCs/IPs under the same conditions as ancestral domains except that these are limited to lands and that these lands are not merely occupied and possessed but are also utilized by the ICCs/IPs under claims of individual or traditional group ownership. These lands include but 117 are not limited to residential lots, rice terraces or paddies, private forests, swidden farms and tree lots. The procedures for claiming ancestral domains and lands are similar to the procedures embodied in Department Administrative Order (DAO) No. 2, series of 1993, signed by then Secretary of the 118 Department of Environment and Natural Resources (DENR) Angel Alcala. DAO No. 2 allowed the delineation of ancestral domains by special task forces and ensured the issuance of Certificates of Ancestral Land Claims (CALC's) and Certificates of Ancestral Domain Claims (CADC's) to IPs. The identification and delineation of these ancestral domains and lands is a power conferred by the IPRA 119 on the National Commission on Indigenous Peoples (NCIP). The guiding principle in identification and 120 delineation is self-delineation. This means that the ICCs/IPs have a decisive role in determining the 121 boundaries of their domains and in all the activities pertinent thereto. The procedure for the delineation and recognition of ancestral domains is set forth in Sections 51 and 52 of the IPRA. The identification, delineation and certification of ancestral lands is in Section 53 of said law. Upon due application and compliance with the procedure provided under the law and upon finding by the NCIP that the application is meritorious, the NCIP shall issue a Certificate of Ancestral Domain Title (CADT) 122 in the name of the community concerned. The allocation of lands within the ancestral domain to any individual or indigenous corporate (family or clan) claimants is left to the ICCs/IPs concerned to decide in 123 accordance with customs and traditions. With respect to ancestral lands outside the ancestral domain, 124 the NCIP issues a Certificate of Ancestral Land Title (CALT). CADT's and CALT's issued under the IPRA shall be registered by the NCIP before the Register of Deeds in 125 the place where the property is situated. (1) Right to Ancestral Domains and Ancestral Lands: How Acquired The rights of the ICCs/IPs to their ancestral domains and ancestral lands may be acquired in two modes: (1) bynative title over both ancestral lands and domains; or (2) by torrens title under the Public Land Act and the Land Registration Act with respect to ancestral lands only. (2) The Concept of Native Title Native title is defined as: "Sec. 3 [l]. Native Title- refers to pre-conquest rights to lands and domains which, as far back as memory reaches, have been held under a claim of private ownership by ICCs/IPs, have never been public lands and 126 are thusindisputably presumed to have been held that way since before the Spanish Conquest." Native title refers to ICCs/IPs' preconquest rights to lands and domains held under a claim of private ownership as far back as memory reaches. These lands are deemed never to have been public lands and are indisputably presumed to have been held that way since before the Spanish Conquest. The rights of ICCs/IPs to their ancestraldomains (which also include ancestral lands) by virtue of native title shall be 127 recognized and respected. Formal recognition, when solicited by ICCs/IPs concerned, shall be embodied in a Certificate of Ancestral Domain Title (CADT), which shall recognize the title of the concerned ICCs/IPs 128 over the territories identified and delineated. Like a torrens title, a CADT is evidence of private ownership of land by native title. Native title, however, is a right of private ownership peculiarly granted to ICCs/IPs over their ancestral lands and domains. The IPRA categorically declares ancestral lands and domains held by native title as never to have been public land. Domains and lands held under native title are, therefore, indisputably presumed to have never been public lands and are private. (a) Cario v. Insular Government
129

The concept of native title in the IPRA was taken from the 1909 case of Cario v. Insular 130 Government. Cario firmly established a concept of private land title that existed irrespective of any royal grant from the State. In 1903, Don Mateo Cario, an Ibaloi, sought to register with the land registration court 146 hectares of land in Baguio Municipality, Benguet Province. He claimed that this land had been possessed and occupied by his ancestors since time immemorial; that his grandfather built fences around the property for the holding of cattle and that his father cultivated some parts of the land. Cario inherited the land in accordance with Igorot custom. He tried to have the land adjusted under the Spanish land laws, but no 131 document issued from the Spanish Crown. In 1901, Cario obtained a possessory title to the land under 132 the Spanish Mortgage Law. The North American colonial government, however, ignored his possessory

L A W O N N A T U R A L R E S O U R C E S a n d E N V I R O N M E N T A L L A W C a s e s a n d S p e c i a l L a w s | 15
title and built a public road on the land prompting him to seek a Torrens title to his property in the land 133 registration court. While his petition was pending, a U.S. military reservation was proclaimed over his land and, shortly thereafter, a military detachment was detailed on the property with orders to keep 134 cattle and trespassers, including Cario, off the land. In 1904, the land registration court granted Cario's application for absolute ownership to the land. Both the Government of the Philippine Islands and the U.S. Government appealed to the C.F.I. of Benguet which reversed the land registration court and dismissed Cario's application. The Philippine Supreme 135 Court affirmed the C.F.I. by applying the Valenton ruling. Cario took the case to the U.S. Supreme 136 Court. On one hand, the Philippine government invoked the Regalian doctrine and contended that Cario failed to comply with the provisions of the Royal Decree of June 25, 1880, which required registration of land claims within a limited period of time. Cario, on the other, asserted that he was the absolute owner of the land jure gentium, and that the land never formed part of the public domain. In a unanimous decision written by Justice Oliver Wendell Holmes, the U.S. Supreme Court held: "It is true that Spain, in its earlier decrees, embodied the universal feudal theory that all lands were held from the Crown, and perhaps the general attitude of conquering nations toward people not recognized as entitled to the treatment accorded to those in the same zone of civilization with themselves. It is true, also, that in legal theory, sovereignty is absolute, and that, as against foreign nations, the United States may assert, as Spain asserted, absolute power. But it does not follow that, as against the inhabitants of the Philippines, the United States asserts that Spain had such power. When theory is left on one side, sovereignty is a question of strength, and may vary in degree. How far a new sovereign shall insist upon the theoretical relation of the subjects to the head in the past, and how far it shall recognize actual facts, 137 are matters for it to decide." The U.S. Supreme Court noted that it need not accept Spanish doctrines. The choice was with the new colonizer. Ultimately, the matter had to be decided under U.S. law. The Cario decision largely rested on the North American constitutionalist's concept of "due process" as 138 well as the pronounced policy "to do justice to the natives." It was based on the strong mandate extended to the Islands via the Philippine Bill of 1902 that "No law shall be enacted in said islands which shall deprive any person of life, liberty, or property without due process of law, or deny to any person therein the equal protection of the laws." The court declared: "The acquisition of the Philippines was not like the settlement of the white race in the United States. Whatever consideration may have been shown to the North American Indians, the dominant purpose of the whites in America was to occupy land. It is obvious that, however stated, the reason for our taking over the Philippines was different. No one, we suppose, would deny that, so far as consistent with paramount necessities, our first object in the internal administration of the islands is to do justice to the natives, not to exploit their country for private gain. By the Organic Act of July 1, 1902, chapter 1369, "Every presumption is and ought to be against the government in a case like the present. It might, perhaps, be proper and sufficient to say that when, as far back as testimony or memory goes, the land has been held by individuals under a claim of private ownership, it will be presumed to have been held in the same way from before the Spanish conquest, and never to have been public land. Certainly in a case like this, if there is doubt or ambiguity in the Spanish law, we ought to give the applicant the benefit 140 of the doubt." The court thus laid down the presumption of a certain title held (1) as far back as testimony or memory went, and (2) under a claim of private ownership. Land held by this title is presumed to "never have been public land." Against this presumption, the U.S. Supreme Court analyzed the Spanish decrees upheld in the 1904 decision ofValenton v. Murciano. The U.S. Supreme Court found no proof that the Spanish decrees did not honor native title. On the contrary, the decrees discussed in Valenton appeared to recognize that the natives owned some land, irrespective of any royal grant. The Regalian doctrine declared in the preamble of the Recopilacion was all "theory and discourse" and it was observed that titles were admitted to exist beyond the powers of the Crown, viz: "If the applicant's case is to be tried by the law of Spain, we do not discover such clear proof that it was bad by that law as to satisfy us that he does not own the land. To begin with, the older decrees and laws cited by the counsel for the plaintiff in error seem to indicate pretty clearly that the natives were recognized as owning some lands, irrespective of any royal grant. In other words, Spain did not assume to convert all the native inhabitants of the Philippines into trespassers or even into tenants at will. For instance, Book 4, title 12, Law 14 of the the Recopilacion de Leyes de las Indias, cited for a contrary conclusion in Valenton v. Murciano, 3 Philippine 537, while it commands viceroys and others, when it seems proper, to call for the exhibition of grants, directs them to confirm those who hold by good grants or justa prescripcion. It is true that it begins by the characteristic assertion of feudal overlordship and section 12 (32 Statutes at Large, 691), all the property and rights acquired there by the United States are to be administered 'for the benefit of the inhabitants thereof.' It is reasonable to suppose that the attitude thus assumed by the United States with regard to what was unquestionably its own is also its attitude in deciding what it will claim for its own. The same statute made a bill of rights, embodying the safeguards of the Constitution, and, like the Constitution, extends those safeguards to all. It provides that 'no law shall be enacted in said islands which shall deprive any person of life, liberty, or property without due process of law, or deny to any person therein the equal protection of the laws.' In the light of the declaration that we have quoted from section 12, it is hard to believe that the United States was ready to declare in the next breath that "any person" did not embrace the inhabitants of Benguet, or that it meant by "property" only that which had become such by ceremonies of which presumably a large part of the inhabitants never had heard, and that it proposed to treat as public land what they, by native custom and 139 by long association,- of the profoundest factors in human thought,- regarded as their own." The Court went further:

L A W O N N A T U R A L R E S O U R C E S a n d E N V I R O N M E N T A L L A W C a s e s a n d S p e c i a l L a w s | 16
the origin of all titles in the King or his predecessors. That was theory and discourse. The fact was that titles were admitted to exist that owed nothing to the powers of Spain beyond this recognition in their 141 books." (Emphasis supplied). The court further stated that the Spanish "adjustment" proceedings never held sway over unconquered territories. The wording of the Spanish laws were not framed in a manner as to convey to the natives that failure to register what to them has always been their own would mean loss of such land. The registration requirement was "not to confer title, but simply to establish it;" it was "not calculated to convey to the mind of an Igorot chief the notion that ancient family possessions were in danger, if he had read every word of it." By recognizing this kind of title, the court clearly repudiated the doctrine of Valenton. It was frank enough, however, to admit the possibility that the applicant might have been deprived of his land under Spanish law because of the inherent ambiguity of the decrees and concomitantly, the various interpretations which may be given them. But precisely because of the ambiguity and of the strong "due 142 process mandate" of the Constitution, the court validated this kind of title. This title was sufficient, even without government administrative action, and entitled the holder to a Torrens certificate. Justice Holmes explained: "It will be perceived that the rights of the applicant under the Spanish law present a problem not without difficulties for courts of a legal tradition. We have deemed it proper on that account to notice the possible effect of the change of sovereignty and the act of Congress establishing the fundamental principles now to be observed. Upon a consideration of the whole case we are of the opinion that law and justice require that the applicant should be granted what he seeks, and should not be deprived of what, by the practice and belief of those among whom he lived, was his property, through a refined interpretation of an almost 143 forgotten law of Spain." Thus, the court ruled in favor of Cario and ordered the registration of the 148 hectares in Baguio 144 Municipality in his name. Examining Cario closer, the U.S. Supreme Court did not categorically refer to the title it upheld as "native title." It simply said: "The Province of Benguet was inhabited by a tribe that the Solicitor-General, in his argument, characterized as a savage tribe that never was brought under the civil or military government of the Spanish Crown. It seems probable, if not certain, that the Spanish officials would not have granted to anyone in that province the registration to which formerly the plaintiff was entitled by the Spanish Laws, and which would have made his title beyond question good. Whatever may have been the technical position of Spain it does not follow that, in the view of the United States, he had lost all rights and was a mere trespasser when the present government seized his land. The argument to that effect seems to amount to a denial of native titles through an important part of the Island of Luzon, at least, for the want of ceremonies which the Spaniards would not have permitted and had not the power to 145 enforce." This is the only instance when Justice Holmes used the term "native title" in the entire length of the Cariodecision. It is observed that the widespread use of the term "native title" may be traced to Professor Owen James Lynch, Jr., a Visiting Professor at the University of the Philippines College of Law from the Yale University Law School. In 1982, Prof. Lynch published an article in the Philippine Law 146 Journal entitled Native Title, Private Right and Tribal Land Law. This article was made after Professor Lynch visited over thirty tribal communities throughout the country and studied the origin and 147 development of Philippine land laws. He discussed Carioextensively and used the term "native title" to refer to Cario's title as discussed and upheld by the U.S. Supreme Court in said case. (b) Indian Title In a footnote in the same article, Professor Lynch stated that the concept of "native title" as defined by 148 Justice Holmes in Cario "is conceptually similar to "aboriginal title" of the American Indians. This is not surprising, according to Prof. Lynch, considering that during the American regime, government policy 149 towards ICCs/IPs was consistently made in reference to native Americans. This was clearly 150 demonstrated in the case of Rubi v. Provincial Board of Mindoro. In Rubi, the Provincial Board of Mindoro adopted a Resolution authorizing the provincial governor to remove the Mangyans from their domains and place them in a permanent reservation in Sitio Tigbao, Lake Naujan. Any Mangyan who refused to comply was to be imprisoned. Rubi and some Mangyans, including one who was imprisoned for trying to escape from the reservation, filed for habeas corpus claiming deprivation of liberty under the Board Resolution. This Court denied the petition on the ground of police power. It upheld government policy promoting the idea that a permanent settlement was the only successful method for educating the Mangyans, introducing civilized customs, improving their health 151 and morals, and protecting the public forests in which they roamed. Speaking through Justice Malcolm, the court said: "Reference was made in the President's instructions to the Commission to the policy adopted by the United States for the Indian Tribes. The methods followed by the Government of the Philippine Islands in its dealings with the so-called non-Christian people is said, on argument, to be practically identical with that followed by the United States Government in its dealings with the Indian tribes. Valuable lessons, it is insisted, can be derived by an investigation of the American-Indian policy. From the beginning of the United States, and even before, the Indians have been treated as "in a state of pupilage." The recognized relation between the Government of the United States and the Indians may be described as that of guardian and ward. It is for the Congress to determine when and how the guardianship shall be terminated. The Indians are always subject to the plenary authority of the United 152 States.

L A W O N N A T U R A L R E S O U R C E S a n d E N V I R O N M E N T A L L A W C a s e s a n d S p e c i a l L a w s | 17
x x x. As to the second point, the facts in the Standing Bear case and the Rubi case are not exactly identical. But even admitting similarity of facts, yet it is known to all that Indian reservations do exist in the United States, that Indians have been taken from different parts of the country and placed on these reservations, without any previous consultation as to their own wishes, and that, when once so located, they have been made to remain on the reservation for their own good and for the general good of the country. If any lesson can be drawn from the Indian policy of the United States, it is that the determination of this policy is for the legislative and executive branches of the government and that when once so decided upon, the courts should not interfere to upset a carefully planned governmental system. Perhaps, just as many forceful reasons exist for the segregation of the Manguianes in Mindoro as existed for the segregation of 153 the different Indian tribes in the United States." Rubi applied the concept of Indian land grants or reservations in the Philippines. An Indian reservation is a part of the public domain set apart by proper authority for the use and occupation of a tribe or tribes of 154 Indians. It may be set apart by an act of Congress, by treaty, or by executive order, but it cannot be 155 established by custom and prescription. Indian title to land, however, is not limited to land grants or reservations. It also covers the "aboriginal 156 right of possession or occupancy." The aboriginal right of possession depends on the actual occupancy of the lands in question by the tribe or nation as their ancestral home, in the sense that such lands 157 constitute definable territory occupied exclusively by the particular tribe or nation. It is a right which exists apart from any treaty, statute, or other governmental action, although in numerous instances treaties have been negotiated with Indian tribes, recognizing their aboriginal possession and delimiting 158 their occupancy rights or settling and adjusting their boundaries. American jurisprudence recognizes the Indians' or native Americans' rights to land they have held and occupied before the "discovery" of the Americas by the Europeans. The earliest definitive statement by the U.S. Supreme Court on the nature of aboriginal title was made in 1823 in Johnson & Graham's 159 Lessee v. M'Intosh. In Johnson, the plaintiffs claimed the land in question under two (2) grants made by the chiefs of two (2) Indian tribes. The U.S. Supreme Court refused to recognize this conveyance, the plaintiffs being private persons. The only conveyance that was recognized was that made by the Indians to the government of the European discoverer. Speaking for the court, Chief Justice Marshall pointed out that the potentates of the old world believed that they had made ample compensation to the inhabitants of the new world by bestowing civilization and Christianity upon them; but in addition, said the court, they found it necessary, in order to avoid conflicting settlements and consequent war, to establish the principle that discovery gives title to the government by whose subjects, or by whose authority, the discovery was made, 160 against all other European governments, which title might be consummated by possession. The exclusion of all other Europeans gave to the nation making the discovery the sole right of acquiring the soil from the natives and establishing settlements upon it. As regards the natives, the court further stated that: "Those relations which were to exist between the discoverer and the natives were to be regulated by themselves. The rights thus acquired being exclusive, no other power could interpose between them. In the establishment of these relations, the rights of the original inhabitants were, in no instance, entirely disregarded; but were necessarily, to a considerable extent, impaired. They were admitted to be the rightful occupants of the soil, with a legal as well as just claim to retain possession of it, and to use itaccording to their own discretion; but their rights to complete sovereignty, as independent nations, were necessarily diminished, and their power to dispose of the soil at their own will, to whomsoever they pleased, was denied by the fundamental principle that discovery gave exclusive title to those who made it. While the different nations of Europe respected the right of the natives as occupants, they asserted the ultimate dominion to be in themselves; and claimed and exercised, as a consequence of this ultimate dominion, a power to grant the soil, while yet in possession of the natives. These grants have been 161 understood by all to convey a title to the grantees, subject only to the Indian right of occupancy." Thus, the discoverer of new territory was deemed to have obtained the exclusive right to acquire Indian land and extinguish Indian titles. Only to the discoverer- whether to England, France, Spain or Holland- did this right belong and not to any other nation or private person. The mere acquisition of the right nonetheless did not extinguish Indian claims to land. Rather, until the discoverer, by purchase or conquest, exercised its right, the concerned Indians were recognized as the "rightful occupants of the soil, with a legal as well as just claim to retain possession of it." Grants made by the discoverer to her subjects of lands occupied by the Indians were held to convey a title to the grantees, subject only to the Indian right of occupancy. Once the discoverer purchased the land from the Indians or conquered them, it was only then that the discoverer gained an absolute title unrestricted by Indian rights. The court concluded, in essence, that a grant of Indian lands by Indians could not convey a title paramount to the title of the United States itself to other parties, saying: "It has never been contended that the Indian title amounted to nothing. Their right of possession has never been questioned. The claim of government extends to the complete ultimate title, charged with 162 this right of possession, and to the exclusive power of acquiring that right." It has been said that the history of America, from its discovery to the present day, proves the universal 163 recognition of this principle. The Johnson doctrine was a compromise. It protected Indian rights and their native lands without having 164 to invalidate conveyances made by the government to many U.S. citizens.

L A W O N N A T U R A L R E S O U R C E S a n d E N V I R O N M E N T A L L A W C a s e s a n d S p e c i a l L a w s | 18
Johnson was reiterated in the case of Worcester v. Georgia. In this case, the State of Georgia enacted a law requiring all white persons residing within the Cherokee nation to obtain a license or permit from the Governor of Georgia; and any violation of the law was deemed a high misdemeanor. The plaintiffs, who were white missionaries, did not obtain said license and were thus charged with a violation of the Act. The U.S. Supreme Court declared the Act as unconstitutional for interfering with the treaties established between the United States and the Cherokee nation as well as the Acts of Congress regulating intercourse with them. It characterized the relationship between the United States government and the Indians as: "The Indian nations were, from their situation, necessarily dependent on some foreign potentate for the supply of their essential wants, and for their protection from lawless and injurious intrusions into their country. That power was naturally termed their protector. They had been arranged under the protection of Great Britain; but the extinguishment of the British power in their neighborhood, and the establishment of that of the United States in its place, led naturally to the declaration, on the part of the Cherokees, that they were under the protection of the United States, and of no other power. They assumed the relation with the United States which had before subsisted with Great Britain. This relation was that of a nation claiming and receiving the protection of one more powerful, not that of 166 individuals abandoning their national character, and submitting as subjects to the laws of a master." It was the policy of the U.S. government to treat the Indians as nations with distinct territorial boundaries and recognize their right of occupancy over all the lands within their domains. Thus: "From the commencement of our government Congress has passed acts to regulate trade and intercourse with the Indians; which treat them as nations, respect their rights, and manifest a firm purpose to afford that protection which treaties stipulate. All these acts, and especially that of 1802, which is still in force, manifestly consider the several Indian nations as distinct political communities, having territorial boundaries, within which their authority is exclusive, and having a right to all the lands within those boundaries, which is not only acknowledged, but guaranteed by the United States. x x x. "The Indian nations had always been considered as distinct, independent political communities, retaining their original natural rights, as the undisputed possessors of the soil from time immemorial,with the single exception of that imposed by irresistible power, which excluded them from intercourse with any other European potentate than the first discoverer of the coast of the particular region claimed: and this was a restriction which those European potentates imposed on themselves, as well as on the Indians. The very term "nation," so generally applied to them, means "a people distinct 167 from others." x x x.
165

The Cherokee nation, then, is a distinct community, occupying its own territory, with boundaries accurately described, in which the laws of Georgia can have no force, and which the citizens of Georgia have no right to enter but with the assent of the Cherokees themselves or in conformity with treaties and with the acts of Congress. The whole intercourse between the United States and this nation is, by our 168 Constitution and laws, vested in the government of the United States." The discovery of the American continent gave title to the government of the discoverer as against all 169 other European governments. Designated as the naked fee, this title was to be consummated by possession and was subject to the Indian title of occupancy. The discoverer acknowledged the Indians' legal and just claim to retain possession of the land, the Indians being the original inhabitants of the land. The discoverer nonetheless asserted the exclusive right to acquire the Indians' land- either by purchase, "defensive" conquest, or cession- and in so doing, extinguish the Indian title. Only the discoverer could extinguish Indian title because it alone asserted ultimate dominion in itself. Thus, while the different nations of Europe respected the rights of the natives as occupants, they all asserted the ultimate 170 dominion and title to be in themselves. As early as the 19th century, it became accepted doctrine that although fee title to the lands occupied by the Indians when the colonists arrived became vested in the sovereign- first the discovering European nation and later the original 13 States and the United States- a right of occupancy in the Indian tribes was nevertheless recognized. The Federal Government continued the policy of respecting the Indian right of occupancy, sometimes called Indian title, which it accorded the protection of complete 171 ownership. But this aboriginal Indian interest simply constitutes "permission" from the whites to occupy 172 the land, and means mere possession not specifically recognized as ownership by Congress. It is clear 173 that this right of occupancy based upon aboriginal possession is not a property right. It is vulnerable to affirmative action by the federal government who, as sovereign, possessed exclusive power to extinguish 174 the right of occupancy at will. Thus, aboriginal title is not the same as legal title. Aboriginal title rests 175 on actual, exclusive and continuous use and occupancy for a long time. It entails that land owned by Indian title must be used within the tribe, subject to its laws and customs, and cannot be sold to another 176 sovereign government nor to any citizen. Such title as Indians have to possess and occupy land is in the tribe, and not in the individual Indian; the right of individual Indians to share in the tribal property usually depends upon tribal membership, the property of the tribe generally being held in communal 177 ownership. As a rule, Indian lands are not included in the term "public lands," which is ordinarily used to designate 178 such lands as are subject to sale or other disposal under general laws. Indian land which has been 179 abandoned is deemed to fall into the public domain. On the other hand, an Indian reservation is a part 180 of the public domain set apart for the use and occupation of a tribe of Indians. Once set apart by proper authority, the reservation ceases to be public land, and until the Indian title is extinguished, no one but 181 Congress can initiate any preferential right on, or restrict the nation's power to dispose of, them. The American judiciary struggled for more than 200 years with the ancestral land claims of indigenous 182 Americans. And two things are clear. First, aboriginal title is recognized. Second, indigenous property

L A W O N N A T U R A L R E S O U R C E S a n d E N V I R O N M E N T A L L A W C a s e s a n d S p e c i a l L a w s | 19
systems are also recognized. From a legal point of view, certain benefits can be drawn from a comparison 183 of Philippine IPs to native Americans. Despite the similarities between native title and aboriginal title, however, there are at present some misgivings on whether jurisprudence on American Indians may be cited authoritatively in the Philippines. The U.S. recognizes the possessory rights of the Indians over their land; title to the land, however, is deemed to have passed to the U.S. as successor of the discoverer. The aboriginal title of ownership is not specifically recognized as ownership by action authorized by 184 Congress. The protection of aboriginal title merely guards against encroachment by persons other than 185 the Federal Government. Although there are criticisms against the refusal to recognize the native 186 Americans' ownership of these lands, the power of the State to extinguish these titles has remained 187 firmly entrenched. Under the IPRA, the Philippine State is not barred form asserting sovereignty over the ancestral domains 188 and ancestral lands. The IPRA, however, is still in its infancy and any similarities between its application in the Philippines vis--vis American Jurisprudence on aboriginal title will depend on the peculiar facts of each case. (c) Why the Cario doctrine is unique In the Philippines, the concept of native title first upheld in Cario and enshrined in the IPRA grants ownership, albeit in limited form, of the land to the ICCs/IPs. Native title presumes that the land is private and was never public. Cario is the only case that specifically and categorically recognizes native title. The long line of cases citing Cario did not touch on native title and the private character of ancestral domains and lands. Cario was cited by the succeeding cases to support the concept of acquisitive prescription under the Public Land Act which is a different matter altogether. Under the Public Land Act, land sought to be registered must be public agricultural land. When the conditions specified in Section 48 [b] of the Public Land Act are complied with, the possessor of the land is deemed to have acquired, by 189 190 operation of law, a right to a grant of the land. The land ceases to be part of the public domain, ipso 191 jure, and is converted to private property by the mere lapse or completion of the prescribed statutory period. It was only in the case of Oh Cho v. Director of Lands that the court declared that the rule that all lands that were not acquired from the government, either by purchase or grant, belong to the public domain has an exception. This exception would be any land that should have been in the possession of an occupant and of his predecessors-in-interest since time immemorial. It is this kind of possession that would justify the presumption that the land had never been part of the public domain or that it had been 193 private property even before the Spanish conquest. Oh Cho, however, was decided under the provisions of the Public Land Act and Cario was cited to support the applicant's claim of acquisitive prescription under the said Act. All these years, Cario had been quoted out of context simply to justify long, continuous, open and adverse possession in the concept of owner of public agricultural land. It is this long, continuous, open 194 and adverse possession in the concept of owner of thirty years both for ordinary citizens and members
192

of the national cultural minorities that converts the land from public into private and entitles the registrant to a torrens certificate of title. (3) The Option of Securing a Torrens Title to the Ancestral Land Indicates that the Land is Private. The private character of ancestral lands and domains as laid down in the IPRA is further strengthened by the option given to individual ICCs/IPs over their individually-owned ancestral lands. For purposes of registration under the Public Land Act and the Land Registration Act, the IPRA expressly converts ancestral land into public agricultural land which may be disposed of by the State. The necessary implication is thatancestral land is private. It, however, has to be first converted to public agricultural land simply for registration purposes. To wit: "Sec. 12. Option to Secure Certificate of Title Under Commonwealth Act 141, as amended, or the Land Registration Act 496- Individual members of cultural communities, with respect to their individuallyowned ancestral lands who, by themselves or through their predecessors-in-interest, have been in continuous possession and occupation of the same in the concept of owner since time immemorial or for a period of not less than thirty (30) years immediately preceding the approval of this Act and uncontested by the members of the same ICCs/IPs shall have the option to secure title to their ancestral lands under the provisions of Commonwealth Act 141, as amended, or the Land Registration Act 496. For this purpose, said individually-owned ancestral lands, which are agricultural in character and actually used for agricultural, residential, pasture, and tree farming purposes, including those with a slope of eighteen percent (18%) or more, are hereby classified as alienable and disposable agricultural lands. The option granted under this section shall be exercised within twenty (20) years from the approval of this 196 Act." ICCs/IPs are given the option to secure a torrens certificate of title over their individually-owned ancestral lands. This option is limited to ancestral lands only, not domains, and such lands must be individually, not communally, owned. Ancestral lands that are owned by individual members of ICCs/IPs who, by themselves or through their predecessors-in-interest, have been in continuous possession and occupation of the same in the concept 197 of owner since time immemorial or for a period of not less than 30 years, which claims are uncontested by the members of the same ICCs/IPs, may be registered under C.A. 141, otherwise known as the Public Land Act, or Act 496, the Land Registration Act. For purposes of registration, the individually-owned ancestral lands are classified as alienable and disposable agricultural lands of the public domain, provided, they are agricultural in character and are actually used for agricultural, residential, pasture and tree farming purposes. These lands shall be classified as public agricultural lands regardless of whether they have a slope of 18% or more.

195

L A W O N N A T U R A L R E S O U R C E S a n d E N V I R O N M E N T A L L A W C a s e s a n d S p e c i a l L a w s | 20
The classification of ancestral land as public agricultural land is in compliance with the requirements of the Public Land Act and the Land Registration Act. C.A. 141, the Public Land Act, deals specifically with 198 lands of the public domain. Its provisions apply to those lands "declared open to disposition or concession" x x x "which have not been reserved for public or quasi-public purposes, nor appropriated by the Government, nor in any manner become private property, nor those on which a private right authorized and recognized by this Act or any other valid law x x x or which having been reserved or 199 appropriated, have ceased to be so." Act 496, the Land Registration Act, allows registration only of private lands and public agricultural lands. Since ancestral domains and lands are private, if the ICC/IP wants to avail of the benefits of C.A. 141 and Act 496, the IPRA itself converts his ancestral land, 200 regardless of whether the land has a slope of eighteen per cent (18%) or over, from private to public agricultural land for proper disposition. The option to register land under the Public Land Act and the Land Registration Act has nonetheless a limited period. This option must be exercised within twenty (20) years from October 29, 1997, the date of approval of the IPRA. Thus, ancestral lands and ancestral domains are not part of the lands of the public domain. They are private and belong to the ICCs/IPs. Section 3 of Article XII on National Economy and Patrimony of the 1987 Constitution classifies lands of the public domain into four categories: (a) agricultural, (b) forest or timber, (c) mineral lands, and (d) national parks. Section 5 of the same Article XII mentions ancestral lands and ancestral domains but it does not classify them under any of the said four categories. To classify them as public lands under any one of the four classes will render the entire IPRA law a nullity. The spirit of the IPRA lies in the distinct concept of ancestral domains and ancestral lands. The IPRA addresses the major problem of the ICCs/IPs which is loss of land. Land and space are of vital concern in terms of 201 sheer survival of the ICCs/IPs. The 1987 Constitution mandates the State to "protect the rights of indigenous cultural communities to their ancestral lands" and that "Congress provide for the applicability of customary laws x x x in 202 determining the ownership and extent of ancestral domain." It is the recognition of the ICCs/IPs distinct rights of ownership over their ancestral domains and lands that breathes life into this constitutional mandate. B. The right of ownership and possession by the ICCs/IPs of their ancestral domains is a limited form of ownership and does not include the right to alienate the same. Registration under the Public Land Act and Land Registration Act recognizes the concept of ownership under thecivil law. This ownership is based on adverse possession for a specified period, and harkens to Section 44 of the Public Land Act on administrative legalization (free patent) of imperfect or incomplete titles and Section 48 (b) and (c) of the same Act on the judicial confirmation of imperfect or incomplete titles. Thus: "Sec. 44. Any natural-born citizen of the Philippines who is not the owner of more than twenty-four hectares and who since July fourth, 1926 or prior thereto, has continuously occupied and cultivated, either by himself or through his predecessors-in-interest, a tract or tracts of agricultural public lands subject to disposition, or who shall have paid the real estate tax thereon while the same has not been occupied by any person shall be entitled, under the provisions of this chapter, to have a free patent issued to him for such tract or tracts of such land not to exceed twenty-four hectares. A member of the national cultural minorities who has continuously occupied and cultivated, either by himself or through his predecessors-in-interest, a tract or tracts of land, whether disposable or not since July 4, 1955, shall be entitled to the right granted in the preceding paragraph of this section:Provided, That at the time he files his free patent application he is not the owner of any real property secured or 203 disposable under the provision of the Public Land Law. x x x. "Sec. 48. The following described citizens of the Philippines, occupying lands of the public domain or claiming to own any such lands or an interest therein, but whose titles have not been perfected or completed, may apply to the Court of First Instance of the province where the land is located for confirmation of their claims and the issuance of a certificate of title therefor, under the Land Registration Act, to wit: (a) [perfection of Spanish titles] xxx. (b) Those who by themselves or through their predecessors-in-interest have been in open, continuous, exclusive, and notorious possession and occupation of agricultural lands of the public domain, under a bona fide claim of acquisition or ownership, for at least thirty years immediately preceding the filing of the application for confirmation of title except when prevented by war or force majeure. These shall be conclusively presumed to have performed all the conditions essential to a Government grant and shall be entitled to a certificate of title under the provisions of this Chapter. (c) Members of the national cultural minorities who by themselves or through their predecessors-in-interest have been in open, continuous, exclusive and notorious possession and occupation of lands of the public domain suitable to agriculture, whether disposable or not, under a bona fide claim of ownership for at least 30 years shall be entitled to the rights 204 granted in sub-section (b) hereof." Registration under the foregoing provisions presumes that the land was originally public agricultural land but because of adverse possession since July 4, 1955 (free patent) or at least thirty years (judicial confirmation), the land has become private. Open, adverse, public and continuous possession is sufficient,

L A W O N N A T U R A L R E S O U R C E S a n d E N V I R O N M E N T A L L A W C a s e s a n d S p e c i a l L a w s | 21
provided, the possessor makes proper application therefor. The possession has to be confirmed judicially or administratively after which a torrens title is issued. A torrens title recognizes the owner whose name appears in the certificate as entitled to all the rights of ownership under the civil law. The Civil Code of the Philippines defines ownership in Articles 427, 428 and 429. This concept is based on Roman Law which the Spaniards introduced to the Philippines through the Civil Code of 1889. Ownership, under Roman Law, may be exercised over things or rights. It primarily includes the right of the owner to enjoy and dispose of the thing owned. And the right to enjoy and 205 dispose of the thing includes the right to receive from the thing what it produces, the right to consume 206 207 the thing by its use, the right to alienate, encumber, transform or even destroy the thing owned, and the right to exclude from the possession of the thing owned by any other person to whom the owner has 208 not transmitted such thing. 1. The Indigenous Concept of Ownership and Customary Law. Ownership of ancestral domains by native title does not entitle the ICC/IP to a torrens title but to a Certificate of Ancestral Domain Title (CADT). The CADT formally recognizes the indigenous concept of ownership of the ICCs/IPs over their ancestral domain. Thus: "Sec. 5. Indigenous concept of ownership.- Indigenous concept of ownership sustains the view that ancestral domains and all resources found therein shall serve as the material bases of their cultural integrity. The indigenous concept of ownership generally holds that ancestral domains are the ICCs/IPs private but community property which belongs to all generations and therefore cannot be sold, disposed or destroyed. It likewise covers sustainable traditional resource rights." The right of ownership and possession of the ICCs/IPs to their ancestral domains is held under the indigenous concept of ownership. This concept maintains the view that ancestral domains are the ICCs/IPs private but community property. It is private simply because it is not part of the public domain. But its private character ends there. The ancestral domain is owned in common by the ICCs/IPs and not by one particular person. The IPRA itself provides that areas within the ancestral domains, whether 209 delineated or not, are presumed to be communally held. These communal rights, however, are not 210 exactly the same as co-ownership rights under the Civil Code. Co-ownership gives any co-owner the right to demand partition of the property held in common. The Civil Code expressly provides that "no coowner shall be obliged to remain in the co-ownership." Each co-owner may demand at any time the 211 partition of the thing in common, insofar as his share is concerned. To allow such a right over ancestral domains may be destructive not only of customary law of the community but of the very community 212 itself. Communal rights over land are not the same as corporate rights over real property, much less corporate condominium rights. A corporation can exist only for a maximum of fifty (50) years subject to an 213 extension of another fifty years in any single instance. Every stockholder has the right to disassociate Following the constitutional mandate that "customary law govern property rights or relations in 216 determining the ownership and extent of ancestral domains," the IPRA, by legislative fiat, introduces a 217 new concept of ownership. This is a concept that has long existed under customary law. Custom, from which customary law is derived, is also recognized under the Civil Code as a source of 218 law. Some articles of the Civil Code expressly provide that custom should be applied in cases where no 219 codal provision is applicable. In other words, in the absence of any applicable provision in the Civil 220 Code, custom, when duly proven, can define rights and liabilities. Customary law is a primary, not secondary, source of rights under the IPRA and uniquely applies to ICCs/IPs. Its recognition does not depend on the absence of a specific provision in the civil law. The indigenous concept of ownership under customary law is specifically acknowledged and recognized, and 221 coexists with the civil law concept and the laws on land titling and land registration. To be sure, the indigenous concept of ownership exists even without a paper title. The CADT is merely a "formal recognition" of native title. This is clear from Section 11 of the IPRA, to wit: "Sec. 11. Recognition of Ancestral Domain Rights.- The rights of ICCs/IPs to their ancestral domains by virtue of Native Title shall be recognized and respected. Formal recognition, when solicited by ICCs/IPs concerned shall be embodied in a Certificate of Ancestral Domain Title, which shall recognize the title of the concerned ICCs/IPs over the territories identified and delineated." The moral import of ancestral domain, native land or being native is "belongingness" to the land, being people of the land- by sheer force of having sprung from the land since time beyond recall, and the faithful nurture of the land by the sweat of one's brow. This is fidelity of usufructuary relation to the landthe possession of stewardship through perduring, intimate tillage, and the mutuality of blessings between 222 man and land; from man, care for land; from the land, sustenance for man. himself from the corporation. 215 involuntarily.
214

Moreover, the corporation itself may be dissolved voluntarily or

Communal rights to the land are held not only by the present possessors of the land but extends to all generations of the ICCs/IPs, past, present and future, to the domain. This is the reason why the ancestral domain must be kept within the ICCs/IPs themselves. The domain cannot be transferred, sold or conveyed to other persons. It belongs to the ICCs/IPs as a community. Ancestral lands are also held under the indigenous concept of ownership. The lands are communal. These lands, however, may be transferred subject to the following limitations: (a) only to the members of the same ICCs/IPs; (b) in accord with customary laws and traditions; and (c) subject to the right of redemption of the ICCs/IPs for a period of 15 years if the land was transferred to a non-member of the ICCs/IPs.

L A W O N N A T U R A L R E S O U R C E S a n d E N V I R O N M E N T A L L A W C a s e s a n d S p e c i a l L a w s | 22
C. Sections 7 (a), 7 (b) and 57 of the IPRA Do Not Violate the Regalian Doctrine Enshrined in Section 2, Article XII of the 1987 Constitution. 1. The Rights of ICCs/IPs Over Their Ancestral Domains and Lands The IPRA grants the ICCs/IPs several rights over their ancestral domains and ancestral lands. Section 7 provides for the rights over ancestral domains: "Sec. 7. Rights to Ancestral Domains.- The rights of ownership and possession of ICCs/IPs to their ancestral domains shall be recognized and protected. Such rights include: a) Right of Ownership.- The right to claim ownership over lands, bodies of water traditionally and actually occupied by ICCs/IPs, sacred places, traditional hunting and fishing grounds, and all improvements made by them at any time within the domains; b) Right to Develop Lands and Natural Resources.- Subject to Section 56 hereof, the right to develop, control and use lands and territories traditionally occupied, owned, or used; to manage and conserve natural resources within the territories and uphold the responsibilities for future generations; to benefit and share the profits from allocation and utilization of the natural resources found therein; the right to negotiate the terms and conditions for the exploration of natural resources in the areas for the purpose of ensuring ecological, environmental protection and the conservation measures, pursuant to national and customary laws; the right to an informed and intelligent participation in the formulation and implementation of any project, government or private, that will affect or impact upon the ancestral domains and to receive just and fair compensation for any damages which they may sustain as a result of the project; and the right to effective measures by the government to prevent any interference with, alienation and encroachment upon these rights;" c) Right to Stay in the Territories.- The right to stay in the territory and not to be removed therefrom. No ICCs/IPs will be relocated without their free and prior informed consent, nor through any means other than eminent domain. x x x; d) Right in Case of Displacement.- In case displacement occurs as a result of natural catastrophes, the State shall endeavor to resettle the displaced ICCs/IPs in suitable areas where they can have temporary life support systems: x x x; e) Right to Regulate the Entry of Migrants. - Right to regulate the entry of migrant settlers and organizations into their domains; f) Right to Safe and Clean Air and Water.-For this purpose, the ICCs/IPs shall have access to integrated systems for the management of their inland waters and air space; h) Right to Resolve Conflict.- Right to resolve land conflicts in accordance with customary laws of the area where the land is located, and only in default thereof shall the complaints be submitted to amicable settlement and to the Courts of Justice whenever necessary." Section 8 provides for the rights over ancestral lands: "Sec. 8. Rights to Ancestral Lands.- The right of ownership and possession of the ICCs/IPs to their ancestral lands shall be recognized and protected. a) Right to transfer land/property.- Such right shall include the right to transfer land or property rights to/among members of the same ICCs/IPs, subject to customary laws and traditions of the community concerned. b) Right to Redemption.- In cases where it is shown that the transfer of land/property rights by virtue of any agreement or devise, to a non-member of the concerned ICCs/IPs is tainted by the vitiated consent of the ICCs/IPs, or is transferred for an unconscionable consideration or price, the transferor ICC/IP shall have the right to redeem the same within a period not exceeding fifteen (15) years from the date of transfer." Section 7 (a) defines the ICCs/IPs the right of ownership over their ancestral domains which covers (a) lands, (b) bodies of water traditionally and actually occupied by the ICCs/IPs, (c) sacred places, (d) traditional hunting and fishing grounds, and (e) all improvements made by them at any time within the domains. The right of ownership includes the following rights: (1) the right to develop lands and natural resources; (b) the right to stay in the territories; (c) the right to resettlement in case of displacement; (d) the right to regulate the entry of migrants; (e) the right to safe and clean air and water; (f) the right to claim parts of the ancestral domains as reservations; and (g) the right to resolve conflict in accordance with customary laws. Section 8 governs their rights to ancestral lands. Unlike ownership over the ancestral domains, Section 8 gives the ICCs/IPs also the right to transfer the land or property rights to members of the same ICCs/IPs or non-members thereof. This is in keeping with the option given to ICCs/IPs to secure a torrens title over the ancestrallands, but not to domains. 2. The Right of ICCs/IPs to Develop Lands and Natural Resources Within the Ancestral Domains Does Not Deprive the State of Ownership Over the Natural Resources and Control and Supervision in their Development and Exploitation. g) Right to Claim Parts of Reservations.- The right to claim parts of the ancestral domains which have been reserved for various purposes, except those reserved and intended for common and public welfare and service;

L A W O N N A T U R A L R E S O U R C E S a n d E N V I R O N M E N T A L L A W C a s e s a n d S p e c i a l L a w s | 23
The Regalian doctrine on the ownership, management and utilization of natural resources is declared in Section 2, Article XII of the 1987 Constitution, viz: "Sec. 2. All lands of the public domain, waters, minerals, coal, petroleum, and other mineral oils, all forces of potential energy, fisheries, forests or timber, wildlife, flora and fauna, and other natural resources are owned by the State. With the exception of agricultural lands, all other natural resources shall not be alienated. The exploration, development, and utilization of natural resources shall be under the full control and supervision of the State. The State may directly undertake such activities, or, it may enter into co-production, joint venture, or production-sharing agreements with Filipino citizens, or corporations or associations at least sixty per centum of whose capital is owned by such citizens. Such agreements may be for a period not exceeding twenty-five years, renewable for not more than twentyfive years, and under such terms and conditions as may be provided by law. In cases of water rights for irrigation, water supply, fisheries, water supply, fisheries, or industrial uses other than the development of water power, beneficial use may be the measure and limit of the grant. The State shall protect the nation's marine wealth in its archipelagic waters, territorial sea, and exclusive economic zone, and reserve its use and enjoyment exclusively to Filipino citizens. The Congress may, by law, allow small-scale utilization of natural resources by Filipino citizens, as well as cooperative fish farming, with priority to subsistence fishermen and fishworkers in rivers, lakes, bays, and lagoons. The President may enter into agreements with foreign-owned corporations involving either technical or financial assistance for large-scale exploration, development, and utilization of minerals, petroleum, and other mineral oils according to the general terms and conditions provided by law, based on real contributions to the economic growth and general welfare of the country. In such agreements, the state shall promote the development and use of local scientific and technical resources. The President shall notify the Congress of every contract entered into in accordance with this provision, 223 within thirty days from its execution." All lands of the public domain and all natural resources- waters, minerals, coal, petroleum, and other mineral oils, all forces of potential energy, fisheries, forests or timber, wildlife, flora and fauna, and other natural resources- are owned by the State. The Constitution provides that in the exploration, development and utilization of these natural resources, the State exercises full control and supervision, and may undertake the same in four (4) modes: 1. The State may directly undertake such activities; or 2. The State may enter into co-production, joint venture or production-sharing agreements with Filipino citizens or qualified corporations; 3. Congress may, by law, allow small-scale utilization of natural resources by Filipino citizens; 4. For the large-scale exploration, development and utilization of minerals, petroleum and other mineral oils, the President may enter into agreements with foreign-owned corporations involving technical or financial assistance. As owner of the natural resources, the State is accorded primary power and responsibility in the exploration, development and utilization of these natural resources . The State may directly undertake the exploitation and development by itself, or, it may allow participation by the private sector through co224 225 226 production, joint venture, or production-sharing agreements. These agreements may be for a period of 25 years, renewable for another 25 years. The State, through Congress, may allow the smallscale utilization of natural resources by Filipino citizens. For the large-scale exploration of these resources, specifically minerals, petroleum and other mineral oils, the State, through the President, may enter into technical and financial assistance agreements with foreign-owned corporations. Under the Philippine Mining Act of 1995, (R.A. 7942) and the People's Small-Scale Mining Act of 1991 (R.A. 7076) the three types of agreements, i.e., co-production, joint venture or production-sharing, may apply 227 228 to both large-scale and small-scale mining. "Small-scale mining" refers to "mining activities which rely heavily on manual labor using simple implements and methods and do not use explosives or heavy mining 229 equipment." Examining the IPRA, there is nothing in the law that grants to the ICCs/IPs ownership over the natural resources within their ancestral domains. The right of ICCs/IPs in their ancestral domains includesownership, but this "ownership" is expressly defined and limited in Section 7 (a) as: "Sec. 7. a) Right of ownership- The right to claim ownership over lands, bodies of water traditionally and actually occupied by ICCs/IPs, sacred places, traditional hunting and fishing grounds, and all improvements made by them at any time within the domains;" The ICCs/IPs are given the right to claim ownership over "lands, bodies of water traditionally and actually occupied by ICCs/IPs, sacred places, traditional hunting and fishing grounds, and all improvements made by them at any time within the domains." It will be noted that this enumeration does not mention bodies of water not occupied by the ICCs/IPs, minerals, coal, wildlife, flora and fauna in the traditional hunting grounds, fish in the traditional fishing grounds, forests or timber in the sacred places, etc. and all other natural resources found within the ancestral domains. Indeed, the right of ownership under Section 7 (a) does not cover "waters, minerals, coal,petroleum and other mineral oils, all forces of potential energy, fisheries, forests or timber, wildlife,flora and fauna and all other natural resources" enumerated in Section 2, Article XII of the 1987 Constitution as belonging to the State. The non-inclusion of ownership by the ICCs/IPs over the natural resources in Section 7(a) complies with the Regalian doctrine.

L A W O N N A T U R A L R E S O U R C E S a n d E N V I R O N M E N T A L L A W C a s e s a n d S p e c i a l L a w s | 24
(a) Section 1, Part II, Rule III of the Implementing Rules Goes Beyond the Parameters of Sec. 7 (a) of the IPRA And is Unconstitutional. The Rules Implementing the IPRA
230

in Section 1, Part II, Rule III reads:

implementation of any project, government or private, that will affect or impact upon the ancestral domains and to receive just and fair compensation for any damages which they may sustain as a result of the project; and the right to effective measures by the government to prevent any interference with, alienation and encroachment upon these rights;" The right to develop lands and natural resources under Section 7 (b) of the IPRA enumerates the following rights: a) the right to develop, control and use lands and territories traditionally occupied; b) the right to manage and conserve natural resources within the territories and uphold the responsibilities for future generations; c) the right to benefit and share the profits from the allocation and utilization of the natural resources found therein; d) the right to negotiate the terms and conditions for the exploration of natural resources for the purpose of ensuring ecological, environmental protection and the conservation measures, pursuant to national and customary laws; e) the right to an informed and intelligent participation in the formulation and implementation of any project, government or private, that will affect or impact upon the ancestral domains and to receive just and fair compensation for any damages which they may sustain as a result of the project; f) the right to effective measures by the government to prevent any interference with, alienation 233 and encroachment upon these rights. Ownership over the natural resources in the ancestral domains remains with the State and the ICCs/IPs are merely granted the right to "manage and conserve" them for future generations, "benefit and share" the profits from their allocation and utilization, and "negotiate the terms and conditions for their exploration" for the purpose of "ensuring ecological and environmental protection and conservation measures." It must be noted that the right to negotiate the terms and conditions over the natural resources covers only their exploration which must be for the purpose of ensuring ecological and environmental protection of, and conservation measures in the ancestral domain. It does not extend to the exploitation and development of natural resources. Simply stated, the ICCs/IPs' rights over the natural resources take the form of management or stewardship. For the ICCs/IPs may use these resources and share in the profits of their utilization or negotiate the terms for their exploration. At the same time, however, the ICCs/IPs must ensure that the natural resources within their ancestral domains are conserved for future generations and that the

"Section 1. Rights of Ownership. ICCs/IPs have rights of ownership over lands, waters, and natural resources and all improvements made by them at any time within the ancestral domains/ lands. These rights shall include, but not limited to, the right over the fruits, the right to possess, the right to use, right to consume, right to exclude and right to recover ownership, and the rights or interests over land and natural resources. The right to recover shall be particularly applied to lands lost through fraud or any form or vitiated consent or transferred for an unconscionable price." Section 1 of the Implementing Rules gives the ICCs/IPs rights of ownership over "lands, waters and natural resources." The term "natural resources" is not one of those expressly mentioned in Section 7 (a) of the law. Our Constitution and jurisprudence clearly declare that the right to claim ownership over land does not necessarily include the right to claim ownership over the natural resources found on or under the 231 land. The IPRA itself makes a distinction between land and natural resources. Section 7 (a) speaks of the right of ownership only over the land within the ancestral domain. It is Sections 7 (b) and 57 of the law that speak of natural resources, and these provisions, as shall be discussed later, do not give the ICCs/IPs the right of ownership over these resources. The constitutionality of Section 1, Part II, Rule III of the Implementing Rules was not specifically and categorically challenged by petitioners. Petitioners actually assail the constitutionality of the 232 Implementing Rules in general. Nevertheless, to avoid any confusion in the implementation of the law, it is necessary to declare that the inclusion of "natural resources" in Section 1, Part II, Rule III of the Implementing Rules goes beyond the parameters of Section 7 (b) of the law and is contrary to Section 2, Article XII of the 1987 Constitution. (b) The Small-Scale Utilization of Natural Resources In Sec. 7 (b) of the IPRA Is Allowed Under Paragraph 3, Section 2 of Article XII of the Constitution. Ownership over natural resources remain with the State and the IPRA in Section 7 (b) merely grants the ICCs/IPs the right to manage them, viz: "Sec. 7 (b) Right to Develop Lands and Natural Resources.- Subject to Section 56 hereof, right to develop, control and use lands and territories traditionally occupied, owned, or used; to manage and conserve natural resourceswithin the territories and uphold the responsibilities for future generations; to benefit and share the profits from allocation and utilization of the natural resources found therein; the right to negotiate the terms and conditions for the exploration of natural resources in the areas for the purpose of ensuring ecological, environmental protection and the conservation measures, pursuant to national and customary laws; the right to an informed and intelligent participation in the formulation and

L A W O N N A T U R A L R E S O U R C E S a n d E N V I R O N M E N T A L L A W C a s e s a n d S p e c i a l L a w s | 25
"utilization" of these resources must not harm the ecology and environment pursuant to national and 234 customary laws. The limited rights of "management and use" in Section 7 (b) must be taken to contemplate small-scale utilization of natural resources as distinguished from large-scale. Small-scale utilization of natural resources is expressly allowed in the third paragraph of Section 2, Article XII of the Constitution "in recognition of the plight of forest dwellers, gold panners, marginal fishermen and others similarly situated 235 who exploit our natural resources for their daily sustenance and survival." Section 7 (b) also expressly mandates the ICCs/IPs to manage and conserve these resources and ensure environmental and ecological protection within the domains, which duties, by their very nature, necessarily reject utilization in a largescale. (c) The Large-Scale Utilization of Natural Resources In Section 57 of the IPRA Is Allowed Under Paragraphs 1 and 4, Section 2, Article XII of the 1987 Constitution. Section 57 of the IPRA provides: "Sec. 57. Natural Resources within Ancestral Domains.- The ICCs/IPs shall have priority rights in theharvesting, extraction, development or exploitation of any natural resources within the ancestral domains. A non-member of the ICCs/IPs concerned may be allowed to take part in the development and utilization of the natural resources for a period of not exceeding twenty-five (25) years renewable for not more than twenty-five (25) years: Provided, That a formal and written agreement is entered into with the ICCs/IPs concerned or that the community, pursuant to its own decision-making process, has agreed to allow such operation: Provided finally, That the NCIP may exercise visitorial powers and take appropriate action to safeguard the rights of the ICCs/IPs under the same contract." Section 57 speaks of the "harvesting, extraction, development or exploitation of natural resources within ancestral domains" and "gives the ICCs/IPs 'priority rights' therein." The terms "harvesting, extraction, development or exploitation" of any natural resources within the ancestral domains obviously refer to large-scale utilization. It is utilization not merely for subsistence but for commercial or other extensive 236 use that require technology other than manual labor. The law recognizes the probability of requiring a non-member of the ICCs/IPs to participate in the development and utilization of the natural resources and thereby allows such participation for a period of not more than 25 years, renewable for another 25 years. This may be done on condition that a formal written agreement be entered into by the non-member and members of the ICCs/IPs. Section 57 of the IPRA does not give the ICCs/IPs the right to "manage and conserve" the natural resources. Instead, the law only grants the ICCs/IPs "priority rights" in the development or exploitation thereof. Priority means giving preference. Having priority rights over the natural resources does not necessarily mean ownership rights. The grant of priority rights implies that there is a superior entity that owns these resources and this entity has the power to grant preferential rights over the resources to whosoever itself chooses. Section 57 is not a repudiation of the Regalian doctrine. Rather, it is an affirmation of the said doctrine that all natural resources found within the ancestral domains belong to the State. It incorporates by implication the Regalian doctrine, hence, requires that the provision be read in the light of Section 2, 237 Article XII of the 1987 Constitution. Interpreting Section 2, Article XII of the 1987 Constitution in relation to Section 57 of IPRA, the State, as owner of these natural resources, may directly undertake the development and exploitation of the natural resources by itself, or in the alternative, it may recognize the priority rights of the ICCs/IPs as owners of the land on which the natural resources are found by entering into a co-production, joint venture, or production-sharing agreement with them. The State may likewise enter into any of said agreements with a non-member of the ICCs/IPs, whether natural or juridical, or enter into agreements with foreign-owned corporations involving either technical or financial assistance for the large-scale exploration, development and utilization of minerals, petroleum, and other mineral oils, or allow such non-member to participate in its agreement with the ICCs/IPs. If the State decides to enter into an agreement with a non-ICC/IP member, the National Commission on Indigenous Peoples (NCIP) shall ensure that the rights of the ICCs/IPs under the agreement shall be protected. The agreement shall be for a period of 25 years, renewable for another 25 years. To reiterate, in the large-scale utilization of natural resources within the ancestral domains, the State, as owner of these resources, has four (4) options: (1) it may, of and by itself, directly undertake the development and exploitation of the natural resources; or (2) it may recognize the priority rights of the ICCs/IPs by entering into an agreement with them for such development and exploitation; or (3) it may enter into an agreement with a non-member of the ICCs/IPs, whether natural or juridical, local or foreign; or (4) it may allow such non-member to participate in the agreement with the ICCs/IPs. The rights granted by the IPRA to the ICCs/IPs over the natural resources in their ancestral domains merely gives the ICCs/IPs, as owners and occupants of the land on which the resources are found, the right to the small-scale utilization of these resources, and at the same time, a priority in their large-scale development and exploitation. Section 57 does not mandate the State to automatically give priority to the ICCs/IPs. The State has several options and it is within its discretion to choose which option to pursue. Moreover, there is nothing in the law that gives the ICCs/IPs the right to solely undertake the large-scale development of the natural resources within their domains. The ICCs/IPs must undertake such endeavour always under State supervision or control. This indicates that the State does not lose control and ownership over the resources even in their exploitation. Sections 7 (b) and 57 of the law simply give due respect to the ICCs/IPs who, as actual occupants of the land where the natural resources lie, have traditionally utilized these resources for their subsistence and survival. Neither is the State stripped of ownership and control of the natural resources by the following provision:

L A W O N N A T U R A L R E S O U R C E S a n d E N V I R O N M E N T A L L A W C a s e s a n d S p e c i a l L a w s | 26
"Section 59. Certification Precondition.- All departments and other governmental agencies shall henceforth be strictly enjoined from issuing, renewing or granting any concession, license or lease, or entering into any production-sharing agreement. without prior certification from the NCIP that the area affected does not overlap with any ancestral domain. Such certification shall only be issued after a fieldbased investigation is conducted by the Ancestral Domains Office of the area concerned: Provided, That no certification shall be issued by the NCIP without the free and prior informed and written consent of the ICCs/IPs concerned: Provided, further, That no department, government agency or government-owned or -controlled corporation may issue new concession, license, lease, or production sharing agreement while there is a pending application for a CADT: Provided, finally, That the ICCs/IPs shall have the right to stop or suspend, in accordance with this Act, any project that has not satisfied the requirement of this consultation process." Concessions, licenses, lease or production-sharing agreements for the exploitation of natural resources shall not be issued, renewed or granted by all departments and government agencies without prior certification from the NCIP that the area subject of the agreement does not overlap with any ancestral domain. The NCIP certification shall be issued only after a field-based investigation shall have been conducted and the free and prior informed written consent of the ICCs/IPs obtained. Non-compliance with the consultation requirement gives the ICCs/IPs the right to stop or suspend any project granted by any department or government agency. As its subtitle suggests, this provision requires as a precondition for the issuance of any concession, license or agreement over natural resources, that a certification be issued by the NCIP that the area subject of the agreement does not lie within any ancestral domain. The provision does not vest the NCIP with power over the other agencies of the State as to determine whether to grant or deny any concession or license or agreement. It merely gives the NCIP the authority to ensure that the ICCs/IPs have been informed of the agreement and that their consent thereto has been obtained. Note that the certification applies to agreements over natural resources that do not necessarily lie within the ancestral domains. For those that are found within the said domains, Sections 7(b) and 57 of the IPRA apply. V. THE IPRA IS A RECOGNITION OF OUR ACTIVE PARTICIPATION IN THE INDIGENOUS INTERNATIONAL MOVEMENT. The indigenous movement can be seen as the heir to a history of anti-imperialism stretching back to prehistoric times. The movement received a massive impetus during the 1960's from two sources. First, the decolonization of Asia and Africa brought into the limelight the possibility of peoples controlling their own destinies. Second, the right of self-determination was enshrined in the UN Declaration on Human 238 Rights. The rise of the civil rights movement and anti-racism brought to the attention of North American Indians, Aborigines in Australia, and Maori in New Zealand the possibility of fighting for fundamental rights and freedoms. In 1974 and 1975, international indigenous organizations were founded, and during the 1980's, indigenous affairs were on the international agenda. The people of the Philippine Cordillera were the first
239

Asians to take part in the international indigenous movement. It was the Cordillera People's Alliance that carried out successful campaigns against the building of the Chico River Dam in 1981-82 and they have 240 since become one of the best-organized indigenous bodies in the world. Presently, there is a growing concern for indigenous rights in the international scene. This came as a result of the increased publicity focused on the continuing disrespect for indigenous human rights and the destruction of the indigenous peoples' environment, together with the national governments' inability to 241 deal with the situation. Indigenous rights came as a result of both human rights and environmental 242 protection, and have become a part of today's priorities for the international agenda. International institutions and bodies have realized the necessity of applying policies, programs and specific rules concerning IPs in some nations. The World Bank, for example, first adopted a policy on IPs as 243 a result of the dismal experience of projects in Latin America. The World Bank now seeks to apply its current policy on IPs to some of its projects in Asia. This policy has provided an influential model for the 244 projects of the Asian Development Bank. The 1987 Philippine Constitution formally recognizes the existence of ICCs/IPs and declares as a State 245 policy the promotion of their rights within the framework of national unity and development. The IPRA 246 amalgamates the Philippine category of ICCs with the international category of IPs, and is heavily influenced by both the International Labor Organization (ILO) Convention 169 and the United Nations 247 (UN) Draft Declaration on the Rights of Indigenous Peoples. ILO Convention No. 169 is entitled the "Convention Concerning Indigenous and Tribal Peoples in 248 Independent Countries" and was adopted on June 27, 1989. It is based on the Universal Declaration of Human Rights, the International Covenant on Economic, Social and Cultural Rights, the International Covenant on Civil and Political Rights, and many other international instruments on the prevention of 249 discrimination. ILO Convention No. 169 revised the "Convention Concerning the Protection and Integration of Indigenous and Other Tribal and Semi-Tribal Populations in Independent Countries" (ILO No. 107) passed on June 26, 1957. Developments in international law made it appropriate to adopt new international standards on indigenous peoples "with a view to removing the assimilationist orientation of the earlier standards," and recognizing the aspirations of these peoples to exercise control over their own 250 institutions, ways of life and economic development." CONCLUSION The struggle of the Filipinos throughout colonial history had been plagued by ethnic and religious differences. These differences were carried over and magnified by the Philippine government through the 251 imposition of a national legal order that is mostly foreign in origin or derivation. Largely unpopulist, the present legal system has resulted in the alienation of a large sector of society, specifically, the indigenous peoples. The histories and cultures of the indigenes are relevant to the evolution of Philippine culture and 252 are vital to the understanding of contemporary problems. It is through the IPRA that an attempt was

L A W O N N A T U R A L R E S O U R C E S a n d E N V I R O N M E N T A L L A W C a s e s a n d S p e c i a l L a w s | 27
made by our legislators to understand Filipino society not in terms of myths and biases but through common experiences in the course of history. The Philippines became a democracy a centennial ago and the decolonization process still continues. If the evolution of the Filipino people into a democratic society is to truly proceed democratically, i.e., if the Filipinos as a whole are to participate fully in the task of 253 continuing democratization, it is this Court's duty to acknowledge the presence of indigenous and customary laws in the country and affirm their co-existence with the land laws in our national legal system. With the foregoing disquisitions, I vote to uphold the constitutionality of the Indigenous Peoples Rights Act of 1997.
Footnotes
1 2 23 24

Chief Judge, US Court of Appeals for the Seventh Circuit; Senior Lecturer, University of Chicago Law School. The University of Chicago Law Review, Vol. 67, Summer 2000, No. 3, p. 573. 3 Dominium is distinguished from imperium which is the government authority possessed by the state expressed in the concept of sovereignty- Lee Hong Hok v. David, 48 SCRA 372, 377 [1972]. 4 Valenton v. Murciano, 3 Phil. 537, 543 [1904]; See also Florencio D.R. Ponce, The Philippine Torrens System, p. 13 [1964]. 5 Antonio H. Noblejas, Land Titles and Deeds, p. 5 [1986]; these grants were better known as repartimientos and encomiendas. Repartimientos were handouts to the military as fitting reward for their services to the Spanish crown. The encomiendas were given to Spaniards to administer and develop with the right to receive and enjoy for themselves the tributes of the natives assigned to them.- Ponce, supra, p. 12, citing Benitez, History of the Philippines, pp. 125-126. 6 Narciso Pena, Registration of Land Titles and Deeds, p. 2 [1994]. 7 The Mortgage Law is a misnomer because it is primarily a law on registration of property and secondarily a mortgage law- Ponce, supra, at 16. 8 Ponce, supra, at 15. 9 3 Phil. 537 [1904]. 10 Id. at 540. 11 Id. at 548. 12 Id. at 543-544. 13 Id. at 543. 14 Id. at 542-543. These comments by the court are clear expressions of the concept that Crown holdings embraced both imperium and dominiumMa. Lourdes Aranal-Sereno and Roan Libarios, The Interface Between National Land Law and Kalinga Land Law, 58 P.L.J. 420, 423 [1983]. 15 Id. at 545-546. 16 Id. at 543. 17 Id. at 557. 18 Id. at 553-554; Valenton was applied in Cansino v. Valdez, 6 Phil. 320 [1906]; Tiglao v. Insular Government, 7 Phil. 80 [1906]; and Cario v. Insular Government, 7 Phil. 132 [1906]; all decided by the Philippine Supreme Court. 19 Please see Section 70, Act 926. 20 Ponce, supra, at 33. 21 Montano v. Insular Government, 12 Phil. 572 [1909]; also cited in Ponce, supra, at 32. 22 Archbishop of Manila v. Director of Lands, 27 Phil. 245 [1914]; also cited in Ponce, supra, at 32.

Antonio H. Noblejas, Land Titles and Deeds, p. 250 [1961]. Ponce, supra, at 32. 25 Pea, Registration of Land Titles and Deeds, p. 26 [1982]; Noblejas, supra, at 32. 26 Noblejas, supra, at 32. 27 Ponce, supra, at 123-124; Noblejas, supra, at 33. 28 2 Aruego, The Framing of the Philippine Constitution, p. 592 [1937]. 29 Id. at 600. 30 Id. at 600-601. 31 Ibid. 32 Section 7. 33 Section 8. 34 Sections 13 to 20. 35 Sections 21 to 28. 36 Sections 29 to 37. 37 Sections 38 and 40. 38 Sections 74 to 77. 39 Section 69. 40 Section 73. 41 Convention Conerning Indigenous and Tribal Peoples in Independent Countries, June 27, 1989. 42 Guide to R.A. 8371, published by the Coalition for Ips Rights and ancestral Domains in cooperation with the ILO and Bilance-Asia Department, p. 4 [1999]hereinafter referred to as Guide to R.A. 8371. 43 Taken from the list of IPs sbmitted by Rep. Andolana to the house of Representatives during the deliberations on H.B. No. 9125Interpellations of Aug. 20, 1997, pp. 00086-00095. "lost tribes" such as the Lutangan and Tatang have not been included. 44 How these people came to the Philippines may be explained by two theories. One view, generally linked to Professor Otley H. Beyer, suggests the "wave theory"a series of arrivals in the archipelago bringing in different types and levels of culture. The Negritos, dark-skinned pygmies, came between 25,000 to 30,000 B.C. Their cultural remains are preserved by the Negrito-type Filipinos found in Luzon, Visayas and Mindanao. Their relatively inferior culture did not enable them to overcome the pressures from the second wave of people, the Indonesians A and B who came in 5,000 and 3,500 B.C. They are represented today by the Kalinga, Gaddang, Isneg, Mangyan, Tagbanua, Manobo, Mandaya, Subanon, and Sama. The first group was pushed inland as the second occupied the coastal and downriver settlements. The last wave involved Malay migrations between 500 B.C. and 1,500 A.D. they had a more advanced culture based on metal age technology. They are represented by the Christianized and Islamized Filipinos who pushed the Indonesian groups inland and occupied much of the coastal, lowland and downstream areas. A second view is postulated by Robert Fox, F. Landa Jocana, Alfredo Evangelista, and Jesus Peralta. Jocano maintains that the Negritos, Indonesians and Malays stand co-equal as ethnic groups without any one being dominant, racially or culturally. The geographic distribution of the ethnolinguistic groups, which shows overlapping of otherwise similar racial strains in both upland and lowland cultures or coastal and inland communities, suggests a random and unstructured advent of different kinds of groups in the archipelagoSamuel K. Tan, A History of the Philippines, published by the Manila Studies Association, Inc. and the Philippine National Historical society, Inc., pp. 33-34 [1997]; Teodoro A. Agoncillo, History of the Filipino People, p. 21 [1990]. 45 Tan, supra, at 35-36. 46 Onofre D. Corpuz, The Roots of the Filipino Nation, Philippine Centennial (1898-1998) Edition, vol. 1, p. 13, Aklahi foundation, Inc. [1989]. It was in 800-1,000 A.D. that the Ifugaos of Northern Luzon built the rice terracesId. at 37.

L A W O N N A T U R A L R E S O U R C E S a n d E N V I R O N M E N T A L L A W C a s e s a n d S p e c i a l L a w s | 28
47 48

Id. at 5-6. Id. at 13. 49 Teodoro A. Agoncillo, History of the Filipino People, p. 54 [1990]. 50 Corpuz, supra, at 5. 51 Id. at 44-45. 52 Agoncillo, supra, at 40. 53 Id. at 40-41. 54 Rafael Iriarte, History of the Judicial System, the Philippine Indigenous Era Prior to 1565, unpublished work submitted as entry to the Centennial Essay-Writing Contest sponsored by the National Centennial Commission and the Supreme Court in 1997, p. 103, citing Perfecto V. Fernandez, Customs Laws in PreConquest Philippines, UP Law Center, p. 10 [1976]. 55 Agoncillo, supra, at 41. 56 Amelia Alonzo, The History of the Judicial System in the Philippines, Indigenous Era Prior to 1565,unpublished work submitted as entry to the Centennial Essay-Writing Contest sponsored by the National Centennial Commission and the Supreme Court in 1997. 57 Agoncillo, supra, at 42. 58 Renato Constantino, A Past Revisited , p. 38 [1975]. 59 Samuel K. Tan, A History of the Philippines, published by the Manila Studies Assn., Inc. and the Phil. National Historical Society, Inc., p. 43 [1997]. 60 Id. 61 Id. at 43-44. 62 Tan, supra, at 47-48. 63 Id. at 48-49. 64 Cacho v. Government of the P.I., 28 Phil. 616, 625-627 [1914]; see also Ponce, The Philippine Torrens System, pp. 11-12 [1964]. In Philippine pre-colonial history, there was only one recorded transaction on the purchase of land. The Maragtas Code tells us of the purchase of Panay Island by ten Bornean datus led by Datu Puti from the Atis under Marikudo in the 13th century. The purchase price for the island was a gold salakot and a long gold necklace Agoncillo, supra, at 25. 65 Constantino, supra, at 38. 66 Corpuz, supra, at 39. 67 Resettlement- "bajo el son de la campana" (under the sound of the bell) or "bajo el toque de la campana"(Under the peal of the bell). 68 People v. Cayat, 68 Phil. 12, 17 [1939]. 69 Id. at 17, citing the Decree of the Governor-General of the Philippines, Jan. 14, 1887. 70 Agoncillo, supra, at 80. 71 Id. at 80. 72 Corpuz, supra, at 277-278. 73 Id. at 277. 74 Id., N.B. But see discussion in Cario v. Insular Government, infra, where the United States Supreme Court found that the Spanish decrees in the Philippines appeared to recognize that the natives owned some land. Whether in the implementation of these decrees the natives ancestral rights to land wereactually respected was not discussed by the U.S. Supreme Court; see also Note 131, infra. 75 Tan, supra, at 49-50. 76 Id. at 67. 77 Id. at 52-53. 78 Id. at 53. 79 Id. at 55.

80 81

People v. Cayat, 68 Phil. 12, 17 [1939]. Memorandum of the Secretary of the Interior, quoted in Rubi v. Provincial Board of Mindoro, 39 Phil. 660, 714 [1919]; also cited in People v. Cayat, supra, at 17-18. 82 Rubi v. Provincial Board of Mindoro, supra, at 693. 83 Charles Macdonald, Indigenous Peoples of the Philippines: Between Segregation and Integration , Indigenous Peoples of Asia, p. 348, ed. by R.H. Barnes, A. Gray and B. Kingsburry, pub. by Association for Asian Studies [1995]. The BNCT made a Bontok and subanon ethnography, a history of Sulu genealogy, and a compilation on unhispanized peoples in northern Luzon.Owen J. Lynch, Jr., The Philippine Colonial Dichotomy: Attraction and Disenfranchisement, 63 P. L. J. 139-140 [1988]. 84 R.A. No. 1888 of 1957. 85 See People v. Cayat, supra, at 21; See also Rubi v. Provincial Board of Mindoro, 39 Phil. 660, 694 [1919] 86 MacDonald, Indigenous Peoples of the Philippines, supra, at 351. 87 The construction of the Ambuklao and Binga dams in the 1950s resulted in the eviction of hundreds of Ibaloi families Cerilo Rico S. Abelardo, Ancestral Domain Rights: Issues, Responses, and Recommendations, Ateneo Law Journal, vol. 38, No. 1, p. 92 [1993]. 88 Section 11, Art. XV, 1973 Constitution. 89 Presidential Decrees Nos. 1017 and 1414. 90 The PANAMIN, however, concentrated funds and resources on image-building, publicity, and impact projects. In Mindanao, the agency resorted to a policy of forced resettlement on reservations, militarization and intimidation- MacDonald, Indigenous Peoples of the Philippines, supra, at 349-350. 91 No occupancy certificates were issued, however, because the government failed to release the decrees implementing rules and regulations- Abelardo, supra, at 120-121. 92 Id., Note 177. 93 Id., at 93-94. 94 MacDonald, Indigenous People of the Philippines, supra, at 351. 95 E.O. Nos. 122-A, 122-B and 122-C. The preamble of E.O. No. 122-B states: "Believing that the new government is committed to formulate more vigorous policies, plans, programs, and projects for tribal Filipinos, otherwise known as Indigenous Cultural Communities, taking into consideration their communal aspirations, customs, traditions, beliefs, and interests, in order to promote and preserve their rich cultural heritage and insure their participation in the countrys development for national unity; xxx" 96 Article II, sec. 22; Article VI, sec. 5, par. 2; Article XII, sec. 5; Article XIII, sec. 6; Article XIV, sec. 17; and Article XVI, sec. 12. 97 MacDonald, Indigenous Peoples of the Philippines, supra, at 345. 98 Samuel K. Tan, A History of the Philippines, p. 54 [1997]. 99 Cordillera Studies Program, Land Use and Ownership and Public Policy in the Cordillera, 29-30 [n.d.]; also cited in Dante B. Gatmaytan, Ancestral Domain Recognition in the Philippines: Trends in Jurisprudence and Legislation, 5 Phil. Nat. Res. L.J. No. 1, pp. 47-48 [1992]. 100 Abelardo, Ancestral Domain Rights, supra, at 98-99, citing Ponciano L. Bennagen, Indigenous Attitudes Toward Land and Natural Resources of Tribal Filipinos, 31 National Council of Churches in the Philippines Newsletter, Oct.-Dec. 1991, at 4-9. 101 Id. at 99, citing June Prill-Brett, Bontok Land Tenure (UP Law library, mimeographed). 102 Ma. Lourdes Aranal-Sereno and Roan Libarios, The Interface of National Land Law and Kalinga Law, 58 P.L.J. 420, 440-441 [1983]. 103 Ibid. 104 Ibid. 105 Ibid.

L A W O N N A T U R A L R E S O U R C E S a n d E N V I R O N M E N T A L L A W C a s e s a n d S p e c i a l L a w s | 29
106 107

Ma. Lourdes Aranal-Sereno and Roan Libarios, The Interface, supra, at 420. Senate Bill No. 1728 was co-sponsored by Senator Macapagal-Arroyo and co-authored by Senators Alvarez, Magsaysay, Revilla, Mercado, Enrile, Honasan, Tatad, Maceda, Shahani, Osmena and Romulo. The Eighth Congress, through Senators Rasul, Estrada and Romulo filed a bill to operationalize the mandate of the 1987 Constitution on indigenous peoples. The bill was reported out, sponsored an interpellated but never enacted into law. In the Ninth Congress, the bill filed by Senators Rasul and Macapagal-Arroyo was never sponsored and deliberated upon in the floor. 108 Sponsorship Speech of Senator Flavier, Legislative History of SBN 1728, Tenth Congress, Second Regular Session, Senate, Oct. 16, 1996, pp. 15-16. 109 Id. at 12. 110 Id. at 17-18. 111 Id. at 13. 112 Journal of the Tenth Congress of the Philippines, Senate, Session No. 5, Aug. 5-6, 1997, pp. 86-87. 113 Co-authors of the bill were Reps. Ermita, Teves, Plaza, Calalay, Recto, Fua, Luciano, Abad, Cosalan, Aumentado, de la Cruz, Bautista, Singson, Damasing, Romualdo, Montilla, Germino, Verceles Proceedings of Sept. 4, 1997, pp. 00107-00108. 114 Sponsorship speech of Rep. Andolana of House Bill No. 9125, March 20, 1997. 115 Interpellation of Aug. 20, 1997, 6:16 p.m., p. 00061. 116 Section 3 [a], IPRA. 117 Section 3 [b], IPRA. 118 Guide to R.A. 8371, p. 14. 119 Section 44 [e], IPRA. 120 Section 51, IPRA. 121 Guide to R.A. 8371, p. 15. 122 A CADT refers to a title formally recognizing the right of possession and ownership of ICCs/IPs over their ancestral domains identified and delineated in acordance with the IPRARule II [c], Rules & Regulations Implementing the IPRA, NCIP Admin. Order No. 1. 123 Section 53 [a], IPRA. 124 A CALT refers to a title formally recognizing the rights of the ICCs/IPs over their ancestral lands- Rule II [d], Implementing Rules, NCIP A.O. No. 1. 125 Section 52 [k], IPRA. 126 Section 3 [1], IPRA. 127 Section 11, IPRA. 128 Ibid. 129 41 Phil. 935 (1909), 212 U.S. 449, 53 L.Ed. 594. 130 Sponsorship Speech of Senator Juan Flavier, Leg. History of SBN 1728, Tenth Congress, Second Regular Session, Oct. 16, 1996, p. 13. 131 It was the practice of the Spanish colonial government not to issue titles to Igorots Owen J. Lynch, Jr., Invisible Peoples and a Hidden Agenda: The Origins of Contemporary Philippine Land Laws (1900-1913), 63 P.L.J. 249, 288 [1988], citing the testimony of Benguet Provincial Governnor William F. Pack, Records at 47, Cario. 132 Maura Law or the Royal Decree of Feb. 13, 1894. 133 Later named Camp John Hay. 134 Lynch, Invisible Peoples, supra, at 288-289. 135 7 Phil. 132 [1906]. 136 In 1901, Cario had entered into a promissory agreement with a U.S. merchant in Manila. The note obliged Cario to sell the land at issue "as soon as he obtains from the Government of the United States, or

its representatives in the Philippines, real and definitive title." See Lynch, Invisible Peoples, supra, at 290, citing Governments Exhibit G, Records, at 137-138, Cario. 137 Cario v. Insular Government, supra, at 939. 138 Ibid. 139 Id. at 940. 140 Id. at 941. 141 Id. at 941-942. 142 Aranal-Sereno and Libarios, The Interface Between Kalinga Land Law, supra at 428-This artcile was one of those circulated among the Constitutional Commissioners in the formulation of Sec. 5, Article XII of the 1987 Constitution (4 Record of the Constitutional Commission 33). 143 Id. at 944. 144 Certificate of Title No. 2 covering the 148 hectares of Baguio Municipality was issued not in the name of Cario who died on June 6, 1908, but to his lawyers John Hausserman and Charles Cohn and his attorney-infact Metcalf Clarke. Hausserman, Cohn and Clarke sold the land to the U.S. Government in a Deed of Quitclaim-Richel B. Langit, Igorot Descendants Claim Rights to Camp John Hay, Manila Times, p. 1, Jan. 12, 1998. 145 Id. at 939. 146 57 P.L.J. 268, 293-296 [1982]. 147 From 1987 to 1988, Prof. Lynch allowed the P.L.J. to publish parts of his doctoral dissertation at the Yale Law School entitled "Invisible Peoples: A History of Philippine Land Law." Please see the Legal Bases of Philippine Colonial Sovereignty: An Inquiry, 62 P.L.J. 279 [1987]; Land Rights, Land Laws and Land Usurpation: The Spanish Era (1568-1898), 63 P.L.J. 82 [1988]; The Colonial Dichotomy: Attraction and Disenfranchisement, 63 P.L.J. 112; Invisible Peoples and a Hidden Agenda: The Origins of Contemporary Philippine Land Laws (1900-1913), 63 P.L.J. 249. 148 "Native title" is a common law recognition of pre-existing aboriginal land interests in Autsralia- Maureen Tehan, Customary Title, Heritage Protection, and Property Rights in Australia: Emerging Patterns of Land Use in the Post-Mabo Era, 7 Pacific Rim Law & Policy Journal, No. 3, p. 765 [June 1998]. 149 Lynch, Native Titles, supra, Note 164, p. 293. 150 39 Phil. 660 [1919]. 151 Id. at 712-713. 152 Id. at 694. 153 Id. at 700. 154 42 C.J.S., Indians, Sec. 29 [1944 ed.]. 155 There are 3 kinds of Indian reservations: (a) those created by treaties prior to 1871; (b) those created by acts of Congress since 1871; and (c) those made by Executive Orders where the President has set apart public lands for the use of the Indians in order to keep them within a certain territory- 42 C.J.S., Indians, Sec. 29 citing Sioux Tribe of Indians v. U.S. 94 Ct. Cl. 150, 170, certiorari granted 62 S. Ct. 631, 315 U.S. 790, 86 L. Ed. 1194, affirmed 62 S. Ct. 1095, 316 U.S. 317, 86 L. Ed. 1501. It is observed that the first two kinds may include lands possessed by aboriginal title. The last kind covers Indian reservations proper. Until 1871, Indian tribes were recognized by the United States as possessing the attributes of nations to the extent that treaties were made with them. In that year, however, Congress, by statute, declared its intention thereafter to make the Indian tribes amenable directly to the power and authority of the United States by the immediate exercise of its legislative power over them, instead of by treaty. Since then, Indian affairs have been regulated by acts if Congress and by contracts with the Indian tribes practically amounting to treaties- 41 Am Jur 2d, Indians, Sec. 55 [1995 ed]. 156 42 C.J.S. Indians, Sec. 28 [1944 ed.].

L A W O N N A T U R A L R E S O U R C E S a n d E N V I R O N M E N T A L L A W C a s e s a n d S p e c i a l L a w s | 30
157 158

Ibid.; see also U.S. v. Santa Fe Pac. R. Co., Ariz., 62 S. Ct. 248, 314 U.S. 339, 86 L. Ed. 260 [1941]. Ibid. 159 8 Wheat 543, 5 L. Ed. 681 [1823]. 160 Id. at 680. 161 Id. at 689. 162 Id. at 696; see also 41 ALR Fed 425, Annotation: Proof and Extinguishment of Aboriginal Title to Indian Lands, Sec. 2[a] [1979]. 163 Buttz v. Northern Pac.R. Co., Dak., 7 S. Ct. 100, 119 U.S. 55, 30 L. Ed. 330, 335 [1886]. 164 Lynch, Native Title, supra, at 293-294; Cohen, Original Indian Title, 32 Minn. L.R. 48-49 [1947]. 165 6 Pet 515, 8 L.Ed. 483 [1832]. 166 Id. at 499. 167 Id. at 500. 168 Id. at 501. 169 The title of the government to Indian lands, the naked fee, is a sovereign title, the government having no landlord from whom it holds the fee- Shoshone Tribe of Indians of Wind River Reservation in Wyoming v. U.S., 85 Ct. Cl. 331, certiorari granted U.S. v. Shoshone Tribe of Indians, 58 S. Ct. 609, 303 U.S. 629, 82 L. Ed. 1090, affirmed 58 S. Ct. 794, 304 U.S. 111, 82 L. Ed. 1213, 1218-1219 [1938]. 170 Buttz v. Northern Pac. R. Co., Dak., at 30 L. Ed. 330, 335; Beecher v. Wetherby, Wis., 95 U.S. 517, 24 L. Ed. 440, 441 [1877]; see also 42 C.J.S., Indians, Sec. 28 [1944 ed.]. 171 Annotation, Proof and Extinguishment of Aboriginal title to Indian Lands, 41 ALR Fed 425, Sec. 2 [b] [1979]- hereinafter cited as Aboriginal Title to Indian Lands. 172 Ibid.; see also Tee Hit Ton Indians v. U.S., 348 U.S. 272, 99 L. Ed. 314, 320, 75 S. Ct. 313 [1955], reh den 348 U.S. 965, 99 L. Ed. 753, 75 S. Ct. 521. 173 Ibid.; Tee Hit Ton Indians v. U.S., at 99 L. Ed. 320. 174 Oneida Indian Nation v. County of Oneida, 414 U.S. 661, 39 L. Ed. 2d 73, 94 S Ct. 772 [1974]; U.S. v. Alcea Bank of Tillamooks, 329 U.S. 40, 91 L. Ed. 29. 67 S. Ct. 167 [1946]. 175 For compensation under the Indian Claims Commission Act, the proof of aboriginal title rests on actual, exclusive and continuous use and occupancy for a long time prior to the loss of the property. (The Indian Claims Commission Act awards compensation to Indians whose aboriginal titles were extinguished by the government through military conquest, creation of a reservation, forced confinement of Indians and removal of Indians from certain portions of the land an the designation of Indian land into forest preserve, grazing district, etc.) - Aboriginal Title to Indian Lands, supra, at Secs. 2[a], 3[a], pp. 431, 433, 437. 176 Aboriginal Title to Indian Lands, supra, at Sec. 2[b], p. 435. 177 41 Am Jr 2d, Indians, Sec. 59 [1995 ed.]. 178 An allotment of Indian land contains restrictions on alienation of the land. These restrictions extend to a devise of the land by will- Missouri, K. & T.R. Co. v. U.S., 235 U.S. 37, 59 L. Ed. 116,. 35 S. Ct. 6 [1914]; A railroad land grant that falls within Indian land is null and void- Northern P. R. Co. v. U.S., 227 U.S. 355, 57 L.Ed. 544,33 S. Ct. 368 [1913]; Portions of Indian land necessary for a railroad right of way were, by the terms of the treaty, declared "public land," implying that land beyond the right of way was privateKindred v. Union P.R. Co., 225 U.S. 582, 56 L. Ed. 1216, 32 S. Ct. 780 [1912]; see also 41 Am Jur 2d, Indians, Sec. 58 [1995 ed]. 179 Aboriginal Title to Indian Lands, supra, at Sec. 2[a], p. 433. 180 42 C.J.S. Indians, Sec. 29 [1944 ed.] 181 Ibid. 182 North American Indians have made much progress in establishing a relationship with the national government and developing their own laws. Some have their own government-recognized constitutions. Usually the recognition of Indian tribes depends on whether the tribe has a reservation. North American

tribes have reached such an advanced stage that the main issues today evolve around complex jurisdictional and litigation matters. Tribes have acquired the status of sovereign nations within another nation, possessing the right to change and grow- Jose Paulo Kastrup, The Internationalization of Indigenous Rights from the Environmental and Human Rights Perspective, Texas International Law Journal, vol. 32: 97, 104 [1997]. 183 Lynch, Native Title, supra, at 293. 184 Dante Gatmaytan, Ancestral Domain Recognition in the Philippines: Trends in Jurisprudence and Legislation, 5 Phil. Nat. Res. L.J. No. 1, pp. 43, 40 [Aug. 1992]; see also Tee Hit Ton Indians v. U.S., supra, at 320. 185 Ibid. 186 D. Gatmaytan, supra, citing Churchill, The Earth is Our Mother: Struggles for American Indian Land and Liberation in the Contemporary United States, The State of Native America: Genocide, Colonization and Resistance 139 (M. Jaimes 1992); and Indian Law Resource Center, United States Denial of Indian Property Rights: A Study in Lawless Power and Racial Discrimination, Rethinking Indian Law 15 (National Lawyers Guild, Committee on Native American Struggles 1982). 187 Id., Note 28, stating that some earlier decisions of the U.S. Supreme Court have held that Congress is subject to the strictures of the Constitution in dealing with Indians. When an Indian property is taken for non-Indian use, the U.S. government is liable for payment of compensation, and an uncompensated taking may be enjoined. F. Cohen, Handbook of Federal Indian Law 217 [1982], citing Shoshone Tribe v. U.S. 299 U.S. 476 [1937]; Choate v. Trapp, 224 U.S. 665 [1912]; and Lane v. Pueblo of Santa Rosa, 249 U.S. 110 [1919]. 188 See Discussion, infra, Part IV (c) (2). 189 Susi v. Razon, 48 Phil. 424 [1925]; Herico v. Dar, 95 SCRA 437 [1980]. 190 Ibid. 191 Director of Lands v. Intermediate Appellate Court, 146 SCRA 509 [1986]; Director of Lands v. Buyco, 216 SCRA 78 [1992]; Republic v. Court of Appeals and Lapina, 235 SCRA 567 [1994]. 192 75 Phil. 890 [1946]. 193 Id. at 892. 194 Sec. 48 [b], C.A. 141. 195 Sec. 48 [c], C.A. 141, as amended. This provision was added in 1964 by R.A. 3872. 196 Section 12, IPRA. 197 "Time immemorial" refers "to a period of time when as far back as memory can go, certain ICCs/Ips are known to have occupied, possessed in the concept of owner, and utilized a defined territory devolved to them, by operation of customary law or inherited from their ancestors, in accordance with their customs and traditions." (Sec. 3 [p], IPRA). 198 Section 2, C.A. 141. 199 Section 8, C.A. 141. 200 The classification of ancestral lands 18% in slope or over as alienable in the IPRA is an exception to Section 15, P.D. 705, the Revised Forestry Code. 201 Charles MacDonald, Indigenous Peoples of the Philippines: Between Segregation and Integration, Indigenous Peoples of Asia, supra, at pp. 345, 350. 202 Section 5, Article XII, 1987 Constitution. 203 Words in bold were amendments introduced by R.A. 3872 in 1964. 204 Words in bold were amendments introduced by R.A. 3872 on June 18, 1964. On January 25, 1977, however, Sec. 48 [b] and 48 [c] were further amended by P.D. 1073 stating that these provisions on cultural minorities apply only to alienable and disposable lands of the public domain- Please see Republic v. CA and Paran, 201 SCRA 1, 10-11 [1991].

L A W O N N A T U R A L R E S O U R C E S a n d E N V I R O N M E N T A L L A W C a s e s a n d S p e c i a l L a w s | 31
205 206

Jus utendi, jus fruendi. Jus abutendi. 207 Jus disponendi. 208 Jus vindicandi. Please see Tolentino, Civil Code, vol. II, pp. 45-46 [1992]; see also Tolentino, vol. I, pp. 1214. 209 Sec. 55, IPRA provides: "Sec. 55. Communal rights.- Subject to Section 56 hereof, areas within the ancestral domains, whether delineated or not, shall be presumed to be communally held: provided, That communal rights under this Act shall not be construed as co-ownership as provided in Republic Act No. 386, otherwise known as the New Civil Code." 210 Ibid. 211 Article 494, Civil Code. 212 Antonio M. La Vina, Arguments for Communal Title, Part II, 2 Phil. Nat. Res. L. J. 23 [Dec. 1989]. 213 Section 11, Corporation Code. 214 Sections 60-72, Corporation Code. 215 Section 117, Corporation Code. Please see also La Vina, Arguments for Communal Title, Part II, supra, at 23. 216 Section 5, par. 2, Article XII, 1987 Constitution. 217 Customary law is recognized by the Local Government Code of 1991 in solving disputes among members of the indigenous communities, viz: "Sec. 412 (c) Conciliation among members of indigenous cultural communities.- The customs and traditions of indigenous cultural communities shall be applied in settling disputes between members of the cultural communities." 218 Law writes custom into contract-Hongkong & Shanghai Bank v. Peters, 16 Phil. 284 [1910]. The Civil Code provides: "Art. 11. Customs which are contrary to law, public order or public policy shall not be countenanced." "Art. 12. A custom must be proved as a fact, according to the rules of evidence." 219 Article 78 on marriages between Mohammedans or pagans who live in the non-Christian provinces- this is now Art. 33 of the Family Code; Art. 118, now Art. 74 of the Family Code on property relations between spouses; Art. 577 on the usufructuary of woodland; Art. 657 on easement of right of way for passage of livestock; Arts. 678, 1315, 1376, 1522, 1564 and 1577. Please see Aquino, Civil Code, vol. 1, p. 25. 220 Castle Bros. v. Gutierrez Hermanos, 11 Phil. 629 [1908]; In Re: Firm Name of Ozaeta Romulo, 92 SCRA 1 [1979]; Yao Kee v. Sy-Gonzales, 167 SCRA 736 [1988]; Please see Aquino, Civil Code, vol. 1, p. 26 for a list of other cases. 221 This situation is analogous to the Muslim code or the Code of Muslim Personal Laws (P.D. 1083) which took effect on February 4, 1977 despite the effectivity of the Civil Code and the Family Code. P.D. 1083 governs persons, family relations and succession among Muslims, the adjudication and settlement of disputes, the organization of the Sharia courts, etc. 222 Mariflor P. Pagusara, The Kalinga Ili: Cultural-Ecological Reflections on Indigenous Theora and Praxis of Man-Nature Relationship, Dakami Ya Nan Dagami, p. 36, Papers and Proceedings of the 1st Cordillera MutiSectoral Land Congress, 11-14 March 1983, Cordillera Consultative Committee [1984]. 223 Section 2, Article XII. 224 A "co-production agreement" is defined as one wherein the government provides input to the mining operation other than the mineral resource- Section 26 (b), R.A. 7942, the Philippine Mining Act of 1995.

225

A "joint venture agreement" is one where a joint-venture company is organized by the government and the contractor with both parties having equity shares, and the government entitled to a share in the gross output- Section 26 (c), R.A. 7942. 226 A mineral "production-sharing agreement" is one where the government grants to the contractor the exclusive right to conduct mining operations within a contract area and shares in the gross output. The contractor provides the financing, technology, management and personnel necessary for the implementation of the agreement- Section 26 (a), R.A. 7942. 227 Section 26, R.A. 7942. 228 Section 3 [d], People's Small-Scale Mining Act of 1991 (R.A. 7076) provides: "Sec. 3 [d] 'Small-scale mining contract' refers to co-production, joint venture or mineral production sharing agreement between the State and a small-scale mining contractor for the small-scale utilization of a plot of mineral land." 229 Section 3 [b], R.A. 7076. 230 NCIP Administrative Order No. 1, Series of 1998. 231 In Republic v. Court of Appeals, 160 SCRA 228, 239 [1988], Cruz, J., ponente, it was declared that if a person is the owner of a piece of agricultural land on which minerals are discovered, his ownership of such land does not give him the right to extract or utilize the said minerals without the permission of the State to which such minerals belong- also cited in H. de Leon, Phil. Constitutional Law, Principles and Cases, vol. 2, pp. 800-801 [1999]. 232 See Ground I, Grounds to Issue Writ of Prohibition, Petition, p. 14. 233 Section 7 (b) is subject to Section 56 of the same law which provides: "Sec. 56. Existing Property Rights Regimes.- Property rights within the ancestral domains already existing and/or vested upon effectivity of this Act, shall be recognized and respected." The law took effect 15 days upon publication in the O.G. or in any 2 newspapers of general circulation (Sec. 84, IPRA). The IPRA was published in the Chronicle and Malaya on Nov. 7, 1997. 234 Section 9 of the IPRA also gives the ICCs/IPs the ff. responsibilities over their ancestral domains: (a) Maintain Ecological Balance- To preserve, restore, and maintain a balanced ecology in the ancestral domain by protecting the flora and fauna, watershed areas, and other reserves; (b) Restore Denuded Areas.- To actively initiate, undertake and participate in the reforestation of denuded areas and other development programs and projects subject to just and reasonable renumeration; (c) Observe Laws.- To observe and comply with the provisions of this Act and the rules and regulations for its effective implementation." Section 58 of the same law also mandates that ancestral domains or portions thereof, which are found to be necessary for critical watersheds, mangroves, wildlife sanctuaries, wilderness, protected areas, forest cover, or reforestation as determined by appropriate agencies with the full participation of the ICCs/IPs concerned shall be maintained, managed and developed for such purposes. The ICCs/IPs concerned shall be given the responsibility to maintain, develop, protect and conserve such areas with the full and effective assistance of government agencies. 235 Hector S. de Leon, Textbook on the New Philippine Constitution pp. 473-474 [1987] citing the 1986 UP Law Constitution Project, The National Economy and Patrimony, p. 11. 236 Under the Small-Scale Mining Act of 1991, "small-scale mining" refers to "mining activities which rely heavily on manual labor using simple implements and methods and do not use explosives or heavy mining equipment"- Section 3 [b], R.A. 7076. 237 See infra., pp. 77-79?. 238 Andrew Gray, The Indigenous Movement in Asia, Indigenous Peoples of Asia, ed. By Barnes, Gray and Kingsbury, pub. By Ass'n. for Asian Studies, at 35, 42 [1995].

L A W O N N A T U R A L R E S O U R C E S a n d E N V I R O N M E N T A L L A W C a s e s a n d S p e c i a l L a w s | 32
239 240

E.g. International Indian Treaty Council, World Council of IPs. Gray, The Indigenous Movement in Asia, supra, at 44, citing the International Work Group for Indigenous Affairs, 1988. 241 Jose Paulo Kastrup, The Internationalization of Indigenous Rights from the Environmental and Human Rights Perspective, 32 Texas International Law Journal 97, 102 [1997]. 242 Benedict Kingsbury, "Indigenous Peoples" in International Law: A Constructivist Approach to the Asian Controversy, The American Journal of International Law, vol. 92: 414, 429 [1998]. 243 The World Bank supported the Chico Dam project. Due to the Kalingas' opposition, the WB pulled out of the project but the conflict between the Philippine government and the natives endured long after- Marcus Colchester, Indigenous Peoples' Rights and Sustainable Resource Use in South and Southeast Asia, Indigenous Peoples of Asia, supra, pp. 59, 71-72. 244 Kingsbury, supra, at 417. 245 Section 22, Article II, 1987 Constitution. 246 Interpellation of Senator Flavier on S.B. No. 1728, Deliberation on Second Reading, November 20, 1996, p. 20. 247 Guide to R.A. 8371, Coalition for IPs Rights and Ancestral Domains, the International Labor Organization, and the ILO-Bilance- Asia Dep't, p. 3 [1999]. 248 Also referred to as the "Indigenous and Tribal Peoples Convention, 1989." 249 See Introduction to ILO Convention No. 169, par. 4. 250 Id., pars. 5 and 6. 251 Perfecto V. Fernandez, Towards a Definition of National Policy on Recognition of Ethnic Law within the Philippine Legal Order, 55 P.L.J. 383, 385 [1980]. 252 Samuel K. Tan, A History of the Philippines, Manila Studies Association, Inc. and the Phil. National Historical Society, Inc., p. 6 [1997]. 253 Fernandez, supra, at 385, 391.

L A W O N N A T U R A L R E S O U R C E S a n d E N V I R O N M E N T A L L A W C a s e s a n d S p e c i a l L a w s | 33
G.R. No. L-30389 December 27, 1972 PEDRO LEE HONG HOK, SIMEON LEE HONG HOK, ROSITA LEE HONG HOK and LEONCIO LEE HONG HOK,petitioners, vs. ANIANO DAVID, THE HON. SECRETARY OF AGRICULTURE AND NATURAL RESOURCES, THE DIRECTOR OF LANDS and COURT OF APPEALS, respondents. Augusto A. Pardalis for petitioners. Luis General, Jr. for respondent Aniano David. Office of the Solicitor General for other respondents. FERNANDO, J.:p Petitioners in this appeal by certiorari would have us reverse a decision of respondent Court of Appeals 2 affirming a lower court judgment dismissing their complaint to have the Torrens Title of respondent Aniano David declared null and void. What makes the task for petitioners quite difficult is that their factual support for their pretension to ownership of such disputed lot through accretion was rejected by respondent Court of Appeals. Without such underpinning, they must perforce rely on a legal theory, which, to put it mildly, is distinguished by unorthodoxy and is therefore far from persuasive. A grant by 3 the government through the appropriate public officials exercising the competence duly vested in them by law is not to be set at naught on the premise, unexpressed but implied, that land not otherwise passing into private ownership may not be disposed of by the state. Such an assumption is at war with settled principles of constitutional law. It cannot receive our assent. We affirm. The decision of respondent Court of Appeals following that of the lower court makes clear that there is no legal justification for nullifying the right of respondent Aniano David to the disputed lot arising from the grant made in his favor by respondent officials. As noted in the decision under review, he "acquired lawful title thereby pursuant to his miscellaneous sales application in accordance with which an order of award and for issuance of a sales patent was made by the Director of Lands on June 18, 1958, covering Lot 2892 containing an area of 226 square meters, which is a portion of Lot 2863 of the Naga Cadastre. On the basis of the order of award of the Director of Lands the Undersecretary of Agriculture and Natural Resources issued on August 26, 1959, Miscellaneous Sales Patent No. V-1209 pursuant to which OCT No. 510 was issued by the Register of Deeds of Naga City to defendant-appellee Aniano David on October 21, 1959. According to the Stipulation of Facts, since the filing of the sales application of Aniano David and during all the proceedings in connection with said application, up to the actual issuance of the sales patent in his favor, the plaintiffs-appellants did not put up any opposition or adverse claim thereto. This is fatal to them because after the registration and issuance of the certificate and duplicate certificate of title based on a public land patent, the land covered thereby automatically comes under the operation of Republic Act
1

496 subject to all the safeguards provided therein.... Under Section 38 of Act 496 any question concerning the validity of the certificate of title based on fraud should be raised within one year from the date of the issuance of the patent. Thereafter the certificate of title based thereon becomes indefeasible.... In this case the land in question is not a private property as the Director of Lands and the Secretary of Agriculture and Natural Resources have always sustained the public character thereof for having been formed by reclamation.... The only remedy therefore, available to the appellants is an action for reconveyance on the ground of fraud. In this case we do not see any fraud committed by defendantappellant Aniano David in applying for the purchase of the land involved through his Miscellaneous Sales Application No. MSA-V-26747, entered in the records of the Bureau of Lands [Miscellaneous Sales] Entry No. V-9033, because everything was done in the open. The notices regarding the auction sale of the land were published, the actual sale and award thereof to Aniano David were not clandestine but open and public official acts of an officer of the Government. The application was merely a renewal of his deceased 4 wife's application, and the said deceased occupied the land since 1938." On such finding of facts, the attempt of petitioners to elicit a different conclusion is likely to be attended with frustration. The first error assigned predicated an accretion having taken place, notwithstanding its rejection by respondent Court of Appeals, would seek to disregard what was accepted by respondent Court as to how the disputed lot came into being, namely by reclamation. It does not therefore call for any further consideration. Neither of the other two errors imputed to respondent Court, as to its holding that authoritative doctrines preclude a party other than the government to dispute the validity of a grant and the recognition of the indefeasible character of a public land patent after one year, is possessed of merit. Consequently, as set forth at the outset, there is no justification for reversal. 1. More specifically, the shaft of criticism was let loose by petitioner aimed at this legal proposition set forth in the exhaustive opinion of then Justice Salvador Esguerra of the Court of Appeals, now a member of this Court: "There is, furthermore, a fatal defect of parties to this action. Only the Government, represented by the Director of Lands, or the Secretary of Agriculture and Natural Resources, can bring an action to cancel a void certificate of title issued pursuant to a void patent (Lucas vs. Durian, 102 Phil. 1157; Director of Lands vs. Heirs of Ciriaco Carlo, G.R. No. L-12485, July 31, 1959). This was not done by said officers but by private parties like the plaintiffs, who cannot claim that the patent and title issued for the land involved are void since they are not the registered owners thereof nor had they been declared as owners in the cadastral proceedings of Naga Cadastre after claiming it as their private property. The cases cited by appellants are not in point as they refer to private registered lands or public lands over which vested rights have been acquired but notwithstanding such fact the Land Department subsequently 5 granted patents to public land applicants." Petitioner ought to have known better. The above excerpt is 6 invulnerable to attack. It is a restatement of a principle that dates back to Maninang v. Consolacion, a 1908 decision. As was there categorically stated: "The fact that the grant was made by the government is undisputed. Whether the grant was in conformity with the law or not is a question which the government may raise, but until it is raised by the government and set aside, the defendant can not question it. The 7 legality of the grant is a question between the grantee and the government." The above citation was 8 repeated ipsissimis verbis inSalazar v. Court of Appeals. Bereft as petitioners were of the right of

L A W O N N A T U R A L R E S O U R C E S a n d E N V I R O N M E N T A L L A W C a s e s a n d S p e c i a l L a w s | 34
ownership in accordance with the findings of the Court of Appeals, they cannot, in the language of Reyes 9 10 v. Rodriguez, "question the [title] legally issued." The second assignment of error is thus disposed of. 2. As there are overtones indicative of skepticism, if not of outright rejection, of the well-known distinction in public law between the government authority possessed by the state which is appropriately embraced in the concept of sovereignty, and its capacity to own or acquire property, it is not inappropriate to pursue the matter further. The former comes under the heading of imperium and the latter of dominium. The use of this term is appropriate with reference to lands held by the state in its proprietary character. In such capacity, it may provide for the exploitation and use of lands and other natural resources, including their disposition, except as limited by the Constitution. Dean Pound did speak of the confusion that existed during the medieval era between such two concepts, but did note the 11 existence of res publicae as a corollary to dominium." As far as the Philippines was concerned, there 12 was a recognition by Justice Holmes in Cario v. Insular Government, a case of Philippine origin, that "Spain in its earlier decrees embodied the universal feudal theory that all lands were held from the 13 14 Crown...." That was a manifestation of the concept of jura regalia, which was adopted by the present Constitution, ownership however being vested in the state as such rather than the head thereof. What was stated by Holmes served to confirm a much more extensive discussion of the matter in the leading 15 case of Valenton v. Murciano, decided in 1904. One of the royal decrees cited was incorporated in the 16 Recopilacion de Leyes de las Indias in these words: "We having acquired full sovereignty over the Indies and all lands, territories, and possessions not heretofore ceded away by our royal predecessors, or by us, or in our name, still pertaining to the royal crown and patrimony, it is our will that all lands which are held without proper and true deeds of grant be restored to us according as they belong to us, in order that after reserving before all what to us or to our viceroys audiences, and governors may seem necessary for public squares, ways, pastures, and commons in those places which are peopled, taking into consideration not only their present condition, but also their future and their probable increase, and after distributing to the natives what may be necessary for tillage and pasturage, confirming them in what they now have and giving them more if necessary, all the rest of said lands may remain free and unencumbered for us to 17 dispose of as we may wish." It could therefore be affirmed in Montano v. Insular Government" that "as to the unappropriated public 19 lands constituting the public domain the sole power of legislation is vested in Congress, ..." They 20 continue to possess that character until severed therefrom by state grant. Where, as in this case, it was found by the Court of Appeals that the disputed lot was the result of reclamation, its being correctly 21 categorized as public land is undeniable. What was held in Heirs of Datu Pendatun v. Director of 22 Lands finds application. Thus: "There being no evidence whatever that the property in question was ever acquired by the applicants or their ancestors either by composition title from the Spanish Government or by possessory information title or by any other means for the acquisition of public lands, 23 the property must be held to be public domain." For it is well-settled "that no public land can be 24 acquired by private persons without any grant, express or implied, from the government." It is indispensable then that there be a showing of a title from the state or any other mode of acquisition 25 recognized by law. The most recent restatement of the doctrine, found in an opinion of Justice J.B.L. 26 Reyes, follows: "The applicant, having failed to establish his right or title over the northern portion of
18

Lot No. 463 involved in the present controversy, and there being no showing that the same has been acquired by any private person from the Government, either by purchase or by grant, the property is and 27 remains part of the public domain." To repeat, the second assignment of error is devoid of merit. 3. The last error assigned would take issue with this portion of the opinion of Justice Esguerra: "According to the Stipulation of Facts, since the filing of the sales application of Aniano David and during all the proceedings in connection with said application, up to the actual issuance of the sales patent in his favor, the plaintiffs-appellants did not put up any opposition or adverse claim thereto. This is fatal to them because after the registration and issuance of the certificate and duplicate certificate of title based on a public land patent, the land covered thereby automatically comes under the operation of Republic Act 496 subject to all the safeguards provided therein ... Under Section 38 of Act 496 any question concerning the validity of the certificate of title based on fraud should be raised within one year from the date of the issuance of 28 the patent. Thereafter the certificate of title based thereon becomes indefeasible ..." Petitioners cannot reconcile themselves to the view that respondent David's title is impressed with the quality of indefeasibility. In thus manifesting such an attitude, they railed to accord deference to controlling precedents. As far back as 1919, in Aquino v. Director of 29 Lands, Justice Malcolm, speaking for the Court, stated: "The proceedings under the Land Registration Law and under the provisions of Chapter VI of the Public Land Law are the same in that both are against the whole world, both take the nature of judicial proceedings, and for both the decree of registration 30 31 issued is conclusive and final." Such a view has been followed since then. The latest case in point 32 is Cabacug v. Lao. There is this revealing excerpt appearing in that decision: "It is said, and with reason, that a holder of a land acquired under a free patent is more favorably situated than that of an owner of registered property. Not only does a free patent have a force and effect of a Torrens Title, but in addition the person to whom it is granted has likewise in his favor the right to repurchase within a period of five 33 years." It is quite apparent, therefore, that petitioners' stand is legally indefensible. WHEREFORE, the decision of respondent Court of Appeals of January 31, 1969 and its resolution of March 14, 1969 are affirmed. With costs against petitioners-appellants. Concepcion, C.J., Makalintal, Zaldivar, Castro, Teehankee Barredo, Makasiar, Antonio and Esguerra, JJ., concur.
Footnotes 1 Petitioners are Pedro Lee Hong Hok, Simeon Lee Hong Hok, Rosita Lee Hong Hok and Leoncio Lee Hong Hok. 2 Original Certificate of Title No. 510 of the Registry of Naga City. 3 The Secretary of Agriculture and Natural Resources and the Director of Lands were likewise named respondents. 4 Decision Appendix A, Brief for the Petitioners, 54-56. 5 Ibid, 56-57. 6 12 Phil. 342. 7 Ibid, 349.

L A W O N N A T U R A L R E S O U R C E S a n d E N V I R O N M E N T A L L A W C a s e s a n d S p e c i a l L a w s | 35
8 87 Phil. 456 (1950). 9 62 Phil. 771 (1936). 10 Ibid, 776. 11 Cf. 3 Pound, Jurisprudence, 108-109 (1959). 12 212 US 449 (1909). 13 Ibid, 458. 14 Cf. Radin, Law Dictionary, 288 (1955). 15 3 Phil. 537. 16 Law I, Title 12, Book 4. 17 3 Phil. 537, 542-543. 18 12 Phil. 572 (1909). 19 Ibid, 579. Cf. Insular Government v. Aldecoa & Co., 19 Phil. 505, (1911); Ankron v. Government of the Philippine Islands, 40 Phil. 10 (1919). 20 Cf. Archbishop of Manila v. Director of Lands, 27 Phil. 245 (1914). 21 Cf. Aragon v. Insular Government, 19 Phil. 223 (1911); Francisco v. Government, 28 Phil. 505 (1914); Government v. Cabangis 53 Phil. 112 (1929). 22 59 Phil. 600 (1934). 23 Ibid, 603. 24 Padilla v. Reyes, 60 Phil. 967, 969 (1934). 25 Cf. Reyes v. Rodriguez, 62 Phil. 771 (1936). 26 Director of Lands v. Court of Appeals, L-29575, April 30, 1971, 38 Phil. 634. 27 Ibid, 639. 28 Decision, Appendix A to Brief for Petitioners, 54-55. 29 39 Phil. 850 (1919). 30 Ibid, 858. 31 Cf. Manalo v. Lukban, 48 Phil. 973, (1924); El Hogar Filipino v. Olviga, 60 Phil. 17 (1934); Republic of the Philippines v. Abacite, L-15415, April 26, 1961, 1 SCRA 1076; Panimdim v. Director of Lands, L-19731, July 31, 1964, 11 SCRA 628; Director of Lands v. The Court of Appeals, L-17696, May 19, 1966, 17 SCRA 71; Antonio v. Barroga, L-23769, April 29, 1968, 23 SCRA 357; Dela Cruz v. Reano, L-29792, August 31, 1970, 34 SCRA 585. 32 L-27036, November 26, 1970, 36 SCRA 92. 33 Ibid, 96.

L A W O N N A T U R A L R E S O U R C E S a n d E N V I R O N M E N T A L L A W C a s e s a n d S p e c i a l L a w s | 36
G.R. No. 127882 January 27, 2004 Administrative Order 96-40, and of the Financial and Technical Assistance Agreement (FTAA) entered into on March 30, 1995 by the Republic of the Philippines and WMC (Philippines), Inc. (WMCP), a corporation organized under Philippine laws. On July 25, 1987, then President Corazon C. Aquino issued Executive Order (E.O.) No. 279 authorizing the DENR Secretary to accept, consider and evaluate proposals from foreign-owned corporations or foreign investors for contracts or agreements involving either technical or financial assistance for large-scale exploration, development, and utilization of minerals, which, upon appropriate recommendation of the Secretary, the President may execute with the foreign proponent. In entering into such proposals, the President shall consider the real contributions to the economic growth and general welfare of the country that will be realized, as well as the development and use of local scientific and technical resources that will be promoted by the proposed contract or agreement. Until Congress shall determine otherwise, large-scale mining, for purpose of this Section, shall mean those proposals for contracts or agreements for mineral resources exploration, development, and utilization involving a committed capital investment in a 7 single mining unit project of at least Fifty Million Dollars in United States Currency (US $50,000,000.00). On March 3, 1995, then President Fidel V. Ramos approved R.A. No. 7942 to "govern the exploration, 8 development, utilization and processing of all mineral resources." R.A. No. 7942 defines the modes of 9 mineral agreements for mining operations, outlines the procedure for their filing and 10 11 12 13 approval, assignment/transfer and withdrawal, and fixes their terms. Similar provisions govern 14 financial or technical assistance agreements. The law prescribes the qualifications of contractors and grants them certain rights, including 16 17 18 19 timber, water and easement rights, and the right to possess explosives. Surface owners, occupants, or concessionaires are forbidden from preventing holders of mining rights from entering private lands and 20 21 concession areas. A procedure for the settlement of conflicts is likewise provided for. The Act restricts the conditions for exploration, quarry and other permits. It regulates the transport, 25 sale and processing of minerals, and promotes the development of mining communities, science and 26 27 mining technology, and safety and environmental protection. The government's share in the agreements is spelled out and allocated, taxes and fees are 29 30 31 imposed, incentives granted. Aside from penalizing certain acts, the law likewise specifies grounds for 32 the cancellation, revocation and termination of agreements and permits. On April 9, 1995, 30 days following its publication on March 10, 1995 in Malaya and Manila Times, two 33 newspapers of general circulation, R.A. No. 7942 took effect. Shortly before the effectivity of R.A. No. 7942, however, or on March 30, 1995, the President entered into an FTAA with WMCP covering 99,387 34 hectares of land in South Cotabato, Sultan Kudarat, Davao del Sur and North Cotabato.
28 22 23 24 15 6

LA BUGAL-B'LAAN TRIBAL ASSOCIATION, INC., represented by its Chairman F'LONG MIGUEL M. LUMAYONG, WIGBERTO E. TAADA, PONCIANO BENNAGEN, JAIME TADEO, RENATO R. CONSTANTINO, JR., F'LONG AGUSTIN M. DABIE, ROBERTO P. AMLOY, RAQIM L. DABIE, SIMEON H. DOLOJO, IMELDA M. GANDON, LENY B. GUSANAN, MARCELO L. GUSANAN, QUINTOL A. LABUAYAN, LOMINGGES D. LAWAY, BENITA P. TACUAYAN, minors JOLY L. BUGOY, represented by his father UNDERO D. BUGOY, ROGER M. DADING, represented by his father ANTONIO L. DADING, ROMY M. LAGARO, represented by his father TOTING A. LAGARO, MIKENY JONG B. LUMAYONG, represented by his father MIGUEL M. LUMAYONG, RENE T. MIGUEL, represented by his mother EDITHA T. MIGUEL, ALDEMAR L. SAL, represented by his father DANNY M. SAL, DAISY RECARSE, represented by her mother LYDIA S. SANTOS, EDWARD M. EMUY, ALAN P. MAMPARAIR, MARIO L. MANGCAL, ALDEN S. TUSAN, AMPARO S. YAP, VIRGILIO CULAR, MARVIC M.V.F. LEONEN, JULIA REGINA CULAR, GIAN CARLO CULAR, VIRGILIO CULAR, JR., represented by their father VIRGILIO CULAR, PAUL ANTONIO P. VILLAMOR, represented by his parents JOSE VILLAMOR and ELIZABETH PUA-VILLAMOR, ANA GININA R. TALJA, represented by her father MARIO JOSE B. TALJA, SHARMAINE R. CUNANAN, represented by her father ALFREDO M. CUNANAN, ANTONIO JOSE A. VITUG III, represented by his mother ANNALIZA A. VITUG, LEAN D. NARVADEZ, represented by his father MANUEL E. NARVADEZ, JR., ROSERIO MARALAG LINGATING, represented by her father RIO OLIMPIO A. LINGATING, MARIO JOSE B. TALJA, DAVID E. DE VERA, MARIA MILAGROS L. SAN JOSE, SR., 1 SUSAN O. BOLANIO, OND, LOLITA G. DEMONTEVERDE, BENJIE L. NEQUINTO, ROSE LILIA S. ROMANO, ROBERTO S. VERZOLA, EDUARDO AURELIO C. REYES, LEAN LOUEL A. PERIA, represented by his father 2 ELPIDIO V. PERIA, GREEN FORUM PHILIPPINES, GREEN FORUM WESTERN VISAYAS, (GF-WV), ENVIRONMETAL LEGAL ASSISTANCE CENTER (ELAC), PHILIPPINE KAISAHAN TUNGO SA KAUNLARAN NG 3 KANAYUNAN AT REPORMANG PANSAKAHAN (KAISAHAN), KAISAHAN TUNGO SA KAUNLARAN NG KANAYUNAN AT REPORMANG PANSAKAHAN (KAISAHAN), PARTNERSHIP FOR AGRARIAN REFORM and RURAL DEVELOPMENT SERVICES, INC. (PARRDS), PHILIPPINE PART`NERSHIP FOR THE DEVELOPMENT OF HUMAN RESOURCES IN THE RURAL AREAS, INC. (PHILDHRRA), WOMEN'S LEGAL BUREAU (WLB), CENTER FOR ALTERNATIVE DEVELOPMENT INITIATIVES, INC. (CADI), UPLAND DEVELOPMENT INSTITUTE (UDI), KINAIYAHAN FOUNDATION, INC., SENTRO NG ALTERNATIBONG LINGAP PANLIGAL (SALIGAN), LEGAL RIGHTS AND NATURAL RESOURCES CENTER, INC. (LRC), petitioners, vs. VICTOR O. RAMOS, SECRETARY, DEPARTMENT OF ENVIRONMENT AND NATURAL RESOURCES (DENR), HORACIO RAMOS, DIRECTOR, MINES AND GEOSCIENCES BUREAU (MGB-DENR), RUBEN TORRES, 4 EXECUTIVE SECRETARY, and WMC (PHILIPPINES), INC. respondents. DECISION CARPIO-MORALES, J.: The present petition for mandamus and prohibition assails the constitutionality of Republic Act No. 5 7942, otherwise known as the PHILIPPINE MINING ACT OF 1995, along with the Implementing Rules and Regulations issued pursuant thereto, Department of Environment and Natural Resources (DENR)

L A W O N N A T U R A L R E S O U R C E S a n d E N V I R O N M E N T A L L A W C a s e s a n d S p e c i a l L a w s | 37
On August 15, 1995, then DENR Secretary Victor O. Ramos issued DENR Administrative Order (DAO) No. 95-23, s. 1995, otherwise known as the Implementing Rules and Regulations of R.A. No. 7942. This was later repealed by DAO No. 96-40, s. 1996 which was adopted on December 20, 1996. On January 10, 1997, counsels for petitioners sent a letter to the DENR Secretary demanding that the 35 DENR stop the implementation of R.A. No. 7942 and DAO No. 96-40, giving the DENR fifteen days from 36 37 receipt to act thereon. The DENR, however, has yet to respond or act on petitioners' letter. Petitioners thus filed the present petition for prohibition and mandamus, with a prayer for a temporary restraining order. They allege that at the time of the filing of the petition, 100 FTAA applications had 38 already been filed, covering an area of 8.4 million hectares, 64 of which applications are by fully foreignowned corporations covering a total of 5.8 million hectares, and at least one by a fully foreign-owned 39 mining company over offshore areas. Petitioners claim that the DENR Secretary acted without or in excess of jurisdiction: I x x x in signing and promulgating DENR Administrative Order No. 96-40 implementing Republic Act No. 7942, the latter being unconstitutional in that it allows fully foreign owned corporations to explore, develop, utilize and exploit mineral resources in a manner contrary to Section 2, paragraph 4, Article XII of the Constitution; II x x x in signing and promulgating DENR Administrative Order No. 96-40 implementing Republic Act No. 7942, the latter being unconstitutional in that it allows the taking of private property without the determination of public use and for just compensation; III x x x in signing and promulgating DENR Administrative Order No. 96-40 implementing Republic Act No. 7942, the latter being unconstitutional in that it violates Sec. 1, Art. III of the Constitution; IV x x x in signing and promulgating DENR Administrative Order No. 96-40 implementing Republic Act No. 7942, the latter being unconstitutional in that it allows enjoyment by foreign citizens as well as fully foreign owned corporations of the nation's marine wealth contrary to Section 2, paragraph 2 of Article XII of the Constitution; (b) Declaring the Philippine Mining Act of 1995 or Republic Act No. 7942 as unconstitutional and null and void; (c) Declaring the Implementing Rules and Regulations of the Philippine Mining Act contained in DENR Administrative Order No. 96-40 and all other similar administrative issuances as unconstitutional and null and void; and (d) Cancelling the Financial and Technical Assistance Agreement issued to Western Mining 41 Philippines, Inc. as unconstitutional, illegal and null and void. Impleaded as public respondents are Ruben Torres, the then Executive Secretary, Victor O. Ramos, the then DENR Secretary, and Horacio Ramos, Director of the Mines and Geosciences Bureau of the DENR. Also impleaded is private respondent WMCP, which entered into the assailed FTAA with the Philippine Government. WMCP is owned by WMC Resources International Pty., Ltd. (WMC), "a wholly owned subsidiary of Western Mining Corporation Holdings Limited, a publicly listed major Australian mining and 42 43 exploration company." By WMCP's information, "it is a 100% owned subsidiary of WMC LIMITED." x x x in recommending approval of and implementing the Financial and Technical Assistance Agreement between the President of the Republic of the Philippines and Western Mining Corporation Philippines Inc. 40 because the same is illegal and unconstitutional. They pray that the Court issue an order: (a) Permanently enjoining respondents from acting on any application for Financial or Technical Assistance Agreements; V x x x in signing and promulgating DENR Administrative Order No. 96-40 implementing Republic Act No. 7942, the latter being unconstitutional in that it allows priority to foreign and fully foreign owned corporations in the exploration, development and utilization of mineral resources contrary to Article XII of the Constitution; VI x x x in signing and promulgating DENR Administrative Order No. 96-40 implementing Republic Act No. 7942, the latter being unconstitutional in that it allows the inequitable sharing of wealth contrary to Sections [sic] 1, paragraph 1, and Section 2, paragraph 4[,] [Article XII] of the Constitution; VII

L A W O N N A T U R A L R E S O U R C E S a n d E N V I R O N M E N T A L L A W C a s e s a n d S p e c i a l L a w s | 38
Respondents, aside from meeting petitioners' contentions, argue that the requisites for judicial inquiry have not been met and that the petition does not comply with the criteria for prohibition and mandamus. Additionally, respondent WMCP argues that there has been a violation of the rule on hierarchy of courts. After petitioners filed their reply, this Court granted due course to the petition. The parties have since filed their respective memoranda. WMCP subsequently filed a Manifestation dated September 25, 2002 alleging that on January 23, 2001, WMC sold all its shares in WMCP to Sagittarius Mines, Inc. (Sagittarius), a corporation organized under 44 Philippine laws. WMCP was subsequently renamed "Tampakan Mineral Resources 45 Corporation." WMCP claims that at least 60% of the equity of Sagittarius is owned by Filipinos and/or Filipino-owned corporations while about 40% is owned by Indophil Resources NL, an Australian 46 company. It further claims that by such sale and transfer of shares, "WMCP has ceased to be connected 47 in any way with WMC." By virtue of such sale and transfer, the DENR Secretary, by Order of December 18, 2001, approved the transfer and registration of the subject FTAA from WMCP to Sagittarius. Said Order, however, was appealed by Lepanto Consolidated Mining Co. (Lepanto) to the Office of the President which upheld it by 49 Decision of July 23, 2002. Its motion for reconsideration having been denied by the Office of the 50 51 President by Resolution of November 12, 2002, Lepanto filed a petition for review before the Court of Appeals. Incidentally, two other petitions for review related to the approval of the transfer and 52 registration of the FTAA to Sagittarius were recently resolved by this Court. It bears stressing that this case has not been rendered moot either by the transfer and registration of the FTAA to a Filipino-owned corporation or by the non-issuance of a temporary restraining order or a 53 preliminary injunction to stay the above-said July 23, 2002 decision of the Office of the President. The validity of the transfer remains in dispute and awaits final judicial determination. This assumes, of course, that such transfer cures the FTAA's alleged unconstitutionality, on which question judgment is reserved. WMCP also points out that the original claimowners of the major mineralized areas included in the WMCP FTAA, namely, Sagittarius, Tampakan Mining Corporation, and Southcot Mining Corporation, are all 54 Filipino-owned corporations, each of which was a holder of an approved Mineral Production Sharing Agreement awarded in 1994, albeit their respective mineral claims were subsumed in the WMCP 55 FTAA; and that these three companies are the same companies that consolidated their interests in 56 Sagittarius to whom WMC sold its 100% equity in WMCP. WMCP concludes that in the event that the FTAA is invalidated, the MPSAs of the three corporations would be revived and the mineral claims would 57 revert to their original claimants. These circumstances, while informative, are hardly significant in the resolution of this case, it involving the validity of the FTAA, not the possible consequences of its invalidation.
48

Of the above-enumerated seven grounds cited by petitioners, as will be shown later, only the first and the last need be delved into; in the latter, the discussion shall dwell only insofar as it questions the effectivity of E. O. No. 279 by virtue of which order the questioned FTAA was forged. I Before going into the substantive issues, the procedural questions posed by respondents shall first be tackled. REQUISITES FOR JUDICIAL REVIEW When an issue of constitutionality is raised, this Court can exercise its power of judicial review only if the following requisites are present: (1) The existence of an actual and appropriate case; (2) A personal and substantial interest of the party raising the constitutional question; (3) The exercise of judicial review is pleaded at the earliest opportunity; and (4) The constitutional question is the lis mota of the case. Respondents claim that the first three requisites are not present. Section 1, Article VIII of the Constitution states that "(j)udicial power includes the duty of the courts of justice to settle actual controversies involving rights which are legally demandable and enforceable." The 59 power of judicial review, therefore, is limited to the determination of actual cases and controversies. An actual case or controversy means an existing case or controversy that is appropriate or ripe for 60 determination, not conjectural or anticipatory, lest the decision of the court would amount to an 61 62 advisory opinion. The power does not extend to hypothetical questions since any attempt at abstraction could only lead to dialectics and barren legal questions and to sterile conclusions unrelated to 63 actualities. "Legal standing" or locus standi has been defined as a personal and substantial interest in the case such that the party has sustained or will sustain direct injury as a result of the governmental act that is being 64 65 challenged, alleging more than a generalized grievance. The gist of the question of standing is whether a party alleges "such personal stake in the outcome of the controversy as to assure that concrete adverseness which sharpens the presentation of issues upon which the court depends for illumination of
58

L A W O N N A T U R A L R E S O U R C E S a n d E N V I R O N M E N T A L L A W C a s e s a n d S p e c i a l L a w s | 39
difficult constitutional questions." Unless a person is injuriously affected in any of his constitutional 67 rights by the operation of statute or ordinance, he has no standing. Petitioners traverse a wide range of sectors. Among them are La Bugal B'laan Tribal Association, Inc., a farmers and indigenous people's cooperative organized under Philippine laws representing a community 68 actually affected by the mining activities of WMCP, members of said cooperative, as well as other 69 residents of areas also affected by the mining activities of WMCP. These petitioners have standing to raise the constitutionality of the questioned FTAA as they allege a personal and substantial injury. They 70 claim that they would suffer "irremediable displacement" as a result of the implementation of the FTAA allowing WMCP to conduct mining activities in their area of residence. They thus meet the appropriate case requirement as they assert an interest adverse to that of respondents who, on the other hand, insist on the FTAA's validity. In view of the alleged impending injury, petitioners also have standing to assail the validity of E.O. No. 279, by authority of which the FTAA was executed. Public respondents maintain that petitioners, being strangers to the FTAA, cannot sue either or both 71 contracting parties to annul it. In other words, they contend that petitioners are not real parties in interest in an action for the annulment of contract. Public respondents' contention fails. The present action is not merely one for annulment of contract but for prohibition and mandamus. Petitioners allege that public respondents acted without or in excess of jurisdiction in implementing the FTAA, which they submit is unconstitutional. As the case involves constitutional questions, this Court is not concerned with whether petitioners are real parties in interest, 72 but with whether they have legal standing. As held in Kilosbayan v. Morato: x x x. "It is important to note . . . that standing because of its constitutional and public policy underpinnings, is very different from questions relating to whether a particular plaintiff is the real party in interest or has capacity to sue. Although all three requirements are directed towards ensuring that only certain parties can maintain an action, standing restrictions require a partial consideration of the merits, as well as broader policy concerns relating to the proper role of the judiciary in certain areas.["] (FRIEDENTHAL, KANE AND MILLER, CIVIL PROCEDURE 328 [1985]) Standing is a special concern in constitutional law because in some cases suits are brought not by parties who have been personally injured by the operation of a law or by official action taken, but by concerned citizens, taxpayers or voters who actually sue in the public interest. Hence, the question in standing is whether such parties have "alleged such a personal stake in the outcome of the controversy as to assure that concrete adverseness which sharpens the presentation of issues upon which the court so largely depends for illumination of difficult constitutional questions." (Baker v. Carr, 369 U.S. 186, 7 L.Ed.2d 633 [1962].)
66

As earlier stated, petitioners meet this requirement. The challenge against the constitutionality of R.A. No. 7942 and DAO No. 96-40 likewise fulfills the requisites of justiciability. Although these laws were not in force when the subject FTAA was entered into, the question as to their validity is ripe for adjudication. The WMCP FTAA provides: 14.3 Future Legislation Any term and condition more favourable to Financial &Technical Assistance Agreement contractors resulting from repeal or amendment of any existing law or regulation or from the enactment of a law, regulation or administrative order shall be considered a part of this Agreement. It is undisputed that R.A. No. 7942 and DAO No. 96-40 contain provisions that are more favorable to WMCP, hence, these laws, to the extent that they are favorable to WMCP, govern the FTAA. In addition, R.A. No. 7942 explicitly makes certain provisions apply to pre-existing agreements. SEC. 112. Non-impairment of Existing Mining/Quarrying Rights. x x x That the provisions of Chapter XIV on government share in mineral production-sharing agreement and of Chapter XVI on incentives of this Act shall immediately govern and apply to a mining lessee or contractor unless the mining lessee or contractor indicates his intention to the secretary, in writing, not to avail of said provisions x x x Provided, finally, That such leases, production-sharing agreements, financial or technical assistance agreements shall comply with the applicable provisions of this Act and its implementing rules and regulations. As there is no suggestion that WMCP has indicated its intention not to avail of the provisions of Chapter XVI of R.A. No. 7942, it can safely be presumed that they apply to the WMCP FTAA. Misconstruing the application of the third requisite for judicial review that the exercise of the review is pleaded at the earliest opportunity WMCP points out that the petition was filed only almost two years after the execution of the FTAA, hence, not raised at the earliest opportunity. The third requisite should not be taken to mean that the question of constitutionality must be raised immediately after the execution of the state action complained of. That the question of constitutionality 73 has not been raised before is not a valid reason for refusing to allow it to be raised later. A contrary rule would mean that a law, otherwise unconstitutional, would lapse into constitutionality by the mere failure of the proper party to promptly file a case to challenge the same. PROPRIETY OF PROHIBITION AND MANDAMUS

L A W O N N A T U R A L R E S O U R C E S a n d E N V I R O N M E N T A L L A W C a s e s a n d S p e c i a l L a w s | 40
Before the effectivity in July 1997 of the Revised Rules of Civil Procedure, Section 2 of Rule 65 read: SEC. 2. Petition for prohibition. When the proceedings of any tribunal, corporation, board, or person, whether exercising functions judicial or ministerial, are without or in excess of its or his jurisdiction, or with grave abuse of discretion, and there is no appeal or any other plain, speedy, and adequate remedy in the ordinary course of law, a person aggrieved thereby may file a verified petition in the proper court alleging the facts with certainty and praying that judgment be rendered commanding the defendant to desist from further proceeding in the action or matter specified therein. Prohibition is a preventive remedy. It seeks a judgment ordering the defendant to desist from continuing 75 with the commission of an act perceived to be illegal. The petition for prohibition at bar is thus an appropriate remedy. While the execution of the contract itself may be fait accompli, its implementation is not. Public respondents, in behalf of the Government, have obligations to fulfill under said contract. Petitioners seek to prevent them from fulfilling such obligations on the theory that the contract is unconstitutional and, therefore, void. The propriety of a petition for prohibition being upheld, discussion of the propriety of the mandamus aspect of the petition is rendered unnecessary. HIERARCHY OF COURTS The contention that the filing of this petition violated the rule on hierarchy of courts does not likewise lie. The rule has been explained thus: Between two courts of concurrent original jurisdiction, it is the lower court that should initially pass upon the issues of a case. That way, as a particular case goes through the hierarchy of courts, it is shorn of all but the important legal issues or those of first impression, which are the proper subject of attention of the appellate court. This is a procedural rule borne of experience and adopted to improve the administration of justice. This Court has consistently enjoined litigants to respect the hierarchy of courts. Although this Court has concurrent jurisdiction with the Regional Trial Courts and the Court of Appeals to issue writs of certiorari, prohibition, mandamus, quo warranto, habeas corpus and injunction, such concurrence does not give a party unrestricted freedom of choice of court forum. The resort to this Court's primary jurisdiction to issue said writs shall be allowed only where the redress desired cannot be obtained in the appropriate courts or where exceptional and compelling circumstances justify such invocation. We held in People v. Cuaresma that: A becoming regard for judicial hierarchy most certainly indicates that petitions for the issuance of extraordinary writs against first level ("inferior") courts should be filed with the Regional Trial Court, and And they likewise claim that the WMC FTAA, which was entered into pursuant to E.O. No. 279, violates Section 2, Article XII of the Constitution because, among other reasons: (1) It allows foreign-owned companies to extend more than mere financial or technical assistance to the State in the exploitation, development, and utilization of minerals, petroleum, and other mineral oils, and even permits foreign owned companies to "operate and manage mining activities." (2) It allows foreign-owned companies to extend both technical and financial assistance, instead of "either technical or financial assistance." To appreciate the import of these issues, a visit to the history of the pertinent constitutional provision, the concepts contained therein, and the laws enacted pursuant thereto, is in order. Section 2, Article XII reads in full: Sec. 2. All lands of the public domain, waters, minerals, coal, petroleum, and other mineral oils, all forces of potential energy, fisheries, forests or timber, wildlife, flora and fauna, and other natural resources are owned by the State. With the exception of agricultural lands, all other natural resources shall not be alienated. The exploration, development, and utilization of natural resources shall be under the full
74

those against the latter, with the Court of Appeals. A direct invocation of the Supreme Court's original jurisdiction to issue these writs should be allowed only where there are special and important reasons therefor, clearly and specifically set out in the petition. This is established policy. It is a policy necessary to prevent inordinate demands upon the Court's time and attention which are better devoted to those matters within its exclusive jurisdiction, and to prevent further over-crowding of the Court's docket x x 76 x. [Emphasis supplied.] The repercussions of the issues in this case on the Philippine mining industry, if not the national economy, as well as the novelty thereof, constitute exceptional and compelling circumstances to justify resort to this Court in the first instance. In all events, this Court has the discretion to take cognizance of a suit which does not satisfy the 77 requirements of an actual case or legal standing when paramount public interest is involved. When the issues raised are of paramount importance to the public, this Court may brush aside technicalities of 78 procedure. II Petitioners contend that E.O. No. 279 did not take effect because its supposed date of effectivity came after President Aquino had already lost her legislative powers under the Provisional Constitution.

L A W O N N A T U R A L R E S O U R C E S a n d E N V I R O N M E N T A L L A W C a s e s a n d S p e c i a l L a w s | 41
control and supervision of the State. The State may directly undertake such activities or it may enter into co-production, joint venture, or production-sharing agreements with Filipino citizens, or corporations or associations at least sixty per centum of whose capital is owned by such citizens. Such agreements may be for a period not exceeding twenty-five years, renewable for not more than twenty-five years, and under such terms and conditions as may be provided by law. In cases of water rights for irrigation, water supply, fisheries, or industrial uses other than the development of water power, beneficial use may be the measure and limit of the grant. The State shall protect the nation's marine wealth in its archipelagic waters, territorial sea, and exclusive economic zone, and reserve its use and enjoyment exclusively to Filipino citizens. The Congress may, by law, allow small-scale utilization of natural resources by Filipino citizens, as well as cooperative fish farming, with priority to subsistence fishermen and fish-workers in rivers, lakes, bays, and lagoons. The President may enter into agreements with foreign-owned corporations involving either technical or financial assistance for large-scale exploration, development, and utilization of minerals, petroleum, and other mineral oils according to the general terms and conditions provided by law, based on real contributions to the economic growth and general welfare of the country. In such agreements, the State shall promote the development and use of local scientific and technical resources. The President shall notify the Congress of every contract entered into in accordance with this provision, within thirty days from its execution. THE SPANISH REGIME AND THE REGALIAN DOCTRINE The first sentence of Section 2 embodies the Regalian doctrine or jura regalia. Introduced by Spain into these Islands, this feudal concept is based on the State's power of dominium, which is the capacity of the 79 State to own or acquire property. In its broad sense, the term "jura regalia" refers to royal rights, or those rights which the King has by virtue of his prerogatives. In Spanish law, it refers to a right which the sovereign has over anything in which a subject has a right of property or propriedad. These were rights enjoyed during feudal times by the king as the sovereign. The theory of the feudal system was that title to all lands was originally held by the King, and while the use of lands was granted out to others who were permitted to hold them under certain conditions, the King theoretically retained the title. By fiction of law, the King was regarded as the original proprietor of all lands, and the true and only source of title, and from him all lands were held. The theory of jura regalia 80 was therefore nothing more than a natural fruit of conquest. The Philippines having passed to Spain by virtue of discovery and conquest, earlier Spanish decrees 82 declared that "all lands were held from the Crown." The Regalian doctrine extends not only to land but also to "all natural wealth that may be found in the 83 bowels of the earth." Spain, in particular, recognized the unique value of natural resources, viewing them, especially minerals, as an abundant source of revenue to finance its wars against other 84 85 nations. Mining laws during the Spanish regime reflected this perspective. THE AMERICAN OCCUPATION AND THE CONCESSION REGIME By the Treaty of Paris of December 10, 1898, Spain ceded "the archipelago known as the Philippine Islands" to the United States. The Philippines was hence governed by means of organic acts that were in 86 the nature of charters serving as a Constitution of the occupied territory from 1900 to 1935. Among the principal organic acts of the Philippines was the Act of Congress of July 1, 1902, more commonly known as the Philippine Bill of 1902, through which the United States Congress assumed the administration of the 87 Philippine Islands. Section 20 of said Bill reserved the disposition of mineral lands of the public domain from sale. Section 21 thereof allowed the free and open exploration, occupation and purchase of mineral deposits not only to citizens of the Philippine Islands but to those of the United States as well: Sec. 21. That all valuable mineral deposits in public lands in the Philippine Islands, both surveyed and unsurveyed, are hereby declared to be free and open to exploration, occupation and purchase, and the land in which they are found, to occupation and purchase, by citizens of the United States or of said Islands: Provided, That when on any lands in said Islands entered and occupied as agricultural lands under the provisions of this Act, but not patented, mineral deposits have been found, the working of such mineral deposits is forbidden until the person, association, or corporation who or which has entered and is occupying such lands shall have paid to the Government of said Islands such additional sum or sums as will make the total amount paid for the mineral claim or claims in which said deposits are located equal to the amount charged by the Government for the same as mineral claims. Unlike Spain, the United States considered natural resources as a source of wealth for its nationals and saw fit to allow both Filipino and American citizens to explore and exploit minerals in public lands, and to 88 grant patents to private mineral lands. A person who acquired ownership over a parcel of private mineral land pursuant to the laws then prevailing could exclude other persons, even the State, from 89 90 exploiting minerals within his property. Thus, earlier jurisprudence held that: A valid and subsisting location of mineral land, made and kept up in accordance with the provisions of the statutes of the United States, has the effect of a grant by the United States of the present and exclusive possession of the lands located, and this exclusive right of possession and enjoyment continues during the entire life of the location. x x x. x x x.
81

L A W O N N A T U R A L R E S O U R C E S a n d E N V I R O N M E N T A L L A W C a s e s a n d S p e c i a l L a w s | 42
The discovery of minerals in the ground by one who has a valid mineral location perfects his claim and his location not only against third persons, but also against the Government. x x x. [Italics in the original.] The Regalian doctrine and the American system, therefore, differ in one essential respect. Under the Regalian theory, mineral rights are not included in a grant of land by the state; under the American 91 doctrine, mineral rights are included in a grant of land by the government. Section 21 also made possible the concession (frequently styled "permit", license" or 92 93 "lease") system. This was the traditional regime imposed by the colonial administrators for the 94 exploitation of natural resources in the extractive sector (petroleum, hard minerals, timber, etc.). Under the concession system, the concessionaire makes a direct equity investment for the purpose of 95 exploiting a particular natural resource within a given area. Thus, the concession amounts to complete control by the concessionaire over the country's natural resource, for it is given exclusive and plenary 96 rights to exploit a particular resource at the point of extraction. In consideration for the right to exploit a natural resource, the concessionaire either pays rent or royalty, which is a fixed percentage of the gross 97 proceeds. Later statutory enactments by the legislative bodies set up in the Philippines adopted the contractual 98 99 framework of the concession. For instance, Act No. 2932, approved on August 31, 1920, which provided for the exploration, location, and lease of lands containing petroleum and other mineral oils and 100 gas in the Philippines, and Act No. 2719, approved on May 14, 1917, which provided for the leasing and 101 development of coal lands in the Philippines, both utilized the concession system. THE 1935 CONSTITUTION AND THE NATIONALIZATION OF NATURAL RESOURCES By the Act of United States Congress of March 24, 1934, popularly known as the Tydings-McDuffie Law, 102 the People of the Philippine Islands were authorized to adopt a constitution. On July 30, 1934, the Constitutional Convention met for the purpose of drafting a constitution, and the Constitution 103 subsequently drafted was approved by the Convention on February 8, 1935. The Constitution was 104 submitted to the President of the United States on March 18, 1935. On March 23, 1935, the President of the United States certified that the Constitution conformed substantially with the provisions of the Act 105 of Congress approved on March 24, 1934. On May 14, 1935, the Constitution was ratified by the Filipino 106 people. The 1935 Constitution adopted the Regalian doctrine, declaring all natural resources of the Philippines, 107 including mineral lands and minerals, to be property belonging to the State. As adopted in a republican system, the medieval concept of jura regalia is stripped of royal overtones and ownership of the land is 108 vested in the State. Section 1, Article XIII, on Conservation and Utilization of Natural Resources, of the 1935 Constitution provided: SECTION 1. All agricultural, timber, and mineral lands of the public domain, waters, minerals, coal, petroleum, and other mineral oils, all forces of potential energy, and other natural resources of the Philippines belong to the State, and their disposition, exploitation, development, or utilization shall be limited to citizens of the Philippines, or to corporations or associations at least sixty per centum of the capital of which is owned by such citizens, subject to any existing right, grant, lease, or concession at the time of the inauguration of the Government established under this Constitution. Natural resources, with the exception of public agricultural land, shall not be alienated, and no license, concession, or lease for the exploitation, development, or utilization of any of the natural resources shall be granted for a period exceeding twenty-five years, except as to water rights for irrigation, water supply, fisheries, or industrial uses other than the development of water power, in which cases beneficial use may be the measure and the limit of the grant. The nationalization and conservation of the natural resources of the country was one of the fixed and 109 dominating objectives of the 1935 Constitutional Convention. One delegate relates: There was an overwhelming sentiment in the Convention in favor of the principle of state ownership of natural resources and the adoption of the Regalian doctrine. State ownership of natural resources was seen as a necessary starting point to secure recognition of the state's power to control their disposition, exploitation, development, or utilization. The delegates of the Constitutional Convention very well knew that the concept of State ownership of land and natural resources was introduced by the Spaniards, however, they were not certain whether it was continued and applied by the Americans. To remove all doubts, the Convention approved the provision in the Constitution affirming the Regalian doctrine. The adoption of the principle of state ownership of the natural resources and of the Regalian doctrine was considered to be a necessary starting point for the plan of nationalizing and conserving the natural resources of the country. For with the establishment of the principle of state ownership of the natural resources, it would not be hard to secure the recognition of the power of the State to control their 110 disposition, exploitation, development or utilization. The nationalization of the natural resources was intended (1) to insure their conservation for Filipino posterity; (2) to serve as an instrument of national defense, helping prevent the extension to the country of foreign control through peaceful economic penetration; and (3) to avoid making the Philippines a 111 source of international conflicts with the consequent danger to its internal security and independence. The same Section 1, Article XIII also adopted the concession system, expressly permitting the State to grant licenses, concessions, or leases for the exploitation, development, or utilization of any of the natural

L A W O N N A T U R A L R E S O U R C E S a n d E N V I R O N M E N T A L L A W C a s e s a n d S p e c i a l L a w s | 43
resources. Grants, however, were limited to Filipinos or entities at least 60% of the capital of which is owned by Filipinos.lawph!l.ne+ The swell of nationalism that suffused the 1935 Constitution was radically diluted when on November 1946, the Parity Amendment, which came in the form of an "Ordinance Appended to the Constitution," 112 was ratified in a plebiscite. The Amendment extended, from July 4, 1946 to July 3, 1974, the right to utilize and exploit our natural resources to citizens of the United States and business enterprises owned 113 or controlled, directly or indirectly, by citizens of the United States: Notwithstanding the provision of section one, Article Thirteen, and section eight, Article Fourteen, of the foregoing Constitution, during the effectivity of the Executive Agreement entered into by the President of the Philippines with the President of the United States on the fourth of July, nineteen hundred and fortysix, pursuant to the provisions of Commonwealth Act Numbered Seven hundred and thirty-three, but in no case to extend beyond the third of July, nineteen hundred and seventy-four, the disposition, exploitation, development, and utilization of all agricultural, timber, and mineral lands of the public domain, waters, minerals, coals, petroleum, and other mineral oils, all forces and sources of potential energy, and other natural resources of the Philippines, and the operation of public utilities, shall, if open to any person, be open to citizens of the United States and to all forms of business enterprise owned or controlled, directly or indirectly, by citizens of the United States in the same manner as to, and under the same conditions imposed upon, citizens of the Philippines or corporations or associations owned or controlled by citizens of the Philippines. The Parity Amendment was subsequently modified by the 1954 Revised Trade Agreement, also known as 114 the Laurel-Langley Agreement, embodied in Republic Act No. 1355. THE PETROLEUM ACT OF 1949 AND THE CONCESSION SYSTEM In the meantime, Republic Act No. 387, June 18, 1949.
115

upon the concessionaire ownership over the petroleum lands and petroleum deposits. However, they did grant concessionaires the right to explore, develop, exploit, and utilize them for the period and under 123 the conditions determined by the law. Concessions were granted at the complete risk of the concessionaire; the Government did not guarantee 124 the existence of petroleum or undertake, in any case, title warranty. Concessionaires were required to submit information as maybe required by the Secretary of Agriculture and Natural Resources, including reports of geological and geophysical examinations, as well as 125 126 127 production reports. Exploration and exploitation concessionaires were also required to submit work programs.lavvphi1.net Exploitation concessionaires, in particular, were obliged to pay an annual exploitation tax, the object of which is to induce the concessionaire to actually produce petroleum, and not simply to sit on the 129 concession without developing or exploiting it. These concessionaires were also bound to pay the Government royalty, which was not less than 12% of the petroleum produced and saved, less that 130 consumed in the operations of the concessionaire. Under Article 66, R.A. No. 387, the exploitation tax may be credited against the royalties so that if the concessionaire shall be actually producing enough oil, 131 it would not actually be paying the exploitation tax. Failure to pay the annual exploitation tax for two consecutive years, or the royalty due to the 133 Government within one year from the date it becomes due, constituted grounds for the cancellation of the concession. In case of delay in the payment of the taxes or royalty imposed by the law or by the 134 concession, a surcharge of 1% per month is exacted until the same are paid. As a rule, title rights to all equipment and structures that the concessionaire placed on the land belong to 135 the exploration or exploitation concessionaire. Upon termination of such concession, the 136 concessionaire had a right to remove the same. The Secretary of Agriculture and Natural Resources was tasked with carrying out the provisions of the law, 137 through the Director of Mines, who acted under the Secretary's immediate supervision and control. The Act granted the Secretary the authority to inspect any operation of the concessionaire and to examine all the books and accounts pertaining to operations or conditions related to payment of taxes and 138 royalties. The same law authorized the Secretary to create an Administration Unit and a Technical Board. The 140 Administration Unit was charged, inter alia, with the enforcement of the provisions of the law. The Technical Board had, among other functions, the duty to check on the performance of concessionaires and to determine whether the obligations imposed by the Act and its implementing regulations were 141 being complied with.
139 132 128

122

also known as the Petroleum Act of 1949, was approved on

The Petroleum Act of 1949 employed the concession system for the exploitation of the nation's petroleum resources. Among the kinds of concessions it sanctioned were exploration and exploitation 116 concessions, which respectively granted to the concessionaire the exclusive right to explore for or 117 develop petroleum within specified areas. Concessions may be granted only to duly qualified persons who have sufficient finances, organization, 119 resources, technical competence, and skills necessary to conduct the operations to be undertaken. Nevertheless, the Government reserved the right to undertake such work itself. This proceeded from the theory that all natural deposits or occurrences of petroleum or natural gas in public and/or private 121 lands in the Philippines belong to the State. Exploration and exploitation concessions did not confer
120 118

L A W O N N A T U R A L R E S O U R C E S a n d E N V I R O N M E N T A L L A W C a s e s a n d S p e c i a l L a w s | 44
Victorio Mario A. Dimagiba, Chief Legal Officer of the Bureau of Energy Development, analyzed the benefits and drawbacks of the concession system insofar as it applied to the petroleum industry: Advantages of Concession. Whether it emphasizes income tax or royalty, the most positive aspect of the concession system is that the State's financial involvement is virtually risk free and administration is simple and comparatively low in cost. Furthermore, if there is a competitive allocation of the resource leading to substantial bonuses and/or greater royalty coupled with a relatively high level of taxation, revenue accruing to the State under the concession system may compare favorably with other financial arrangements. Disadvantages of Concession. There are, however, major negative aspects to this system. Because the Government's role in the traditional concession is passive, it is at a distinct disadvantage in managing and developing policy for the nation's petroleum resource. This is true for several reasons. First, even though most concession agreements contain covenants requiring diligence in operations and production, this establishes only an indirect and passive control of the host country in resource development. Second, and more importantly, the fact that the host country does not directly participate in resource management decisions inhibits its ability to train and employ its nationals in petroleum development. This factor could delay or prevent the country from effectively engaging in the development of its resources. Lastly, a direct role in management is usually necessary in order to obtain a knowledge of the international petroleum industry which is important to an appreciation of the host country's resources in relation to those of other 142 countries. Other liabilities of the system have also been noted: x x x there are functional implications which give the concessionaire great economic power arising from its exclusive equity holding. This includes, first, appropriation of the returns of the undertaking, subject to a modest royalty; second, exclusive management of the project; third, control of production of the natural resource, such as volume of production, expansion, research and development; and fourth, exclusive responsibility for downstream operations, like processing, marketing, and distribution. In short, even if nominally, the state is the sovereign and owner of the natural resource being exploited, it has been shorn of all elements of control over such natural resource because of the exclusive nature of the contractual regime of the concession. The concession system, investing as it does ownership of natural resources, constitutes a consistent inconsistency with the principle embodied in our Constitution that natural resources belong to the state and shall not be alienated, not to mention the fact that the concession was 143 the bedrock of the colonial system in the exploitation of natural resources. Eventually, the concession system failed for reasons explained by Dimagiba: Notwithstanding the good intentions of the Petroleum Act of 1949, the concession system could not have properly spurred sustained oil exploration activities in the country, since it assumed that such a capitalintensive, high risk venture could be successfully undertaken by a single individual or a small company. In effect, concessionaires' funds were easily exhausted. Moreover, since the concession system practically closed its doors to interested foreign investors, local capital was stretched to the limits. The old system also failed to consider the highly sophisticated technology and expertise required, which would be 144 available only to multinational companies. A shift to a new regime for the development of natural resources thus seemed imminent. PRESIDENTIAL DECREE NO. 87, THE 1973 CONSTITUTION AND THE SERVICE CONTRACT SYSTEM The promulgation on December 31, 1972 of Presidential Decree No. 87, otherwise known as The Oil Exploration and Development Act of 1972 signaled such a transformation. P.D. No. 87 permitted the 146 government to explore for and produce indigenous petroleum through "service contracts." "Service contracts" is a term that assumes varying meanings to different people, and it has carried many names in different countries, like "work contracts" in Indonesia, "concession agreements" in Africa, "production-sharing agreements" in the Middle East, and "participation agreements" in Latin 147 America. A functional definition of "service contracts" in the Philippines is provided as follows: A service contract is a contractual arrangement for engaging in the exploitation and development of petroleum, mineral, energy, land and other natural resources by which a government or its agency, or a private person granted a right or privilege by the government authorizes the other party (service contractor) to engage or participate in the exercise of such right or the enjoyment of the privilege, in that the latter provides financial or technical resources, undertakes the exploitation or production of a given resource, or directly manages the productive enterprise, operations of the exploration and exploitation of 148 the resources or the disposition of marketing or resources. In a service contract under P.D. No. 87, service and technology are furnished by the service contractor for 149 which it shall be entitled to the stipulated service fee. The contractor must be technically competent 150 and financially capable to undertake the operations required in the contract. Financing is supposed to be provided by the Government to which all petroleum produced belongs. In case the Government is unable to finance petroleum exploration operations, the contractor may furnish services, technology and financing, and the proceeds of sale of the petroleum produced under the contract shall be the source of funds for payment of the service fee and the operating expenses due the 152 contractor. The contractor shall undertake, manage and execute petroleum operations, subject to the 153 government overseeing the management of the operations. The contractor provides all necessary services and technology and the requisite financing, performs the exploration work obligations, and assumes all exploration risks such that if no petroleum is produced, it will not be entitled to 154 reimbursement. Once petroleum in commercial quantity is discovered, the contractor shall operate the 155 field on behalf of the government.
151 145

L A W O N N A T U R A L R E S O U R C E S a n d E N V I R O N M E N T A L L A W C a s e s a n d S p e c i a l L a w s | 45
P.D. No. 87 prescribed minimum terms and conditions for every service contract. It also granted the 157 contractor certain privileges, including exemption from taxes and payment of tariff duties, and 158 permitted the repatriation of capital and retention of profits abroad. Ostensibly, the service contract system had certain advantages over the concession regime. It has been opined, though, that, in the Philippines, our concept of a service contract, at least in the petroleum 160 industry, was basically a concession regime with a production-sharing element. On January 17, 1973, then President Ferdinand E. Marcos proclaimed the ratification of a new 161 Constitution. Article XIV on the National Economy and Patrimony contained provisions similar to the 1935 Constitution with regard to Filipino participation in the nation's natural resources. Section 8, Article XIV thereof provides: Sec. 8. All lands of the public domain, waters, minerals, coal, petroleum and other mineral oils, all forces of potential energy, fisheries, wildlife, and other natural resources of the Philippines belong to the State. With the exception of agricultural, industrial or commercial, residential and resettlement lands of the public domain, natural resources shall not be alienated, and no license, concession, or lease for the exploration, development, exploitation, or utilization of any of the natural resources shall be granted for a period exceeding twenty-five years, renewable for not more than twenty-five years, except as to water rights for irrigation, water supply, fisheries, or industrial uses other than the development of water power, in which cases beneficial use may be the measure and the limit of the grant. While Section 9 of the same Article maintained the Filipino-only policy in the enjoyment of natural resources, it also allowed Filipinos, upon authority of the Batasang Pambansa, to enter into service contracts with any person or entity for the exploration or utilization of natural resources. Sec. 9. The disposition, exploration, development, exploitation, or utilization of any of the natural resources of the Philippines shall be limited to citizens, or to corporations or associations at least sixty per centum of which is owned by such citizens. The Batasang Pambansa, in the national interest, may allow such citizens, corporations or associations to enter into service contracts for financial, technical, management, or other forms of assistance with any person or entity for the exploration, or utilization of any of the natural resources. Existing valid and binding service contracts for financial, technical, management, or other forms of assistance are hereby recognized as such. [Emphasis supplied.] The concept of service contracts, according to one delegate, was borrowed from the methods followed by 162 India, Pakistan and especially Indonesia in the exploration of petroleum and mineral oils. The provision allowing such contracts, according to another, was intended to "enhance the proper development of our natural resources since Filipino citizens lack the needed capital and technical know-how which are essential in the proper exploration, development and exploitation of the natural resources of the 163 country."
159 156

The original idea was to authorize the government, not private entities, to enter into service contracts 164 with foreign entities. As finally approved, however, a citizen or private entity could be allowed by the 165 National Assembly to enter into such service contract. The prior approval of the National Assembly was 166 deemed sufficient to protect the national interest. Notably, none of the laws allowing service contracts were passed by the Batasang Pambansa. Indeed, all of them were enacted by presidential decree. On March 13, 1973, shortly after the ratification of the new Constitution, the President promulgated 167 Presidential Decree No. 151. The law allowed Filipino citizens or entities which have acquired lands of the public domain or which own, hold or control such lands to enter into service contracts for financial, technical, management or other forms of assistance with any foreign persons or entity for the 168 exploration, development, exploitation or utilization of said lands. Presidential Decree No. 463, also known as The Mineral Resources Development Decree of 1974, was enacted on May 17, 1974. Section 44 of the decree, as amended, provided that a lessee of a mining claim may enter into a service contract with a qualified domestic or foreign contractor for the exploration, development and exploitation of his claims and the processing and marketing of the product thereof. Presidential Decree No. 704 (The Fisheries Decree of 1975), approved on May 16, 1975, allowed Filipinos engaged in commercial fishing to enter into contracts for financial, technical or other forms of assistance with any foreign person, corporation or entity for the production, storage, marketing and 171 processing of fish and fishery/aquatic products. Presidential Decree No. 705 (The Revised Forestry Code of the Philippines), approved on May 19, 1975, allowed "forest products licensees, lessees, or permitees to enter into service contracts for financial, technical, management, or other forms of assistance . . . with any foreign person or entity for the 173 exploration, development, exploitation or utilization of the forest resources." Yet another law allowing service contracts, this time for geothermal resources, was Presidential Decree 174 No. 1442, which was signed into law on June 11, 1978. Section 1 thereof authorized the Government to enter into service contracts for the exploration, exploitation and development of geothermal resources with a foreign contractor who must be technically and financially capable of undertaking the operations required in the service contract. Thus, virtually the entire range of the country's natural resources from petroleum and minerals to geothermal energy, from public lands and forest resources to fishery products was well covered by apparent legal authority to engage in the direct participation or involvement of foreign persons or corporations (otherwise disqualified) in the exploration and utilization of natural resources through 175 service contracts. THE 1987 CONSTITUTION AND TECHNICAL OR FINANCIAL ASSISTANCE AGREEMENTS
172 170 169

L A W O N N A T U R A L R E S O U R C E S a n d E N V I R O N M E N T A L L A W C a s e s a n d S p e c i a l L a w s | 46
After the February 1986 Edsa Revolution, Corazon C. Aquino took the reins of power under a revolutionary government. On March 25, 1986, President Aquino issued Proclamation No. 176 3, promulgating the Provisional Constitution, more popularly referred to as the Freedom Constitution. By authority of the same Proclamation, the President created a Constitutional Commission (CONCOM) to 177 draft a new constitution, which took effect on the date of its ratification on February 2, 1987. The 1987 Constitution retained the Regalian doctrine. The first sentence of Section 2, Article XII states: "All lands of the public domain, waters, minerals, coal, petroleum, and other mineral oils, all forces of potential energy, fisheries, forests or timber, wildlife, flora and fauna, and other natural resources are owned by the State." Like the 1935 and 1973 Constitutions before it, the 1987 Constitution, in the second sentence of the same provision, prohibits the alienation of natural resources, except agricultural lands. The third sentence of the same paragraph is new: "The exploration, development and utilization of natural resources shall be under the full control and supervision of the State." The constitutional policy of the State's "full control and supervision" over natural resources proceeds from the concept of jura regalia, as well as the recognition of the importance of the country's natural resources, not only for national 178 economic development, but also for its security and national defense. Under this provision, the State 179 assumes "a more dynamic role" in the exploration, development and utilization of natural resources. Conspicuously absent in Section 2 is the provision in the 1935 and 1973 Constitutions authorizing the State to grant licenses, concessions, or leases for the exploration, exploitation, development, or utilization of natural resources. By such omission, the utilization of inalienable lands of public domain through 180 "license, concession or lease" is no longer allowed under the 1987 Constitution. Having omitted the provision on the concession system, Section 2 proceeded to introduce "unfamiliar 181 language": The State may directly undertake such activities or it may enter into co-production, joint venture, or production-sharing agreements with Filipino citizens, or corporations or associations at least sixty per centum of whose capital is owned by such citizens. Consonant with the State's "full supervision and control" over natural resources, Section 2 offers the State 182 two "options." One, the State may directly undertake these activities itself; or two, it may enter into coproduction, joint venture, or production-sharing agreements with Filipino citizens, or entities at least 60% of whose capital is owned by such citizens. A third option is found in the third paragraph of the same section: Sixth, the agreements must contain rudimentary stipulations for the promotion of the development and use of local scientific and technical resources. The Congress may, by law, allow small-scale utilization of natural resources by Filipino citizens, as well as cooperative fish farming, with priority to subsistence fishermen and fish-workers in rivers, lakes, bays, and lagoons. While the second and third options are limited only to Filipino citizens or, in the case of the former, to corporations or associations at least 60% of the capital of which is owned by Filipinos, a fourth allows the participation of foreign-owned corporations. The fourth and fifth paragraphs of Section 2 provide: The President may enter into agreements with foreign-owned corporations involving either technical or financial assistance for large-scale exploration, development, and utilization of minerals, petroleum, and other mineral oils according to the general terms and conditions provided by law, based on real contributions to the economic growth and general welfare of the country. In such agreements, the State shall promote the development and use of local scientific and technical resources. The President shall notify the Congress of every contract entered into in accordance with this provision, within thirty days from its execution. Although Section 2 sanctions the participation of foreign-owned corporations in the exploration, development, and utilization of natural resources, it imposes certain limitations or conditions to agreements with such corporations. First, the parties to FTAAs. Only the President, in behalf of the State, may enter into these agreements, and only with corporations. By contrast, under the 1973 Constitution, a Filipino citizen, corporation or association may enter into a service contract with a "foreign person or entity." Second, the size of the activities: only large-scale exploration, development, and utilization is 183 allowed. The term "large-scale usually refers to very capital-intensive activities." Third, the natural resources subject of the activities is restricted to minerals, petroleum and other mineral oils, the intent being to limit service contracts to those areas where Filipino capital 184 may not be sufficient. Fourth, consistency with the provisions of statute. The agreements must be in accordance with the terms and conditions provided by law. Fifth, Section 2 prescribes certain standards for entering into such agreements. The agreements must be based on real contributions to economic growth and general welfare of the country.

L A W O N N A T U R A L R E S O U R C E S a n d E N V I R O N M E N T A L L A W C a s e s a n d S p e c i a l L a w s | 47
Seventh, the notification requirement. The President shall notify Congress of every financial or technical assistance agreement entered into within thirty days from its execution. Finally, the scope of the agreements. While the 1973 Constitution referred to "service contracts for financial, technical, management, or other forms of assistance" the 1987 Constitution provides for "agreements. . . involving either financial or technical assistance." It bears noting that the phrases "service contracts" and "management or other forms of assistance" in the earlier constitution have been omitted. By virtue of her legislative powers under the Provisional Constitution, President Aquino, on July 10, 1987, signed into law E.O. No. 211 prescribing the interim procedures in the processing and approval of applications for the exploration, development and utilization of minerals. The omission in the 1987 Constitution of the term "service contracts" notwithstanding, the said E.O. still referred to them in Section 2 thereof: Sec. 2. Applications for the exploration, development and utilization of mineral resources, including renewal applications and applications for approval of operating agreements and mining service contracts, shall be accepted and processed and may be approved x x x. [Emphasis supplied.] The same law provided in its Section 3 that the "processing, evaluation and approval of all mining applications . . . operating agreements and service contracts . . . shall be governed by Presidential Decree No. 463, as amended, other existing mining laws, and their implementing rules and regulations. . . ." As earlier stated, on the 25th also of July 1987, the President issued E.O. No. 279 by authority of which the subject WMCP FTAA was executed on March 30, 1995. On March 3, 1995, President Ramos signed into law R.A. No. 7942. Section 15 thereof declares that the Act "shall govern the exploration, development, utilization, and processing of all mineral resources." Such declaration notwithstanding, R.A. No. 7942 does not actually cover all the modes through which the State may undertake the exploration, development, and utilization of natural resources. The State, being the owner of the natural resources, is accorded the primary power and responsibility in the exploration, development and utilization thereof. As such, it may undertake these activities through four modes: The State may directly undertake such activities. (2) The State may enter into co-production, joint venture or production-sharing agreements with Filipino citizens or qualified corporations. (3) Congress may, by law, allow small-scale utilization of natural resources by Filipino citizens.
185

(4) For the large-scale exploration, development and utilization of minerals, petroleum and other mineral oils, the President may enter into agreements with foreign-owned corporations involving 186 technical or financial assistance. Except to charge the Mines and Geosciences Bureau of the DENR with performing researches and 187 188 surveys, and a passing mention of government-owned or controlled corporations, R.A. No. 7942 does not specify how the State should go about the first mode. The third mode, on the other hand, is governed 189 190 by Republic Act No. 7076 (the People's Small-Scale Mining Act of 1991) and other pertinent laws. R.A. No. 7942 primarily concerns itself with the second and fourth modes. Mineral production sharing, co-production and joint venture agreements are collectively classified by R.A. 191 No. 7942 as "mineral agreements." The Government participates the least in a mineral production 192 sharing agreement (MPSA). In an MPSA, the Government grants the contractor the exclusive right to 193 194 conduct mining operations within a contract area and shares in the gross output. The MPSA contractor provides the financing, technology, management and personnel necessary for the agreement's 195 implementation. The total government share in an MPSA is the excise tax on mineral products under 196 197 Republic Act No. 7729, amending Section 151(a) of the National Internal Revenue Code, as amended. In a co-production agreement (CA), the Government provides inputs to the mining operations other 199 than the mineral resource, while in a joint venture agreement (JVA), where the Government enjoys the greatest participation, the Government and the JVA contractor organize a company with both parties 200 having equity shares. Aside from earnings in equity, the Government in a JVA is also entitled to a share 201 202 203 in the gross output. The Government may enter into a CA or JVA with one or more contractors. The Government's share in a CA or JVA is set out in Section 81 of the law: The share of the Government in co-production and joint venture agreements shall be negotiated by the Government and the contractor taking into consideration the: (a) capital investment of the project, (b) the risks involved, (c) contribution of the project to the economy, and (d) other factors that will provide for a fair and equitable sharing between the Government and the contractor. The Government shall also be entitled to compensations for its other contributions which shall be agreed upon by the parties, and shall consist, among other things, the contractor's income tax, excise tax, special allowance, withholding tax due from the contractor's foreign stockholders arising from dividend or interest payments to the said foreign stockholders, in case of a foreign national and all such other taxes, duties and fees as provided for under existing laws. All mineral agreements grant the respective contractors the exclusive right to conduct mining operations 204 and to extract all mineral resources found in the contract area. A "qualified person" may enter into any 205 of the mineral agreements with the Government. A "qualified person" is any citizen of the Philippines with capacity to contract, or a corporation, partnership, association, or cooperative organized or authorized for the purpose of engaging in mining, with technical and financial
198

L A W O N N A T U R A L R E S O U R C E S a n d E N V I R O N M E N T A L L A W C a s e s a n d S p e c i a l L a w s | 48
capability to undertake mineral resources development and duly registered in accordance with law at 206 least sixty per centum (60%) of the capital of which is owned by citizens of the Philippines x x x. The fourth mode involves "financial or technical assistance agreements." An FTAA is defined as "a contract involving financial or technical assistance for large-scale exploration, development, and 207 utilization of natural resources." Any qualified person with technical and financial capability to undertake large-scale exploration, development, and utilization of natural resources in the Philippines 208 may enter into such agreement directly with the Government through the DENR. For the purpose of granting an FTAA, a legally organized foreign-owned corporation (any corporation, partnership, association, or cooperative duly registered in accordance with law in which less than 50% of the capital is 209 210 owned by Filipino citizens) is deemed a "qualified person." Other than the difference in contractors' qualifications, the principal distinction between mineral agreements and FTAAs is the maximum contract area to which a qualified person may hold or be 211 granted. "Large-scale" under R.A. No. 7942 is determined by the size of the contract area, as opposed to the amount invested (US $50,000,000.00), which was the standard under E.O. 279. Like a CA or a JVA, an FTAA is subject to negotiation. The Government's contributions, in the form of taxes, in an FTAA is identical to its contributions in the two mineral agreements, save that in an FTAA: The collection of Government share in financial or technical assistance agreement shall commence after the financial or technical assistance agreement contractor has fully recovered its pre-operating expenses, 213 exploration, and development expenditures, inclusive. III Having examined the history of the constitutional provision and statutes enacted pursuant thereto, a consideration of the substantive issues presented by the petition is now in order. THE EFFECTIVITY OF EXECUTIVE ORDER NO. 279 Petitioners argue that E.O. No. 279, the law in force when the WMC FTAA was executed, did not come into effect. E.O. No. 279 was signed into law by then President Aquino on July 25, 1987, two days before the opening 214 of Congress on July 27, 1987. Section 8 of the E.O. states that the same "shall take effect immediately." 215 This provision, according to petitioners, runs counter to Section 1 of E.O. No. 200, which provides: SECTION 1. Laws shall take effect after fifteen days following the completion of their publication either in the Official Gazette or in a newspaper of general circulation in the Philippines, unless it is otherwise 216 provided. [Emphasis supplied.]
212

On that premise, petitioners contend that E.O. No. 279 could have only taken effect fifteen days after its publication at which time Congress had already convened and the President's power to legislate had ceased. Respondents, on the other hand, counter that the validity of E.O. No. 279 was settled in Miners Association of the Philippines v. Factoran, supra. This is of course incorrect for the issue in Miners Association was not the validity of E.O. No. 279 but that of DAO Nos. 57 and 82 which were issued pursuant thereto. Nevertheless, petitioners' contentions have no merit. It bears noting that there is nothing in E.O. No. 200 that prevents a law from taking effect on a date other than even before the 15-day period after its publication. Where a law provides for its own date of effectivity, such date prevails over that prescribed by E.O. No. 200. Indeed, this is the very essence of the phrase "unless it is otherwise provided" in Section 1 thereof. Section 1, E.O. No. 200, therefore, applies only when a statute does not provide for its own date of effectivity. What is mandatory under E.O. No. 200, and what due process requires, as this Court held in Taada v. 217 Tuvera, is the publication of the law for without such notice and publication, there would be no basis for the application of the maxim "ignorantia legis n[eminem] excusat." It would be the height of injustice to punish or otherwise burden a citizen for the transgression of a law of which he had no notice whatsoever, not even a constructive one. While the effectivity clause of E.O. No. 279 does not require its publication, it is not a ground for its invalidation since the Constitution, being "the fundamental, paramount and supreme law of the nation," 218 219 is deemed written in the law. Hence, the due process clause, which, so Taada held, mandates the publication of statutes, is read into Section 8 of E.O. No. 279. Additionally, Section 1 of E.O. No. 200 which provides for publication "either in the Official Gazette or in a newspaper of general circulation in the Philippines," finds suppletory application. It is significant to note that E.O. No. 279 was actually published 220 in the Official Gazette on August 3, 1987. From a reading then of Section 8 of E.O. No. 279, Section 1 of E.O. No. 200, and Taada v. Tuvera, this Court holds that E.O. No. 279 became effective immediately upon its publication in the Official Gazette on August 3, 1987. That such effectivity took place after the convening of the first Congress is irrelevant. At the time President Aquino issued E.O. No. 279 on July 25, 1987, she was still validly exercising legislative powers 221 under the Provisional Constitution. Article XVIII (Transitory Provisions) of the 1987 Constitution explicitly states:

L A W O N N A T U R A L R E S O U R C E S a n d E N V I R O N M E N T A L L A W C a s e s a n d S p e c i a l L a w s | 49
Sec. 6. The incumbent President shall continue to exercise legislative powers until the first Congress is convened. The convening of the first Congress merely precluded the exercise of legislative powers by President Aquino; it did not prevent the effectivity of laws she had previously enacted. There can be no question, therefore, that E.O. No. 279 is an effective, and a validly enacted, statute. THE CONSTITUTIONALITY OF THE WMCP FTAA MR. VILLEGAS. That is right. Petitioners submit that, in accordance with the text of Section 2, Article XII of the Constitution, FTAAs should be limited to "technical or financial assistance" only. They observe, however, that, contrary to the language of the Constitution, the WMCP FTAA allows WMCP, a fully foreign-owned mining corporation, to extend more than mere financial or technical assistance to the State, for it permits WMCP to manage and 222 operate every aspect of the mining activity. Petitioners' submission is well-taken. It is a cardinal rule in the interpretation of constitutions that the 223 instrument must be so construed as to give effect to the intention of the people who adopted it. This intention is to be sought in the constitution itself, and the apparent meaning of the words is to be taken as expressing it, except in cases where that assumption would lead to absurdity, ambiguity, or 224 contradiction. What the Constitution says according to the text of the provision, therefore, compels acceptance and negates the power of the courts to alter it, based on the postulate that the framers and 225 the people mean what they say. Accordingly, following the literal text of the Constitution, assistance accorded by foreign-owned corporations in the large-scale exploration, development, and utilization of petroleum, minerals and mineral oils should be limited to "technical" or "financial" assistance only. WMCP nevertheless submits that the word "technical" in the fourth paragraph of Section 2 of E.O. No. 279 encompasses a "broad number of possible services," perhaps, "scientific and/or technological in 226 basis." It thus posits that it may also well include "the area of management or operations . . . so long as such assistance requires specialized knowledge or skills, and are related to the exploration, development 227 and utilization of mineral resources." This Court is not persuaded. As priorly pointed out, the phrase "management or other forms of assistance" in the 1973 Constitution was deleted in the 1987 Constitution, which allows only "technical or financial assistance." Casus omisus pro omisso habendus est. A person, object or thing omitted from an 228 enumeration must be held to have been omitted intentionally. As will be shown later, the management or operation of mining activities by foreign contractors, which is the primary feature of service contracts, was precisely the evil that the drafters of the 1987 Constitution sought to eradicate. Respondents insist that "agreements involving technical or financial assistance" is just another term for service contracts. They contend that the proceedings of the CONCOM indicate "that although the SR. TAN. So those are the safeguards[?] MR. VILLEGAS. Yes. There was no law at all governing service contracts before. SR. TAN. Thank you, Madam President.
230

terminology 'service contract' was avoided [by the Constitution], the concept it represented was not." They add that "[t]he concept is embodied in the phrase 'agreements involving financial or technical 229 assistance.'" And point out how members of the CONCOM referred to these agreements as "service contracts." For instance: SR. TAN. Am I correct in thinking that the only difference between these future service contracts and the past service contracts under Mr. Marcos is the general law to be enacted by the legislature and the notification of Congress by the President? That is the only difference, is it not?

[Emphasis supplied.]

WMCP also cites the following statements of Commissioners Gascon, Garcia, Nolledo and Tadeo who alluded to service contracts as they explained their respective votes in the approval of the draft Article: MR. GASCON. Mr. Presiding Officer, I vote no primarily because of two reasons: One, the provision on service contracts. I felt that if we would constitutionalize any provision on service contracts, this should always be with the concurrence of Congress and not guided only by a 231 general law to be promulgated by Congress. x x x. [Emphasis supplied.] x x x. MR. GARCIA. Thank you. I vote no. x x x. Service contracts are given constitutional legitimization in Section 3, even when they have been proven to be inimical to the interests of the nation, providing as they do the legal loophole for the exploitation of our natural resources for the benefit of foreign interests. They constitute a serious negation of Filipino control on the use and disposition of the nation's natural resources, 232 especially with regard to those which are nonrenewable. [Emphasis supplied.] xxx

L A W O N N A T U R A L R E S O U R C E S a n d E N V I R O N M E N T A L L A W C a s e s a n d S p e c i a l L a w s | 50
MR. NOLLEDO. While there are objectionable provisions in the Article on National Economy and Patrimony, going over said provisions meticulously, setting aside prejudice and personalities will reveal that the article contains a balanced set of provisions. I hope the forthcoming Congress will implement such provisions taking into account that Filipinos should have real control over our economy and patrimony, and if foreign equity is permitted, the same must be subordinated to the imperative demands of the national interest. x x x. It is also my understanding that service contracts involving foreign corporations or entities are resorted to only when no Filipino enterprise or Filipino-controlled enterprise could possibly undertake the exploration or exploitation of our natural resources and that compensation under such contracts cannot and should not equal what should pertain to ownership of capital. In other words, the service contract should not be an instrument to circumvent the basic provision, that the exploration and exploitation of natural resources should be truly for the benefit of Filipinos. Thank you, and I vote yes. x x x. MR. TADEO. Nais ko lamang ipaliwanag ang aking boto. Matapos suriin ang kalagayan ng Pilipinas, ang saligang suliranin, pangunahin ang salitang "imperyalismo." Ang ibig sabihin nito ay ang sistema ng lipunang pinaghaharian ng iilang monopolyong kapitalista at ang salitang "imperyalismo" ay buhay na buhay sa National Economy and Patrimony na nating ginawa. Sa pamamagitan ng salitang "based on," naroroon na ang free trade sapagkat tayo ay mananatiling tagapagluwas ng hilaw na sangkap at tagaangkat ng yaring produkto. Pangalawa, naroroon pa rin ang parity rights, ang service contract, ang 60-40 equity sa natural resources. Habang naghihirap ang sambayanang Pilipino, ginagalugad naman ng mga dayuhan ang ating likas na yaman. Kailan man ang Article on National Economy and Patrimony ay hindi nagpaalis sa pagkaalipin ng ating ekonomiya sa kamay ng mga dayuhan. Ang solusyon sa suliranin ng bansa ay dalawa lamang: ang pagpapatupad ng tunay na reporma sa lupa at ang national industrialization. Ito ang tinatawag naming pagsikat ng araw sa Silangan. Ngunit ang mga landlords and big businessmen at ang mga komprador ay nagsasabi na ang free trade na ito, ang kahulugan para sa amin, ay ipinipilit sa ating sambayanan na ang araw ay sisikat sa Kanluran. 234 Kailan man hindi puwedeng sumikat ang araw sa Kanluran. I vote no. [Emphasis supplied.] This Court is likewise not persuaded. As earlier noted, the phrase "service contracts" has been deleted in the 1987 Constitution's Article on National Economy and Patrimony. If the CONCOM intended to retain the concept of service contracts As the following question of Commissioner Quesada and Commissioner Villegas' answer shows the drafters intended to do away with service contracts which were used to circumvent the capitalization (60%-40%) requirement: MS. QUESADA. The 1973 Constitution used the words "service contracts." In this particular Section 3, is there a safeguard against the possible control of foreign interests if the Filipinos go into coproduction with them? MR. VILLEGAS. Yes. In fact, the deletion of the phrase "service contracts" was our first attempt to avoid some of the abuses in the past regime in the use of service contracts to go around the 6040 arrangement. The safeguard that has been introduced and this, of course can be refined is found in Section 3, lines 25 to 30, where Congress will have to concur with the President on any agreement entered into between a foreign-owned corporation and the government involving technical or financial assistance for large-scale exploration, development and utilization of 237 natural resources. [Emphasis supplied.] In a subsequent discussion, Commissioner Villegas allayed the fears of Commissioner Quesada regarding the participation of foreign interests in Philippine natural resources, which was supposed to be restricted to Filipinos. MS. QUESADA. Another point of clarification is the phrase "and utilization of natural resources shall be under the full control and supervision of the State." In the 1973 Constitution, this was
233

under the 1973 Constitution, it could have simply adopted the old terminology ("service contracts") instead of employing new and unfamiliar terms ("agreements . . . involving either technical or financial assistance"). Such a difference between the language of a provision in a revised constitution and that of a 235 similar provision in the preceding constitution is viewed as indicative of a difference in purpose. If, as respondents suggest, the concept of "technical or financial assistance" agreements is identical to that of "service contracts," the CONCOM would not have bothered to fit the same dog with a new collar. To uphold respondents' theory would reduce the first to a mere euphemism for the second and render the change in phraseology meaningless. An examination of the reason behind the change confirms that technical or financial assistance agreements are not synonymous to service contracts. [T]he Court in construing a Constitution should bear in mind the object sought to be accomplished by its adoption, and the evils, if any, sought to be prevented or remedied. A doubtful provision will be examined in light of the history of the times, and the condition and circumstances under which the Constitution was framed. The object is to ascertain the reason which induced the framers of the Constitution to enact the particular provision and the purpose sought to be accomplished thereby, in order to construe the whole 236 as to make the words consonant to that reason and calculated to effect that purpose.

[Emphasis supplied.]

L A W O N N A T U R A L R E S O U R C E S a n d E N V I R O N M E N T A L L A W C a s e s a n d S p e c i a l L a w s | 51
limited to citizens of the Philippines; but it was removed and substituted by "shall be under the full control and supervision of the State." Was the concept changed so that these particular resources would be limited to citizens of the Philippines? Or would these resources only be under the full control and supervision of the State; meaning, noncitizens would have access to these natural resources? Is that the understanding? MR. VILLEGAS. No, Mr. Vice-President, if the Commissioner reads the next sentence, it states: MR. DAVIDE. It will not take three minutes. Such activities may be directly undertaken by the State, or it may enter into co-production, joint venture, production-sharing agreements with Filipino citizens. So we are still limiting it only to Filipino citizens. x x x. MS. QUESADA. Going back to Section 3, the section suggests that: The exploration, development, and utilization of natural resources may be directly undertaken by the State, or it may enter into co-production, joint venture or production-sharing agreement with . . . corporations or associations at least sixty per cent of whose voting stock or controlling interest is owned by such citizens. Lines 25 to 30, on the other hand, suggest that in the large-scale exploration, development and utilization of natural resources, the President with the concurrence of Congress may enter into agreements with foreign-owned corporations even for technical or financial assistance. I wonder if this part of Section 3 contradicts the second part. I am raising this point for fear that foreign investors will use their enormous capital resources to facilitate the actual exploitation or exploration, development and effective disposition of our natural resources to the detriment of Filipino investors. I am not saying that we should not consider borrowing money from foreign sources. What I refer to is that foreign interest should be allowed to participate only to the extent that they lend us money and give us technical assistance with the appropriate government permit. In this way, we can insure the enjoyment of our natural resources by our own people. MR. VILLEGAS. Actually, the second provision about the President does not permit foreign investors to participate. It is only technical or financial assistance they do not own anything but on conditions that have to be determined by law with the concurrence of Congress. So, it is very restrictive. If the Commissioner will remember, this removes the possibility for service contracts which we said 238 yesterday were avenues used in the previous regime to go around the 60-40 requirement. [Emphasis supplied.] The Commission had just approved the Preamble. In the Preamble we clearly stated that the Filipino people are sovereign and that one of the objectives for the creation or establishment of a government is to conserve and develop the national patrimony. The implication is that the national patrimony or our natural resources are exclusively reserved for the Filipino people. No alien must be allowed to enjoy, exploit and develop our natural resources. As a matter of fact, that principle proceeds from the fact that our natural resources are gifts from God to the Filipino people and it would be a breach of that special blessing from God if we will allow aliens to exploit our natural resources. I voted in favor of the Jamir proposal because it is not really exploitation that we granted to the alien corporations but only for them to render financial or technical assistance. It is not for them to enjoy our natural resources. Madam President, our natural resources are depleting; our population is increasing by leaps and bounds. Fifty years from now, if we will allow these aliens to exploit our natural resources, there will be no more natural resources for the next generations of Filipinos. It may last long if we will begin now. Since 1935 the aliens have been allowed to enjoy to a certain extent the exploitation of our natural resources, and we became victims of foreign dominance and control. The aliens are interested in coming to the Philippines because they would like to enjoy the bounty of nature exclusively intended for Filipinos by God. And so I appeal to all, for the sake of the future generations, that if we have to pray in the Preamble "to preserve and develop the national patrimony for the sovereign Filipino people and for the generations to come," we must at this time decide once and for all that our natural resources must be reserved only to Filipino citizens. Thank you.
239

The present Chief Justice, then a member of the CONCOM, also referred to this limitation in scope in proposing an amendment to the 60-40 requirement: MR. DAVIDE. May I be allowed to explain the proposal? MR. MAAMBONG. Subject to the three-minute rule, Madam President.

[Emphasis supplied.]
240

The opinion of another member of the CONCOM is persuasive and leaves no doubt as to the intention of the framers to eliminate service contracts altogether. He writes: Paragraph 4 of Section 2 specifies large-scale, capital-intensive, highly technological undertakings for which the President may enter into contracts with foreign-owned corporations, and enunciates strict conditions that should govern such contracts. x x x.

L A W O N N A T U R A L R E S O U R C E S a n d E N V I R O N M E N T A L L A W C a s e s a n d S p e c i a l L a w s | 52
This provision balances the need for foreign capital and technology with the need to maintain the national sovereignty. It recognizes the fact that as long as Filipinos can formulate their own terms in their own territory, there is no danger of relinquishing sovereignty to foreign interests. Are service contracts allowed under the new Constitution? No. Under the new Constitution, foreign investors (fully alien-owned) can NOT participate in Filipino enterprises except to provide: (1) Technical Assistance for highly technical enterprises; and (2) Financial Assistance for large-scale enterprises. The intent of this provision, as well as other provisions on foreign investments, is to prevent the practice (prevalent in the Marcos government) of skirting the 60/40 equation using the cover of service 241 contracts. [Emphasis supplied.] Furthermore, it appears that Proposed Resolution No. 496, which was the draft Article on National Economy and Patrimony, adopted the concept of "agreements . . . involving either technical or financial assistance" contained in the "Draft of the 1986 U.P. Law Constitution Project" (U.P. Law draft) which was 243 taken into consideration during the deliberation of the CONCOM. The former, as well as Article XII, as adopted, employed the same terminology, as the comparative table below shows:
242

production, joint venture, production sharing agreements with Filipino citizens or corporations or associations sixty per cent of whose voting stock or controlling interest is owned by such citizens for a period of not more than twenty-five years, renewable for not more than twenty-five years and under such terms and conditions as may be provided by law. In case as to water rights for irrigation, water supply, fisheries, or industrial uses other than the development of water power, beneficial use may be the measure and limit of the grant. The National Assembly may by law allow small scale utilization of natural resources by Filipino citizens. The National Assembly, may, by two-thirds vote of all its members by special law provide the terms and conditions under which a foreign-owned corporation may enter into agreements with the government involving either technical or financial assistance for large-scale exploration, development, or utilization of natural resources. [Emphasis supplied.]

production-sharing agreements with Filipino citizens or corporations or associations at least sixty per cent of whose voting stock or controlling interest is owned by such citizens. Such agreements shall be for a period of twenty-five years, renewable for not more than twenty-five years, and under such term and conditions as may be provided by law. In cases of water rights for irrigation, water supply, fisheries or industrial uses other than the development for water power, beneficial use may be the measure and limit of the grant. The Congress may by law allow small-scale utilization of natural resources by Filipino citizens, as well as cooperative fish farming in rivers, lakes, bays, and lagoons. The President with the concurrence of Congress, by special law, shall provide the terms and conditions under which a foreign-owned corporation may enter into agreements with the government involving either technical or financial assistance for largescale exploration, development, and utilization of natural resources. [Emphasis supplied.]

joint venture, or productionsharing agreements with Filipino citizens, or corporations or associations at least sixty per centum of whose capital is owned by such citizens. Such agreements may be for a period not exceeding twenty-five years, renewable for not more than twenty-five years, and under such terms and conditions as may be provided by law. In case of water rights for irrigation, water supply, fisheries, or industrial uses other than the development of water power, beneficial use may be the measure and limit of the grant. The State shall protect the nation's marine wealth in its archipelagic waters, territorial sea, and exclusive economic zone, and reserve its use and enjoyment exclusively to Filipino citizens. The Congress may, by law, allow small-scale utilization of natural resources by Filipino citizens, as well as cooperative fish farming, with priority to subsistence fishermen and fish-workers in rivers, lakes, bays, and lagoons. The President may enter into agreements with foreign-owned corporations involving either technical or financial assistance for large-scale exploration, development, and utilization of minerals, petroleum, and other mineral oils

DRAFT OF THE UP CONSTITUTION PROJECT

LAW

PROPOSED RESOLUTION NO. 496 OF THE CONSTITUTIONAL COMMISSION

ARTICLE XII OF CONSTITUTION

THE

1987

Sec. 1. All lands of the public domain, waters, minerals, coal, petroleum and other mineral oils, all forces of potential energy, fisheries, flora and fauna and other natural resources of the Philippines are owned by the State. With the exception of agricultural lands, all other natural resources shall not be alienated. The exploration, development and utilization of natural resources shall be under the full control and supervision of the State. Such activities may be directly undertaken by the state, or it may enter into co-

Sec. 3. All lands of the public domain, waters, minerals, coal, petroleum and other mineral oils, all forces of potential energy, fisheries, forests, flora and fauna, and other natural resources are owned by the State. With the exception of agricultural lands, all other natural resources shall not be alienated. The exploration, development, and utilization of natural resources shall be under the full control and supervision of the State. Such activities may be directly undertaken by the State, or it may enter into coproduction, joint venture,

Sec. 2. All lands of the public domain, waters, minerals, coal, petroleum, and other mineral oils, all forces of potential energy, fisheries, forests or timber, wildlife, flora and fauna, and other natural resources are owned by the State. With the exception of agricultural lands, all other natural resources shall not be alienated. The exploration, development, and utilization of natural resources shall be under the full control and supervision of the State. The State may directly undertake such activities or it may enter into co-production,

L A W O N N A T U R A L R E S O U R C E S a n d E N V I R O N M E N T A L L A W C a s e s a n d S p e c i a l L a w s | 53
5. Ownership of equipment, machinery, fixed assets, and other properties remain with contractor (Sec. 12, P.D. 87); 6. Repatriation of capital and retention of profits abroad guaranteed to the contractor (Sec. 13, P.D. 87); and 7. While title to the petroleum discovered may nominally be in the name of the government, the contractor has almost unfettered control over its disposition and sale, and even the domestic requirements of the country is relegated to a pro rata basis (Sec. 8). In short, our version of the service contract is just a rehash of the old concession regime x x x. Some people have pulled an old rabbit out of a magician's hat, and foisted it upon us as a new and different animal. The service contract as we know it here is antithetical to the principle of sovereignty over our natural resources restated in the same article of the [1973] Constitution containing the provision for service contracts. If the service contractor happens to be a foreign corporation, the contract would also run counter to the constitutional provision on nationalization or Filipinization, of the exploitation of our 245 natural resources. [Emphasis supplied. Underscoring in the original.] Professor Merlin M. Magallona, also a member of the working group, was harsher in his reproach of the system: x x x the nationalistic phraseology of the 1935 [Constitution] was retained by the [1973] Charter, but the essence of nationalism was reduced to hollow rhetoric. The 1973 Charter still provided that the exploitation or development of the country's natural resources be limited to Filipino citizens or corporations owned or controlled by them. However, the martial-law Constitution allowed them, once these resources are in their name, to enter into service contracts with foreign investors for financial, technical, management, or other forms of assistance. Since foreign investors have the capital resources, the actual exploitation and development, as well as the effective disposition, of the country's natural resources, would be under their direction, and control, relegating the Filipino investors to the role of second-rate partners in joint ventures. Through the instrumentality of the service contract, the 1973 Constitution had legitimized at the highest level of state policy that which was prohibited under the 1973 Constitution, namely: the exploitation of the country's natural resources by foreign nationals. The drastic impact of [this] constitutional change becomes more pronounced when it is considered that the active party to any service contract may be a corporation wholly owned by foreign interests. In such a case, the citizenship requirement is completely set aside, permitting foreign corporations to obtain actual possession, control, and [enjoyment] of the 246 country's natural resources. [Emphasis supplied.]

according to the general terms and conditions provided by law, based on real contributions to the economic growth and general welfare of the country. In such agreements, the State shall promote the development and use of local scientific and technical resources. [Emphasis supplied.] The President shall notify the Congress of every contract entered into in accordance with this provision, within thirty days from its execution.

The insights of the proponents of the U.P. Law draft are, therefore, instructive in interpreting the phrase "technical or financial assistance." In his position paper entitled Service Contracts: Old Wine in New Bottles?, Professor Pacifico A. Agabin, who was a member of the working group that prepared the U.P. Law draft, criticized service contracts for they "lodge exclusive management and control of the enterprise to the service contractor, which is reminiscent of the old concession regime. Thus, notwithstanding the provision of the Constitution that natural resources belong to the State, and that these shall not be alienated, the service contract system 244 renders nugatory the constitutional provisions cited." He elaborates: Looking at the Philippine model, we can discern the following vestiges of the concession regime, thus: 1. Bidding of a selected area, or leasing the choice of the area to the interested party and then negotiating the terms and conditions of the contract; (Sec. 5, P.D. 87) 2. Management of the enterprise vested on the contractor, including operation of the field if petroleum is discovered; (Sec. 8, P.D. 87) 3. Control of production and other matters such as expansion and development; (Sec. 8) 4. Responsibility for downstream operations marketing, distribution, and processing may be with the contractor (Sec. 8);

L A W O N N A T U R A L R E S O U R C E S a n d E N V I R O N M E N T A L L A W C a s e s a n d S p e c i a l L a w s | 54
Accordingly, Professor Agabin recommends that: Recognizing the service contract for what it is, we have to expunge it from the Constitution and reaffirm ownership over our natural resources. That is the only way we can exercise effective control over our natural resources. This should not mean complete isolation of the country's natural resources from foreign investment. Other contract forms which are less derogatory to our sovereignty and control over natural resources like technical assistance agreements, financial assistance [agreements], co-production agreements, joint ventures, production-sharing could still be utilized and adopted without violating constitutional provisions. In other words, we can adopt contract forms which recognize and assert our sovereignty and ownership over natural resources, and where the foreign entity is just a pure contractor instead of the 247 beneficial owner of our economic resources. [Emphasis supplied.] Still another member of the working group, Professor Eduardo Labitag, proposed that: 2. Service contracts as practiced under the 1973 Constitution should be discouraged, instead the government may be allowed, subject to authorization by special law passed by an extraordinary majority to enter into either technical or financial assistance. This is justified by the fact that as presently worded in the 1973 Constitution, a service contract gives full control over the contract area to the service contractor, for him to work, manage and dispose of the proceeds or production. It was a subterfuge to get around the 248 nationality requirement of the constitution. [Emphasis supplied.] In the annotations on the proposed Article on National Economy and Patrimony, the U.P. Law draft summarized the rationale therefor, thus: 5. The last paragraph is a modification of the service contract provision found in Section 9, Article XIV of the 1973 Constitution as amended. This 1973 provision shattered the framework of nationalism in our fundamental law (see Magallona, "Nationalism and its Subversion in the Constitution"). Through the service contract, the 1973 Constitution had legitimized that which was prohibited under the 1935 constitutionthe exploitation of the country's natural resources by foreign nationals. Through the service contract, acts prohibited by the Anti-Dummy Law were recognized as legitimate arrangements. Service contracts lodge exclusive management and control of the enterprise to the service contractor, not unlike the old concession regime where the concessionaire had complete control over the country's natural resources, having been given exclusive and plenary rights to exploit a particular resource and, in effect, having been assured of ownership of that resource at the point of extraction (see Agabin, "Service Contracts: Old Wine in New Bottles"). Service contracts, hence, are antithetical to the principle of sovereignty over our natural resources, as well as the constitutional provision on nationalization or Filipinization of the exploitation of our natural resources. Under the proposed provision, only technical assistance or financial assistance agreements may be entered into, and only for large-scale activities. These are contract forms which recognize and assert our sovereignty and ownership over natural resources since the foreign entity is just a pure contractor and not a beneficial owner of our economic resources. The proposal recognizes the need for capital and technology to develop our natural resources without sacrificing our sovereignty and control over such resources by the safeguard of a special law which requires two-thirds vote of all the members of the Legislature. This will ensure that such agreements will be debated upon exhaustively and thoroughly in 249 the National Assembly to avert prejudice to the nation. [Emphasis supplied.] The U.P. Law draft proponents viewed service contracts under the 1973 Constitution as grants of beneficial ownership of the country's natural resources to foreign owned corporations. While, in theory, the State owns these natural resources and Filipino citizens, their beneficiaries service contracts actually vested foreigners with the right to dispose, explore for, develop, exploit, and utilize the same. Foreigners, not Filipinos, became the beneficiaries of Philippine natural resources. This arrangement is clearly incompatible with the constitutional ideal of nationalization of natural resources, with the Regalian doctrine, and on a broader perspective, with Philippine sovereignty. The proponents nevertheless acknowledged the need for capital and technical know-how in the largescale exploitation, development and utilization of natural resources the second paragraph of the proposed draft itself being an admission of such scarcity. Hence, they recommended a compromise to reconcile the nationalistic provisions dating back to the 1935 Constitution, which reserved all natural resources exclusively to Filipinos, and the more liberal 1973 Constitution, which allowed foreigners to participate in these resources through service contracts. Such a compromise called for the adoption of a new system in the exploration, development, and utilization of natural resources in the form of technical agreements or financial agreements which, necessarily, are distinct concepts from service contracts. The replacement of "service contracts" with "agreements involving either technical or financial assistance," as well as the deletion of the phrase "management or other forms of assistance," assumes greater significance when note is taken that the U.P. Law draft proposed other equally crucial changes that were obviously heeded by the CONCOM. These include the abrogation of the concession system and the adoption of new "options" for the State in the exploration, development, and utilization of natural resources. The proponents deemed these changes to be more consistent with the State's ownership of, and its "full control and supervision" (a phrase also employed by the framers) over, such resources. The Project explained: 3. In line with the State ownership of natural resources, the State should take a more active role in the exploration, development, and utilization of natural resources, than the present practice of granting licenses, concessions, or leases hence the provision that said activities shall be under the full control and supervision of the State. There are three major schemes by which the State could undertake these activities: first, directly by itself; second, by virtue of co-production, joint venture, production sharing agreements with Filipino citizens or corporations or associations sixty per cent (60%) of the voting stock or controlling interests of which are owned by such citizens; or third, with a foreign-owned corporation, in

L A W O N N A T U R A L R E S O U R C E S a n d E N V I R O N M E N T A L L A W C a s e s a n d S p e c i a l L a w s | 55
cases of large-scale exploration, development, or utilization of natural resources through agreements involving either technical or financial assistance only. x x x. At present, under the licensing concession or lease schemes, the government benefits from such benefits only through fees, charges, ad valorem taxes and income taxes of the exploiters of our natural resources. Such benefits are very minimal compared with the enormous profits reaped by theses licensees, grantees, concessionaires. Moreover, some of them disregard the conservation of natural resources and do not protect the environment from degradation. The proposed role of the State will enable it to a greater share in the profits it can also actively husband its natural resources and engage in developmental programs that will be beneficial to them. 4. Aside from the three major schemes for the exploration, development, and utilization of our natural resources, the State may, by law, allow Filipino citizens to explore, develop, utilize natural resources in small-scale. This is in recognition of the plight of marginal fishermen, forest dwellers, gold panners, and 250 others similarly situated who exploit our natural resources for their daily sustenance and survival. Professor Agabin, in particular, after taking pains to illustrate the similarities between the two systems, concluded that the service contract regime was but a "rehash" of the concession system. "Old wine in new bottles," as he put it. The rejection of the service contract regime, therefore, is in consonance with the abolition of the concession system. In light of the deliberations of the CONCOM, the text of the Constitution, and the adoption of other proposed changes, there is no doubt that the framers considered and shared the intent of the U.P. Law proponents in employing the phrase "agreements . . . involving either technical or financial assistance." While certain commissioners may have mentioned the term "service contracts" during the CONCOM deliberations, they may not have been necessarily referring to the concept of service contracts under the 1973 Constitution. As noted earlier, "service contracts" is a term that assumes different meanings to 251 different people. The commissioners may have been using the term loosely, and not in its technical and legal sense, to refer, in general, to agreements concerning natural resources entered into by the Government with foreign corporations. These loose statements do not necessarily translate to the adoption of the 1973 Constitution provision allowing service contracts. It is true that, as shown in the earlier quoted portions of the proceedings in CONCOM, in response to Sr. Tan's question, Commissioner Villegas commented that, other than congressional notification, the only difference between "future" and "past" "service contracts" is the requirement of a general law as there 252 were no laws previously authorizing the same. However, such remark is far outweighed by his more categorical statement in his exchange with Commissioner Quesada that the draft article "does not permit foreign investors to participate" in the nation's natural resources which was exactly what service 253 contracts did except to provide "technical or financial assistance." In the case of the other commissioners, Commissioner Nolledo himself clarified in his work that the 254 present charter prohibits service contracts. Commissioner Gascon was not totally averse to foreign 255 participation, but favored stricter restrictions in the form of majority congressional concurrence. On the other hand, Commissioners Garcia and Tadeo may have veered to the extreme side of the spectrum and their objections may be interpreted as votes against any foreign participation in our natural resources whatsoever. WMCP cites Opinion No. 75, s. 1987, and Opinion No. 175, s. 1990 of the Secretary of Justice, expressing the view that a financial or technical assistance agreement "is no different in concept" from the service contract allowed under the 1973 Constitution. This Court is not, however, bound by this interpretation. When an administrative or executive agency renders an opinion or issues a statement of policy, it merely interprets a pre-existing law; and the administrative interpretation of the law is at best 258 advisory, for it is the courts that finally determine what the law means. In any case, the constitutional provision allowing the President to enter into FTAAs with foreign-owned corporations is an exception to the rule that participation in the nation's natural resources is reserved exclusively to Filipinos. Accordingly, such provision must be construed strictly against their enjoyment by 259 non-Filipinos. As Commissioner Villegas emphasized, the provision is "very restrictive." Commissioner Nolledo also remarked that "entering into service contracts is an exception to the rule on protection of natural resources for the interest of the nation and, therefore, being an exception, it should be subject, 260 whenever possible, to stringent rules." Indeed, exceptions should be strictly but reasonably construed; they extend only so far as their language fairly warrants and all doubts should be resolved in favor of the 261 general provision rather than the exception. With the foregoing discussion in mind, this Court finds that R.A. No. 7942 is invalid insofar as said Act authorizes service contracts. Although the statute employs the phrase "financial and technical agreements" in accordance with the 1987 Constitution, it actually treats these agreements as service contracts that grant beneficial ownership to foreign contractors contrary to the fundamental law. Section 33, which is found under Chapter VI (Financial or Technical Assistance Agreement) of R.A. No. 7942 states: SEC. 33. Eligibility.Any qualified person with technical and financial capability to undertake large-scale exploration, development, and utilization of mineral resources in the Philippines may enter into a financial or technical assistance agreement directly with the Government through the Department. [Emphasis supplied.] "Exploration," as defined by R.A. No. 7942, means the searching or prospecting for mineral resources by geological, geochemical or geophysical surveys, remote sensing, test pitting, trending, drilling, shaft sinking, tunneling or any other means for the
256 257

L A W O N N A T U R A L R E S O U R C E S a n d E N V I R O N M E N T A L L A W C a s e s a n d S p e c i a l L a w s | 56
purpose of determining the existence, extent, quantity and quality thereof and the feasibility of mining 262 them for profit. A legally organized foreign-owned corporation may be granted an exploration permit, which vests it 264 with the right to conduct exploration for all minerals in specified areas, i.e., to enter, occupy and 265 explore the same. Eventually, the foreign-owned corporation, as such permittee, may apply for a 266 financial and technical assistance agreement. "Development" is the work undertaken to explore and prepare an ore body or a mineral deposit for 267 mining, including the construction of necessary infrastructure and related facilities. "Utilization" "means the extraction or disposition of minerals." A stipulation that the proponent shall dispose of the minerals and byproducts produced at the highest price and more advantageous terms and conditions as provided for under the implementing rules and regulations is required to be incorporated in 269 every FTAA. A foreign-owned/-controlled corporation may likewise be granted a mineral processing 270 permit. "Mineral processing" is the milling, beneficiation or upgrading of ores or minerals and rocks or 271 by similar means to convert the same into marketable products. An FTAA contractor makes a warranty that the mining operations shall be conducted in accordance with 272 the provisions of R.A. No. 7942 and its implementing rules and for work programs and minimum 273 expenditures and commitments. And it obliges itself to furnish the Government records of geologic, 274 accounting, and other relevant data for its mining operation. "Mining operation," as the law defines it, means mining activities involving exploration, feasibility, 275 development, utilization, and processing. The underlying assumption in all these provisions is that the foreign contractor manages the mineral resources, just like the foreign contractor in a service contract. Furthermore, Chapter XII of the Act grants foreign contractors in FTAAs the same auxiliary mining rights 276 that it grants contractors in mineral agreements (MPSA, CA and JV). Parenthetically, Sections 72 to 75 use the term "contractor," without distinguishing between FTAA and mineral agreement contractors. And so does "holders of mining rights" in Section 76. A foreign contractor may even convert its FTAA into a mineral agreement if the economic viability of the contract area is found to be inadequate to justify large277 scale mining operations, provided that it reduces its equity in the corporation, partnership, association 278 or cooperative to forty percent (40%). Finally, under the Act, an FTAA contractor warrants that it "has or has access to all the financing, 279 managerial, and technical expertise. . . ." This suggests that an FTAA contractor is bound to provide
268 263

some management assistance a form of assistance that has been eliminated and, therefore, proscribed by the present Charter. By allowing foreign contractors to manage or operate all the aspects of the mining operation, the abovecited provisions of R.A. No. 7942 have in effect conveyed beneficial ownership over the nation's mineral resources to these contractors, leaving the State with nothing but bare title thereto. Moreover, the same provisions, whether by design or inadvertence, permit a circumvention of the constitutionally ordained 60%-40% capitalization requirement for corporations or associations engaged in the exploitation, development and utilization of Philippine natural resources. In sum, the Court finds the following provisions of R.A. No. 7942 to be violative of Section 2, Article XII of the Constitution: (1) The proviso in Section 3 (aq), which defines "qualified person," to wit: Provided, That a legally organized foreign-owned corporation shall be deemed a qualified person for purposes of granting an exploration permit, financial or technical assistance agreement or mineral processing permit. (2) Section 23, which specifies the rights and obligations of an exploration permittee, insofar as said section applies to a financial or technical assistance agreement, (3) Section 33, which prescribes the eligibility of a contractor in a financial or technical assistance agreement; (4) Section 35, which enumerates the terms and conditions for every financial or technical assistance agreement; (5) Section 39, which allows the contractor in a financial and technical assistance agreement to convert the same into a mineral production-sharing agreement; (6) Section 56, which authorizes the issuance of a mineral processing permit to a contractor in a financial and technical assistance agreement; The following provisions of the same Act are likewise void as they are dependent on the foregoing provisions and cannot stand on their own: (1) Section 3 (g), which defines the term "contractor," insofar as it applies to a financial or technical assistance agreement.
284 283 282 281 280

L A W O N N A T U R A L R E S O U R C E S a n d E N V I R O N M E N T A L L A W C a s e s a n d S p e c i a l L a w s | 57
Section 34, which prescribes the maximum contract area in a financial or technical assistance agreements; Section 36,
286 285

which allows negotiations for financial or technical assistance agreements;

(d) have the right of possession of the Contract Area, with full right of ingress and egress and the right to occupy the same, subject to the provisions of Presidential Decree No. 512 (if applicable) and not be prevented from entry into private ands by surface owners and/or occupants thereof when prospecting, exploring and exploiting for minerals therein; xxx (f) to construct roadways, mining, drainage, power generation and transmission facilities and all other types of works on the Contract Area; (g) to erect, install or place any type of improvements, supplies, machinery and other equipment relating to the Mining Operations and to use, sell or otherwise dispose of, modify, remove or diminish any and all parts thereof; (h) enjoy, subject to pertinent laws, rules and regulations and the rights of third Parties, easement rights and the use of timber, sand, clay, stone, water and other natural resources in the Contract Area without cost for the purposes of the Mining Operations; xxx (i) have the right to mortgage, charge or encumber all or part of its interest and obligations under this Agreement, the plant, equipment and infrastructure and the Minerals produced from the Mining Operations; x x x.
295

Section 37, which prescribes the procedure for filing and evaluation of financial or technical assistance agreement proposals; Section 38,
288

287

which limits the term of financial or technical assistance agreements;

Section 40, which allows the assignment or transfer of financial or technical assistance agreements; Section 41,
290

289

which allows the withdrawal of the contractor in an FTAA;


291

The second and third paragraphs of Section 81, which provide for the Government's share in a financial and technical assistance agreement; and Section 90, which provides for incentives to contractors in FTAAs insofar as it applies to said contractors; When the parts of the statute are so mutually dependent and connected as conditions, considerations, inducements, or compensations for each other, as to warrant a belief that the legislature intended them as a whole, and that if all could not be carried into effect, the legislature would not pass the residue independently, then, if some parts are unconstitutional, all the provisions which are thus dependent, 293 conditional, or connected, must fall with them. There can be little doubt that the WMCP FTAA itself is a service contract. Section 1.3 of the WMCP FTAA grants WMCP "the exclusive right to explore, exploit, utilise[,] process and dispose of all Minerals products and by-products thereof that may be produced from the Contract 294 Area." The FTAA also imbues WMCP with the following rights: (b) to extract and carry away any Mineral samples from the Contract area for the purpose of conducting tests and studies in respect thereof; (c) to determine the mining and treatment processes to be utilised during the Development/Operating Period and the project facilities to be constructed during the Development and Construction Period;
292

All materials, equipment, plant and other installations erected or placed on the Contract Area remain the property of WMCP, which has the right to deal with and remove such items within twelve months from 296 the termination of the FTAA. Pursuant to Section 1.2 of the FTAA, WMCP shall provide "[all] financing, technology, management and personnel necessary for the Mining Operations." The mining company binds itself to "perform all Mining 297 Operations . . . providing all necessary services, technology and financing in connection therewith," and to "furnish all materials, labour, equipment and other installations that may be required for carrying on all 298 Mining Operations." > WMCP may make expansions, improvements and replacements of the mining 299 facilities and may add such new facilities as it considers necessary for the mining operations. These contractual stipulations, taken together, grant WMCP beneficial ownership over natural resources that properly belong to the State and are intended for the benefit of its citizens. These stipulations are abhorrent to the 1987 Constitution. They are precisely the vices that the fundamental law seeks to avoid,

L A W O N N A T U R A L R E S O U R C E S a n d E N V I R O N M E N T A L L A W C a s e s a n d S p e c i a l L a w s | 58
the evils that it aims to suppress. Consequently, the contract from which they spring must be struck down. In arguing against the annulment of the FTAA, WMCP invokes the Agreement on the Promotion and Protection of Investments between the Philippine and Australian Governments, which was signed in Manila on January 25, 1995 and which entered into force on December 8, 1995. x x x. Article 2 (1) of said treaty states that it applies to investments whenever made and thus the fact that [WMCP's] FTAA was entered into prior to the entry into force of the treaty does not preclude the Philippine Government from protecting [WMCP's] investment in [that] FTAA. Likewise, Article 3 (1) of the treaty provides that "Each Party shall encourage and promote investments in its area by investors of the other Party and shall [admit] such investments in accordance with its Constitution, Laws, regulations and investment policies" and in Article 3 (2), it states that "Each Party shall ensure that investments are accorded fair and equitable treatment." The latter stipulation indicates that it was intended to impose an obligation upon a Party to afford fair and equitable treatment to the investments of the other Party and that a failure to provide such treatment by or under the laws of the Party may constitute a breach of the treaty. Simply stated, the Philippines could not, under said treaty, rely upon the inadequacies of its own laws to deprive an Australian investor (like [WMCP]) of fair and equitable treatment by invalidating [WMCP's] FTAA without likewise nullifying the service contracts entered into before the enactment of RA 7942 such as those mentioned in PD 87 or EO 279. This becomes more significant in the light of the fact that [WMCP's] FTAA was executed not by a mere Filipino citizen, but by the Philippine Government itself, through its President no less, which, in entering into said treaty is assumed to be aware of the existing Philippine laws on service contracts over the exploration, development and utilization of natural resources. The execution of the FTAA by the Philippine Government assures the Australian Government that the FTAA is in accordance with existing Philippine 300 laws. [Emphasis and italics by private respondents.] The invalidation of the subject FTAA, it is argued, would constitute a breach of said treaty which, in turn, would amount to a violation of Section 3, Article II of the Constitution adopting the generally accepted principles of international law as part of the law of the land. One of these generally accepted principles is pacta sunt servanda, which requires the performance in good faith of treaty obligations. Even assuming arguendo that WMCP is correct in its interpretation of the treaty and its assertion that "the Philippines could not . . . deprive an Australian investor (like [WMCP]) of fair and equitable treatment by invalidating [WMCP's] FTAA without likewise nullifying the service contracts entered into before the enactment of RA 7942 . . .," the annulment of the FTAA would not constitute a breach of the treaty invoked. For this decision herein invalidating the subject FTAA forms part of the legal system of the 301 302 Philippines. The equal protection clause guarantees that such decision shall apply to all contracts belonging to the same class, hence, upholding rather than violating, the "fair and equitable treatment" stipulation in said treaty. One other matter requires clarification. Petitioners contend that, consistent with the provisions of Section 2, Article XII of the Constitution, the President may enter into agreements involving "either technical or financial assistance" only. The agreement in question, however, is a technical and financial assistance agreement. Petitioners' contention does not lie. To adhere to the literal language of the Constitution would lead to 303 absurd consequences. As WMCP correctly put it: x x x such a theory of petitioners would compel the government (through the President) to enter into contract with two (2) foreign-owned corporations, one for financial assistance agreement and with the other, for technical assistance over one and the same mining area or land; or to execute two (2) contracts with only one foreign-owned corporation which has the capability to provide both financial and technical assistance, one for financial assistance and another for technical assistance, over the same mining area. Such an absurd result is definitely not sanctioned under the canons of constitutional 304 construction. [Underscoring in the original.] Surely, the framers of the 1987 Charter did not contemplate such an absurd result from their use of "either/or." A constitution is not to be interpreted as demanding the impossible or the impracticable; and 305 unreasonable or absurd consequences, if possible, should be avoided. Courts are not to give words a meaning that would lead to absurd or unreasonable consequences and a literal interpretation is to be 306 rejected if it would be unjust or lead to absurd results. That is a strong argument against its 307 adoption. Accordingly, petitioners' interpretation must be rejected. The foregoing discussion has rendered unnecessary the resolution of the other issues raised by the petition. WHEREFORE, the petition is GRANTED. The Court hereby declares unconstitutional and void: (1) The following provisions of Republic Act No. 7942: (a) The proviso in Section 3 (aq), (b) Section 23, (c) Section 33 to 41, (d) Section 56, (e) The second and third paragraphs of Section 81, and

L A W O N N A T U R A L R E S O U R C E S a n d E N V I R O N M E N T A L L A W C a s e s a n d S p e c i a l L a w s | 59
(f) Section 90. (2) All provisions of Department of Environment and Natural Resources Administrative Order 9640, s. 1996 which are not in conformity with this Decision, and (3) The Financial and Technical Assistance Agreement between the Government of the Republic of the Philippines and WMC Philippines, Inc. SO ORDERED. Davide, Jr., C.J., Puno, Quisumbing, Carpio, Corona, Callejo, Sr., and Tinga. JJ., Vitug, J., see Separate Panganiban, J., see Separate Ynares-Santiago, Sandoval-Gutierrez and Austria-Martinez, JJ., joins J., Panganiban's separate Azcuna, no part, one of the parties was a client.
Footnotes
1 2 18 19

concur. Opinion. Opinion. opinion.

Appears as "Nequito" in the caption of the Petition but "Nequinto" in the body. (Rollo, p. 12.) As appears in the body of the Petition. (Id., at 13.) The caption of the petition does not include Louel A. Peria as one of the petitioners but the name of his father Elpidio V. Peria appears therein. 3 Appears as "Kaisahan Tungo sa Kaunlaran ng Kanayunan at Repormang Pansakahan (KAISAHAN)" in the caption of the Petition by "Philippine Kaisahan Tungo sa Kaunlaran ng Kanayunan at Repormang Pansakahan (KAISAHAN)" in the body. (Id., at 14.) 4 Erroneously designated in the Petition as "Western Mining Philippines Corporation." (Id., at 212.) Subsequently, WMC (Philippines), Inc. was renamed "Tampakan Mineral Resources Corporation." (Id., at 778.) 5 An Act Instituting A New System of Mineral Resources Exploration, Development, Utilization and Conservation. 6 Authorizing the Secretary of Environment and Natural Resources to Negotiate and Conclude Joint Venture, Co-Production, or Production-Sharing Agreements for the Exploration, Development and Utilization of Mineral Resources, and Prescribing the Guidelines for such Agreements and those Agreements involving Technical or Financial Assistance by Foreign-Owned Corporations for Large-Scale Exploration, Development and Utilization of Minerals. 7 Exec. Order No. 279 (1987), sec. 4. 8 Rep. Act No. 7942 (1995), sec. 15. 9 Id., sec. 26 (a)-(c). 10 Id., sec. 29. 11 Id., sec. 30. 12 Id., sec. 31. 13 Id., sec. 32. 14 Id., ch. VI. 15 Id., secs. 27 and 33 in relation to sec. 3 (aq). 16 Id., sec. 72. 17 Id., sec. 73.

Id., sec. 75. Id., sec. 74. 20 Id., sec. 76. 21 Id., ch. XIII. 22 Id., secs. 20-22. 23 Id., secs. 43, 45. 24 Id., secs. 46-49, 51-52. 25 Id., ch. IX. 26 Id., ch. X. 27 Id., ch. XI. 28 Id., ch. XIV. 29 Id., ch. XV. 30 Id., ch. XVI. 31 Id., ch. XIX. 32 Id., ch. XVII. 33 Section 116, R.A. No. 7942 provides that the Act "shall take effect thirty (30) days following its complete publication in two (2) newspapers of general circulation in the Philippines." 34 WMCP FTAA, sec. 4.1. 35 Rollo, p. 22. 36 Ibid. 37 Ibid. 38 Ibid. The number has since risen to 129 applications when the petitioners filed their Reply. (Rollo, p. 363.) 39 Id., at 22. 40 Id., at 23-24. 41 Id., at 52-53. Emphasis and underscoring supplied. 42 WMCP FTAA, p. 2. 43 Rollo, p. 220. 44 Id., at 754. 45 Vide Note 4. 46 Rollo, p. 754. 47 Id., at 755. 48 Id., at 761-763. 49 Id., at 764-776. 50 Id., at 782-786. 51 Docketed as C.A.-G. R. No. 74161. 52 G.R. No. 153885, entitled Lepanto Consolidated Mining Company v. WMC Resources International Pty. Ltd., et al., decided September 24, 2003 and G.R. No. 156214, entitled Lepanto Mining Company v. WMC Resources International Pty. Ltd., WMC (Philippines), Inc., Southcot Mining Corporation, Tampakan Mining Corporation and Sagittarius Mines, Inc., decided September 23, 2003. 53 Section 12, Rule 43 of the Rules of Court, invoked by private respondent, states, " The appeal shall not stay the award, judgment, final order or resolution sought to be reviewed unless the Court of Appeals shall direct otherwise upon such terms as it may deem just." 54 WMCP's Reply (dated May 6, 2003) to Petitioners' Comment (to the Manifestation and Supplemental Manifestation), p. 3. 55 Ibid. 56 Ibid.

L A W O N N A T U R A L R E S O U R C E S a n d E N V I R O N M E N T A L L A W C a s e s a n d S p e c i a l L a w s | 60
57

WMCP's Reply (dated May 6, 2003) to Petitioners' Comment (to the Manifestation and Supplemental Manifestation), p. 4. 58 Philippine Constitution Association v. Enriquez, 235 SCRA 506 (1994); National Economic Protectionism Association v. Ongpin, 171 SCRA 657 (1989); Dumlao v. COMELEC, 95 SCRA 392 (1980). 59 Dumlao v. COMELEC, supra. 60 Board of Optometry v. Colet, 260 SCRA 88 (1996). 61 Dumlao v. COMELEC, supra. 62 Subic Bay Metropolitan Authority v. Commission on Elections, 262 SCRA 492 (1996). 63 Angara v. Electoral Commission, 63 Phil. 139 (1936). 64 Integrated Bar of the Philippines v. Zamora, 338 SCRA 81, 100 (2000); Dumlao v. COMELEC, supra; People v. Vera, 65 Phil. 56 (1937). 65 Dumlao v. COMELEC, supra. 66 Integrated Bar of the Philippines v. Zamora, supra. 67 Ermita-Malate Hotel and Motel Operators Association, Inc. v. City Mayor of Manila 21 SCRA 449 (1967). 68 Petitioners Roberto P. Amloy, Raqim L. Dabie, Simeon H. Dolojo, Imelda Gandon, Leny B. Gusanan, Marcelo L. Gusanan, Quintol A. Labuayan, Lomingges Laway, and Benita P. Tacuayan. 69 Petitioners F'long Agutin M. Dabie, Mario L. Mangcal, Alden S. Tusan, Sr. Susuan O. Bolanio, OND, Lolita G. Demonteverde, Benjie L. Nequinto, Rose Lilia S. Romano and Amparo S. Yap. 70 Rollo, p. 6. 71 Id. at 337, citing Malabanan v. Gaw Ching, 181 SCRA 84 (1990). 72 246 SCRA 540 (1995). 73 People v. Vera, supra. 74 Militante v. Court of Appeals, 330 SCRA 318 (2000). 75 Ibid. 76 Cruz v. Secretary of Environment and Natural Resources, 347 SCRA 128 (2000), Kapunan, J., Separate Opinion. [Emphasis supplied.] 77 Joya v. Presidential Commission on Good Government, 225 SCRA 568 (1993). 78 Integrated Bar of the Philippines v. Zamora, supra. 79 J. Bernas, S.J., The 1987 Constitution of the Philippines: A Commentary 1009 (1996). 80 Cruz v. Secretary of Environment and Natural Resources, supra, Kapunan, J., Separate Opinion. 81 Id., Puno, J., Separate Opinion, and Panganiban, J., Separate Opinion. 82 Cario v. Insular Government, 212 US 449, 53 L.Ed. 595 (1909). For instance, Law 14, Title 12, Book 4 of the Recopilacion de Leyes de las Indias proclaimed: We having acquired full sovereignty over the Indies, and all lands, territories, and possessions not heretofore ceded away by our royal predecessors, or by us, or in our name, still pertaining to the royal crown and patrimony, it is our will that all lands which are held without proper and true deeds of grant be restored to us according as they belong to us, in order that after reserving before all what to us or to our viceroys, audiencias, and governors may seem necessary for public squares, ways, pastures, and commons in those places which are peopled, taking into consideration not only their present condition, but also their future and their probable increase, and after distributing to the natives what may be necessary for tillage and pasturage, confirming them in what they now have and giving them more if necessary, all the rest of said lands may remain free and unencumbered for us to dispose of as we may wish. 83 Republic v. Court of Appeals, 160 SCRA 228 (1988). It has been noted, however, that "the prohibition in the [1935] Constitution against alienation by the state of mineral lands and minerals is not properly a part of the Regalian doctrine but a separate national policy designed to conserve our mineral resources and

prevent the state from being deprived of such minerals as are essential to national defense." (A. Noblejas, Philippine Law on Natural Resources 126-127 [1959 ed.], citing V. Francisco, The New Mining Law.) 84 Cruz v. Secretary of Environment and Natural Resources, supra, Kapunan, J., Separate Opinion, citing A. Noblejas, Philippine Law on Natural Resources 6 (1961). Noblejas continues: Thus, they asserted their right of ownership over mines and minerals or precious metals, golds, and silver as distinct from the right of ownership of the land in which the minerals were found. Thus, when on a piece of land mining was more valuable than agriculture, the sovereign retained ownership of mines although the land has been alienated to private ownership. Gradually, the right to the ownership of minerals was extended to base metals. If the sovereign did not exploit the minerals, they grant or sell it as a right separate from the land. (Id., at 6.) 85 In the unpublished case of Lawrence v. Garduo (L-10942, quoted in V. Francisco, Philippine Law on Natural Resources 14-15 [1956]), this Court observed: The principle underlying Spanish legislation on mines is that these are subject to the eminent domain of the state. The Spanish law of July 7, 1867, amended by the law of March 4, 1868, in article 2 says: "The ownership of the substances enumerated in the preceding article (among them those of inflammable nature), belong[s] to the state, and they cannot be disposed of without the government authority." The first Spanish mining law promulgated for these Islands (Decree of Superior Civil Government of January 28, 1864), in its Article I, says: "The supreme ownership of mines throughout the kingdom belong[s] to the crown and to the king. They shall not be exploited except by persons who obtained special grant from this superior government and by those who may secure it thereafter, subject to this regulation." Article 2 of the royal decree on ownership of mines in the Philippine Islands, dated May 14, 1867, which was the law in force at the time of the cession of these Islands to the Government of the United States, says: "The ownership of the substances enumerated in the preceding article (among them those of inflammable nature) belongs to the state, and they cannot be disposed of without an authorization issued by the Superior Civil Governor." Furthermore, all those laws contained provisions regulating the manner of prospecting, locating and exploring mines in private property by persons other than the owner of the land as well as the granting of concessions, which goes to show that private ownership of the land did not include, without express grant, the mines that might be found therein. Analogous provisions are found in the Civil Code of Spain determining the ownership of mines. In its Article 339 (Article 420, New Civil Code) enumerating properties of public ownership, the mines are included, until specially granted to private individuals. In its article 350 (Art. 437, New Civil Code) declaring that the proprietor of any parcel of land is the owner of its surface and of everything under it, an exception is made as far as mining laws are concerned. Then in speaking of minerals, the Code in its articles 426 and 427 (Art. 519, New Civil Code) provides rules governing the digging of pits by third persons on private-owned lands for the purpose of prospecting for minerals. 86 Atok Big-Wedge Mining Co. v. Intermediate Appellate Court, 261 SCRA 528 (1996). 87 Ibid. 88 Cruz v. Secretary of Environment and Natural Resources, supra, Kapunan, J., Separate Opinion. 89 Ibid. 90 McDaniel v. Apacible and Cuisia, 42 Phil. 749 (1922). 91 Noblejas, supra, at 5. 92 V. M. A. Dimagiba, Service Contract Concepts in Energy, 57 Phil. L. J. 307, 313 (1982).

L A W O N N A T U R A L R E S O U R C E S a n d E N V I R O N M E N T A L L A W C a s e s a n d S p e c i a l L a w s | 61
93

P. A. Agabin, Service Contracts: Old Wine in New Bottles?, in II Draft Proposal of the 1986 U.P. Law Constitution Project 3. 94 Id., at 2-3. 95 Id., at 3. 96 Ibid. 97 Ibid. 98 Ibid. 99 An Act to Provide for the Exploration, Location and Lease of Lands Containing Petroleum and other Mineral Oils and Gas in the Philippine Islands. 100 An Act to Provide for the Leasing and Development of Coal Lands in the Philippine Islands. 101 Agabin, supra, at 3. 102 People v. Linsangan, 62 Phil. 646 (1935). 103 Ibid. 104 Ibid. 105 Ibid. 106 Ibid. 107 Atok Big-Wedge Mining Co. v. Intermediate Appellate Court, supra. 108 Bernas, S.J., supra, at 1009-1010, citing Lee Hong Hok v. David, 48 SCRA 372 (1972). 109 II J. Aruego, The Framing of the Philippine Constitution 592 (1949). 110 Id., at 600-601. 111 Id., at 604. Delegate Aruego expounds: At the time of the framing of the Philippine Constitution, Filipino capital had been known to be rather shy. Filipinos hesitated as a general rule to invest a considerable sum of their capital for the development, exploitation, and utilization of the natural resources of the country. They had not as yet been so used to corporate enterprises as the peoples of the West. This general apathy, the delegates knew, would mean the retardation of the development of the natural resources, unless foreign capital would be encouraged to come in and help in that development. They knew that the nationalization of the natural resources would certainly not encourage the investment of foreign capital into them. But there was a general feeling in the Convention that it was better to have such development retarded or even postponed altogether until such time when the Filipinos would be ready and willing to undertake it rather than permit the natural resources to be placed under the ownership or control of foreigners in order that they might be immediately developed, with the Filipinos of the future serving not as owners but at most as tenants or workers under foreign masters. By all means, the delegates believed, the natural resources should be conserved for Filipino posterity. The nationalization of natural resources was also intended as an instrument of national defense. The Convention felt that to permit foreigner to own or control the natural resources would be to weaken the national defense. It would be making possible the gradual extension of foreign influence into our politics, thereby increasing the possibility of foreign control. x x x. Not only these. The nationalization of the natural resources, it was believed, would prevent making the Philippines a source of international conflicts with the consequent danger to its internal security and independence. For unless the natural resources were nationalized, with the nationals of foreign countries having the opportunity to own or control them, conflicts of interest among them might arise inviting danger to the safety and independence of the nation. (Id., at 605-606.) 112 Palting v. San Jose Petroleum Inc., 18 SCRA 924 (1966); Republic v. Quasha, 46 SCRA 160 (1972). 113 Atok Big-Wedge Mining Co. v. Intermediate Appellate Court, supra.

114

Article VI thereof provided: 1. The disposition, exploitation, development and utilization of all agricultural, timber, and mineral lands of the public domain, waters, minerals, coal, petroleum and other mineral oils, all forces and sources of potential energy, and other natural resources of either Party, and the operation of public utilities, shall, if open to any person, be open to citizens of the other Party and to all forms of business enterprise owned or controlled directly or indirectly, by citizens of such other Party in the same manner as to and under the same conditions imposed upon citizens or corporations or associations owned or controlled by citizens of the Party granting the right. 2. The rights provided for in Paragraph 1 may be exercised x x x in the case of citizens of the United States, with respect to natural resources in the public domain in the Philippines, only through the medium of a corporation organized under the laws of the Philippines and at least 60% of the capital stock of which is owned or controlled by citizens of the United States x x x. 3. The United States of America reserves the rights of the several States of the United States to limit the extent to which citizens or corporations or associations owned or controlled by citizens of the Philippines may engage in the activities specified in this Article. The Republic of the Philippines reserves the power to deny any of the rights specified in this Article to citizens of the United States who are citizens of States, or to corporations or associations at least 60% of whose capital stock or capital is owned or controlled by citizens of States, which deny like rights to citizens of the Philippines, or to corporations or associations which ore owned or controlled by citizens of the Philippines x x x. 115 An Act to Promote the Exploration, Development, Exploitation, and Utilization of the Petroleum Resources of the Philippines; to Encourage the Conservation of such Petroleum Resources; to Authorize the Secretary of Agriculture and Natural Resources to Create an Administration Unit and a Technical Board in the Bureau of Mines; to Appropriate Funds therefor; and for other purposes. 116 Rep. Act No. 387 (1949), as amended, art. 10 (b). 117 Id., art. 10 (c). 118 Id., art. 5. 119 Id., art. 31. The same provision recognized the rights of American citizens under the Parity Amendment: During the effectivity and subject to the provisions of the ordinance appended to the Constitution of the Philippines, citizens of the United States and all forms of business enterprises owned and controlled, directly or indirectly, by citizens of the United States shall enjoy the same rights and obligations under the provisions of this Act in the same manner as to, and under the same conditions imposed upon, citizens of the Philippines or corporations or associations owned or controlled by citizens of the Philippines. 120 Id., art. 10. 121 Id., art. 3. 122 Id., art. 9. 123 Ibid. 124 Rep. Act No. 387 (1949), as amended, art. 8. 125 Id., art. 25. 126 Id., art. 47. 127 Id., art. 60. 128 Id., art. 64. Article 49, R.A. No. 387 originally imposed an annual exploration tax on exploration concessionaires but this provision was repealed by Section 1, R.A. No. 4304. 129 Francisco, supra, at 103. 130 Rep. Act No. 387 (1949), as amended, art. 65. 131 Francisco, supra, at 103.

L A W O N N A T U R A L R E S O U R C E S a n d E N V I R O N M E N T A L L A W C a s e s a n d S p e c i a l L a w s | 62
132 133

Rep. Act No. 387 (1949), as amended, art. 90 (b) 3. Id., art. 90 (b) 4. 134 Id., art. 93-A. 135 Id., art. 93. 136 Ibid. 137 Rep. Act No. 387 (1949), as amended, art. 94. 138 Id., art. 106. 139 Id., art. 95. 140 Ibid. 141 Rep. Act No. 387 (1949), as amended, art. 95 (e). 142 Dimagiba, supra, at 315, citing Fabrikant, Oil Discovery and Technical Change in Southeast Asia, Legal Aspects of Production Sharing Contracts in the Indonesian Petroleum Industry, 101-102, sections 13C.24 and 13C.25 (1972). 143 Agabin, supra, at 4. 144 Dimagiba, supra, at 318. 145 Amending Presidential Decree No. 8 issued on October 2, 1972, and Promulgating an Amended Act to Promote the Discovery and Production of Indigenous Petroleum and Appropriate Funds Therefor. 146 Pres. Decree No. 87 (1972), sec. 4. 147 Agabin, supra, at 6. 148 M. Magallona, Service Contracts in Philippine Natural Resources, 9 World Bull. 1, 4 (1993). 149 Pres. Decree No. 87 (1972), sec. 6. 150 Id., sec. 4. 151 Id., sec. 6. 152 Id., sec. 7. 153 Id., sec. 8. 154 Ibid. 155 Ibid. 156 Pres. Decree No. 87 (1972), sec. 9. 157 Id., sec. 12. 158 Id., sec. 13. 159 Dimagiba draws the following comparison between the service contract scheme and the concession system: In both the concession system and the service contract scheme, work and financial obligations are required of the developer. Under Republic Act No. 387 and Presidential Decree No. 87, the concessionaire and the service contractors are extracted certain taxes in favor of the government. In both arrangements, the explorationist/developer is given incentives in the form of tax exemptions in the importation or disposition of machinery, equipment, materials and spare parts needed in petroleum operations. The concessionaire and the service contractor are required to keep in their files valuable data and information and may be required to submit need technological or accounting reports to the Government. Duly authorized representatives of the Government could, under the law, inspect or audit the books of accounts of the contract holder. In both systems, signature, discovery or production bonuses may be given by the developer to the host Government. The concession system, however, differs considerably from the service contract system in important areas of the operations. In the concession system, the Government merely receives fixed royalty which is a certain percentage of the crude oil produced or other units of measure,

regardless of whether the concession holder makes profits or not. This is not so in the service contract system. A certain percentage of the gross production is set aside for recoverable expenditures by the contractor. Of the net proceeds the parties are entitled percentages of share that will accrue to each of them. In the royalty system, the concessionaire may be discouraged to produce more for the reason that since the royalty paid to the host country is closely linked to the volume of production, the greater the produce, the more amount or royalty would be allocated to the Government. This is not so in the production sharing system. The share of the Government depends largely on the net proceeds of production after reimbursing the service contractor of its recoverable expenses. As a general rule, the Government plays a passive role in the concession system, more particularly, interested in receiving royalties from the concessionaire. In the production-sharing arrangement, the Government plays a more active role in the management and monitoring of oil operations and requires the service contractor entertain obligations designed to bring more economic and technological benefits to the host country. (Dimagiba, supra, at 330-331.) 160 Agabin, supra, at 6. 161 The antecedents leading to the Proclamation are narrated in Javellana v. Executive Secretary, 50 SCRA 55 (1973): On March 16, 1967, Congress of the Philippines passed Resolution No. 2, which was amended by Resolution No. 4, of said body, adopted on June 17, 1969, calling a convention to propose amendments to the Constitution of the Philippines. Said Resolution No. 2, as amended, was implemented by Republic Act No. 6132 approved on August 24, 1970, pursuant to the provisions of which the election of delegates to said convention was held on November 10, 1970, and the 1971 Convention began to perform its functions on June 1, 1971. While the Convention was in session on September 21, 1972, the President issued Proclamation No. 1081 placing the entire Philippines under Martial Law. On November 29, 1972, the President of the Philippines issued Presidential Decree No. 73, submitting to the Filipino people for ratification or rejection the Constitution of the Republic of the Philippines proposed by the 1971 Constitutional Convention, and appropriating funds therefor, as well as setting the plebiscite for such ratification on January 15, 1973. On January 17, 1973, the President issued Proclamation No. 1102 certifying and proclaiming that the Constitution proposed by the 1971 Constitutional Convention "has been ratified by an overwhelming majority of all the votes cast by the members of all the Barangays (Citizens Assemblies) throughout the Philippines, and has thereby come into effect." 162 Bernas, S.J., supra, at 1016, Note 28, citing Session of November 25, 1972. 163 Agabin, supra, at 1, quoting Sanvictores, The Economic Provisions in the 1973 Constitution, in Espiritu, 1979 Philconsa Reader on Constitutional and Policy Issues 449. 164 Bernas, S.J., supra, at 1016, Note 28, citing Session of November 25, 1972. 165 Ibid. 166 Ibid. 167 Allowing Citizens of the Philippines or Corporations or Associations at least Sixty Per Centum of the Capital of which is Owned by such Citizens to Enter into Service Contracts with Foreign Persons, Corporations for the Exploration, Development, Exploitation or Utilization of Lands of the Public Domain, Amending for the purpose certain provisions of Commonwealth Act No. 141. 168 Pres. Decree No. 151 (1973), sec. 1. 169 Providing for A Modernized System of Administration and Disposition of Mineral Lands and to Promote and Encourage the Development and Exploitation thereof. 170 Revising and Consolidating All Laws and Decrees Affecting Fishing and Fisheries.

L A W O N N A T U R A L R E S O U R C E S a n d E N V I R O N M E N T A L L A W C a s e s a n d S p e c i a l L a w s | 63
171 172

Pres. Decree No. 704 (1975), sec. 21. Revising Presidential Decree No. 389, otherwise known as The Forestry Reform Code of the Philippines. 173 Pres. Decree No. 705 (1975), sec. 62. 174 An Act to Promote the Exploration and Development of Geothermal Resources. 175 Magallona, supra, at 6. 176 Declaring a National Policy to Implement the Reforms Mandated by the People, Protecting their Basic Rights, Adopting a Provisional Constitution, and Providing for an Orderly Transition to a Government under a New Constitution. 177 Const., art. XVIII, sec. 27; De Leon v. Esguerra, 153 SCRA 602 (1987). 178 Miners Association of the Philippines, Inc. v. Factoran, Jr., 240 SCRA 100 (1995). 179 Ibid. 180 Ibid. 181 J. Bernas, S.J., The Intent of the 1986 Constitution Writers 812 (1995). 182 Miners Association of the Philippines, Inc. v. Factoran, Jr., supra. 183 III Records of the Constitutional Commission 255. 184 Id., at 355-356. 185 Const. (1986), art. II, sec. 1. 186 Cruz v. Secretary of Environment and Natural Resources, supra, Puno, J., Separate Opinion. 187 Rep. Act No. 7942 (1995), sec. 9. 188 SEC. 82. Allocation of Government Share.The Government share as referred to in the preceding sections shall be shared and allocated in accordance with Sections 290 and 292 of Republic Act No. 7160 otherwise known as the Local Government Code of 1991. In case the development and utilization of mineral resources is undertaken by a government-owned or -controlled corporation, the sharing and allocation shall be in accordance with Sections 291 and 292 of the said Code. 189 An Act Creating A People's Small-Scale Mining Program and for other purposes. 190 Rep. Act No. 7942 (1995), sec. 42. 191 Id., secs. 3 (ab) and 26. 192 "Contractor" means a qualified person acting alone or in consortium who is a party to a mineral agreement or to a financial or technical assistance agreement. (Id., sec. 3[g].) 193 "Contract area" means land or body water delineated for purposes of exploration, development, or utilization of the minerals found therein. (Id., sec. 3[f].) 194 "Gross output" means the actual market value of minerals or mineral products from its mining area as defined in the National Internal Revenue Code (Id., sec. 3[v]). 195 Id., sec. 26 (a). 196 An Act Reducing Excise Tax Rates on Metallic and Non-Metallic Minerals and Quarry Resources, amending for the purpose Section 151 (a) of the National Internal Revenue Code, as amended. 197 Rep. Act No. 7942 (1995), sec. (80). 198 Id., Sec. 26 (b). 199 "Mineral resource" means any concentration of minerals/rocks with potential economic value. (Id., sec. 3[ad].) 200 Id., sec. 26 (c). 201 Ibid. 202 Id., sec. 3 (h). 203 Id., sec. 3 (x). 204 Id., sec. 26, last par. 205 Id., sec. 27. 206 Id., sec. 3 (aq).

207 208

Id., sec. 3 (r). Id., sec. 33. 209 Id., sec. 3 (t). 210 Id., sec. 3 (aq). 211 The maximum areas in cases of mineral agreements are prescribed in Section 28 as follows: SEC. 28. Maximum Areas for Mineral Agreement. The maximum area that a qualified person may hold at any time under a mineral agreement shall be: (a) Onshore, in any one province (1) For individuals, ten (10) blocks; and (2) For partnerships, cooperatives, associations, or corporations, one hundred (100) blocks. (b) Onshore, in the entire Philippines (1) For individuals, twenty (20) blocks; and (2) For partnerships, cooperatives, associations, or corporations, two hundred (200) blocks. (c) Offshore, in the entire Philippines (1) For individuals, fifty (50) blocks; (2) For partnerships, cooperatives, associations, or corporations, five hundred (500) blocks; and (3) For the exclusive economic area, a larger area to be determined by the Secretary. The maximum areas mentioned above that a contractor may hold under a mineral agreement shall not include mining/quarry areas under operating agreements between the contractor and a claimowner/lessee/permittee/licensee entered into under Presidential Decree No. 463. On the other hand, Section 34, which governs the maximum area for FTAAs provides: SEC. 34. Maximum Contract Area. The maximum contract area that may be granted per qualified person, subject to relinquishment shall be: (a) 1,000 meridional blocks onshore; (b) 4,000 meridional blocks offshore; or (c) Combinations of (a) and (b) provided that it shall not exceed the maximum limits for onshore and offshore areas. 212 Id., sec. 33. 213 Id., sec. 81. 214 Kapatiran v. Tan, 163 SCRA 371 (1988). 215 Providing for the Publication of Laws either in the Official Gazette or in a Newspaper of General Circulation in the Philippines as a Requirement for their Effectivity. 216 Section 1, E.O. No. 200 was subsequently incorporated in the Administrative Code of 1987 (Executive Order No. 292 as Section 18, Chapter 5 (Operation and Effect of Laws), Book 1 (Sovereignty and General Administration). 217 136 SCRA 27 (1985). 218 Manila Prince Hotel v. Government Service Insurance System, 267 SCRA 408 (1997). 219 Const., art. 3, sec. 1. 220 83 O.G. (Suppl.) 3528-115 to 3528-117 (August 1987). 221 Miners Association of the Philippines, Inc. v. Factoran, Jr., supra. 222 Petitioners note in their Memorandum that the FTAA:

L A W O N N A T U R A L R E S O U R C E S a n d E N V I R O N M E N T A L L A W C a s e s a n d S p e c i a l L a w s | 64
x x x guarantees that wholly foreign owned [WMCP] entered into the FTAA in order to facilitate "the large scale exploration, development and commercial exploitation of mineral deposits that may be found to exist within the Contract area." [Section 1.1] As a contractor it also has the "exclusive right to explore, exploit, utilize, process and dispose of all mineral products and byproducts thereof that may be derived or produced from the Contract Area." [Section 1.3] Thus, it is divided into an "exploration and feasibility phase" [Section 3.2 (a)] and a "construction, development and production phase." [Section 3. 2 (b).] Thus, it is this wholly foreign owned corporation that, among other things: (a) operates within a prescribed contract area [Section 4], (b) opts to apply for a Mining Production Sharing Agreement [Section 4.2], (c) relinquishes control over portions thereof at their own choice [Section 4.6], (d) submits work programs, incurs expenditures, and makes reports during the exploration period [Section 5], (e) submits a Declaration of Mining Feasibility [Sections 5.4 and 5.5], (f) during the development period, determines the timetable, submits work programs, provides the reports and determines and executes expansions, modifications, improvements and replacements of new mining facilities within the area [Section 6], (g) complies with the conditions for environmental protection and industrial safety, posts the necessary bonds and makes representations and warranties to the government [Section 10.5]. The contract subsists for an initial term of twenty-five (25) years from the date of its effectivity [Section 3.1] and renewable for a further period of twenty-five years under the same terms and conditions upon application by private respondent [Section 3.3]. (Rollo, pp. 458-459.) 223 H. C. Black, Handbook on the Construction and Interpretation of the Laws 8. 224 Ibid. 225 J. M. Tuason & Co., Inc. v. Land Tenure Association, 31 SCRA 413 (1970). 226 Rollo, p. 580. 227 Ibid. Emphasis supplied. 228 People v. Manantan, 115 Phil. 657 (1962); Commission on Audit of the Province of Cebu v. Province of Cebu, 371 SCRA 196 (2001). 229 Rollo, p. 569. 230 III Record of the Constitutional Commission 351-352. 231 V Record of the Constitutional Commission 844. 232 Id., at 841. 233 Id., at 842. 234 Id. at 844. 235 Vide Cherey v. Long Beach, 282 NY 382, 26 NE 2d 945, 127 ALR 1210 (1940), cited in 16 Am Jur 2d Constitutional Law 79. 236 Civil Liberties Union v. Executive Secretary, 194 SCRA 317, 325 (1991). 237 III Record of the Constitutional Commission 278. 238 Id., at 316-317. 239 III Record of the Constitutional Commission 358-359. 240 Vera v. Avelino, 77 Phil. 192 (1946). 241 J. Nolledo, The New Constitution of the Philippines Annotated 924-926 (1990). 242 Resolution to Incorporate in the New Constitution an Article on National Economy and Patrimony. 243 The Chair of the Committee on National Economy and Patrimony, alluded to it in the discussion on the capitalization requirement: MR. VILLEGAS. We just had a long discussion with the members of the team from the UP Law Center who provided us a draft. The phrase that is contained here which we adopted from the UP draft is "60 percent of voting stock." (III Record of the Constitutional Commission 255.) Likewise, in explaining the reasons for the deletion of the term "exploitation": MR. VILLEGAS. Madam President, following the recommendation in the UP draft, we omitted "exploitation" first of all because it is believed to be subsumed under "development" and secondly because it has a derogatory connotation. (Id., at 358.) 244 Id., at 12. 245 Id., at 15-16. 246 M. Magallona, Nationalism and Its Subversion in the Constitution 5, in II Draft Proposal of the 1986 U.P. Law Constitution Project. 247 Agabin, supra, at 16. 248 E. Labitag, Philippine Natural Resources: Some Problems and Perspectives 17 in II Draft Proposal of the 1986 U.P. Law Constitution Project. 249 I Draft Proposal of the 1986 U.P. Law Constitution Project 11-13. 250 Id., at 9-11. Professor Labitag also suggests that: x x x. The concession regime of natural resources disposition should be discontinued. Instead the State shall enter into such arrangements and agreements like co-production, joint ventures, etc. as shall bring about effective control and a larger share in the proceeds, harvest or production. (Labitag, supra, at 17.) 251 Vide Note 147. 252 Vide Note 230. The question was posed before the Jamir amendment and subsequent proposals introducing other limitations. Comm. Villegas' response that there was no requirement in the 1973 Constitution for a law to govern service contracts and that, in fact, there were then no such laws is inaccurate. The 1973 Charter required similar legislative approval, although it did not specify the form it should take: "The Batasang Pambansa, in the national interest, may allow such citizens to enter into service contracts." As previously noted, however, laws authorizing service contracts were actually enacted by presidential decree. 253 Vide Note 238. 254 Vide Note 241. 255 Vide Note 231. 256 Dated July 28, 1987. 257 Dated October 3, 1990. 258 Peralta v. Civil Service Commission, 212 SCRA 425 (1992). 259 Vide Note 238. 260 III Record of the Constitutional Commission 354. 261 Salaysay v. Castro, 98 Phil. 364 (1956). 262 Rep. Act No. 7942 (1995), sec. 3 (q). 263 Id., sec. 3 (aq). 264 Id., sec. 20. 265 Id., sec. 23, first par. 266 Id., sec. 23, last par. 267 Id., sec. 3 (j). 268 Id., sec. 3 (az). 269 Id., sec. 35 (m). 270 Id., secs. 3 (aq) and 56.

L A W O N N A T U R A L R E S O U R C E S a n d E N V I R O N M E N T A L L A W C a s e s a n d S p e c i a l L a w s | 65
271 272

Id., sec. 3 (y). Id., sec. 35 (g). 273 Id., sec. 35 (h). 274 Id., sec. 35 (l). 275 Id., sec. 3 (af). 276 SEC. 72. Timber Rights.Any provision of the law to the contrary notwithstanding, a contractor may be granted a right to cut trees or timber within his mining area as may be necessary for his mining operations subject to forestry laws, rules and regulations: Provided, That if the land covered by the mining area is already covered by exiting timber concessions, the volume of timber needed and the manner of cutting and removal thereof shall be determined by the mines regional director, upon consultation with the contractor, the timber concessionaire/permittee and the Forest Management Bureau of the Department: Provided, further, That in case of disagreement between the contractor and the timber concessionaire, the matter shall be submitted to the Secretary whose decision shall be final. The contractor shall perform reforestation work within his mining area in accordance with forestry laws, rules and regulations. [Emphasis supplied.] SEC. 73. Water Rights.A contractor shall have water rights for mining operations upon approval of application with the appropriate government agency in accordance with existing water laws, rules and regulations promulgated thereunder: Provided, That water rights already granted or vested through long use, recognized and acknowledged by local customs, laws and decisions of courts shall not thereby be impaired: Provided, further, That the Government reserves the right to regulate water rights and the reasonable and equitable distribution of water supply so as to prevent the monopoly of the use thereof. [Emphasis supplied.] SEC. 74. Right to Possess Explosives.A contractor/exploration permittee shall have the right to possess and use explosives within his contract/permit area as may be necessary for his mining operations upon approval of an application with the appropriate government agency in accordance with existing laws, rules and regulations promulgated thereunder: Provided, That the Government reserves the right to regulate and control the explosive accessories to ensure safe mining operations. [Emphasis supplied.] SEC. 75. Easement Rights.When mining areas are so situated that for purposes of more convenient mining operations it is necessary to build, construct or install on the mining areas or lands owned, occupied or leased by other persons, such infrastructure as roads, railroads, mills, waste dump sites, tailings ponds, warehouses, staging or storage areas and port facilities, tramways, runways, airports, electric transmission, telephone or telegraph lines, dams and their normal flood and catchment areas, sites for water wells, ditches, canals, new river beds, pipelines, flumes, cuts, shafts, tunnels, or mills, the contractor, upon payment of just compensation, shall be entitled to enter and occupy said mining areas or lands. [Emphasis supplied.] SEC. 76. Entry into Private Lands and Concession Areas.Subject to prior notification, holders of mining rights shall not be prevented from entry into private lands and concession areas by surface owners, occupants, or concessionaires when conducting mining operations therein: Provided, That any damage done to the property of the surface owner, occupant, or concessionaire as a consequence of such operations shall be properly compensated as may be bee provided for in the implementing rules and regulations: Provided, further, That to guarantee such compensation, the person authorized to conduct mining operation shall, prior thereto, post a bond with the regional director based on the type of properties, the prevailing prices in and around the area where the mining operations are to be conducted, with surety or sureties satisfactory to the regional director. [Emphasis supplied.] 277 Id., sec. 39, first par. 278 Id., sec. 39, second par.

279 280

Id., sec. 35 (e). SEC. 23. Rights and Obligations of the Permittee.x x x. The permittee may apply for a mineral production sharing agreement, joint venture agreement, co-production agreement or financial or technical assistance agreement over the permit area, which application shall be granted if the permittee meets the necessary qualifications and the terms and conditions of any such agreement: Provided, That the exploration period covered by the exploration period of the mineral agreement or financial or technical assistance agreement. 281 SEC. 35. Terms and Conditions. The following terms, conditions, and warranties shall be incorporated in the financial or technical assistance agreement, to wit: (a) A firm commitment in the form of a sworn statement, of an amount corresponding to the expenditure obligation that will be invested in the contract area: Provided, That such amount shall be subject to changes as may be provided for in the rules and regulations of this Act; (b) A financial guarantee bond shall be posted in favor of the Government in an amount equivalent to the expenditure obligation of the applicant for any year; (c) Submission of proof of technical competence, such as, but not limited to, its track record in mineral resource exploration, development, and utilization; details of technology to be employed in the proposed operation; and details of technical personnel to undertake the operation; (d) Representations and warranties that the applicant has all the qualifications and none of the disqualifications for entering into the agreement; (e) Representations and warranties that the contractor has or has access to all the financing, managerial and technical expertise and, if circumstances demand, the technology required to promptly and effectively carry out the objectives of the agreement with the understanding to timely deploy these resources under its supervision pursuant to the periodic work programs and related budgets, when proper, providing an exploration period up to two (2) years, extendible for another two (2) years but subject to annual review by the Secretary in accordance with the implementing rules and regulations of this Act, and further, subject to the relinquishment obligations; (f) Representations and warranties that, except for paymets for dispositions for its equity, foreign investments in local enterprises which are qualified for repatriation, and local supplier's credits and such other generally accepted and permissible financial schemes for raising funds for valid business purposes, the conractor shall not raise any form of financing from domestic sources of funds, whether in Philippine or foreign currency, for conducting its mining operations for and in the contract area; (g) The mining operations shall be conducted in accordance with the provisions of this Act and its implementing rules and regulations; (h) Work programs and minimum expenditures commitments; (i) Preferential use of local goods and services to the maximum extent practicable; (j) A stipulation that the contractors are obligated to give preference to Filipinos in all types of mining employment for which they are qualified and that technology shall be transferred to the same; (k) Requiring the proponent to effectively use appropriate anti-pollution technology and facilities to protect the environment and to restore or rehabilitate mined out areas and other areas affected by mine tailings and other forms of pollution or destruction; (l) The contractors shall furnish the Government records of geologic, accounting, and other relevant data for its mining operations, and that book of accounts and records shall be open for inspection by the government;

L A W O N N A T U R A L R E S O U R C E S a n d E N V I R O N M E N T A L L A W C a s e s a n d S p e c i a l L a w s | 66
(m) Requiring the proponent to dispose of the minerals and byproducts produced under a financial or technical assistance agreement at the highest price and more advantageous terms and conditions as provided for under the rules and regulations of this Act; (n) Provide for consultation and arbitration with respect to the interpretation and implementation of the terms and conditions of the agreements; and (o) Such other terms and conditions consistent with the Constitution and with this Act as the Secretary may deem to be for the best interest of the State and the welfare of the Filipino people. 282 SEC. 39. Option to Convert into a Mineral Agreement. The contractor has the option to convert the financial or technical assistance agreement to a mineral agreement at any time during the term of the agreement, if the economic viability of the contract area is found to be inadequate to justify large-scale mining operations, after proper notice to the Secretary as provided for under the implementing rules and regulations; Provided, That the mineral agreement shall only be for the remaining period of the original agreement. In the case of a foreign contractor, it shall reduce its equity to forty percent (40%) in the corporation, partnership, association, or cooperative. Upon compliance with this requirement by the contractor, the Secretary shall approve the conversion and execute the mineral productionsharing agreement. 283 SEC. 56. Eligibility of Foreign-owned/-controlled Corporation.A foreign owned/ -controlled corporation may be granted a mineral processing permit. 284 SEC. 3. Definition of Terms. As used in and for purposes of this Act, the following terms, whether in singular or plural, shall mean: xxx (g) "Contractor" means a qualified person acting alone or in consortium who is a party to a mineral agreement or to a financial or technical assistance agreement. 285 SEC. 34. Maximum Contract Area. The maximum contract area that may be granted per qualified person, subject to relinquishment shall be: (a) 1,000 meridional blocks onshore; (b) 4,000 meridional blocks offshore; or (c) Combinations of (a) and (b) provided that it shall not exceed the maximum limits for onshore and offshore areas. 286 SEC. 36. Negotiations. A financial or technical assistance agreement shall be negotiated by the Department and executed and approved by the President. The President shall notify Congress of all financial or technical assistance agreements within thirty (30) days from execution and approval thereof. 287 SEC. 37. Filing and Evaluation of Financial or Technical Assistance Agreement Proposals. All financial or technical assistance agreement proposals shall be filed with the Bureau after payment of the required processing fees. If the proposal is found to be sufficient and meritorious in form and substance after evaluation, it shall be recorded with the appropriate government agency to give the proponent the prior right to the area covered by such proposal: Provided, That existing mineral agreements, financial or technical assistance agreements and other mining rights are not impaired or prejudiced thereby. The Secretary shall recommend its approval to the President. 288 SEC. 38. Term of Financial or Technical Assistance Agreement. A financial or technical assistance agreement shall have a term not exceeding twenty-five (25) years to start from the execution thereof, renewable for not more than twenty-five (25) years under such terms and conditions as may be provided by law. 289 SEC. 40. Assignment/Transfer. A financial or technical assistance agreement may be assigned or transferred, in whole or in part, to a qualified person subject to the prior approval of the President: Provided, That the President shall notify Congress of every financial or technical assistance agreement assigned or converted in accordance with this provision within thirty (30) days from the date of the approval thereof. 290 SEC. 41. Withdrawal from Financial or Technical Assistance Agreement. The contractor shall manifest in writing to the Secretary his intention to withdraw from the agreement, if in his judgment the mining project is no longer economically feasible, even after he has exerted reasonable diligence to remedy the cause or the situation. The Secretary may accept the withdrawal: Provided, That the contractor has complied or satisfied all his financial, fiscal or legal obligations. 291 SEC. 81. Government Share in Other Mineral Agreements.x x x. The Government share in financial or technical assistance agreement shall consist of, among other things, the contractor's corporate income tax, excise tax, special allowance, withholding tax due from the contractor's foreign stockholders arising from dividend or interest payments to the said foreign stockholder in case of a foreign national and all such other taxes, duties and fees as provided for under existing laws. The collection of Government share in financial or technical assistance agreement shall commence after the financial or technical assistance agreement contractor has fully recovered its pre-operating expenses, exploration, and development expenditures, inclusive. 292 SEC. 90. Incentives.The contractors in mineral agreements, and financial or technical assistance agreements shall be entitled to the applicable fiscal and non-fiscal incentives as provided for under Executive Order No. 226, otherwise known as the Omnibus Investments Code of 1987: Provided, That holders of exploration permits may register with the Board of Investments and be entitled to the fiscal incentives granted under the said Code for the duration of the permits or extensions thereof: Provided, further, That mining activities shall always be included in the investment priorities plan. 293 Lidasan v. Commission on Elections, 21 SCRA 496 (1967). 294 Vide also WMCP FTAA, sec. 10.2 (a). 295 WMCP, sec. 10.2. 296 Id., sec. 11. 297 Id., sec. 10.1(a). 298 Id., sec. 10.1(c). 299 Id., sec. 6.4. 300 Rollo, pp. 563-564. 301 Civil Code, art. 8. 302 Const., art III, sec. 1. 303 Vide Note 223. 304 Rollo, p. 243. 305 Civil Liberties Union v. Executive Secretary, supra. 306 Automotive Parts & Equipment Company, Inc. v. Lingad, 30 SCRA 248 (1969). 307 Ibid.

L A W O N N A T U R A L R E S O U R C E S a n d E N V I R O N M E N T A L L A W C a s e s a n d S p e c i a l L a w s | 67
G.R. No. L-50464 January 29, 1990 SUNBEAM CONVENIENCE FOODS INC., CORAL BEACH DEVELOPMENT CORP., and the REGISTER OF DEEDS OF BATAAN, petitioners, vs. HON. COURT OF APPEALS and THE REPUBLIC OF THE PHILIPPINES, respondents. Filoteo T. Banzon for petitioners. 2. The title issued to SUNBEAM and CORAL BEACH had become indefeasible and imprescriptible; SARMIENTO, J.: In this petition for review on certiorari, Convenience Foods Corporation (hereafter simply SUNBEAM) and Coral Beach Development Corporation (hereafter simply CORAL BEACH) bring to our attention the decision rendered by the Court of Appeals in "Republic of the Philippines v. Hon. Pedro T. Santiago, et al.," disposing as follows: WHEREFORE, the writ prayed for is granted. The order of the respondent judge dated October 7, 1977, dismissing Civil Case No. 4062 is set aside, and respondent judge is ordered to require private respondents to file their answer to the complaint in said Civil Case No. 4062 and thereafter to proceed with the trial of the case on the merits and to render judgment thereon.' The following facts stated by the respondent Court in its decision and restated by the petitioners in their petition are accurate: (a) On April 29, 1963, the Director of Lands caused the issuance of a Sales Patent in favor of defendant Sunbeam Convenience Foods, Inc., over the parcels of land both situated in Mariveles, Bataan and more particularly described and bounded as follows: Lot 1-Sgs-2409 (area 3,113,695 sq. m ) Lot 2-Sgs-2409 area 1,401,855 sq. m (b) On May 3, 1963, the aforesaid Sales Patent was registered with the defendant Register of Deeds of Bataan who in turn issued Original Certificate of Title No. Sp-24 in favor of defendant Sunbeam Convenience Foods, Inc., for the two parcels of land above-described; (c) Subsequently, Original Certificate of Title No. Sp-24 was cancelled and in lieu thereof, Transfer Certificate of Title No. T-12421 was issued over Lot 1, Sgs-2409, while Transfer Certificate of Title No. 12422 was issued over Lot 2, Sgs-2409, both in favor of defendant Coral Beach Development Corporation I b. Finding that Lots I and 2 are alienable and disposable lands of the public domain under the jurisdiction of the Director of Lands despite clear and positive evidence to the contrary. c. Concluding that the complaint for reversion is defective as it was not initiated by the Director of Lands. d. Finding that the complaint for reversion states no cause of action for alleged failure of petitioner to 6 exhaust administrative remedies. The Court of Appeals gave due course to the petition for certiorari, set aside the Order of Dismissal rendered by the Court of First Instance in Civil Case No. 4062, and ordered the presiding judge Hon. Pedro 3. The action for reversion was defective, having been initiated by the Solicitor General and not by the 3 Director of Lands. The then Court of First Instance of Bataan dismissed the complaint in the Order of 4 October 7, 1977, adopting mainly the theory that since the titles sought to be cancelled emanated from the administrative act of the Bureau of Lands Director, the latter, not the courts, had jurisdiction over the disposition of the land. The Solicitor General received the copy of the Order on October 11, 1977 and filed a Notice of Appeal 5 dated October 25, 1977 . The Solicitor General then moved for an extension of thirty days within which to file the Record on Appeal and to pay the docket fee in order to perfect the appeal. This was to be followed by another motion for extension filed by the Solicitor General, resulting in the Court of Appeals granting the petitioner another extension of fifteen days from December 10, 1977. Finally before this period of extension lapsed, instead of an appeal, a petition for certiorari with the respondent Court of Appeals was filed. According to the Solicitor General, the Court of First Instance committed grave abuse of discretion in dismissing the complaint and in a. Not finding that since the lower court acted in a Motion to Dismiss, the correctness of its decision must be decided in the assumed truth and accuracy of the allegations of the complaint. The complaint alleges that the lands in question are forest lands; hence, inalienable. (d) On May 11, 1976, the Solicitor General in the name of the Republic of the Philippines instituted before 2 the Court of First Instance of Bataan, an action for reversion docketed as Civil Case No. 4062. SUNBEAM and CORAL BEACH filed a Motion to Dismiss on the following grounds: 1. The Republic of the Philippines should have exhausted all administrative remedies before filing the case in court;

L A W O N N A T U R A L R E S O U R C E S a n d E N V I R O N M E N T A L L A W C a s e s a n d S p e c i a l L a w s | 68
T. Santiago to receive the answers of the private respondents SUNBEAM and CORAL BEACH in the action for reversion. Hence Sunbeam and Coral Beach filed this petition for review. A review is not a matter of right but of sound judicial discretion, and is granted only when there are special and important reasons therefore. The following, while neither controlling nor fully measuring the Court's discretion, enumerates the premises for granting a review: (a) When the Court of Appeals has decided a question of substance, not theretofore determined by the Supreme Court or has decided it in a way probably not in accord with law or the applicable decisions of the Supreme Court; and (b) When the Court of Appeals has so far departed from the accepted and usual course of judicial 7 proceedings or so far sanctioned such departure by a lower court as to call for supervision . We agree with the Court of Appeals' granting of the petition filed by the Republic of the Philippines charging the then Court of First Instance with grave abuse of discretion. The filing of the Motion to Dismiss the complaint for reversion by SUNBEAM and CORAL BEACH on the ground of lack of cause of action, necessarily carried with it the admission, for purposes of the motion, of the truth of all material facts pleaded in the complaint instituted by the Republic. An important factual issue raised in the complaint was the classification of the lands as forest lands. This 9 material allegation stated in the Republic's complaint' was never denied specifically by the defendants (petitioners herein) SUNBEAM and CORAL BEACH. If it is true that the lands are forest lands, then all these proceedings become moot and academic. Land 10 remains unclassified land until it is released therefrom and rendered open to disposition. Our adherence to the Regalian doctrine subjects all agricultural, timber, and mineral lands to the 11 dominion of the State. Thus, before any land may be declassified from the forest group and converted into alienable or disposable land for agricultural or other purposes, there must be a positive act from the government. Even rules on the confirmation of imperfect titles do not apply unless and until the land classified as forest land is released in an official proclamation to that effect so that it may form part of the 12 disposable agricultural lands of the public domain. The mere fact that a title was issued by the Director of Lands does not confer any validity on such title if 13 the property covered by the title or patent is part of the public forest. The only way to resolve this question of fact as to the classification of the land is by remanding the case to the lower court for a full- dress trial on the issues involved. Generally, the rules of procedure must be observed so that the efficient administration of justice is ensured. However, the rules of procedure should be viewed as mere tools designed to facilitate the 14 attainment of justice. They must lead to the proper and just determination of litigation, without tying the hands of the law or making it indifferent to realities. Certiorari is one such remedy. Considered extraordinary, it is made available only when there is no 15 appeal, nor any plain, speedy or adequate remedy in the ordinary course of the law. The long line of decisions denying the petition for certiorari, either before appeal was availed of or specially in instances where the appeal period has lapsed, far outnumbers the instances when certiorari was given due course. The few significant exceptions were: when public welfare and the advancement of public policy dictate; or 16 when the broader interests of justice so require, or when the writs issued are null, or when the 17 questioned order amounts to an oppressive exercise of judicial authority. We find nothing disagreeable with the action of the Court of Appeals to give due course to the petition considering that the issue affected a matter of public concern which is the disposition of the lands of our matrimony No less than the Constitution protects its policy. We therefore find no compelling reason to disturb the findings of the appellate court, in the absence of a clear showing that the Court of Appeals has decided a question of substance in a manner inconsistent with jurisprudence, or that the respondent Court has departed from the accepted and usual course of 18 judicial proceedings. In sum, no reversible error has been committed by the respondent court. WHEREFORE, the petition is DENIED and the decision of the Court of Appeals is affirmed. Costs against the petitioners. SO ORDERED. Melencio-Herrera (Chairperson), Paras, Padilla and Regalado, JJ., concur.

L A W O N N A T U R A L R E S O U R C E S a n d E N V I R O N M E N T A L L A W C a s e s a n d S p e c i a l L a w s | 69
G.R. No. 124293 January 31, 2005 PNB, the National Government's shareholdings in PHILSECO increased to 97.41% thereby reducing KAWASAKI's shareholdings to 2.59%. In the interest of the national economy and the government, the COP and the APT deemed it best to sell the National Government's share in PHILSECO to private entities. After a series of negotiations between the APT and KAWASAKI, they agreed that the latter's right of first refusal under the JVA be "exchanged" for the right to top by five percent (5%) the highest bid for the said shares. They further agreed that KAWASAKI would be entitled to name a company in which it was a stockholder, which could exercise the 1 right to top. On September 7, 1990, KAWASAKI informed APT that Philyards Holdings, Inc. (PHI) would exercise its right to top. At the pre-bidding conference held on September 18, 1993, interested bidders were given copies of the JVA between NIDC and KAWASAKI, and of the Asset Specific Bidding Rules (ASBR) drafted for the National Government's 87.6% equity share in PHILSECO. The provisions of the ASBR were explained to the interested bidders who were notified that the bidding would be held on December 2, 1993. A portion of the ASBR reads: 1.0 The subject of this Asset Privatization Trust (APT) sale through public bidding is the National Government's equity in PHILSECO consisting of 896,869,942 shares of stock (representing 87.67% of PHILSECO's outstanding capital stock), which will be sold as a whole block in accordance with the rules herein enumerated. xxx xxx xxx 2.0 The highest bid, as well as the buyer, shall be subject to the final approval of both the APT Board of Trustees and the Committee on Privatization (COP). 2.1 APT reserves the right in its sole discretion, to reject any or all bids. 1.4 Neither party shall sell, transfer or assign all or any part of its interest in SNS [PHILSECO] to any third party without giving the other under the same terms the right of first refusal. This provision shall not apply if the transferee is a corporation owned or controlled by the GOVERNMENT or by a KAWASAKI affiliate. On November 25, 1986, NIDC transferred all its rights, title and interest in PHILSECO to the Philippine National Bank (PNB). Such interests were subsequently transferred to the National Government pursuant to Administrative Order No. 14. On December 8, 1986, President Corazon C. Aquino issued Proclamation No. 50 establishing the Committee on Privatization (COP) and the Asset Privatization Trust (APT) to take title to, and possession of, conserve, manage and dispose of non-performing assets of the National Government. Thereafter, on February 27, 1987, a trust agreement was entered into between the National Government and the APT wherein the latter was named the trustee of the National Government's share in PHILSECO. In 1989, as a result of a quasi-reorganization of PHILSECO to settle its huge obligations to 3.0 This public bidding shall be on an Indicative Price Bidding basis. The Indicative price set for the National Government's 87.67% equity in PHILSECO is PESOS: ONE BILLION THREE HUNDRED MILLION (P1,300,000,000.00). xxx xxx xxx 6.0 The highest qualified bid will be submitted to the APT Board of Trustees at its regular meeting following the bidding, for the purpose of determining whether or not it should be endorsed by the APT Board of Trustees to the COP, and the latter approves the same. The APT shall advise Kawasaki Heavy Industries, Inc. and/or its nominee, [PHILYARDS] Holdings, Inc., that the highest bid is acceptable to the National Government. Kawasaki Heavy Industries, Inc. and/or [PHILYARDS] Holdings, Inc. shall then have a period of thirty (30) calendar days from the date of receipt of such advice from APT within which to

J.G. SUMMIT HOLDINGS, INC., petitioner, vs. COURT OF APPEALS; COMMITTEE ON PRIVATIZATION, its Chairman and Members; ASSET PRIVATIZATION TRUST; and PHILYARDS HOLDINGS, INC., respondents. RESOLUTION PUNO, J.: For resolution before this Court are two motions filed by the petitioner, J.G. Summit Holdings, Inc. for reconsideration of our Resolution dated September 24, 2003 and to elevate this case to the Court En Banc. The petitioner questions the Resolution which reversed our Decision of November 20, 2000, which in turn reversed and set aside a Decision of the Court of Appeals promulgated on July 18, 1995. I. Facts The undisputed facts of the case, as set forth in our Resolution of September 24, 2003, are as follows: On January 27, 1997, the National Investment and Development Corporation (NIDC), a government corporation, entered into a Joint Venture Agreement (JVA) with Kawasaki Heavy Industries, Ltd. of Kobe, Japan (KAWASAKI) for the construction, operation and management of the Subic National Shipyard, Inc. (SNS) which subsequently became the Philippine Shipyard and Engineering Corporation (PHILSECO). Under the JVA, the NIDC and KAWASAKI will contribute P330 million for the capitalization of PHILSECO in the proportion of 60%-40% respectively. One of its salient features is the grant to the parties of the right of first refusal should either of them decide to sell, assign or transfer its interest in the joint venture, viz:

L A W O N N A T U R A L R E S O U R C E S a n d E N V I R O N M E N T A L L A W C a s e s a n d S p e c i a l L a w s | 70
exercise their "Option to Top the Highest Bid" by offering a bid equivalent to the highest bid plus five (5%) percent thereof. 6.1 Should Kawasaki Heavy Industries, Inc. and/or [PHILYARDS] Holdings, Inc. exercise their "Option to Top the Highest Bid," they shall so notify the APT about such exercise of their option and deposit with APT the amount equivalent to ten percent (10%) of the highest bid plus five percent (5%) thereof within the thirty (30)-day period mentioned in paragraph 6.0 above. APT will then serve notice upon Kawasaki Heavy Industries, Inc. and/or [PHILYARDS] Holdings, Inc. declaring them as the preferred bidder and they shall have a period of ninety (90) days from the receipt of the APT's notice within which to pay the balance of their bid price. 6.2 Should Kawasaki Heavy Industries, Inc. and/or [PHILYARDS] Holdings, Inc. fail to exercise their "Option to Top the Highest Bid" within the thirty (30)-day period, APT will declare the highest bidder as the winning bidder. xxx xxx xxx 12.0 The bidder shall be solely responsible for examining with appropriate care these rules, the official bid forms, including any addenda or amendments thereto issued during the bidding period. The bidder shall likewise be responsible for informing itself with respect to any and all conditions concerning the PHILSECO Shares which may, in any manner, affect the bidder's proposal. Failure on the part of the bidder to so examine and inform itself shall be its sole risk and no relief for error or omission will be given by APT or COP. . . . At the public bidding on the said date, petitioner J.G. Summit Holdings, Inc. submitted a bid of Two Billion and Thirty Million Pesos (P2,030,000,000.00) with an acknowledgment of KAWASAKI/[PHILYARDS'] right to top, viz: 4. I/We understand that the Committee on Privatization (COP) has up to thirty (30) days to act on APT's recommendation based on the result of this bidding. Should the COP approve the highest bid, APT shall advise Kawasaki Heavy Industries, Inc. and/or its nominee, [PHILYARDS] Holdings, Inc. that the highest bid is acceptable to the National Government. Kawasaki Heavy Industries, Inc. and/or [PHILYARDS] Holdings, Inc. shall then have a period of thirty (30) calendar days from the date of receipt of such advice from APT within which to exercise their "Option to Top the Highest Bid" by offering a bid equivalent to the highest bid plus five (5%) percent thereof. As petitioner was declared the highest bidder, the COP approved the sale on December 3, 1993 "subject to the right of Kawasaki Heavy Industries, Inc./[PHILYARDS] Holdings, Inc. to top JGSMI's bid by 5% as specified in the bidding rules."
2

On December 29, 1993, petitioner informed APT that it was protesting the offer of PHI to top its bid on the grounds that: (a) the KAWASAKI/PHI consortium composed of KAWASAKI, [PHILYARDS], Mitsui, Keppel, SM Group, ICTSI and Insular Life violated the ASBR because the last four (4) companies were the losing bidders thereby circumventing the law and prejudicing the weak winning bidder; (b) only KAWASAKI could exercise the right to top; (c) giving the same option to top to PHI constituted unwarranted benefit to a third party; (d) no right of first refusal can be exercised in a public bidding or auction sale; and (e) the JG Summit consortium was not estopped from questioning the proceedings. On February 2, 1994, petitioner was notified that PHI had fully paid the balance of the purchase price of the subject bidding. On February 7, 1994, the APT notified petitioner that PHI had exercised its option to top the highest bid and that the COP had approved the same on January 6, 1994. On February 24, 1994, the APT and PHI executed a Stock Purchase Agreement. Consequently, petitioner filed with this Court a Petition for Mandamus under G.R. No. 114057. On May 11, 1994, said petition was referred to the Court of Appeals. On July 18, 1995, the Court of Appeals denied the same for lack of merit. It ruled that the petition for mandamus was not the proper remedy to question the constitutionality or legality of the right of first refusal and the right to top that was exercised by KAWASAKI/PHI, and that the matter must be brought "by the proper party in the proper forum at the proper time and threshed out in a full blown trial." The Court of Appeals further ruled that the right of first refusal and the right to top are prima facie legal and that the petitioner, "by participating in the public bidding, with full knowledge of the right to top granted to KAWASAKI/*PHILYARDS+ isestopped from questioning the validity of the award given to [PHILYARDS] after the latter exercised the right to top and had paid in full the purchase price of the subject shares, pursuant to the ASBR." Petitioner filed a Motion for Reconsideration of said Decision which was denied on March 15, 1996. Petitioner thus filed a Petition for Certiorari with this Court alleging grave abuse of discretion on the part of the appellate court. On November 20, 2000, this Court rendered x x x [a] Decision ruling among others that the Court of Appeals erred when it dismissed the petition on the sole ground of the impropriety of the special civil action of mandamus because the petition was also one of certiorari. It further ruled that a shipyard like PHILSECO is a public utility whose capitalization must be sixty percent (60%) Filipino-owned. Consequently, the right to top granted to KAWASAKI under the Asset Specific Bidding Rules (ASBR) drafted for the sale of the 87.67% equity of the National Government in PHILSECO is illegal not only because it violates the rules on competitive bidding but more so, because it allows foreign corporations to own more than 40% equity in the shipyard. It also held that "although the petitioner had the opportunity to examine the ASBR before it participated in the bidding, it cannot be estopped from questioning the unconstitutional, illegal and inequitable provisions thereof." Thus, this Court voided the transfer of the national government's 87.67% share in PHILSECO to Philyard[s] Holdings, Inc., and upheld the right of JG Summit, as the highest bidder, to take title to the said shares, viz: WHEREFORE, the instant petition for review on certiorari is GRANTED. The assailed Decision and Resolution of the Court of Appeals are REVERSED and SET ASIDE. Petitioner is ordered to pay to APT its bid price of Two Billion Thirty Million Pesos (P2,030,000,000.00), less its bid deposit plus interests upon the finality of this Decision. In turn, APT is ordered to:

L A W O N N A T U R A L R E S O U R C E S a n d E N V I R O N M E N T A L L A W C a s e s a n d S p e c i a l L a w s | 71
(a) accept the said amount of P2,030,000,000.00 less bid deposit and interests from petitioner; (b) execute a Stock Purchase Agreement with petitioner; Motion to Elevate this Case to the (c) cause the issuance in favor of petitioner of the certificates of stocks representing 87.6% of PHILSECO's total capitalization; (d) return to private respondent PHGI the amount of Two Billion One Hundred Thirty-One Million Five Hundred Thousand Pesos (P2,131,500,000.00); and (e) cause the cancellation of the stock certificates issued to PHI. SO ORDERED. In separate Motions for Reconsideration, respondents submit[ted] three basic issues for x x x resolution: (1) Whether PHILSECO is a public utility; (2) Whether under the 1977 JVA, KAWASAKI can exercise its right of first refusal only up to 40% of the total capitalization of PHILSECO; and (3) Whether the right to top 3 granted to KAWASAKI violates the principles of competitive bidding. (citations omitted) In a Resolution dated September 24, 2003, this Court ruled in favor of the respondents. On the first issue, we held that Philippine Shipyard and Engineering Corporation (PHILSECO) is not a public utility, as by 4 5 nature, a shipyard is not a public utility and that no law declares a shipyard to be a public utility. On the second issue, we found nothing in the 1977 Joint Venture Agreement (JVA) which prevents Kawasaki Heavy Industries, Ltd. of Kobe, Japan (KAWASAKI) from acquiring more than 40% of PHILSECOs total 6 capitalization. On the final issue, we held that the right to top granted to KAWASAKI in exchange for its 7 right of first refusal did not violate the principles of competitive bidding. On October 20, 2003, the petitioner filed a Motion for Reconsideration and a Motion to Elevate This Case 9 to the Court En Banc. Public respondents Committee on Privatization (COP) and Asset Privatization Trust (APT), and private respondent Philyards Holdings, Inc. (PHILYARDS) filed their Comments on J.G. Summit Holdings, Inc.s (JG Summits) Motion for Reconsideration and Motion to Elevate This Case to the Court En Banc on January 29, 2004 and February 3, 2004, respectively. II. Issues Based on the foregoing, the relevant issues to resolve to end this litigation are the following: 1. Whether there are sufficient bases to elevate the case at bar to the Court en banc.
8

2. Whether the motion for reconsideration raises any new matter or cogent reason to warrant a reconsideration of this Courts Resolution of September 24, 2003.

Court En Banc The petitioner prays for the elevation of the case to the Court en banc on the following grounds: 1. The main issue of the propriety of the bidding process involved in the present case has been confused with the policy issue of the supposed fate of the shipping industry which has never 10 been an issue that is determinative of this case. 2. The present case may be considered under the Supreme Court Resolution dated February 23, 1984 which included among en banc cases those involving a novel question of law and those where a doctrine or principle laid down by the Court en banc or in division may be modified or 11 reversed. 3. There was clear executive interference in the judicial functions of the Court when the Honorable Jose Isidro Camacho, Secretary of Finance, forwarded to Chief Justice Davide, a memorandum dated November 5, 2001, attaching a copy of the Foreign Chambers Report dated October 17, 2001, which matter was placed in the agenda of the Court and noted by it in a formal 12 resolution dated November 28, 2001. Opposing J.G. Summits motion to elevate the case en banc, PHILYARDS points out the petitioners inconsistency in previously opposing PHILYARDS Motion to Refer the Case to the Court En Banc. PHILYARDS contends that J.G. Summit should now be estopped from asking that the case be referred to the Court en banc. PHILYARDS further contends that the Supreme Court en banc is not an appellate court to which decisions or resolutions of its divisions may be appealed citing Supreme Court 13 Circular No. 2-89 dated February 7, 1989. PHILYARDS also alleges that there is no novel question of law involved in the present case as the assailed Resolution was based on well-settled jurisprudence. Likewise, PHILYARDS stresses that the Resolution was merely an outcome of the motions for reconsideration filed by it and the COP and APT and is "consistent with t he inherent power of courts to amend and control its 14 process and orders so as to make them conformable to law and justice. (Rule 135, sec. 5)" Private respondent belittles the petitioners allegations regarding the change in ponente and the alleged executive interference as shown by former Secretary of Finance Jose Isidro Camachos memorandum dated November 5, 2001 arguing that these do not justify a referral of the present case to the Court en banc. In insisting that its Motion to Elevate This Case to the Court En Banc should be granted, J.G. Summit further argued that: its Opposition to the Office of the Solicitor Generals Motion to Refer is different from

L A W O N N A T U R A L R E S O U R C E S a n d E N V I R O N M E N T A L L A W C a s e s a n d S p e c i a l L a w s | 72
its own Motion to Elevate; different grounds are invoked by the two motions; there was unwarranted "executive interference"; and the change in ponente is merely noted in asserting that this case should be 15 decided by the Court en banc. We find no merit in petitioners contention that the propriety of the bidding process involved in the present case has been confused with the policy issue of the fate of the shipping industry which, petitioner maintains, has never been an issue that is determinative of this case. The Courts Resolution of September 24, 2003 reveals a clear and definitive ruling on the propriety of the bidding process. In discussing whether the right to top granted to KAWASAKI in exchange for its right of first refusal violates the principles of competitive bidding, we made an exhaustive discourse on the rules and principles of public 16 bidding and whether they were complied with in the case at bar. This Court categorically ruled on the petitioners argument that PHILSECO, as a shipyard, is a public utility which should maintain a 60% -40% Filipino-foreign equity ratio, as it was a pivotal issue. In doing so, we recognized the impact of our ruling 17 on the shipbuilding industry which was beyond avoidance. We reject petitioners argument that the present case may be considered under the Supreme Court Resolution dated February 23, 1984 which included among en banc cases those involving a novel question of law and those where a doctrine or principle laid down by the court en banc or in division may be modified or reversed. The case was resolved based on basic principles of the right of first refusal in commercial law and estoppel in civil law. Contractual obligations arising from rights of first refusal are not 18 new in this jurisdiction and have been recognized in numerous cases. Estoppel is too known a civil law concept to require an elongated discussion. Fundamental principles on public bidding were likewise used to resolve the issues raised by the petitioner. To be sure, petitioner leans on the right to top in a public bidding in arguing that the case at bar involves a novel issue. We are not swayed. The right to top was merely a condition or a reservation made in the bidding rules which was fully disclosed to all bidding 19 parties. In Bureau Veritas, represented by Theodor H. Hunermann v. Office of the President, et al ., we dealt with this conditionality, viz: x x x It must be stressed, as held in the case of A.C. Esguerra & Sons v. Aytona, et al., (L-18751, 28 April 1962, 4 SCRA 1245), that in an "invitation to bid, there is a condition imposed upon the bidders to the effect that the bidding shall be subject to the right of the government to reject any and all bids subject to its discretion. In the case at bar, the government has made its choice and unless an unfairness or injustice is shown, the losing bidders have no cause to complain nor right to dispute that choice. This is a well-settled doctrine in this jurisdiction and elsewhere." The discretion to accept or reject a bid and award contracts is vested in the Government agencies entrusted with that function. The discretion given to the authorities on this matter is of such wide latitude that the Courts will not interfere therewith, unless it is apparent that it is used as a shield to a fraudulent award (Jalandoni v. NARRA, 108 Phil. 486 [1960]). x x x The exercise of this discretion is a policy decision that necessitates prior inquiry, investigation, comparison, evaluation, and deliberation. This task can best be discharged by the Government agencies concerned, not by the Courts. The role of the Courts is to ascertain whether a branch or instrumentality of the Government has transgressed its constitutional boundaries. But the Courts will not interfere with executive or legislative discretion exercised within those boundaries. Otherwise, it strays into the realm of policy decision-making. It is only upon a clear showing of grave abuse of discretion that the Courts will set aside the award of a contract made by a government entity. Grave abuse of discretion implies a capricious, arbitrary and whimsical exercise of power (Filinvest Credit Corp. v. Intermediate Appellate Court, No. 65935, 30 September 1988, 166 SCRA 155). The abuse of discretion must be so patent and gross as to amount to an evasion of positive duty or to a virtual refusal to perform a duty enjoined by law, as to act at all in contemplation of law, where the power is exercised in an arbitrary and despotic manner by reason of passion or hostility (Litton Mills, Inc. v. Galleon Trader, Inc., et al[.], L-40867, 26 July 1988, 163 SCRA 489). The facts in this case do not indicate any such grave abuse of discretion on the part of public respondents when they awarded the CISS contract to Respondent SGS. In the "Invitation to Prequalify and Bid" (Annex "C," supra), the CISS Committee made an express reservation of the right of the Government to "reject any or all bids or any part thereof or waive any defects contained thereon and accept an offer most advantageous to the Government." It is a well-settled rule that where such reservation is made in an Invitation to Bid, the highest or lowest bidder, as the case may be, is not entitled to an award as a matter of right (C & C Commercial Corp. v. Menor, L-28360, 27 January 1983, 120 SCRA 112). Even the lowest Bid or any Bid may be rejected or, in the exercise of sound discretion, the award may be made to another than the lowest bidder (A.C. Esguerra & Sons v. Aytona, supra, citing 43 Am. Jur., 788). (emphases supplied)1awphi1.nt Like the condition in the Bureau Veritas case, the right to top was a condition imposed by the government in the bidding rules which was made known to all parties. It was a condition imposed on all bidders equally, based on the APTs exercise of its discretion in deciding on how best to privatize the governments shares in PHILSECO. It was not a whimsical or arbitrary condition plucked from the ether and inserted in the bidding rules but a condition which the APT approved as the best way the government could comply with its contractual obligations to KAWASAKI under the JVA and its mandate of getting the most advantageous deal for the government. The right to top had its history in the mutual right of first refusal in the JVA and was reached by agreement of the government and KAWASAKI. Further, there is no "executive interference" in the functions of this Court by the mere filing of a memorandum by Secretary of Finance Jose Isidro Camacho. The memorandum was merely "noted" to acknowledge its filing. It had no further legal significance. Notably too, the assailed Resolution dated September 24, 2003 was decided unanimously by the Special First Division in favor of the respondents . Again, we emphasize that a decision or resolution of a Division is that of the Supreme Court and the 21 Court en banc is not an appellate court to which decisions or resolutions of a Division may be appealed. For all the foregoing reasons, we find no basis to elevate this case to the Court en banc.
20

L A W O N N A T U R A L R E S O U R C E S a n d E N V I R O N M E N T A L L A W C a s e s a n d S p e c i a l L a w s | 73
Motion for Reconsideration Three principal arguments were raised in the petitioners Motion for Reconsideration. First, that a fair resolution of the case should be based on contract law, not on policy considerations; the contracts do not 22 authorize the right to top to be derived from the right of first refusal. Second, that neither the right of first refusal nor the right to top can be legally exercised by the consortium which is not the proper party 23 granted such right under either the JVA or the Asset Specific Bidding Rules (ASBR). Third, that the maintenance of the 60%-40% relationship between the National Investment and Development Corporation (NIDC) and KAWASAKI arises from contract and from the Constitution because PHILSECO is a landholding corporation and need not be a public utility to be bound by the 60%-40% constitutional 24 limitation. On the other hand, private respondent PHILYARDS asserts that J.G. Summit has not been able to show 25 compelling reasons to warrant a reconsideration of the Decision of the Court. PHILYARDS denies that the Decision is based mainly on policy considerations and points out that it is premised on principles governing obligations and contracts and corporate law such as the rule requiring respect for contractual stipulations, upholding rights of first refusal, and recognizing the assignable nature of contracts 26 rights. Also, the ruling that shipyards are not public utilities relies on established case law and fundamental rules of statutory construction. PHILYARDS stresses that KAWASAKIs right of first refusal or 27 even the right to top is not limited to the 40% equity of the latter. On the landholding issue raised by J.G. Summit, PHILYARDS emphasizes that this is a non-issue and even involves a question of fact. Even assuming that this Court can take cognizance of such question of fact even without the benefit of a trial, PHILYARDS opines that landholding by PHILSECO at the time of the bidding is irrelevant because what is essential is that ultimately a qualified entity would eventually hold PHILSECOs real estate 28 properties. Further, given the assignable nature of the right of first refusal, any applicable nationality restrictions, including landholding limitations, would not affect the right of first refusal itself, but only the 29 manner of its exercise. Also, PHILYARDS argues that if this Court takes cognizance of J.G. Summits allegations of fact regarding PHILSECOs landholding, it must also recognize PHILYARDS assertions that 30 PHILSECOs landholdings were sold to another corporation. As regards the right of first refusal, private respondent explains that KAWASAKIs reduced shareholdings (from 40% to 2.59%) did not translate to a 31 deprivation or loss of its contractually granted right of first refusal. Also, the bidding was valid because PHILYARDS exercised the right to top and it was of no moment that losing bidders later joined PHILYARDS 32 in raising the purchase price. In cadence with the private respondent PHILYARDS, public respondents COP and APT contend: b. The landholding issue does not pose questions of fact. 1. The conversion of the right of first refusal into a right to top by 5% does not violate any provision in the JVA between NIDC and KAWASAKI. 2. PHILSECO is not a public utility and therefore not governed by the constitutional restriction on foreign ownership. c. That PHILSECO owned land at the time that the right of first refusal was agreed upon and at the time of the bidding are most relevant. d. Whether a shipyard is a public utility is not the core issue in this case. 3. The petitioner is legally estopped from assailing the validity of the proceedings of the public bidding as it voluntarily submitted itself to the terms of the ASBR which included the provision on the right to top. 4. The right to top was exercised by PHILYARDS as the nominee of KAWASAKI and the fact that PHILYARDS formed a consortium to raise the required amount to exercise the right to top the highest bid by 5% does not violate the JVA or the ASBR. 5. The 60%-40% Filipino-foreign constitutional requirement for the acquisition of lands does not apply to PHILSECO because as admitted by petitioner itself, PHILSECO no longer owns real property. 6. Petitioners motion to elevate the case to the Court en banc is baseless and would only delay 33 the termination of this case. In a Consolidated Comment dated March 8, 2004, J.G. Summit countered the arguments of the public and private respondents in this wise: 1. The award by the APT of 87.67% shares of PHILSECO to PHILYARDS with losing bidders through the exercise of a right to top, which is contrary to law and the constitution is null and void for being violative of substantive due process and the abuse of right provision in the Civil Code. a. The bidders*+ right to top was actually exercised by losing bidders. b. The right to top or the right of first refusal cannot co-exist with a genuine competitive bidding. c. The benefits derived from the right to top were unwarranted. 2. The landholding issue has been a legitimate issue since the start of this case but is shamelessly ignored by the respondents. a. The landholding issue is not a non-issue.

L A W O N N A T U R A L R E S O U R C E S a n d E N V I R O N M E N T A L L A W C a s e s a n d S p e c i a l L a w s | 74
3. Fraud and bad faith attend the alleged conversion of an inexistent right of first refusal to the right to top. a. The history behind the birth of the right to top shows fraud and bad faith. b. The right of first refusal was, indeed, "effectively useless." 4. Petitioner is not legally estopped to challenge the right to top in this case. a. Estoppel is unavailing as it would stamp validity to an act that is prohibited by law or against public policy. b. Deception was patent; the right to top was an attractive nuisance. c. The 10% bid deposit was placed in escrow. J.G. Summits insistence that the right to top cannot be sourced from the right of first refusal is not new and we have already ruled on the issue in our Resolution of September 24, 2003. We upheld the mutual 34 right of first refusal in the JVA. We also ruled that nothing in the JVA prevents KAWASAKI from acquiring 35 more than 40% of PHILSECOs total capitalization. Likewise, nothing in the JVA or ASBR bars the conversion of the right of first refusal to the right to top. In sum, nothing new and of significance in the petitioners pleading warrants a reconsideration of our ruling. Likewise, we already disposed of the argument that neither the right of first refusal nor the right to top can legally be exercised by the consortium which is not the proper party granted such right under either the JVA or the ASBR. Thus, we held: The fact that the losing bidder, Keppel Consortium (composed of Keppel, SM Group, Insular Life Assurance, Mitsui and ICTSI), has joined PHILYARDS in the latter's effort to raise P2.131 billion necessary in exercising the right to top is not contrary to law, public policy or public morals. There is nothing in the ASBR that bars the losing bidders from joining either the winning bidder (should the right to top is not exercised) or KAWASAKI/PHI (should it exercise its right to top as it did), to raise the purchase price. The petitioner did not allege, nor was it shown by competent evidence, that the participation of the losing bidders in the public bidding was done with fraudulent intent. Absent any proof of fraud, the formation by [PHILYARDS] of a consortium is legitimate in a free enterprise system. The appellate court is thus correct in holding the petitioner estopped from questioning the validity of the transfer of the National 36 Government's shares in PHILSECO to respondent. Further, we see no inherent illegality on PHILYARDS act in seeking funding from parties who were losing bidders. This is a purely commercial decision over which the State should not interfere absent any legal infirmity. It is emphasized that the case at bar involves the disposition of shares in a corporation which the government sought to privatize. As such, the persons with whom PHILYARDS desired to enter into business with in order to raise funds to purchase the shares are basically its business. This is in contrast to a case involving a contract for the operation of or construction of a government infrastructure where the identity of the buyer/bidder or financier constitutes an important consideration. In such cases, the government would have to take utmost precaution to protect public interest by ensuring that the parties with which it is contracting have the ability to satisfactorily construct or operate the infrastructure. On the landholding issue, J.G. Summit submits that since PHILSECO is a landholding company, KAWASAKI could exercise its right of first refusal only up to 40% of the shares of PHILSECO due to the constitutional prohibition on landholding by corporations with more than 40% foreign-owned equity. It further argues that since KAWASAKI already held at least 40% equity in PHILSECO, the right of first refusal was inutile and 37 as such, could not subsequently be converted into the right to top. Petitioner also asserts that, at present, PHILSECO continues to violate the constitutional provision on landholdings as its shares are more 38 than 40% foreign-owned. PHILYARDS admits that it may have previously held land but had already 39 divested such landholdings. It contends, however, that even if PHILSECO owned land, this would not affect the right of first refusal but only the exercise thereof. If the land is retained, the right of first refusal, being a property right, could be assigned to a qualified party. In the alternative, the land could be divested before the exercise of the right of first refusal. In the case at bar, respondents assert that since the right of first refusal was validly converted into a right to top, which was exercised not by KAWASAKI, but by PHILYARDS which is a Filipino corporation (i.e., 60% of its shares are owned by Filipinos), then there is no 40 violation of the Constitution. At first, it would seem that questions of fact beyond cognizance by this Court were involved in the issue. However, the records show that PHILYARDS admits it had owned land 41 up until the time of the bidding. Hence, the only issue is whether KAWASAKI had a valid right of first refusal over PHILSECO shares under the JVA considering that PHILSECO owned land until the time of the bidding and KAWASAKI already held 40% of PHILSECOs equity. We uphold the validity of the mutual rights of first refusal under the JVA between KAWASAKI and NIDC. First of all, the right of first refusal is a property right of PHILSECO shareholders, KAWASAKI and NIDC, under the terms of their JVA. This right allows them to purchase the shares of their co-shareholder before they are offered to a third party. The agreement of co-shareholders to mutually grant this right to each other, by itself, does not constitute a violation of the provisions of the Constitution limiting land ownership to Filipinos and Filipino corporations. As PHILYARDS correctly puts it, if PHILSECO still owns land, the right of first refusal can be validly assigned to a qualified Filipino entity in order to maintain the 60%-40% ratio. This transfer, by itself, does not amount to a violation of the Anti-Dummy Laws, absent proof of any fraudulent intent. The transfer could be made either to a nominee or such other party which the holder of the right of first refusal feels it can comfortably do business with. Alternatively, PHILSECO may divest of its landholdings, in which case KAWASAKI, in exercising its right of first refusal, can exceed 40% of PHILSECOs equity. In fact, it can even be said that if the foreign shareholdings of a landholding corporation exceeds 40%, it is not the foreign stockholde rs ownership of the shares which is adversely affected but the capacity of the corporation to own land that is, the corporation becomes disqualified to own land. This finds support under the basic corporate law principle that the corporation and its stockholders are separate juridical entities. In this vein, the right of first refusal over shares pertains to the

L A W O N N A T U R A L R E S O U R C E S a n d E N V I R O N M E N T A L L A W C a s e s a n d S p e c i a l L a w s | 75
shareholders whereas the capacity to own land pertains to the corporation. Hence, the fact that PHILSECO owns land cannot deprive stockholders of their right of first refusal. No law disqualifies a person from purchasing shares in a landholding corporation even if the latter will exceed the allowed foreign equity, what the law disqualifies is the corporation from owning land . This is the clear import of the following provisions in the Constitution: Section 2. All lands of the public domain, waters, minerals, coal, petroleum, and other mineral oils, all forces of potential energy, fisheries, forests or timber, wildlife, flora and fauna, and other natural resources are owned by the State. With the exception of agricultural lands, all other natural resources shall not be alienated. The exploration, development, and utilization of natural resources shall be under the full control and supervision of the State. The State may directly undertake such activities, or it may enter into co-production, joint venture, or production-sharing agreements with Filipino citizens, or corporations or associations at least sixty per centum of whose capital is owned by such citizens. Such agreements may be for a period not exceeding twenty-five years, renewable for not more than twentyfive years, and under such terms and conditions as may be provided by law. In cases of water rights for irrigation, water supply, fisheries, or industrial uses other than the development of water power, beneficial use may be the measure and limit of the grant. xxx xxx xxx Section 7. Save in cases of hereditary succession, no private lands shall be transferred or conveyed except to individuals, corporations, or associations qualified to acquire or hold lands of the public 42 domain. (emphases supplied) The petitioner further argues that "an option to buy land is void in itself (Philippine Banking Corporation v. Lui She, 21 SCRA 52 [1967]). The right of first refusal granted to KAWASAKI, a Japanese corporation, is 43 similarly void. Hence, the right to top, sourced from the right of first refusal, is also void." Contrary to the contention of petitioner, the case of Lui She did not that say "an option to buy land is void in itself," for we ruled as follows: x x x To be sure, a lease to an alien for a reasonable period is valid. So is an option giving an alien the right to buy real property on condition that he is granted Philippine citizenship. As this Court said in Krivenko vs. Register of Deeds: [A]liens are not completely excluded by the Constitution from the use of lands for residential purposes. Since their residence in the Philippines is temporary, they may be granted temporary rights such as a lease contract which is not forbidden by the Constitution. Should they desire to remain here forever and share our fortunes and misfortunes, Filipino citizenship is not impossible to acquire. But if an alien is given not only a lease of, but also an option to buy, a piece of land, by virtue of which the Filipino owner cannot sell or otherwise dispose of his property, this to last for 50 years, then it becomes clear that the arrangement is a virtual transfer of ownership whereby the owner divests himself in stages not only of the right to enjoy the land (jus possidendi, jus utendi, jus fruendi and jus abutendi) but also of the right to dispose of it (jus disponendi) rights the sum total of which make up ownership. It is just as if today the possession is transferred, tomorrow, the use, the next day, the disposition, and so on, until ultimately all the rights of which ownership is made up are consolidated in an alien. And yet this is just exactly what the parties in this case did within this pace of one year, with the result that Justina Santos'[s] ownership of her property was reduced to a hollow concept. If this can be done, then the Constitutional ban against alien landholding in the Philippines, as announced in Krivenko 44 vs. Register of Deeds, is indeed in grave peril. (emphases supplied; Citations omitted) In Lui She, the option to buy was invalidated because it amounted to a virtual transfer of ownership as the owner could not sell or dispose of his properties. The contract in Lui She prohibited the owner of the land from selling, donating, mortgaging, or encumbering the property during the 50-year period of the option to buy. This is not so in the case at bar where the mutual right of first refusal in favor of NIDC and KAWASAKI does not amount to a virtual transfer of land to a non-Filipino. In fact, the case at bar involves a right of first refusal over shares of stockwhile the Lui She case involves an option to buy the land itself. As discussed earlier, there is a distinction between the shareholders ownership of shares and the corporations ownership of land arising from the separate juridical personalities of the corporation and its shareholders. We note that in its Motion for Reconsideration, J.G. Summit alleges that PHILSECO continues to violate the Constitution as its foreign equity is above 40% and yet owns long-term leasehold rights which are 45 real rights. It cites Article 415 of the Civil Code which includes in the definition of immovable property, 46 "contracts for public works, and servitudes and other real rights over immovable property." Any existing 47 landholding, however, is denied by PHILYARDS citing its recent financial statements. First, these are questions of fact, the veracity of which would require introduction of evidence. The Court needs to validate these factual allegations based on competent and reliable evidence. As such, the Court cannot resolve the questions they pose. Second, J.G. Summit misreads the provisions of the Constitution cited in its own pleadings, to wit: 29.2 Petitioner has consistently pointed out in the past that private respondent is not a 60%-40% corporation, and this violates the Constitution x x x The violation continues to this day because under the law, it continues to own real property xxx xxx xxx 32. To review the constitutional provisions involved, Section 14, Article XIV of the 1973 Constitution (the JVA was signed in 1977), provided: "Save in cases of hereditary succession, no private lands shall be transferred or conveyed except to individuals, corporations, or associations qualified to acquire or hold lands of the public domain."

L A W O N N A T U R A L R E S O U R C E S a n d E N V I R O N M E N T A L L A W C a s e s a n d S p e c i a l L a w s | 76
32.1 This provision is the same as Section 7, Article XII of the 1987 Constitution. 32.2 Under the Public Land Act, corporations qualified to acquire or hold lands of the public domain are corporations at least 60% of which is owned by Filipino citizens (Sec. 22, Commonwealth Act 141, as amended). (emphases supplied) As correctly observed by the public respondents, the prohibition in the Constitution applies only to 48 ownership of land. It does not extend to immovable or real property as defined under Article 415 of the Civil Code.Otherwise, we would have a strange situation where the ownership of immovable property 49 such as trees, plants and growing fruit attached to the land would be limited to Filipinos and Filipino corporations only. III. WHEREFORE, in view of the foregoing, the petitioners Motion for Reconsideration is DENIED WITH FINALITY and the decision appealed from is AFFIRMED. The Motion to Elevate This Case to the Court En Banc is likewise DENIED for lack of merit. SO ORDERED. Davide, Jr., C.J., (Chairman), Ynares-Santiago, Corona, and Tinga, JJ., concur.
Footnotes
1 2

Also referred to in this Resolution as "PHILYARDS." Also referred to as J G Summit. 3 Resolution promulgated on September 24, 2003, pp. 2 10. 4 Id. at 10 13. 5 Id. at 14 22. 6 Id. at 22 25. 7 Id. at 26 32. 8 Rollo, p. 1854. 9 Rollo, p. 1876. 10 J.G. Summits Motion to Elevate this Case to the Court En Banc dated October 17, 2003, p. 3; Rollo, p. 1878. 11 Id. 12 Id. 13 2. A decision or resolution of a Division of the Court, when concurred in by a majority of its Members who actually took part in the deliberations on the issues in a case and voted thereon, and in no case without the concurrence of at least three of such Members, is a decision or resolution of the Supreme Court (Section 4[3], Article VIII, 1987 Constitution). 3. The Court en banc is not an Appellate Court to which decisions or resolutions of a Division may be appealed. xxx xxx xxx

5. A resolution of the Division denying a partys motion for referral to the Court en banc of any Division case, shall be final and not appealable to the Court en banc. 6. When a decision or resolution is referred by a Division to the Court en banc, the latter may, in the absence of sufficiently important reasons, decline to take cognizance of the same, in which case, the decision or resolution shall be returned to the referring Division. 7. No motion for reconsideration of the action of the Court en banc declining to take cognizance of a referral by a Division, shall be entertained. 14 PHILYARDS Comment dated February 3, 2004, pp. 26-27; Rollo, pp. 1996-1997. 15 J.G. Summits Consolidated Comment dated March 8, 2004. 16 Resolution dated September 24, 2003, pp. 26-32. 17 Id., pp. 10-22. 18 See Bastida and Ysmael & Co., Inc. v. Dy Buncio & Co., Inc., 93 Phil. 195 (1953); Garcia v. Burgos , 291 SCRA 546 (1998); Sadhwani v. CA , 281 SCRA 75 (1997); Paraaque Kings Enterprises, Incorporated v. CA , 268 SCRA 727 (1997); Polytechnic University of the Philippines v. CA , 368 SCRA 691 (2001); and Guzman, Bocaling & Co. v. Bonnevie, 206 SCRA 668 (1992). 19 G.R. No. 101678, February 3, 1992, 205 SCRA 705. 20 Sec. 4(3), Art. VIII, Constitution. 21 Supreme Court Circular No. 2-89, February 7, 1989. 22 J.G. Summits Motion for Reconsideration dated October 17, 2003, pp. 8-9; Rollo, pp. 1861-1862. 23 Id. at 10-13; Rollo, pp. 1863-1866. 24 Id. at 13-19; Rollo, pp. 1866-1872. 25 PHILYARDS Comment dated February 3, 2004, p. 1; Rollo, p. 1971. 26 Id. at 2; Rollo, p. 1972. 27 Id. at 5; Rollo, p. 1975. 28 Id. at 9; Rollo, p. 1979. 29 Id. at 12; Rollo, p. 1982. 30 Id. 31 Id. at 14; Rollo, p. 1984. 32 Id. at 19; Rollo, p. 1989. 33 COP and APTs Comment dated January 14, 2004, pp. 14-15; Rollo, pp. 1927-1928. 34 Resolution dated September 24, 2003, pp. 23-24. 35 Id. at 22. 36 Resolution dated September 24, 2003, pp. 31-32. 37 J.G. Summits Consolidated Reply dated March 11, 2004, p. 14; Rollo, p. 2109. 38 Id. 39 PHILYARDS Manifestation and Comment dated June 26, 2002, p. 10; Rollo, p. 1334. 40 PHILYARDS Comment dated February 3, 2004, pp. 8-16; Rollo, pp. 1978-1986. 41 PHILYARDS Manifestation and Comment dated June 26, 2002, p. 10; Rollo, p. 1334. 42 Constitution, Article XII, National Economy and Patrimony. 43 J.G. Summits Consolidated Comment dated March 8, 2004, p. 17; Rollo, p. 2112. 44 Philippine Banking Corporation v. Lui She, No. L-17587, September 12, 1967, 21 SCRA 52. 45 J.G. Summits Motion for Reconsideration dated October 17, 2003, p. 14; Rollo, p. 1867. 46 Id. at 15; Rollo, p. 1868. 47 PHILYARDS Manifestation and Comment dated June 26, 2002, p. 10; Rollo, p. 1334. 48 COP and APTs Comment dated January 14, 2004, p. 36; Rollo, p. 1949. 49 Art. 415(2), Civil Code.

L A W O N N A T U R A L R E S O U R C E S a n d E N V I R O N M E N T A L L A W C a s e s a n d S p e c i a l L a w s | 77
G.R. No. L-46729 November 19, 1982 LAUSAN AYOG, BENITO AYOG, DAMASO AYOG, JULIO AYOG, SEGUNDA AYOG, VICENTE ABAQUETA, BERNARDINO ADORMEO, VIDAL ALBANO, FELICIANO ARIAS, ANTONIO BALDOS, MAXIMO BALDOS, ROMERO BINGZON, EMILIO CADAYDAY, FRUCTUOSO CHUA, SR., HERACLEO CHUA, GUILLERMO DAGOY, ABDON DEIMOS, NICASIO DE LEON, JULIANA VDA. DE DIANNA, DEMOCRITO DEVERO, ALFREDO DIVINAGRACIA, ESTEBAN DIVINAGRACIA, LEODEGARDIO DIVINAGRACIA, NELLO DIVINAGRACIA, MERQUIADES EMBERADOR, JESUS EMPERADO, PORFERIO ENOC, SOFRONIO ENOC, RAFAEL GAETOS, NICOLAS GARLET, TRINIDAD GARLET, FORTUNATA GEONZON, NICOLADA NAQUILA, TORIBIO NAQUILA, EFREN OKAY, ELPIDIO OKAY, SR., DIEGO ONGRIA, ERNESTO PANARES, VICENTE PATULOT, IGNACIA RIBAO, JUANO RICO, JESUS ROSALITA, ARMANDO TANTE and ANSELMO VALMORES, petitioners, vs. JUDGE VICENTE N. CUSI, JR., Court of First Instance of Davao, Branch I, PROVINCIAL SHERIFF OF DAVAO, and BINAN DEVELOPMENT CO., INC., respondents. MINISTER OF NATURAL RESOURCES and DIRECTOR OF LANDS, intervenors. AQUINO, J.: This case is about the application of section 11, Article XIV of the 1973 Constitution (disqualifying a private corporation from purchasing public lands) to a 1953 sales award made by the Bureau of Lands, for which a sales patent and Torrens title were issued in 1975, and to the 1964 decision of the trial court, ejecting some of the petitioners from the land purchased, which decision was affirmed in 1975 by the Court of Appeals. That legal question arises under the following facts: On January 21, 1953, the Director of Lands, after a bidding, awarded to Bian Development Co., Inc. on the basis of its 1951 Sales Application No. V-6834 Cadastral Lot No. 281 located at Barrio Tamugan, Guianga (Baguio District), Davao City with an area of about two hundred fifty hectares. Some occupants of the lot protested against the sale. The Director of Lands in his decision of August 30, 1957 dismissed the protests and ordered the occupants to vacate the lot and remove their improvements. No appeal was made from that decision. The Director found that the protestants (defendants in the 1961 ejectment suit, some of whom are now petitioners herein) entered the land only after it was awarded to the corporation and, therefore, they could not be regarded as bona fide occupants thereof. The Director characterized them as squatters. He found that some claimants were fictitious persons (p. 30, Rollo of L-43505, Okay vs. CA). He issued a writ of execution but the protestants defied the writ and refused to vacate the land (p. 28, Rollo of L-43505, Okay vs. CA). ** Because the alleged occupants refused to vacate the land, the corporation filed against them on February 27, 1961 in the Court of First Instance of Davao, Civil Case No. 3711, an ejectment suit ( accion publiciana). The forty defendants were Identified as follows: 1. Vicente Abaqueta 21. Eniego Garlic 2. Candido Abella 22. Nicolas Garlic 3. Julio Ayog 23. Rufo Garlic 4. Arcadio Ayong 24. Alfonso Ibales 5. Generoso Bangonan 25. Julian Locacia 6. Lomayong Cabao 26. Filomeno Labantaban 7. Jose Catibring 27. Arcadio Lumantas 8. Teodolfo Chua 28. Santos Militante 9. Guillermo Dagoy 29. Toribio Naquila 10. Anastacia Vda. de Didal 30. Elpidio Okay 11. Alfredo Divinagracia 31. Guillermo Omac 12. Silverio Divinagracia 32. Emilio Padayday 13. Galina Edsa 33. Marcosa Vda. de Rejoy 14. Jesus Emperado 34. Lorenzo Rutsa 15. Porfirio Enoc 35. Ramon Samsa 16. Benito Ente 36. Rebecca Samsa 17. German Flores 37. Alfeao Sante 18. Ciriaco Fuentes 38. Meliton Sante 19. Pulong Gabao 39. Amil Sidaani 20. Constancio Garlic 40. Cosme Villegas

L A W O N N A T U R A L R E S O U R C E S a n d E N V I R O N M E N T A L L A W C a s e s a n d S p e c i a l L a w s | 78
That ejectment suit delayed the issuance of the patent. The trial court found that the protests of twenty of the abovenamed defendants were among those that were dismissed by the Director of Lands in his 1957 decision already mentioned. On July 18, 1961 the purchase price of ten thousand pesos was fully paid by Binan Development Co., Inc. OnNovember 10, 1961, an official of the Bureau of Lands submitted a final investigation report wherein it was stated that the corporation had complied with the cultivation and other requirements under the Public Land Law and had paid the purchase price of the land (p. 248, Rollo). It was only more than thirteen years later or on August 14, 1975 when Sales Patent No. 5681 was issued to the corporation for that lot with a reduced area of 175.3 hectares. The patent was registered. Original Certificate of Title No. P-5176 was issued to the patentee. The Director of Lands in his memorandum dated June 29, 1974 for the Secretary of Natural Resources, recommending approval of the sales patent, pointed out that the purchaser corporation had complied with the said requirements long before the effectivity of the Constitution, that the land in question was free from claims and conflicts and that the issuance of the patent was in conformity with the guidelines prescribed in Opinion No. 64, series of 1973, of Secretary of Justice Vicente Abad Santos and was an exception to the prohibition in section 11, Article XIV of the Constitution (p. 258, Rollo). Secretary of Natural Resources Jose J. Leido, Jr., in approving the patent on August 14, 1975, noted that the applicant had acquired a nested right to its issuance (p. 259, Rollo). Before that patent was issued, there was a trial in the ejectment suit. Fifteen defendants (out of forty), namely, Julio Ayog, Guillermo Bagoy, Generoso Bangonan, Jose Catibring, Porfirio Enoc, Jose Emperado, Arcadio Lomanto, Toribio Naquila, Elpidio Okay, Alfeo Sante, Meliton Sante, Ramon Samsa, Rebecca Samsa, Arcadio Sarumines and Felix Tahantahan, testified that they entered the disputed land long before 1951 and that they planted it to coconuts, coffee, jackfruit and other fruit trees. (p. 28, Record on Appeal). The trial court did not give credence to their testimonies. It believed the report of an official of the Bureau of Lands that in 1953 the land was free from private claims and conflicts and it gave much weight to the decision of the Director of Lands dismissing the protests of the defendants against the sales award (p. 30, Record on Appeal). Furthermore, the trial court during its ocular inspection of the land on November 8, 1964 found that the plantings on the land could not be more than ten years old, meaning that they were not existing in 1953 when the sales award was made. Hence, the trial court ordered the defendants to vacate the land and to restore the possession thereof to tile company. The Court of Appeals affirmed that judgment on December 5, 1975 in its decision in Binan Development Co., Inc. vs, Sante, CA-G.R. No. 37142- R. The review of the decision was denied by this Court on May 17, 1976 in Elpidio Okay vs. Court of Appeals, L43505. After the record was remanded to the trial court, the corporation filed a motion for execution. The defendants, some of whom are now petitioners herein, opposed the motion. They contended that the adoption of the Constitution, which took effect on January 17, 1973, was a supervening fact which rendered it legally impossible to execute the lower court's judgment. They invoked the constitutional prohibition, already mentioned, that "no private corporation or association may hold alienable lands of the public domain except by lease not to exceed one thousand hectares in area." The lower court suspended action on the motion for execution because of the manifestation of the defendants that they would file a petition for prohibition in this Court. On August 24, 1977, the instant prohibition action was filed. Some of the petitioners were not defendants in the ejectment case. We hold that the said constitutional prohibition has no retroactive application to the sales application of Bian Development Co., Inc. because it had already acquired a vested right to the land applied for at the time the 1973 Constitution took effect. That vested right has to be respected. lt could not be abrogated by the new Constitution. Section 2, Article XIII of the 1935 Constitution allows private corporations to purchase public agricultural lands not exceeding one thousand and twenty-four hectares. Petitioners' prohibition action is barred by the doctrine of vested rights in constitutional law. "A right is vested when the right to enjoyment has become the property of some particular person or persons as a present interest" (16 C.J.S. 1173). It is "the privilege to enjoy property legally vested, to enforce contracts, and enjoy the rights of property conferred by the existing law" (12 C.J. 955, Note 46, No. 6) or "some right or interest in property which has become fixed and established and is no longer open to doubt or controversy" (Downs vs. Blount 170 Fed. 15, 20, cited in Balboa vs. Farrales, 51 Phil. 498, 502). The due process clause prohibits the annihilation of vested rights. "A state may not impair vested rights by legislative enactment, by the enactment or by the subsequent repeal of a municipal ordinance, or by a change in the constitution of the State, except in a legitimate exercise of the police power" (16 C.J.S. 1177-78). It has been observed that, generally, the term "vested right" expresses the concept of present fixed interest, which in right reason and natural justice should be protected against arbitrary State action, or an innately just and imperative right which an enlightened free society, sensitive to inherent and irrefragable individual rights, cannot deny (16 C.J.S. 1174, Note 71, No. 5, citing Pennsylvania Greyhound Lines, Inc. vs. Rosenthal, 192 Atl. 2nd 587).

L A W O N N A T U R A L R E S O U R C E S a n d E N V I R O N M E N T A L L A W C a s e s a n d S p e c i a l L a w s | 79
Secretary of Justice Abad Santos in his 1973 opinion ruled that where the applicant, before the Constitution took effect, had fully complied with all his obligations under the Public Land Act in order to entitle him to a sales patent, there would seem to be no legal or equitable justification for refusing to issue or release the sales patent (p. 254, Rollo). In Opinion No. 140, series of 1974, he held that as soon as the applicant had fulfilled the construction or cultivation requirements and has fully paid the purchase price, he should be deemed to have acquired by purchase the particular tract of land and to him the area limitation in the new Constitution would not apply. In Opinion No. 185, series of 1976, Secretary Abad Santos held that where the cultivation requirements were fulfilled before the new Constitution took effect but the full payment of the price was completed after January 17, 1973, the applicant was, nevertheless, entitled to a sales patent (p. 256, Rollo). Such a contemporaneous construction of the constitutional prohibition by a high executive official carries great weight and should be accorded much respect. It is a correct interpretation of section 11 of Article XIV. In the instant case, it is incontestable that prior to the effectivity of the 1973 Constitution the right of the corporation to purchase the land in question had become fixed and established and was no longer open to doubt or controversy. Its compliance with the requirements of the Public Land Law for the issuance of a patent had the effect of segregating the said land from the public domain. The corporation's right to obtain a patent for that land is protected by law. It cannot be deprived of that right without due process (Director of Lands vs. CA, 123 Phil. 919). As we cannot review the factual findings of the trial court and the Court of Appeals, we cannot entertain petitioners' contention that many of them by themselves and through their predecessors-in-interest have possessed portions of land even before the war. They should have filed homestead or free patent applications. Our jurisdiction is limited to the resolution of the legal issue as to whether the 1973 Constitution is an obstacle to the implementation of the trial court's 1964 final and executory judgment ejecting the petitioners. On that issue, we have no choice but to sustain its enforceability. Nevertheless, in the interest of social justice, to avoid agrarian unrest and to dispel the notion that the law grinds the faces of the poor, the administrative authorities should find ways and means of accommodating some of the petitioners if they are landless and are really tillers of the soil who in the words of President Magsaysay deserve a little more food in their stomachs, a little more shelter over their heads and a little more clothing on their backs. The State should endeavor to help the poor who find it difficult to make both ends meet and who suffer privations in the universal struggle for existence. A tiller of the soil is entitled to enjoy basic human rights, particularly freedom from want. The common man should be assisted in possessing and cultivating a piece of land for his sustenance, to give him social security and to enable him to achieve a dignified existence and become an independent, self-reliant and responsible citizen in our democratic society. To guarantee him that right is to discourage him from becoming a subversive or from rebelling against a social order where, as the architect of the French Revolution observed, the rich are choking with the superfluities of life but the famished multitude lack the barest necessities. Indeed, one purpose of the constitutional prohibition against purchases of public agricultural lands by private corporations is to equitably diffuse land ownership or to encourage "owner-cultivatorship and the economic family- size farm" and to prevent a recurrence of cases like the instant case. Huge landholdings by corporations or private persons had owned social unrest. Petitioners' counsel claims that Bian Development Co., Inc. seeks to execute the judgment in Civil Case No. 3711, the ejectment suit from which this prohibition case arose, against some of the petitioners who were not defendants in that suit (p. 126, Rollo). Those petitioners are not successors-in-interest of the defendants in the ejectment suit. Nor do they derive their right of possession from the said defendants. Those petitioners occupy portions of the disputed land distinct and separate from the portions occupied by the said defendants. We hold that judgment cannot be enforced against the said petitioners who were not defendants in that litigation or who were not summoned and heard in that case. Generally, "it is an axiom of the law that no man shall be affected by proceedings to which he is a stranger" (Ed. A. Keller & Co. vs Ellerman & Bucknall Steamship Co., 38 Phil. 514, 520). To enforce the judgment against those who were not parties to the case and who occupy portions of the disputed land distinct and separate from the portions occupied by the defendants in the ejectment suit, would be violative of due process of law, the law which, according to Daniel Webster in his argument in the Dartmouth College case, is the law of the land, a law which hears before it condemns, which proceeds upon inquiry and renders judgment only after trial. "The meaning is, that every citizen shall hold his life, liberty, property, and immunities, under the protection of the general rules which govern society." (Cited in Lopez vs. Director of Lands, 47 Phil. 23, 32. See Gatchalian vs. Arlegui, L-35615 and Tang Tee vs. Arlegui, L-41360, February 17, 1977, 75 SCRA 234 and Berses vs. Villanueva, 25 Phil. 473.) Contempt incident.-During the pendency of this case, or at about four o'clock in the morning of December 12, 1978, Ciriaco Tebayan, Domingo Nevasca, Rogelio Duterte and Sofronio Etac, employees of the Crown

L A W O N N A T U R A L R E S O U R C E S a n d E N V I R O N M E N T A L L A W C a s e s a n d S p e c i a l L a w s | 80
Fruits and Cannery Corporation, plowed or bulldozed with their tractors a portion of the disputed land which was occupied by Melquiades Emberador, one of the petitioners herein. The disputed land was leased by Bian Development Co., Inc. to the canning corporation. The four tractor drivers destroyed the improvements thereon worth about five thousand pesos consisting of coffee, coconut and banana plants. Emberador was in the hospital at the time the alleged destruction of the improvements occurred. However, it should be noted that Emberador was not expressly named as a defendant in the ejectment suit. Apparently, he is not included in the trial court's decision although he was joined as a co-petitioner in this prohibition case. The petitioners in their motion of January 11, 1979 asked that the four tractor drivers and Honesto Garcia, the manager of Bian Development Co., Inc., be declared in contempt of court for having disregarded the restraining order issued by this Court on August 29, 1977, enjoining specifically Judge Vicente N. Cusi and the provincial sheriff from enforcing the decision in the ejectment suit, Civil Case No. 3711 (pp. 46-47, 138- 141, Rollo). Garcia and the four drivers answered the motion. The incident was assigned for hearing to Judge Antonio M. Martinez of the Court of First Instance of Davao. Judge Martinez found that the plowing was made at the instance of Garcia who told the barrio captain, petitioner Lausan Ayog, a Bagobo, that he (Garcia) could not wait anymore for the termination of this case. The record shows that on April 30, 1979 or four months after the said incident, Emberador, in consideration of P3,500, as the value of the improvements on his land, executed a quitclaim in favor of the Crown Fruits and Cannery Corporation (Exh. 1, 2 and 3). We hold that no contempt was committed. The temporary restraining order was not directed to Bian Development Co., Inc. its officers, agents or privies. Emberador was not named specifically in the trial court's judgment as one of the occupants to be ejected. For the redress of whatever wrong or delict was committed against Emberador by reason of the destruction of his improvements, his remedy is not in a contempt proceeding but in some appropriate civil and criminal actions against the destroyer of the improvements. In resume, we find that there is no merit in the instant prohibition action. The constitutional prohibition relied upon by the petitioners as a ground to stop the execution of the judgment in the ejectment suit has no retroactive application to that case and does not divest the trial court of jurisdiction to enforce that judgment. WHEREFORE, the petition is dismissed for lack of merit but with the clarification that the said judgment cannot be enforced against those petitioners herein who were not defendants in the ejectment case, Civil Case No. 3711, and over whom the lower court did not acquire jurisdiction. The contempt proceeding is also dismissed. No costs. SO ORDERED. Concepcion, Jr., Guerrero, Abad Santos, Relova and Gutierrez, Jr., JJ., concur. Escolin, J., took no part. Separate Opinions VASQUEZ, J., concurring: I concur with the very ably written main opinion. However, I wish to erase any possible erroneous impression that may be derived from the dispositive portion insofar as it declares that the judgment in the ejectment cage may not be enforced against the petitioners who were not defendants in Civil Case No. 3711 and over whom the lower court did not acquire jurisdiction. The judgment in any case is binding and enforceable not only against the parties thereto but also against "their successors in interest by title subsequent to the commencement of the action" (Sec. 49[b], Rule 39, Rules of Court). We have previously held that the judgment in an ejectment case may be enforced not only against the defendants therein but also against the members of their family, their relatives or privies who derive their right of possession from the defendants (Ariem vs. Delos Angeles, 49 SCRA 343). A further clarification of the dispositive portion is apparently needed to exclude from the effect of the judgment in the ejectment case only the petitioners who do not derive their right of possession from any of the defendants in the ejectment suit. Fernando, C.J., Teehankee, Melencio-Herrera, Plana, Makasiar and De Castro, JJ., concurs.
Footnotes * According to respondent corporation, some of the adverse claimants or protestants were not landless farmers but were well-educated persons belonging to the middle class. Thus, Elpidio Okay was an elementary school principal. Vicente Rehoy was a landowner and barrio captain. Patricio de Leon was a cashier and later assistant branch manager of the Philippine National Baank. Ernesto Paares was a high school teacher and later a college professor. Francisco Mateo was a former college dean (p. 105, Rollo). According to the 44 petitioners, they are tillers of the soil (p. 126, Rollo).

L A W O N N A T U R A L R E S O U R C E S a n d E N V I R O N M E N T A L L A W C a s e s a n d S p e c i a l L a w s | 81
G.R. No. L-69997 September 30, 1987 UNGAY MALOBAGO MINES, INC., petitioner, vs. HON. INTERMEDIATE APPELLATE COURT, DIRECTOR OF LANDS, GREGORIA BOLANOS, AUREA ARAOJO, GERVACIO ARAOJO, MARIA BERNAL, FELIX DETECIO, JESUS ASUNCION, MELENCIO ASUNCION and BIENVENIDO ASUNCION, respondents. GUTIERREZ, JR., J.: Before us is a petition which seeks to set aside the decision of the then Intermediate Appellate Court affirming the dismissal of the petitioner's action for annulment and cancellation of free patents granted to the private respondents on the ground that the petitioner has no personality to file an action for reversion, the lands involved being public In character. On July 20, 1962, the President of the Philippines granted the following mining patents on mineral claims located at Ungay Malobago, Rapu-Rapu Albay. 1. lode patent No. V-52 to John Canson, Jr., on mineral claim known as "Catanduandes;" 2. lode patent No. V-48 to petitioner, on mineral claims known as "Junior;" 3. lode patent No. V-53 to John Canson, Jr., on mineral claims known as "Oas;" 4. lode patent No. V - 46 to petitioner on mineral claim known as "Ester;" 5. lode patent No. V - 51 to Carlos Stilianopulos on mineral claim known as "Jovellar;" 6. lode patent No. V - 49 to petitioner, in mineral claim known as "Manila;" 7. lode patent No. V - 50 to Carlos Stilianopulos on mineral claim known as "Polangui;" and 8. lode patent No. V - 47 to petitioner on mineral claim known as "Ligao;"(pp. 5-7, Decision Annex 1, Petition) Way back on October 30, 1959, John Canson, Jr. and Carlos Stilianopulos assigned their rights to their mining claims in favor of the petitioner. The assignment of rights was recorded in the Office of the Mining Recorder of Albay on December 2, 1959. 6. Free Patent No. 0663 dated March 25, 1974 and the corresponding Certificate of Title No. VH-19333 to appellee Bienvenido Asuncion. (Rollo, pp. 200-201) All of the above patents covered portions of the lots covered by the patents belonging to the petitioner. The petitioner filed a complaint for annulment and cancellation of patents against the private respondents and prayed that all the free patent titles issued in their favor for properties over which original certificates of title had already been issued in its favor be declared null and void. The Director of Lands, who was impleaded as a formal defendant, filed his answer alledging, among others, that the petitioner has no personality to institute the cancellation proceedings inasmuch as the government is the grantor and not the petitioner, and it should be the grantor who should institute the cancellation proceedings. On January 25, 1980, the trial court rendered a decision dismissing the complaint. It ruled that since the disputed properties form part of disposable land of the public domain, the action for reversion should be 1. Free Patent No. 458143 dated October 3, 1968 and corresponding Certificate of Title No. VH-12195 to appellee Felix Detecio; 2. Free Patent No. 427824 dated November 21, 1968 and corresponding Certificate of Title No. VH-12256 to appellee Melencio Asuncion; 3. Free Patent No. 433318 dated January 10, 1969 and corresponding Certificate of Title No. VH-12198 to appellee Jesus Asuncion; 4. Free Patents No. 422847 dated November 11, 1968 and No. 421947 dated October 28, 1969 and corresponding Certificates of Title Nos. VH-12185 and 12186, respectively, to appellee Maria Bernal; 5. Free Patent No. 408568 dated July 8, 1968 and corresponding Certificate of Title No. VH-11591 to appellee Gregorio Bolanos; and The aforestated mining patents, after their issuance on July 20, 1962, were all recorded in the Office of the Mining Recorder of Albay on August 28, 1962 and transcribed on September 4, 1962 in the Registration Book of the Registry of Deeds of Albay. Consequently, the Register of Deeds of Albay issued the respective original certificates of titles pursuant to Section 122 of Act No. 496 in the names of John Canson, Jr., Carlos Stilianopulos, and the petitioner. Subsequently, or from 1968 to 1974, the following free patents were granted by the respondent Director of Lands and the corresponding original certificates of titles were issued by the Register of Deeds of Albay:

L A W O N N A T U R A L R E S O U R C E S a n d E N V I R O N M E N T A L L A W C a s e s a n d S p e c i a l L a w s | 82
instituted by the Solicitor General in the name of the Republic of the Philippines and that, therefore, the petitioner lacks personality to institute the annulment proceedings. The petitioner appealed to the then Intermediate Appellate Court. On April 5, 1984, the appellate court affirmed the decision of the trial court. It ruled that the titles issued to the petitioner cover mineral lands which belong to the public domain and that these cannot be the subject of private ownership. According to the Court, under Section 101 of the Public Land Law, only the Solicitor General or the officer acting in his stead has the authority to institute an action on behalf of the Republic for the cancellation of the respondents' titles and for reversion of their homesteads to the Government. In this instant petition, the petitioner raises two issues: a) Whether or not the appellate court committed an error of law when it ruled that the lands in question belong to the public domain; and b) whether or not the appellate court erred in discussing the complaint on the ground that the petitioner had no personality to institute the same. With regard to the first issue, the petitioner maintains that since its mining claims were perfected prior to November 15, 1935, the date when the 1935 Constitution took effect, the applicable law is the Philippine Bill of 1902 and that under this Act, a valid location of a mining claim segregates the area from the public domain. (Gold Creek Mining Corporation v. Rodriguez, 66 Phil. 259). The Solicitor-General, on the other hand, argues that the petitioner's mining patents covered by Torrens Titles were granted only in 1962 by the President of the Philippines, by authority of the Constitution of the Philippines. Under the then Constitution, except for public agricultural lands, natural resources which includes all mineral lands, shall not be alienated. (Art. XIII, Section 1, 1935 Constitution) Therefore, what the mining patents issued in 1962 conveyed to petitioner was only the ownership of, and the right to extract and utilize, the minerals within the area covered by the petitioner's Torrens Titles but not the ownership of the land where the minerals are found. We rule for the private respondents. The petitioner has been beguiling, less than candid, and inexplicably silent as to material dates in the presentation of its case. Nowhere in the records of this petition is there any mention of a date before November 15, 1935 as to when essential acts regarding its mining claims were executed. It is silent as to when the land was entered, measured, and plotted; when the legal posts and notices were put up; when the claim was registered with the mining recorder; whether or not the annual amount of labor or development, and other requirements under the Philippine Bill of 1902 were followed. These may have been complied with but not necessarily before 1935. A mere mention in the Torrens title that the provisions of the Philippine Bill of 1902 were followed is not sufficient. The Philippine Bill provides the procedures for the perfection of mining claims but not the dates when such procedures were undertaken by any prospector or claimant. The same procedures would have to be followed even after the Jones Law of 1916 and the Constitution of 1935 were promulgated, but subject to the restrictions of the fundamental law. The petitioner has failed to state if and when new procedures, different from the 1902 procedures, were provided by law to give a little substance to its case. The petitioner is completely and strangely silent about these vital aspects of its petition. Petitioner has not established by clear and convincing evidence that the locations of its mining claims were perfected prior to November 15,1935 when the Government of Commonwealth was inaugurated. In fact neither the original complaint nor the amended one alleged the perfection of petitioner's mining rights prior to November 15, 1935. All that petitioner offers as evidence of its claims were the original certificates of titles covering mining patents which embodied a uniform "WHEREAS" clause stating that the petitioner "has fully complied with all the conditions, requirements, and provisions of the Act of the United States of Congress of July 1, 1902, as amended, ..." In the absence of proof that the petitioner's claims were perfected prior to the 1935 Constitution, the provision of the latter with regard to inalienable lands of the public domain will apply. Article XIII, Section I of the 1935 Constitution provides: All agricultural timber, and mineral lands of the public domain, waters, minerals, coal, petroleum, and other mineral oils, all forces of potential energy, and other natural resources of the Philippines belong to the State, and their disposition, exploitation, development, or utilization shall be limited to citizens of the Philippines, or to corporations or associations at least sixty per centum of the capital of which is owned by such citizens, subject to any existing right, grant, lease, or concession at the time of the inauguration of the Government established under this Constitution. Natural resources, with the exception of public agricultural land, shall not be alienated and no license, concession, or lease for the exploitation, development, or utilization of any of the natural resources shall be granted for a period exceeding twenty-five years, renewable for another twenty-five years, except as to water rights for irrigation, water supply, fisheries, or industrial uses other than the development of water power, in which cases beneficial use may be the measure and the at of the grant. (Emphasis supplied) Therefore, applying the aforequoted provision to the case at bar, we conclude that the issuance of the lode patents on mineral claims by the President of the Philippines in 1962 in favor of the petitioner granted to it only the right to extract or utilize the minerals which may be found on or under the surface of the land. On the other hand, the issuance of the free patents by the respondent Director of Lands in 1979 in favor of the private respondents granted to them the ownership and the right to use the land for agricultural purposes but excluding the ownership of, and the right to extract or utilize, the minerals which may be found on or under the surface.

L A W O N N A T U R A L R E S O U R C E S a n d E N V I R O N M E N T A L L A W C a s e s a n d S p e c i a l L a w s | 83
There is no basis in the records for the petitioner's stand that it acquired the right to the mineral lands prior to the effectivity of the 1935 Constitution, thus, making such acquisition outside its purview and scope. Every application for a concession of public land has to be viewed in the light of its peculiar circumstances. (Director of Lands v. Funtilar 142 SCRA 57, 69). In the case at bar, although the original certificates of titles of the petitioner were issued prior to the titles of the private respondents, the former cannot prevail over the latter for the provisions of the Constitution which governed at the time of their issuance prohibited the alienation of mineral lands of the public domain. In the case of Republic v. Animas (56 SCRA 499), this Court ruled that a grantee does not become the owner of a land illegally included in the grant just because title has been issued in his favor:. A patent is void at law if the officer who issued the patent had no authority to do so (Knight v. Land Ass. 142 U.S. 161, 12 Sup. Ct., 258, 35L ED. 974; emphasis supplied). If a person obtains a title under the Public Land Act which includes, by mistake or oversight, lands which cannot be registered under the Torrens System, or when the Director of Lands did not have jurisdiction over the same because it is a public forest, the grantee does not, by virtue of said certificate of title alone, become the owner of the land illegally included. (See Ledesma vs. Municipality of Iloilo, 49 Phil. 769) Moreover, patents and land grants are construed favorably in favor of the Government, and most strongly against the grantee. Any doubt as to the intention or extent of the grant, or the intention of the Government, is to be resolved in its favor. (See Republic v. Court of Appeals, 73 SCRA 146, 156). Hence, as earlier stated, in the absence of proof that the petitioner acquired the right of ownership over the mineral lands prior to the 1935 Constitution, the titles issued in its favor must be construed as conveying only the right to extract and utilize the minerals thereon. The appellate court did not likewise err in concluding that the petitioner has no personality to institute the action below for annulment and cancellation of patents. The mineral lands over which it has a right to extract minerals remained part of the inalienable lands of the public domain and thus, only the Solicitor General or the person acting in his stead can bring an action for reversion. (See Sumail v. Judge of the Court of First Instance of Cotabato, et al., 96 Phil. 946; and Heirs of Tanak Pangawaran Patiwayan v. Martinez, 142 SCRA 252). WHEREFORE, the petition is hereby DISMISSED for lack of merit. The decision of the Intermediate Appellate Court is AFFIRMED. Costs against the petitioner. SO ORDERED. Fernan, Feliciano, Bidin and Cortes, JJ., concur.

L A W O N N A T U R A L R E S O U R C E S a n d E N V I R O N M E N T A L L A W C a s e s a n d S p e c i a l L a w s | 84
G.R. No. 137887 February 28, 2000 SO ORDERED.
3

REPUBLIC OF THE PHILIPPINES, petitioner, vs. DAMIAN ERMITAO DE GUZMAN, DEOGRACIAS ERMITAO DE GUZMAN, ZENAIDA ERMITAO DE GUZMAN, ALICIA ERMITAO DE GUZMAN, SALVADOR ERMITAO DE GUZMAN, DOMINGA ERMITAO, NATIVIDAD ENCARNACION, MELBA E. TORRES, FLORA MANALO, SOCORRO DELA ROSA, JOSE ERMITAO, ESMERANDO ERMITAO, TRICOM DEVELOPMENT CORPORATION and FILOMENO ERMITAO, respondents. YNARES-SANTIAGO, J.: Before us is a Petition for Review on Certiorari of a decision of the Court of Appeals affirming the 2 judgment of the Regional Trial Court of Tagaytay, Branch 18, in LRC Cases No. TG-362 and TG-396. The facts are simple: Conflicting applications for confirmation of imperfect title were filed by Norma Almanzor and private respondent Salvador De Guzman over parcels of land located in Silang, Cavite. After trial on the merits, the lower court rendered judgment in favor of private respondent De Guzman, to wit WHEREFORE, judgment is hereby rendered by this Court as follows: (1) In LRC Case No. TG-362, this Court hereby denies the application for registration of the parcels of land mentioned therein by applicant Norma R. Almanzor for lack of factual and legal bases; (2) In LRC Case No. 396, this Court hereby approves the petition for registration and thus places under the operation of Act 141, Act 946 and/or P.D. 1529, otherwise known as the Property Registration Law, the land described in Plan Psu-67537-Amd-2 and containing an area of 308,638 square meters, as supported by its technical descriptions now forming parts of the records of these cases, in addition to other proofs adduced in the names of petitioners Damian Ermitao De Guzman, Deogracias Ermitao De Guzman, Zenaida Ermitao De Guzman, Alicia Ermitao De Guzman and Salvador De Guzman, all married, of legal age and with residence and postal addresses at Magallanes Street, Carmona, Cavite, subject to the claims of oppositors Dominga Ermitao, Natividad Encarnacion, Melba E. Torres, Flora Manalo, Socorro de la Rosa, Jose Ermitao and Esmeranso Ermitao under an instrument entitled "Waiver of Rights with Conformity" the terms and conditions of which are hereby ordered by this Court to be annotated at the back of the certificates of title to be issued to the petitioners pursuant to the judgment of this Court.
1

As earlier mentioned, on appeal to the Court of Appeals, said judgment was affirmed and the petition for registration of private respondents over the subject parcels of land was approved. Hence, the instant Petition, anchored upon the following assignments of error I THE TRIAL COURT ERRED IN NOT FINDING THAT THE DE GUZMANS HAVE NOT SUBMITTED PROOF OF THEIR FEE SIMPLE TITLE OR POSSESSION IN THE MANNER AND FOR THE LENGTH OF TIME REQUIRED BY LAW TO JUSTIFY CONFIRMATION OF AN IMPERFECT TITLE. II THE TRIAL COURT ERRED IN NOT DECLARING THAT THE DE GUZMANS HAVE NOT OVERTHROWN THE PRESUMPTION THAT THE LANDS ARE PORTIONS OF THE PUBLIC DOMAIN BELONGING TO 4 THE REPUBLIC OF THE PHILIPPINES. We find merit in the instant Petition. It is not disputed that the subject parcels of land were released as agricultural land only in 1965 while the 6 petition for confirmation of imperfect title was filed by private respondents only in 1991. Thus the period of occupancy of the subject parcels of land from 1965 until the time the application was filed in 1991 was only twenty six (26) years, four (4) years short of the required thirty (30) year period possession requirement under Sec. 14, P.D. 29 andR.A. No. 6940. In finding that private respondents' possession of the subject property complied with law, the Court of Appeals reasoned out that (W)hile it is true that the land became alienable and disposable only in December, 1965, however, records indicate that as early as 1928, Pedro Ermitao, appellees' predecessor-ininterest, was already in possession of the property, cultivating it and planting various crops thereon. It follows that appellees' possession as of the time of the filing of the petition in 1991 when tacked to Pedro Ermitao's possession is 63 years or more than the required 30 years 7 period of possession. The land, which is agricultural, has been converted to private property. We disagree.
5

L A W O N N A T U R A L R E S O U R C E S a n d E N V I R O N M E N T A L L A W C a s e s a n d S p e c i a l L a w s | 85
The Court of Appeals' consideration of the period of possession prior to the time the subject land was released as agricultural is in direct contravention of the pronouncement in Almeda vs. Court of 8 Appeals, to wit The Court of Appeals correctly ruled that the private respondents had not qualified for a grant under Section 48(b) of the Public Land Act because their possession of the land while it was still inalienable forest land, or before it was declared alienable and disposable land of the public domain on January 13, 1968, could not ripen into private ownership, and should be excluded from the computation of the 30-year open and continuous possession in concept of owner required under Section 48(b) of Com. Act 141. It accords with our ruling in Director of Lands vs. Court of Appeals, Ibarra Bishar, et al., 178 SCRA 708, that: Unless and until the land classified as forest is released in an official proclamation to that effect so that it may form part of the disposable lands of the public domain, the rules on confirmation of imperfect title do not apply (Amunategui vs. Director of Forestry, 126 SCRA 69; Director of Lands vs. Court of Appeals, 129 SCRA 689; Director of Lands vs. Court of Appeals, 133 SCRA 701; Republic vs. Court of Appeals, 148 SCRA 480; Vallarta vs. Intermediate Appellate Court, 151 SCRA 679). Thus possession of forest lands, however long, cannot ripen into private ownership (Vamo vs. Government, 41 Phil. 161 [1920]; Adorable vs. Director of Forestry, 17 Phil. 410 [1960]). A parcel of forest land is within the exclusive jurisdiction of the Bureau of Forestry and beyond the power and jurisdiction of the cadastral court to register under the Torrens System (Republic vs. Court of Appeals, 89 SCRA 648; Republic vs. Vera, 120 SCRA 210 [1983]; Director of Lands vs. Court of Appeals, 129 SCRA 689 [1984]). (emphasis ours) So, too, is the Court of Appeals' reliance on the case of Director of Land Management vs. Court of 9 Appeals misplaced. There, while the period of possession of the applicant's predecessor-in-interest was tacked to his own possession to comply with the required thirty year period possession requirement, the land involved therein was not forest land but alienable public land. On the other hand, in the case before us, the property subject of private respondents' application was only declared alienable in 1965. Prior to such date, the same was forest land incapable of private appropriation. It was not registrable and possession thereof, no matter how lengthy, could not convert it into private property, (unless) and until 10 such lands were reclassified and considered disposable and alienable. In summary, therefore, prior to its declaration as alienable land in 1965, any occupation or possession thereon cannot be considered in the counting of the thirty year possession requirement. This is in accord with the ruling inAlmeda vs. Court of Appeals, (supra), and because the rules on the confirmation of imperfect titles do not apply unless and until the land classified as forest land is released in an official proclamation to that effect so that it may form part of the disposable agricultural lands of the public 11 domain. While we acknowledge the Court of Appeals' finding that private respondents and their predecessors-ininterest have been in possession of the subject land for sixty three (63) years at the time of the application of their petition, our hands are tied by the applicable laws and jurisprudence in giving practical relief to them. The fact remains that from the time the subject land was declared alienable until the time of their application, private respondents' occupation thereof was only twenty six (26) years. We cannot consider their thirty seven (37) years of possession prior to the release of the land as alienable because absent the fact of declassification prior to the possession and cultivation in good faith by petitioner, the property occupied by him remained classified as forest or timberland, which he could not have acquired by prescription. Further, jurisprudence is replete with cases which reiterate that forest lands or forest reserves are not capable of private appropriation and possession thereof, however long, cannot convert them into private property. Possession of the land by private respondents, whether spanning decades or centuries, could never ripen into ownership. This Court is constrained to abide by the latin maxim "(d)ura 12 lex, sed lex". WHEREFORE, the instant Petition is GRANTED and the February 26, 1998 decision of the Court of Appeals in CA-G.R. CV No. 48785 as well as that of the Regional Trial Court of Cavite, Branch 38, in LRC Case No. TG-396 are both REVERSED. Judgment is rendered dismissing LRC Case No. 396 for failure of the applicants therein to comply with the thirty year occupancy and possessory requirements of law for confirmation of imperfect title. No pronouncement as to costs.1wphi1.nt SO ORDERED. Davide, Jr., C.J., Puno, Kapunan and Pardo, JJ., concur.
Footnotes
1 2

CA-G.R. CV No. 48785, dated 26 February 1998; Petition, Annex "A"; Rollo, p. 24-38. Dated 8 September 1994; Records, LRC Case No. TG-362, pp. 440-454. 3 Id., at p. 14; Rollo, p. 454. 4 Petition, pp. 7-8; Rollo, pp. 12-13. 5 See Exhibit "S-4"; Records, p. 98. 6 See Petition; Records, LRC Case No. TG-396, pp. 1-18. 7 See Note 1, at p. 10; Rollo, p. 33. 8 G.R. No. 85322, 196 SCRA 476, 480 [1991]. 9 G.R. No. 94525, 205 SCRA 486 [1992]. 10 Palomo vs. Court of Appeals, G.R. No. 95608, 266 SCRA 392, 401 [1997]. 11 Ituralde vs. Falcasantos, G.R. No. 128017, 301 SCRA 293, 296 [1999], citing Sunbeam Convenience Foods, Inc. vs. Court of Appeals, 181 SCRA 443, 448 [1990]. 12 De la Cruz vs. Court of Appeals, G.R. No. 120652, 286 SCRA 230, 235 [1998].

L A W O N N A T U R A L R E S O U R C E S a n d E N V I R O N M E N T A L L A W C a s e s a n d S p e c i a l L a w s | 86
G.R. No. 133250 November 11, 2003 According to the zonal valuation of the Bureau of Internal Revenue, the value of the Property is Seven Thousand Eight Hundred Pesos (P7,800.00) per square meter. The Municipal Assessor of Paraaque, Metro Manila, where the Property is located, pegs the market value of the Property at Six Thousand Pesos (P6,000.00) per square meter. Based on these alone, the price at which PEA agreed to convey the property is a pittance. And PEA cannot claim ignorance of these valuations, at least not those of the Municipal Assessors office, since it has been trying to convince the Office of the Municipal Assessor of Paraaque to reduce the valuation of various reclaimed properties thereat in order for PEA to save on accrued real property taxes. PEAs justification for the purchase price are various appraisal reports, particularly the following: (1) An appraisal by Vic T. Salinas Realty and Consultancy Services concluding that the Property is worth P500.00 per square meter for the smallest island and P750.00 per square meter for the two other islands, or a total of P1,170,000.00 as of 22 February 1995; (2) An appraisal by Valencia Appraisal Corporation concluding that the Property is worth P850 per square meter for Island I, P800 per square meter for Island II and P600 per square meter for the smallest island, or a total of P1,289,732,000, also as of 22 February 1995; and (3) An Appraisal by Asian Appraisal Company, Inc. (AACI), stating that the Property is worth approximately P1,000 per square meter for Island I, P950 per square meter for Island II and P600 per square meter for Island III, or a total of P1,518,805,000 as of 27 February 1995. The credibility of the foregoing appraisals, however, are [sic] greatly impaired by a subsequent appraisal report of AACI stating that the property is worth P4,500.00 per square meter as of 26 March 1996. Such discrepancies in the appraised value as appearing in two different reports by the same appraisal company submitted within a span of one year render all such appraisal reports unworthy of even the slightest consideration. Furthermore, the appraisal report submitted by the Commission on Audit estimates the value of the Property to be approximately P33,673,000,000.00, or P21,333.07 per square meter. There were also other offers made for the property from other parties which indicate that the Property has been undervalued by PEA. For instance, on 06 March 1995, Mr. Young D. See, President of Saeil Heavy Industries Co., Ltd., (South Korea), offered to buy the property at P1,400.00 and expressed its willingness to issue a stand-by letter of credit worth $10 million. PEA did not consider this offer and instead finalized the JVA with AMARI. Other offers were made on various dates by Aspac Management and Development Group Inc. (for P1,600 per square meter), Universal Dragon Corporation (for P1,600 per square meter), Cleene Far East Manila

FRANCISCO I. CHAVEZ, petitioner, vs. PUBLIC ESTATES AUTHORITY and AMARI COASTAL BAY DEVELOPMENT CORPORATION, respondents. RESOLUTION CARPIO, J.: This Court is asked to legitimize a government contract that conveyed to a private entity 157.84 hectares of reclaimed public lands along Roxas Boulevard in Metro Manila at the negotiated price of P1,200 per square meter. However, published reports place the market price of land near that area at that time at a 1 high of P90,000 per square meter. The difference in price is a staggering P140.16 billion, equivalent to the budget of the entire Judiciary for seventeen years and more than three times the Marcos Swiss deposits that this Court forfeited in favor of the government. Many worry to death that the private investors will lose their investments, at most not more than one2 half billion pesos in legitimate expenses, if this Court voids the contract. No one seems to worry about the more than tens of billion pesos that the hapless Filipino people will lose if the contract is allowed to stand. There are those who question these figures, but the questions arise only because the private entity somehow managed to inveigle the government to sell the reclaimed lands without public bidding in patent violation of the Government Auditing Code. Fortunately for the Filipino people, two Senate Committees, the Senate Blue Ribbon Committee and the Committee on Accountability of Public Officers, conducted extensive public hearings to determine the actual market value of the public lands sold to the private entity. The Senate Committees established the clear, indisputable and unalterable fact that the sale of the public lands is grossly and unconscionably undervalued based on official documents submitted by the proper government agencies during the Senate investigation. We quote the joint report of these two Senate Committees, Senate Committee 3 Report No. 560, as approved by the Senate in plenary session on 27 September 1997: The Consideration for the Property PEA, under the JVA, obligated itself to convey title and possession over the Property, consisting of approximately One Million Five Hundred Seventy Eight Thousand Four Hundred Forty One (1,578,441) Square Meters for a total consideration of One Billion Eight Hundred Ninety Four Million One Hundred Twenty Nine Thousand Two Hundred (P1,894,129,200.00) Pesos, or a price of One Thousand Two Hundred (P1,200.00) Pesos per square meter.

L A W O N N A T U R A L R E S O U R C E S a n d E N V I R O N M E N T A L L A W C a s e s a n d S p e c i a l L a w s | 87
Incorporated and Hyosan Prime Construction Co. Ltd. which had prepared an Irrevocable Clean Letter of Credit for P100,000,000. In addition, AMARI agreed to pay huge commissions and bonuses to various persons, amounting to P1,596,863,050.00 (P1,754,707,150.00 if the bonus is included), as will be discussed fully below, which indicate that AMARI itself believed the market value to be much higher than the agreed purchase price. If such commissions are added to the purchase price, AMARIs acquisition cost for the Property will add-up to P3,490,992,250.00 (excluding the bonus). If AMARI was willing to pay such amount for the Property, why was PEA willing to sell for only P1,894,129,200.00, making the Government stand to lose approximately P1,596,863,050.00? x x x government lands, is grossly and unconscionably undervalued. The authoritative appraisal, of course, is that of the Commission on Audit which valued the 157.84 hectares at P21,333.07 per square meter or a total of P33.673 billion. Thus, based on the official appraisal of the Commission on Audit, the independent constitutional body that safeguards government assets, the actual loss to the Filipino people is a shocking P31.779 billion. This gargantuan monetary anomaly, aptly earning the epithet "Grandmother of All Scams," is not the major defect of this government contract. The major flaw is not even the P1.754 billion in commissions the Senate Committees discovered the private entity paid to various persons to secure the 5 contract, described in Senate Report No. 560 as follows: A Letter-Agreement dated 09 June 1995 signed by Messrs. Premchai Karnasuta and Emmanuel Sy for and in behalf of AMARI, on the one hand, and stockholders of AMARI namely, Mr. Chin San Cordova (a.k.a. Benito Co) and Mr. Chua Hun Siong (a.k.a. Frank Chua), on the other, sets forth various payments AMARI paid or agreed to pay the aforesaid stockholders by way of fees for "professional efforts and services in successfully negotiating and securing for AMARI the Joint Venture Agreement", as follows: Form of Payment Managers Checks Managers Checks Clearly, the purchase price agreed to by PEA is way below the actual value of the Property, thereby subjecting the Government to grave injury and enabling AMARI to enjoy tremendous benefit and advantage. (Emphasis supplied) The Senate Committee Report No. 560 attached the following official documents from the Bureau of Internal Revenue, the Municipal Assessor of Paraaque, Metro Manila, and the Commission on Audit: 1. Annex "M," Certified True Copy of BIR Zonal Valuations as certified by Antonio F. Montemayor, Revenue District Officer. This official document fixed the market value of the 157.84 hectares at P7,800 per square meter. 2. Annex "N," Certification of Soledad S. Medina-Cue, Municipal Assessor, Paraaque, dated 10 December 1996. This official document fixed the market value at P6,000 per square meter. 3. Exhibit "1-Engr. Santiago," the Appraisal Report of the Commission on Audit. This official document fixed the market value at P21,333.07 per square meter. Whether based on the official appraisal of the BIR, the Municipal Assessor or the Commission on Audit, the P1,200 per square meter purchase price, or a total of P1.894 billion for the 157.84 hectares of 10 Post Dated Checks (PDCs) 24 PDCs 48 PDCs Cash bonus Developed land from Project Paid/Payable On 28 April 1995 Upon signing of letter 60 days from date of letter 31 Aug. 95 to 31 Jan. 98 Amount P 400,000,000.00 262,500,000.00 127,000,000.00 150,000,000.00
4

Even if we simply assume that the market value of the Property is half of the market value fixed by the Municipal Assessors Office of Paraaque for lands along Roxas Boulevard, or P3,000.00 per square meter, the Government now stands to lose approximately P2,841,193,800.00. But an even better assumption would be that the value of the Property is P4,500.00 per square meter, as per the AACI appraisal report dated 26 March 1996, since this is the valuation used to justify the issuance of P4 billion worth of shares of stock of Centennial City Inc. (CCI) in exchange for 4,800,000 AMARI shares with a total par value of only P480,000,000.00. With such valuation, the Governments loss will amount to P5,208,855,300.00.

Monthly, over a 12-month pd. 357,363,050.00 from date of letter When sale of land begins Upon completion phase TOTAL not exceeding 157,844,100.00 of each Costing 300,000,000.00 P1,754,707,150.00 ==============

Mr. Luis Benitez of SGV, the external auditors of AMARI, testified that said Letter-Agreement 6 was approved by the AMARI Board. (Emphasis supplied)

L A W O N N A T U R A L R E S O U R C E S a n d E N V I R O N M E N T A L L A W C a s e s a n d S p e c i a l L a w s | 88
The private entity that purchased the reclaimed lands for P1.894 billion expressly admitted before the Senate Committees that it spent P1.754 billion in commissions to pay various individuals for " professional efforts and services in successfully negotiating and securing " the contract. By any legal or moral yardstick, the P1.754 billion in commissions obviously constitutes bribe money. Nonetheless, there are those who insist that the billions in investments of the private entity deserve protection by this Court. Should this Court establish a new doctrine by elevating grease money to the status of legitimate investments deserving of protection by the law? Should this Court reward the patently illegal and grossly unethical business practice of the private entity in securing the contract? Should we allow those with hands dripping with dirty money equitable relief from this Court? Despite these revolting anomalies unearthed by the Senate Committees, the fatal flaw of this contract is that it glaringly violates provisions of the Constitution expressly prohibiting the alienation of lands of the public domain. Thus, we now come to the resolution of the second Motions for Reconsideration filed by public respondent Public Estates Authority ("PEA") and private respondent Amari Coastal Bay Development Corporation ("Amari"). As correctly pointed out by petitioner Francisco I. Chavez in his Consolidated 8 Comment, the second Motions for Reconsideration raise no new issues. However, the Supplement to "Separate Opinion, Concurring and Dissenting" of Justice Josue N. Bellosillo brings to the Courts attention the Resolutions of thi s Court on 3 February 1965 and 24 June 1966 in L21870 entitled"Manuel O. Ponce, et al. v. Hon. Amador Gomez, et al." and No. L-22669 entitled "Manuel O. Ponce, et al. v. The City of Cebu, et al." ("Ponce Cases") . In effect, the Supplement to the Dissenting Opinion claims that these two Resolutions serve as authority that a single private corporation like Amari may acquire hundreds of hectares of submerged lands, as well as reclaimed submerged lands, within Manila Bay under the Amended Joint Venture Agreement ("Amended JVA"). We find the cited Ponce Cases inapplicable to the instant case. First, as Justice Bellosillo himself states in his supplement to his dissent, the Ponce Cases admit 9 that "submerged lands still belong to the National Government. " The correct formulation, however, is that submerged lands are owned by the State and are inalienable . Section 2, Article XII of the 1987 Constitution provides: All lands of the public domain, waters, minerals, coal, petroleum, and other mineral oils, all forces of potential energy, fisheries, forests or timber, wildlife, flora and fauna, and other natural resources are owned by the State. With the exception of agricultural lands, all other natural resources shall not be alienated. x x x. (Emphasis supplied) Submerged lands, like the waters (sea or bay) above them, are part of the States inalienable natural resources. Submerged lands are property of public dominion, absolutely inalienable and outside the
7

commerce of man. This is also true with respect to foreshore lands. Any sale of submerged or foreshore 11 lands is void being contrary to the Constitution. This is why the Cebu City ordinance merely granted Essel, Inc. an "irrevocable option" to purchase the foreshore lands after the reclamation and did not actually sell to Essel, Inc. the still to be reclaimed foreshore lands. Clearly, in the Ponce Cases the option to purchase referred to reclaimed lands, and not to foreshore lands which are inalienable. Reclaimed lands are no longer foreshore or submerged lands, and thus may qualify as alienable agricultural lands of the public domain provided the requirements of public land laws are met. In the instant case, the bulk of the lands subject of the Amended JVA are still submerged lands even to this very day, and therefore inalienable and outside the commerce of man. Of the 750 hectares subject of the Amended JVA, 592.15 hectares or 78% of the total area are still submerged, permanently under the waters of Manila Bay. Under the Amended JVA, the PEA conveyed to Amari the submerged lands even before their actual reclamation, although the documentation of the deed of transfer and issuance of the certificates of title would be made only after actual reclamation. The Amended JVA states that the PEA "hereby contributes to the Joint Venture its rights and privileges to 12 perform Rawland Reclamation and Horizontal Development as well as own the Reclamation Area." The Amended JVA further states that "the sharing of the Joint Venture Proceeds shall be based on the ratio of 13 thirty percent (30%) for PEA and seventy percent (70%) for AMARI." The Amended JVA also provides that the PEA "hereby designates AMARI to perform PEAs rights and privileges to reclaim, own and 14 develop the Reclamation Area." In short, under the Amended JVA the PEA contributed its rights, privileges and ownership over the Reclamation Area to the Joint Venture which is 70% owned by Amari. Moreover, the PEA delegated to Amari the right and privilege to reclaim the submerged lands. The Amended JVA mandates that the PEA had "the duty to execute without delay the necessary deed of transfer or conveyance of the title pertaining to AMARIs Land share based on the Land Allocation 15 Plan." The Amended JVA also provides that "PEA, when requested in writing by AMARI, shall then cause the issuance and delivery of the proper certificates of title covering AMARIs Land Share in the name of 16 AMARI, x x x." In the Ponce Cases, the City of Cebu retained ownership of the reclaimed foreshore lands and Essel, Inc. only had an "irrevocable option" to purchase portions of the foreshore lands once actually reclaimed. In sharp contrast, in the instant case ownership of the reclamation area, including the submerged lands, was immediately transferred to the joint venture. Amari immediately acquired the absolute right to own 70% percent of the reclamation area, with the deeds of transfer to be documented and the certificates of title to be issued upon actual reclamation. Amaris right to own the submerged lands is immediately effective upon the approval of the Amended JVA and not merely an option to be exercised in the future if and when the reclamation is actually realized. The submerged lands, being inalienable and outside the commerce of man, could not be the subject of the commercial transactions specified in the Amended JVA.

10

L A W O N N A T U R A L R E S O U R C E S a n d E N V I R O N M E N T A L L A W C a s e s a n d S p e c i a l L a w s | 89
Second, in the Ponce Cases the Cebu City ordinance granted Essel, Inc. an "irrevocable option" to purchase from Cebu City not more than 70% of the reclaimed lands. The ownership of the reclaimed lands remained with Cebu City until Essel, Inc. exercised its option to purchase. With the subsequent enactment of the Government Auditing Code (Presidential Decree No. 1445) on 11 June 1978, any sale of government land must be made only through public bidding. Thus, such an "irrevocable option" to purchase government land would now be void being contrary to the requirement of public bidding 17 expressly required in Section 79 of PD No. 1445. This requirement of public bidding is reiterated in 18 19 Section 379 of the 1991 Local Government Code. Obviously, the ingenious reclamation scheme adopted in the Cebu City ordinance can no longer be followed in view of the requirement of public bidding in the sale of government lands. In the instant case, the Amended JVA is a negotiated contract which clearly contravenes Section 79 of PD No. 1445. Third, Republic Act No. 1899 authorized municipalities and chartered cities to reclaim foreshore lands. The two Resolutions in the Ponce Cases upheld the Cebu City ordinance only with respect to foreshore areas, and nullified the same with respect to submerged areas. Thus, the 27 June 1965 Resolution made the injunction of the trial court against the City of Cebu "permanent insofar x x x as the area outside or beyond the foreshore land proper is concerned." As we held in the 1998 case of Republic Real Estate Corporation v. Court of Appeals , citing the Ponce Cases, RA No. 1899 applies only to foreshore lands, not to submerged lands. In his concurring opinion inRepublic Real Estate Corporation, Justice Reynato S. Puno stated that under Commonwealth Act No. 141, "foreshore and lands under water were not to be alienated and sold to private parties," and that such lands "remained property of the State." Justice Puno emphasized that "Commonwealth Act No. 141 has remained in effect at present." The instant case involves principally submerged lands within Manila Bay. On this score, the Ponce Cases, which were decided based on RA No. 1899, are not applicable to the instant case. Fourth, the Ponce Cases involve the authority of the City of Cebu to reclaim foreshore areas pursuant to a general law, RA No. 1899. The City of Cebu is a public corporation and is qualified, under the 1935, 1973, and 1987 Constitutions, to hold alienable or even inalienable lands of the public domain. There is no dispute that a public corporation is not covered by the constitutional ban on acquisition of alienable public lands. Both the 9 July 2002 Decision and the 6 May 2003 Resolution of this Court in the instant case expressly recognize this. Cebu City is an end user government agency, just like the Bases Conversion and Development Authority or 21 the Department of Foreign Affairs. Thus, Congress may by law transfer public lands to the City of Cebu to be used for municipal purposes, which may be public or patrimonial. Lands thus acquired by the City of Cebu for a public purpose may not be sold to private parties. However, lands so acquired by the City of Cebu for a patrimonial purpose may be sold to private parties, including private corporations. However, in the instant case the PEA is not an end user agency with respect to the reclaimed lands under the Amended JVA. As we explained in the 6 May 2003 Resolution:
20

PEA is the central implementing agency tasked to undertake reclamation projects nationwide. PEA took the place of the Department of Environment and Natural Resources ("DENR" for brevity) as the government agency charged with leasing or selling all reclaimed lands of the public domain. In the hands of PEA, which took over the leasing and selling functions of DENR, reclaimed foreshore (or submerged lands) lands are public lands in the same manner that these same lands would have been public lands in the hands of DENR. (Emphasis supplied) Our 9 July 2002 Decision explained the rationale for treating the PEA in the same manner as the DENR with respect to reclaimed foreshore or submerged lands in this wise: To allow vast areas of reclaimed lands of the public domain to be transferred to PEA as private lands will sanction a gross violation of the constitutional ban on private corporations from acquiring any kind of alienable land of the public domain. PEA will simply turn around, as PEA has now done under the Amended JVA, and transfer several hundreds of hectares of these reclaimed and still to be reclaimed lands to a single private corporation in only one transaction. This scheme will effectively nullify the constitutional ban in Section 3, Article XII of the 1987 Constitution which was intended to diffuse equitably the ownership of alienable lands of the public domain among Filipinos, now numbering over 80 million strong. (Emphasis supplied) Finally, the Ponce Cases were decided under the 1935 Constitution which allowed private corporations to acquire alienable lands of the public domain. However, the 1973 Constitution prohibited private corporations from acquiring alienable lands of the public domain, and the 1987 Constitution reiterated this prohibition. Obviously, the Ponce Cases cannot serve as authority for a private corporation to acquire alienable public lands, much less submerged lands, since under the present Constitution a private corporation like Amari is barred from acquiring alienable lands of the public domain. Clearly, the facts in the Ponce Cases are different from the facts in the instant case. Moreover, the governing constitutional and statutory provisions have changed since the Ponce Cases were disposed of in 1965 and 1966 through minute Resolutions of a divided (6 to 5) Court. This Resolution does not prejudice any innocent third party purchaser of the reclaimed lands covered by the Amended JVA. Neither the PEA nor Amari has sold any portion of the reclaimed lands to third parties. Title to the reclaimed lands remains with the PEA. As we stated in our 9 July 2002 Decision: In the instant case, the only patent and certificates of title issued are those in the name of PEA, a wholly government owned corporation performing public as well as proprietary functions. No patent or certificate of title has been issued to any private party. No one is asking the Director of Lands to cancel PEAs patent or certificates of title. In fact, the thrust of the instant petition is that PEAs certificates of title should remain with PEA, and the land covered by these certificates, being alienable lands of the public domain, should not be sold to a private corporation.

L A W O N N A T U R A L R E S O U R C E S a n d E N V I R O N M E N T A L L A W C a s e s a n d S p e c i a l L a w s | 90
As we held in our 9 July 2002 Decision, the Amended JVA "violates glaringly Sections 2 and 3, Article XII of the 1987 Constitution." In our 6 May 2003 Resolution, we DENIED with FINALITY respondents Motions for Reconsideration. Litigations must end some time. It is now time to write finis to this "Grandmother of All Scams." WHEREFORE, the second Motions for Reconsideration filed by Public Estates Authority and Amari Coastal Bay Development Corporation are DENIED for being prohibited pleadings. In any event, these Motions for Reconsideration have no merit. No further pleadings shall be allowed from any of the parties. SO ORDERED. Davide, Jr., C.J., Panganiban, Austria-Martinez, Carpio-Morales, and Callejo, Sr., JJ., concur. Bellosillo, J., voted to grant reconsideration, pls. see dissenting opinion. Puno, J., maintains previous qualified opinion. Vitug, J., pls. see separate(concurring) opinion. Quisumbing, J., voted to allow reconsideration, see separate opinion. Ynares-Santiago, Sandoval-Gutierrez, and Corona, JJ., maintains their dissent. Azcuna, J., no part. Tinga, J., see dissenting opinion.
Footnotes
1

Form of Payment Managers Checks Managers Checks 10 Post Dated Checks (PDCs) 24 PDCs 48 PDCs Cash bonus Developed land from Project

Paid/Payable On 28 April 1995 Upon signing of letter 60 days from date of letter 31 Aug. 95 to 31 Jan. 98

Amount P 400,000,000.00 262,500,000.00 127,000,000.00 150,000,000.00

Monthly, over a 12-month pd. 357,363,050.00 from date of letter When sale of land begins Upon completion of each Phase TOTAL not exceeding 157,844,100.00 Costing 300,000,000.00 P1,754,707,150.00

See "The Grandmother of All Scams" by Sheila S. Coronel and Ellen Tordesillas, 18-20 March 1998, Philippine Center for Investigative Journalism. This report won the 1st Prize in the 1998 JVO Investigative Journalism Awards. 2 6 May 2003 Resolution, p. 13. 3 PEAs Memorandum dated 4 August 1999 quoted extensively, in its Statement of Facts and the Case, the Statement of Facts in Senate Committee Report No. 560 dated 16 September 1997. Moreover, the existence of this report is a matter of judicial notice pursuant to Section 1, Rule 129 of the Rules of Court which provides, "A court shall take judicial notice, without the introduction of evidence, of x x x the official acts of the legislature." 4 9 July 2002 Decision, p. 4. 5 Senate Committee Report No. 560, p. 48. 6 A more detailed discussion on this matter in Senate Report No. 560 reads as follows: The Commissions A Letter-Agreement dated 09 June 1995 signed by Messrs. Premchai Karnasuta and Emmanuel Sy for and in behalf of AMARI, on the one hand, and stockholders of AMARI namely, Mr. Chin San Cordova (a.k.a. Benito Co) and Mr. Chua Hun Siong (a.k.a. Frank Chua), on the other, sets forth various payments AMARI paid or agreed to pay the aforesaid stockholders by way of fees for "professional efforts and services in successfully negotiating and securing for AMARI the Joint Venture Agreement", as follows:

============== Mr. Luis Benitez of SGV, the external auditors of AMARI, testified that said Letter-Agreement was approved by the AMARI Board. On the first payment of P400 million, records show that P300 million was paid in managers checks of Citibank-Makati, while the balance of P100 million was deposited to the account of the two Chinese in a Hongkong bank. On the basis of a Memorandum Order dated April 28, 1995 issued by Messrs. Karnasuta and Emmanuel Sy, and upon the instruction of Messrs. Chin San Cordova and Chua Hun Siong, 31 managers checks in the total amount of P300 million were issued by Citibank-Makati in favor of a Mr. George Trivio, a Dominican Republic national, broken down as follows: 1) Twenty-nine (29) managers checks at P10 million each; 2) One (1) managers check at P7 million; and, One (1) managers check at P3 million. All these checks were indorsed by Mr. Trivio. Mr. Sy could not satisfactorily answer why Mr. Trivio was made payee of the Managers Checks when he had nothing to do with the transactions. Neither could he provide information regarding the said Mr. Trivio. Mr. Emmanuel Sy admitted signing several blank checks as special request from Messrs. Co and Chua and issuing said checks as follows: 1) Ten (10) Managers checks dated 60 days from the June 9 letter amounting to P127 million; 2) Twenty-four (24) blank checks amounting to P150 million dated from 31 August 1995 up to 31 January 1998; and, 3) Forty (40) blank checks amounting to P357 million. In this regard, the pertinent portion of the 9 June 1995 letter-agreement provides as follows: "3. Upon signing of this letter-agreement AMARI shall (a) pay to you (in cash in the form of Bank Managers Checks) the sum of Two Hundred Sixty Two Million Five Hundred Thousand Pesos (Pesos 262,500,000) and (b) pay and deliver to you the following checks:

L A W O N N A T U R A L R E S O U R C E S a n d E N V I R O N M E N T A L L A W C a s e s a n d S p e c i a l L a w s | 91
"3.1 Ten (10) checks dated sixty (60) days from date of this letter agreement in the total amount of One Hundred Twenty Seven Million Pesos (Pesos 127,000,000); "3.2 Twenty-Four (24) checks in the total amount of One Hundred Fifty Million Pesos (Pesos 150,000,000) as follows: DUE DATE OF CHECK AMOUNT August 31, 1995 March 31, 1996 April 30, 1996 May 31, 1996 June 30, 1996 July 31, 1996 August 31, 1996 September 30, 1996 October 31, 1996 November 30, 1996 December 31, 1996 January 31, 1997 February 28, 1997 March 31,1997 April 30, 1997 May 31, 1997 June 30, 1997 July 31, 1997 August 31, 1997 September 30, 1997 October 31, 1997 November 30, 1997 December 31, 1997 January 31, 1998 Total P 6,250,000 6,250,000 6,250,000 6,250,000 6,250,000 6,250,000 6,250,000 6,250,000 6,250,000 6,250,000 6,250,000 6,250,000 6,250,000 6,250,000 6,250,000 6,250,000 6,250,000 6,250,000 6,250,000 6,250,000 6,250,000 6,250,000 6,250,000 6,250,000 P150,000,000 "Each monthly payment to consist of Four (4) checks, three (3) checks of which shall each bear the amount of P7,250,000 and one (1) check of which shall bear the amount of P8,000,000 for a total monthly amount of P29,750,000. These monthly payment of four (4) checks each shall be dated the last date of the thirteen, fourteen, fifteen, sixteen, seventeen, eighteen, nineteen, twenty, twenty-one, twenty-two, twenty-three, and twenty-four months from the date of this letter agreement. The last issued check hereunder shall bear the sum of P8,363,050." The Provisional Receipt shows that Mr. Chin San Cordova and Mr. Chua Hun Siong received the amount of P896,863,050.00 as of 09 June 1995. Based on the submitted photocopies of the returned checks issued by AMARI vis-a-vis item 3(b) of the quoted Letter-Agreement, the following persons were made payees: Emmanuel Sy, Manuel Sy, Sy Pio Lato, International Merchandising and Development Corporation, Golden Star Industrial Corporation, Chin San Cordova, EY, and Wee Te Lato. Other payments were made payable to Cash (bearer instruments). Each person was thus named payee to the following amounts: 1. Emmanuel Sy: Citibank Check No. 000019 dated 10/31/96 2. Manuel Sy: Citibank Check No. 000007 dated 8/8/95 3. Sy Pio Lato: Citibank Check No. 000008 dated 8/8/95 000009 dated 8/8/95 000010 dated 8/8/95 4. International Merchandising and Development Corporation: Citibank Check No. 000013 dated 4/30/96 000014 dated 5/31/96 000015 dated 6/30/96 000016 dated 7/31/96 000045 dated 9/30/96 5. Golden Star Industrial Corporation: Citibank Check No. 000018 dated 9/30/96 6. Chin San Cordova: Citibank Check No. 000041 dated 8/31/96 000043 dated 9/30/96 7. EY: Citibank Check No. 000047 dated 10/31/96 000049 dated 10/31/96 8. Wee Te Lato: 7,250,000 7,250,000 7,250,000 7,250,000 6,250,000 6,250,000 6,250,000 6,250,000 6,250,000 7,250,000 12,700,000 12,700,000 12,700,000 12,700,000 P 6,250,000

========== "3.3 Forty Eight (48) checks in the total amount of Three Hundred Fifty Seven Million Three Hundred Sixty Three Thousand Fifty Pesos (Pesos 357,363,050) payable over a period of twelve (12) months as follows:

L A W O N N A T U R A L R E S O U R C E S a n d E N V I R O N M E N T A L L A W C a s e s a n d S p e c i a l L a w s | 92
Citibank Check No. 000048 dated 10/31/96 9. Bearer Instruments: CASH: Citibank Check No. 000001 dated 8/8/95 000002 dated 8/8/95 000003 dated 8/8/95 000004 dated 8/8/95 000005 dated 8/8/95 000006 dated 8/8/95 000012 dated 3/31/96 000017 dated 8/31/96 000039 dated 8/31/96 000040 dated 8/31/96 000042 dated 8/31/95 000044 dated 9/30/96 000046 dated 9/30/96 000050 dated 10/31/96 10. Payees Name Not Legible: Citibank Check No. 000011 dated 8/31/96 6,250,000 On the other hand, Ms. Aurora Montano, a cousin of Mr. Justiniano Montano IV, was asked by a Mr. Ben Cuevo if she knew anybody from PEA, and she answered: "Yes, I know Mr. Justiniano Montano IV." For this answer, and for introducing the AMARI representative to Mr. Montano, she received P10 million in cash and P20 million in postdated managers checks in the office of Mr. Benito Co and in the presence of, aside from Mr. Benito Co, Mr. Ben Cuevo and Mr. Frank Chua. Ms. Montano, however, insisted that she actually received only P10 million. Ms. Montano furthermore admitted that, through Mr. Ben Cuevo, she met Messrs. Chin San Cordova and Chua Hun Siong in 1994 for this transaction. In Executive Session, Mr. Ben Cuevo admitted to having encashed two checks at Pilipinas Bank, worth P12.5 million. According to him, the two checks form part of the P150 million worth of post-dated checks (PDCs), with a face value of P6.25 million per check, described in the LetterAgreement. Of this P150 million, Mr. Cuevo actually received five (5) PDCs worth P31 million, but he was only able to encash 2 checks at P12.5 million. Still in Executive Session, Mr. Ben Cuevo also admitted receiving a check worth P6.25 million payable to his company, International Merchandising and Development Corporation. This was deposited in his Current Account No. 604010562-A, and the amount was transferred by credit memo to Mr. Montano IVs account at Pilipinas Bank. Mr. Montano IV admitted that he has an account with Pilipinas Bank, but invoked his constitutional right against self-incrimination when asked if he received the amount of P6.25 million transferred to his account. The Pilipinas Bank Credit Advice dated May 6, 1996, marked as Exhibit 1-Montano IV, indicating the transfer of the amount of P6.25 million was presented by 12,700,000 12,700,000 12,700,000 12,700,000 12,700,000 12,700,000 6,250,000 6,250,000 7,250,000 7,250,000 8,000,000 7,250,000 7,250,000 8,000,000 7,250,000 Senator Drilon. Once or twice, a certain Ms. Polly Tragico accompanied Mr. Montano IV to withdraw funds from Pilipinas Bank-Pavilion. 7 Both filed on 26 May 2003. On 6 June 2003 Amari filed a Supplement to its second Motion for Reconsideration. 8 Filed on 19 August 2003. 9 Decision dated 17 January 1964 of Judge Amador E. Gomez. Also quoted in Justice Josue N. Bellosillos Supplement to Separate Opinion, Concurring and Dissenting. 10 Sections 2 and 3, Article XII of the 1987 Constitution. 11 Article 112 , Civil Code of the Philippines. 12 Section 3.2 (a), Amended JVA. 13 Section 3.3 (a), Amended JVA. 14 Section 2.2, Amended JVA. 15 Section 5.2 (c), Amended JVA. 16 Ibid. 17 SECTION 79. Destruction or sale of unserviceable property. When government property has become unserviceable for any cause, or is no longer needed, it shall, upon application of the officer accountable therefor, be inspected by the head of the agency or his duly authorized representative in the presence of the auditor concerned and, if found to be valueless or unsalable, it may be destroyed in their presence. If found to be valuable, it may be sold at public auction to the highest bidder under the supervision of the proper committee on awards or similar body in the presence of the auditor concerned or other duly authorized representative of the Commission, after advertising by printed notice in the Official Gazette, or for not less than three consecutive days in any newspaper of general circulation, or where the value of the property does not warrant the expense of publication, by notices posted for a like period in at least three public places in the locality where the property is to be sold. In the event that the public auction fails, the property may be sold at a private sale at such price as may be fixed by the same committee or body concerned and approved by the Commission. 18 SECTION 379. Property Disposal. When property of any local government unit has become unserviceable for any cause or is no longer needed, it shall upon application of the officer accountable therefor, be inspected and appraised by the provincial, city or municipal auditor, as the case may be, or his duly authorized representative or that of the Commission on Audit and, if found valueless or unusable, shall be destroyed in the presence of the inspecting officer. If found valuable, the same shall be sold at public auction to the highest bidder under the supervision of the committee on awards and in the presence of the provincial, city or municipal auditor or his duly authorized representative. Notice of the public auction shall be posted in at least three (3) publicly accessible and conspicuous places, and if the acquisition cost exceeds One hundred thousand pesos (P100,000.00) in the case of provinces and cities, and Fifty thousand pesos (P50,000.00) in the case of municipalities, notice of auction shall be published at least two (2) times within a reasonable period in a newspaper of general circulation in the locality. 19 Under Section 380 of the 1991 Local Government Code, local governments can sell real property through negotiated sale only with the approval of the Commission on Audit. Under paragraph 2 (a) of COA Circular No. 89-296, on "Sale Thru Negotiation," a negotiated sale may be resorted to only if "[T]here was a failure of public auction." The Commission on Audit enforces the express requirement in Section 79 of the Government Auditing Code that a negotiated sale is possible only after there is a failure of public auction. 20 359 Phil. 530 (1998). 21 Laurel v. Garcia, G.R. No. 92013, 25 July 1990, 187 SCRA 797.

L A W O N N A T U R A L R E S O U R C E S a n d E N V I R O N M E N T A L L A W C a s e s a n d S p e c i a l L a w s | 93
November 7, 1936 December 1, 1936 CHAPTER II Classification, Delimitation, and Survey of Lands of the Public Domain, for the Concession Thereof SECTION 6. The President, upon the recommendation of the Secretary of Agriculture and Commerce, shall from time to time classify the lands of the public domain into COMMONWEALTH ACT NO. 141* (a) (b) AN ACT TO AMEND AND COMPILE THE LAWS RELATIVE TO LANDS OF THE PUBLIC DOMAIN (c) Mineral lands, Alienable or disposable; Timber, and

TITLE I TITLE AND APPLICATION OF THE ACT, LANDS TO WHICH IT REFERS, AND CLASSIFICATION, DELIMITATION, AND SURVEY THEREOF FOR CONCESSION

and may at any time and in a like manner transfer such lands from one class to another, for the purposes of their administration and disposition. SECTION 7. For the purposes of the administration and disposition of alienable or disposable public lands, the President, upon recommendation by the Secretary of Agriculture and Commerce, shall from time to time declare what lands are open to disposition or concession under this Act. SECTION 8. Only those lands shall be declared open to disposition or concession which have been officially delimited and classified and, when practicable, surveyed, and which have not been reserved for public or quasi-public uses, nor appropriated by the Government, nor in any manner become private property, nor those on which a private right authorized and recognized by this Act or any other valid law may be claimed, or which, having been reserved or appropriated, have ceased to be so However, the President may, for reasons of public interest, declare lands of the public domain open to disposition before the same have had their boundaries established or been surveyed, or may, for the same reason, suspend their concession or disposition until they are again declared open to concession or disposition by proclamation duly published or by Act of the National Assembly. SECTION 9. For the purpose of their administration and disposition, the lands of the public domain alienable or open to disposition shall be classified, according to the use or purposes to which such lands are destined, as follows: (a) (b) (c) (d) Agricultural Residential commercial industrial or for similar productive purposes Educational, charitable, or other similar purposes Reservations for town sites and for public and quasi-public uses.

CHAPTER I Short Title of the Act, Lands to Which it Applies, and Officers Charged With Its Execution SECTION 1. The short title of this Act shall be "The Public Land Act. SECTION 2. The provisions of this Act shall apply to the lands of the public domain; but timber and mineral lands shall be governed by special laws and nothing in this Act provided shall be understood or construed to change or modify the administration and disposition of the lands commonly called "friar lands'' and those which, being privately owned, have reverted to or become the property of the Commonwealth of the Philippines, which administration and disposition shall be governed by the laws at present in force or which may hereafter be enacted. SECTION 3. The Secretary of Agriculture and Commerce shall be the executive officer charged with carrying out the provisions of this Act through the Director of Lands, who shall act under his immediate control. SECTION 4. Subject to said control, the Director of Lands shall have direct executive control of the survey, classification, lease, sale or any other form of concession or disposition and management of the lands of the public domain, and his decisions as to questions of fact shall be conclusive when approved by the Secretary of Agriculture and Commerce. SECTION 5. The Director of Lands, with the approval of the Secretary of Agriculture and Commerce shall prepare and issue such forms, instructions, rules, and regulations consistent with this Act, as may be necessary and proper to carry into effect the provisions thereof and for the conduct of proceedings arising under such provisions.

L A W O N N A T U R A L R E S O U R C E S a n d E N V I R O N M E N T A L L A W C a s e s a n d S p e c i a l L a w s | 94
The President, upon recommendation by the Secretary of Agriculture and Commerce, shall from time to time make the classifications provided for in this section, and may, at any time and in a similar manner, transfer lands from one class to another. SECTION 10. The words "alienation, "'disposition, or "concession" as used in this Act, shall mean any of the methods authorized by this Act for the acquisition, lease, use, or benefit of the lands of the public domain other than timber or mineral lands. SECTION 14. No certificate shall be given or patent issued for the land applied for until at least one-fifth of the land has been improved and cultivated. The period within which the land shall be cultivated shall not be less than one or more than five years, from and after the date of the approval of the application. The applicant shall, within the said period, notify the Director of Lands as soon as he is ready to acquire the title. If at the date of such notice, the applicant shall prove to the satisfaction of the Director of Lands, that he has resided continuously for at least one year in the municipality in which the land is located, or in a municipality adjacent to the same, and has cultivated at least one-fifth of the land continuously since the approval of the application, and shall make affidavit that no part of said land has been alienated or encumbered, and that he has complied with all the requirements of this Act, then, upon the payment of five pesos, as final fee, he shall be entitled to a patent. SECTION 15. At the option of the applicant, payment of the fees required in this chapter may be made to the municipal treasurer of the locality, who, in turn, shall forward them to the provincial treasurer. In case of delinquency of the applicant, the Director of Lands may, sixty days after such delinquency has occurred, either cancel the application or grant an extension of time not to exceed one hundred and twenty days for the payment of the sum due. SECTION 16. If at any time before the expiration of the period allowed by law for the making of final proof, it shall be proven to the satisfaction of the Director of Lands, after due notice to the homesteader, that the land entered is under the law not subject to home-stead entry, or that the homesteader has actually changed his residence, or voluntarily abandoned the land for more than six months at any one time during the years of residence and occupation herein required, or has otherwise failed to comply with the requirements of this Act, the Director of Lands may cancel the entry. SECTION 17. Before final proof shall be submitted by any person claiming to have complied with the provisions of this chapter, due notice, as prescribed by the Secretary of Agriculture and Commerce shall be given to the public of his intention to make such proof, stating therein the name and address of the homesteader, the description of the land, with its boundaries and area, the names of the witness by whom it is expected that the necessary facts will be established, and the time and place at which, and the name of the officer before whom, such proof will be made. SECTION 18. In case the homesteader shall suffer from mental alienation, or shall for any other reason be incapacitated from exercising his rights personally, the person legally representing him may offer and submit the final proof on behalf of such incapacitated person. SECTION 19. Not more than one homestead entry shall be allowed to any one person, and no person to whom a homestead patent has been issued by virtue of the provisions of this Act regardless of the area of his original homestead, may again acquire a homestead; Provided, however, That any previous homesteader who has been issued a patent for less than twenty-four hectares and otherwise qualified to make a homestead entry, may be allowed another homestead which, together with his previous homestead shall not exceed an area of twenty-four hectares.

TITLE II Agricultural Public Lands

CHAPTER III Forms of Concession of Agricultural Lands SECTION 11. Public lands suitable for agricultural purposes can be disposed of only as follows, and not otherwise: (1) (2) (3) (4) For homestead settlement By sale By lease By confirmation of imperfect or incomplete titles: (a) (b) By judicial legalization By administrative legalization (free patent).

SECTION 12. Any citizen of the Philippines over the age of eighteen years, or the head of a family, who does not own more than twenty-four hectares of land in the Philippines or has not had the benefit of any gratuitous allotment of more than twenty-four hectares of land since the occupation of the Philippines by the United States, may enter a homestead of not exceeding twentyfour hectares of agricultural land of the public domain. SECTION 13. Upon the filing of an application for a homestead, the Director of Lands, if he finds that the application should be approved, shall do so and authorize the applicant to take possession of the land upon the payment of five pesos, Philippine currency, as entry fee. Within six months from and after the date of the approval of the of the application, the applicant shall begin to work the homestead, otherwise he shall lose his prior right to the land.

L A W O N N A T U R A L R E S O U R C E S a n d E N V I R O N M E N T A L L A W C a s e s a n d S p e c i a l L a w s | 95
SECTION 20. If at any time after the approval of the application and before the patent is issued, the applicant shall prove to the satisfaction of the Director of Lands that he has complied with all the requirements of the law, but cannot continue with his homestead, through no fault of his own, and there is a bona fide purchaser for the rights and improvements of the applicant on the land, and that the conveyance is not made for purposes of speculation, then the applicant, with the previous approval of the Director of Lands may transfer his rights to the land and improvements to any person legally qualified to apply for a homestead, and immediately after such transfer, the purchaser shall file a homestead application for the land so acquired and shall succeed the original homesteader in his rights and obligations beginning with the date of the approval of said application of the purchaser. Any person who has so transferred his rights may not again apply for a new homestead. Every transfer made without the previous approval of the Director of Lands shall be null and void and shall result in the cancellation of the entry and the refusal of the patent. SECTION 21. Any non-Christian Filipino who has not applied for a home-stead, desiring to live upon or occupy land on any of the reservations set aside for the so-called "non-Christian tribes" may request a permit of occupation for any tract of land of the public domain reserved for said nonChristian tribes under this Act, the area of which shall not exceed four hectares. It shall be an essential condition that the applicant for the permit cultivate and improve the land, and if such cultivation has not been begun within six months from and after the date on which the permit was received, the permit shall be cancelled. The permit shall be for a term of one year. If at the expiration of this term or at any time prior thereto, the holder of the permit shall apply for a homestead under the provisions of this chapter, including the portion for which a permit was granted to him, he shall have the priority, otherwise the land shall be again open to disposition at the expiration of the permit. For each permit the sum of one peso shall be paid. SECTION 23. No person, corporation, association, or partnership other than those mentioned in the last preceding section may acquire or own agricultural public land or land of any other denomination or classification, which is at the time or was originally, really or presumptively, of the public domain, or any permanent improvement thereon, or any real right on such land and improvement: Provided, however, That persons, corporations, associations or partnerships which, at the date upon which the Philippine Constitution took effect, held agricultural public lands or land of any other denomination, that belonged originally, really or presumptively, to the public domain, or permanent improvements on such lands, or a real right upon such lands and Constitution took improvements, having acquired the same under the laws and regulations in force at the date of such acquisition, shall be authorized to continue holding the same as if such persons, corporations, associations, or partnerships were qualified under the last preceding section; but they shall not encumber, convey, or alienate the same to persons, corporations, associations, or partnerships not included in section twenty-two of this Act, except by reason of hereditary succession, duly legalized and acknowledged by competent courts. SECTION 24. Lands sold under the provisions of this chapter must be appraised in accordance with section one hundred and sixteen of this Act. The Director of Lands shall announce the sale thereof by publishing the proper notice once a week for six consecutive weeks in the Official Gazette, and in two newspapers one published in Manila and the other published in the municipality or in the province where the lands are located, or in a neighboring province, and the same notice shall be posted on the bulletin board of the Bureau Of Lands in Manila, and in the most conspicuous place in the provincial building and the municipal building of the province and municipality, respectively, where the land is located, and, if practicable, on the land itself; but if the value of the land does not exceed two hundred and forty pesos, the publication in the Official Gazette and newspapers may be omitted. The notices shall be published one in English and the other in Spanish or in the local dialect, and shall fix a date not earlier than sixty days after the date of the notice upon which the land will be awarded to the highest bidder, or public bids will be called for, or other action will be taken as provided in this chapter. SECTION 25. Public agricultural lands which are not located within ten (10) kilometers from the boundaries of the city proper in chartered cities or within five (5) kilometers from the municipal hall or town occupants plaza of any municipality may be sold to actual occupants who do not own any parcel of land or whose total land holdings do not exceed five hectares and who comply with the minimum requirements of Commonwealth Act numbered one hundred forty-one, as amended, and who have resided on the land applied for at least two years prior to the date of the application. cdt All bids must be sealed and addressed to the Director of Lands and must have inclosed therewith cash or certified check, treasury warrant, or post-office money order payable to the order of the Director of Lands for ten per centum of the amount of the bid, which amount shall be retained in case the bid is accepted as part payment of the purchase price: Provided, That no bid shall be considered the amount of which is less than the appraised value of the land. In addition to existing publication requirements in section twenty-four of Commonwealth Act Numbered one hundred forty-one, as amended, notices and of applications shall be posted for a

CHAPTER IV Sale SECTION 22. Any citizen of lawful age of the Philippines, and any such citizen not of lawful age who is a head of a family, and any corporation or association of which at least sixty per centum of the capital stock or of any interest in said capital stock belongs wholly to citizens of the Philippines, and which is organized and constituted under the laws of Philippines, and corporate bodies organized in the Philippines authorized under their charters to do so; may purchase any tract of public agricultural land disposable under this Act, not to exceed one hundred and forty-four hectares in the case of an individual and one thousand and twenty-four hectares in that of a corporation or association, by proceeding as prescribed in this chapter: Provided, That partnerships shall be entitled to purchase not to exceed one hundred and forty-four hectares for each member thereof. but the total area so purchased shall in no case exceed the one thousand and twenty-four hectares authorized in this section for associations and corporations.

L A W O N N A T U R A L R E S O U R C E S a n d E N V I R O N M E N T A L L A W C a s e s a n d S p e c i a l L a w s | 96
period of not less than thirty days in at least three conspicuous places in the municipality where the parcel of land is located, one of which shall be at the municipal building, and other, in the barrio council building of the barrio where the land is located. SECTION 26. Upon the opening of the bids, the land shall be awarded to the highest bidder. If there are two or more equal bids which are higher than the others, and one of such equal bids is that of the applicant, his bid shall be accepted. If, however, the bid of the applicant is not one of such equal and higher bids, the Director of Lands shall at once submit the land for public bidding, and to the person making the highest bid on such public auction the land shall be awarded. In any case, the applicant shall always have the option of raising his bid to equal that of the highest bidder, and in this case the land shall be awarded to him. No bid received at such public auction shall be finally accepted until the bidder shall have deposited ten per centum of his bid, as required in Section twenty-five of this Act. In case none of the tracts of land that are offered for sale or the purchase of which has been applied for, has an area in excess of twenty-four hectares, the Director of Lands may delegate to the District Land Officer concerned the power of receiving bids, holding the auction, and proceeding in accordance with the provisions of this Act, but the District Land Officer shall submit his recommendation to the Director of Lands, for the final decision of the latter in the case. The District Land Officer shall accept and process any application for the purchase of public lands not exceeding five hectares subject to the approval of the Director of Lands within sixty days after receipt of the recommendation of said District Land Officer. SECTION 27. The purchase price shall be paid as follows: The balance of the purchase price after deducting the amount paid at the time of submitting the bid, may be paid in full upon the making of the award, or in not more than ten equal annual installments from the date of the award. SECTION 28. The purchaser shall have not less than one-fifth of the land broken and cultivated within five years after the date of the award; and before any patent is issued, the purchaser must show of occupancy, cultivation, and improvement of at least one-fifth of the land applied for until the date on which final payment is made: Provided, however, That in case land purchased is to be devoted to pasture, it shall be sufficient compliance with this condition if the purchaser shall graze on the land as many heads of his cattle as will occupy at least one-half of the entire area at the rate of one head per hectare. SECTION 29. After title has been granted, the purchaser may not, within a period of ten years from such cultivation or grant, convey or encumber or dispose said lands or rights thereon to any person, corporation or association, without prejudice to any right or interest of the Government in the land: Provided, That any sale and encumbrance made in violation of the provisions of this section, shall be null and void and shall produce the effect of annulling the acquisition and reverting the property and all rights thereto to the State, and all payments on the purchase price theretofore made to the Government shall be forfeited. SECTION 30. If at any time after the date of the award and before the issuance of patent, it is proved to the satisfaction of the Director of Lands, after due notice to the purchaser, that the purchaser has voluntarily abandoned the land for more than one year at any one time, or has otherwise failed to comply with the requirements of the law, then the land shall revert to the State, and all prior payments made by the purchaser and all improvements existing on the land shall be forfeited. SECTION 31. No person, corporation, association, or partnership shall be permitted, after the approval of this Act, to acquire the title to or possess as owner any lands of the public domain if such lands, added to other lands belonging to such person, corporation, association, or partnership shall give a total area greater than area the acquisition of which by purchase is authorized under this Act. Any excess in area over this maximum and all right, title, interest, claim or action held by any person, corporation, association, or partnership resulting directly or indirectly in such excess shall revert to the State. This section shall, however, not be construed to prohibit any person, corporation, association, or partnership authorized by this Act to require lands of the public domain from making loans upon real necessary for the recovery of such loans; but in this case, as soon as the excess above referred to occurs, such person, corporation, association, or partnership shall dispose of such lands within five years, for the purpose of removing the excess mentioned. Upon the land in excess of the limit there shall be paid, so long as the same is not disposed of, for the first year a surtax of fifty per centum additional to the ordinary tax to which such property shall be subject, and for each succeeding year fifty per centum shall be added to the last preceding annual tax rate, until the property shall have been disposed of. The person, corporation, association, or partnership owning the land in excess of the limit established by this Act shall determine the portion of land to be segregated. At the request of Secretary of Agriculture and Commerce, the Solicitor-General or the officer acting in his stead shall institute the necessary proceedings in the proper court for the purpose of determining the excess portion to be segregated, as well as the disposal of such portion in the exclusive interest of the Government. SECTION 32. This chapter shall be held to authorize only one purchase of the maximum amount of land hereunder by the same person, corporation, association, or partnership; and no corporation, association, or partnership, any member of which shall have received the benefits of this chapter or of the next following chapter, either as an individual or as a member of any other corporation, association, or partnership, shall purchase any other lands of the public domain under this chapter. But any purchaser of public land, after having made the last payment upon and cultivated at least one-fifth of the land purchased, if the same shall be less than the maximum allowed by this Act, may purchase successively additional agricultural public land adjacent to or not distant from the land first purchased, until the total area of such purchases shall reach the maximum established in this chapter: Provided, That in making such additional purchase or purchases, the same conditions shall be complied with as prescribed by this Act for the first purchase.

CHAPTER V

L A W O N N A T U R A L R E S O U R C E S a n d E N V I R O N M E N T A L L A W C a s e s a n d S p e c i a l L a w s | 97
Lease SECTION 33. Any citizen of lawful age of the Philippines, and any corporation or association of which at least sixty per centum of the capital stock or of any interest in said capital stock belongs wholly to citizens of the Philippines, and which is organized and constituted under the laws of the Philippines, may lease any tract of agricultural public land available for lease under the provisions of this Act, not exceeding a total of one thousand and twenty-four hectares. If the land leased is adapted to and be devoted for grazing purposes, an area not exceeding two thousand hectares may be granted. No member, stockholder, of officer, representative, attorney, agent, employee or bondholder of any corporation or association holding or controlling agricultural public land shall apply, directly or indirectly, for agricultural public land except under the homestead and free patent provisions of this Act: Provided, That no lease shall be permitted to interfere with any prior claim by settlement or occupation, until the consent of the occupant or settler is first had, or until such claim shall be legally extinguished, and no person, corporation, or association shall be permitted to lease lands hereunder which are not reasonably necessary to carry on his business in case of an individual, or the business for which it was lawfully created and which it may lawfully pursue in the Philippines, if an association or corporation. SECTION 34. A notice of the date and place of the auction of the right to lease the land shall be published and announced in the same manner as that prescribed for the publication and announcement of the notice of sale, in section twenty-four of this Act. SECTION 35. All bids must be sealed and addressed to the Director of Lands and must have enclosed therewith cash or a certified check, Treasury warrant, or post-office money order payable to the order of the Director of Lands, for a sum equivalent to the rental for at least, the first three months of the lease: Provided, That no bid shall be considered in which the proposed annual rental is less than three per centum of the value of the land according to the appraisal made in conformity with section one hundred and sixteen of this Act. SECTION 36. The auction of the right to lease the land shall be conducted under the same procedure as that prescribed for the auction sale of agricultural lands as described in section twentysix of this Act: Provided, That no bid shall be accepted until the bidder shall have deposited the rental for at least the first three months of the lease. SECTION 37. The annual rental of the land leased shall not be less than three per centum of the value of the land, according to the appraisal and reappraisal made in accordance with section one hundred sixteen of this Act; except for lands reclaimed by the Government, which shall not be less than four per centum of the appraised and reappraised value of the land: Provided, That one-fourth of the annual rental of these lands reclaimed prior to the approval of this Act shall accrue to the construction and improvement portion of the Portworks Funds: And provided, further, That the annual rental of not less than four per centum of the appraised and reappraised value of the lands reclaimed using the Portworks Fund after the approval of this Act shall all accrue to the construction and improvement portion of the Portworks Fund. But if the land leased is adapted to and be devoted for granting purposes, the annual rental shall be not less than two per centum of-the appraised and reappraised value thereof- Every contract of lease under the provisions of this chapter shall contain a cause to the effect that are appraisal of the land leased shall be made every ten years from the date of the approval of the lease, if the term of the same shall be in excess of ten years. In case the lessee is not agreeable to the reappraisal and prefers to give up his contract of lease, he shall notify the Director of Lands of his desire within the six months next preceding the date on which the reappraisal takes effect, and in case his request is approved, the Director of Lands may, if the lessee should so desire, proceed in accordance with section one hundred of this Act. SECTION 38. Leases shall run for a period of not more than twenty-five years, but may be renewed once for another period of not to exceed twenty-five years, in case the lessee shall have made important improvements which, in the discretion of the Secretary of Agriculture and Commerce justify a renewal. Upon the final expiration of the lease, all buildings and other permanent improvements made by the lessee, his heirs, executors, administrators, successors, or assigns shall become the property of the Government, and the land together with the said improvements shall be disposed of in accordance with the provisions of chapter five of this Act. SECTION 39. It shall be an inherent and essential condition of the lease that the lessee shall have not less than one-third of the land broken and cultivated within five years after the date of the approval of the lease: Provided, however, That in case the land leased is to be devoted to pasture, it shall be sufficient compliance with this condition if the lessee shall graze on the land as many heads of cattle as will occupy at least one-half of the entire area at the rate of one head per hectare. SECTION 40. The lessee shall not assign, encumber, or sublet his rights without the consent of the Secretary of Agriculture and Commerce, and the violation of this condition shall avoid the contract: Provided, That assignment, encumbrance, or subletting for purposes of speculation shall not be permitted in any case: Provided, further, That nothing contained in this section shall be understood or construed to permit the assignment, encumbrance, or subletting of lands leased under this Act, or under any previous Act, to persons, corporations, or associations which under this Act, are not authorized to lease public lands. SECTION 41. The lease of any lands under this chapter shall not confer the right to remove or dispose of any valuable timber except as provided in the regulations of the Bureau of Forestry for cutting timber upon such lands. Nor shall such lease confer the right to remove or dispose of stone, oil, coal, salts. or other minerals, or medicinal mineral waters existing upon the same. The lease as to the part of the land which shall be mineral may be canceled by the Secretary of Agriculture and Commerce, after notice to the lessee, whenever the said part of the land is more valuable for agricultural purposes. The commission of waste or violation of the forestry regulations by the lessee shall work a forfeiture of his last payment of rent and render him liable to immediate dispossession and suit for damage. SECTION 42. After having paid rent for at least the first two years of the lease, and having complied with the requirements prescribed in section thirty nine, the lessee of agricultural public land with an area than the maximum allowed by law, may lease successively additional agricultural public land adjacent to or near the land originally leased until the total- area of such leases shall reach

L A W O N N A T U R A L R E S O U R C E S a n d E N V I R O N M E N T A L L A W C a s e s a n d S p e c i a l L a w s | 98
the maximum established in this chapter: Provided, That in making such additional lease, the same conditions shall be complied with as prescribed by this Act for the first lease. SECTION 43. During the life of the lease, any lessee who shall have complied with all the conditions thereof and shall have the qualifications required by section twenty-two, shall have the option of purchasing the land leased subject to the restrictions of chapter five of this Act. city council and barangay council affected, and copies thereof shall be posted on the bulletin board of the Bureau of Lands at Manila and at conspicuous places in the provincial building and at the municipal building and barangay hall or meeting place. It shall moreover, be announced by government radio whenever available, in each of the barrios of the municipality. SECTION 46. If, after the filing of the application and the investigation, the Director of Lands shall be satisfied of the truth of the allegations contained the application and that the applicant comes within the provisions chapter, he shall cause a patent to issue to the applicant or his legal successor for the tract so occupied and cultivated, provided its area does not exceed twenty-four hectares: Provided, That no application shall be finally acted upon until notice thereof has been published in the municipality and barrio in which the land is located and adverse claimants have had an opportunity to present their claims.

CHAPTER VI Free Patents SECTION 44. Any natural-born citizen of the Philippines who is not the owner of more than twenty-four hectares and who since July fourth, nineteen hundred and twenty-six or prior thereto, has continuously occupied and cultivated, either by himself or through his predecessors-in-interest, a tract or tracts of agricultural public lands subject to disposition, or who shall have paid the real estate tax thereon while same has not been occupied by any person shall be entitled, under the provisions of this chapter, to have a free patent issued to him for such tract or tracts of such land not to exceed twenty-four hectares. A member of the national cultural minorities who has continuously occupied and cultivated, either by himself or through his predecessors-in-interest, a tract or tracts of land, whether disposable or not since July 4, 1955, shall be entitled to the right granted in the preceding paragraph of this section: Provided, That at the time he files his free patent application he is not the owner of any real property secured or disposable under this provision of the Public Land Law SECTION 45. The President of the Philippines (Prime Minister), upon recommendation of the Secretary of Natural Resources, shall from time to time fix by proclamation the period which applications for Proclamation free patents may be filed in the district, chartered city, of period province, municipality or region specified in such proclamation, and upon the expiration of the period so designated, unless the same be extended by the President (Prime Minister) all the land comprised within such district, chartered city, province, municipality or region subject thereto under the provisions of this chapter may be disposed of as agricultural public land without prejudice to the prior right of the occupant and cultivator to acquire such land under this Act by means other than free patent. The time to be fixed in the entire Archipelago for the filing of applications under this Chapter shall not extend beyond December 31, 1987, except in the provinces of Agusan del Norte, Agusan del Sur, Cotabato, South Cotabato, Bukidnon, Lanao del Norte, Lanao del Sur, Davao del Norte, Davao del Sur, Davao Oriental, Sulu, Mt. Province, Benguet, Kalinga-Apayao, and Ifugao where the President of the Philippines, upon recommendation of the Secretary of Natural Resources, shall determine or fix the time beyond which the filing of applications under this Chapter shall not extend. The period fixed for any district, chartered city, province, or municipality shall begin to run thirty days after the publication of the proclamation in the Official Gazette and if available in one newspaper of general circulation in the city, province or municipality concerned. A certified copy of said proclamation shall be furnished by the Secretary of Natural Resources within 30 days counted from the date of the presidential proclamation to the Director of Lands and to the provincial board, the municipal board or

CHAPTER VII Judicial Confirmation of Imperfect or Incomplete Titles SECTION 47. The persons specified in the next following section are hereby granted time, not to extend beyond December 31, 1987 within which to take advantage of the benefit of this chapter: Provided, That this extension shall apply only where the area applied for does not exceed 144 hectares. Provided, further, That the several periods of time designated by the President in accordance with section forty-five of this Act shall apply also to the lands comprised in the provisions of this chapter, but this section shall not be construed as prohibiting any of said persons from acting under this chapter at any time prior to the period fixed by the President. SECTION 48. The following-described citizens of the Philippines, occupying lands of the public domain or claiming to own any such lands or an interest therein, but whose titles have not been perfected or completed, may apply to the Court of First Instance of the province where the land is located for confirmation of their claims and the issuance of a certificate of title therefor, under the Land Registration Act , to wit: (a) Those who prior to the transfer of sovereignty from Spain to the prior United States have applied for the purchase, composition or other form of grant of lands of the public domain under the laws and royal decrees then in force and have instituted and prosecuted the proceedings in connection therewith, but have with or without default upon their part, or for any other cause, not received title therefor, if such applicants or grantees and their heirs have occupied and cultivated said lands continuously since the filing of their applications. (b) Those who by themselves or through their predecessors in interest have been in open, continuous, exclusive, and notorious possession and occupation of agricultural lands of the public domain, under a bona fide claim of acquisition or ownership, for at least thirty years immediately preceding the filing of the application for confirmation of title except

L A W O N N A T U R A L R E S O U R C E S a n d E N V I R O N M E N T A L L A W C a s e s a n d S p e c i a l L a w s | 99
when prevented by war or force majeure. These shall be conclusively presumed to have performed all the conditions essential to a Government grant and shall be entitled to a certificate of title under the provisions of this chapter. (c) Members of the national cultural minorities who by themselves or through their predecessors-in-interest have been in open, continuous, exclusive and notorious possession and occupation of lands of the public domain suitable to agriculture, whether disposable or not, under a bona fide claim of ownership for at least 30 years shall be entitled to the rights granted in sub-section (b) hereof. SECTION 49. No person claiming title to lands of the public domain not possession of the qualifications specified in the last preceding section may apply for the benefits of this chapter. SECTION 50. Any person or persons, or their legal representatives or successors in right, claiming any lands or interest in lands under the provisions of this chapter, must in every case present an application to the proper Court of First Instance, praying that the validity of the alleged title or claim be inquired into and that a certificate of title be issued to them under the provisions of the Land Registration Act. The application shall conform as nearly as may be in its material allegations to the requirements of an application for registration under the Land Registration Act, and shall be accompanied by a plan of the land and all documents evidencing a right on the part of the applicant to the land claimed. The application shall also state the citizenship of the applicant and shall set forth fully the nature of the claim and when based upon proceeding initiated under Spanish laws, it shall specify as exactly as possible the date and form of application for purchase composition or other form of grant, the extent of the compliance with the conditions required by the Spanish laws and royal decrees for the acquisition of legal title, and if not fully complied with, the reason for such noncompliance, together with a statement of the length of time such land or any portion thereof has been actually occupied by the claimant or his predecessors in interest; the use made of the land, and the nature of the inclosure, if any. The fees provided to be paid for the registration of lands under the Land Registration Act shall be collected from applicants under this chapter. SECTION 51. Applications for registration under this chapter shall be heard in the Court of First Instance in the same manner and shall be subject to the same procedure as established in the Land Registration Act for other applications, except that a notice of all such applications, together with a plan of the lands claimed, shall be immediately forwarded to the Director of Lands, who may appear as a party in such cases: Provided, That prior to the publication for hearing, all of the papers in said case shall be transmitted papers by the clerk to the Solicitor General or officer acting in his stead, in order that he may, if he deems it advisable for the interests of the Government, investigate all of the facts alleged in the application or otherwise brought to his attention. The Solicitor-General shall return such papers to the clerk as soon as practicable within three months. The final decree of the court shall in every case be the basis for the original certificate of title in favor of the person entitled to the property under the procedure prescribed in section forty-one of the Land Registration Act. SECTION 52. In cadastral proceedings, instead of an application, an answer or claim may be filed with the same effect as in the procedure provided in the last preceding two sections. SECTION 53. It shall be lawful for the Director of Lands, whenever in the opinion of the President the public interests shall require it, to cause to be filed in the proper Court of First Instance, through the Solicitor-General or the officer acting in his stead, a petition against the holder, claimant, possessor, or occupant of any land who shall not have voluntarily come in under the provisions of this chapter or of the Land Registration Act, stating in substance that the title of such holder, claimant, possessor, or occupant is open to discussion; or that the boundaries of any such land which has not been brought into court as aforesaid are open to question; or that it is advisable that the title to such lands be settled and adjudicated, and praying that the title to any such land or the boundaries thereof or the right to occupancy thereof be settled and adjudicated. The judicial proceedings under this section shall be in accordance with the laws on adjudication of title in cadastral proceedings. SECTION 54. If in the hearing of any application arising under this chapter the court shall find that more than one person or claimant has an interest in the land, such conflicting interests shall be adjudicated by the court and decree awarded in favor of the person or persons entitled to the land according to the laws, but if none of said person is entitled to the land, or if the person who might be entitled to the same lacks the qualifications required by this Act for acquiring agricultural land of the public domain, the decision shall be in favor of the Government. SECTION 55. Whenever, in any proceedings under this chapter to secure registration of an incomplete or imperfect claim of title initiated prior to the transfer of sovereignty from Spain to the United States, it shall appear that had such claims been prosecuted to completion under the laws prevailing when instituted, and under the conditions of the grant then contemplated, the conveyance of such land to the applicant would not have been gratuitous, but would have involved payment therefor to the Government, then and in that event the court shall, after decreeing in whom title should vest, further determine the amount to be paid as a condition for the registration of the land. Such judgment shall be certified to the Director of Lands by the clerk of the court for collection of the amount due from the person entitled to conveyance. Upon payment to the Director of Lands of the price specified in the judgment, he shall so certify to the proper Court of First Instance and said court shall forthwith order the registration of the land in favor of the competent person entitled thereto. If said person shall fail to pay the amount of money required by the decree within a reasonable time fixed in the same, the court shall order the proceeding to stand dismissed and the title to the land shall then be in the State free from any claim of the applicant. SECTION 56. Whenever any judgment of confirmation or other decree of the court under this chapter shall become final, the clerk of the court concerned shall certify that fact to the Director

L A W O N N A T U R A L R E S O U R C E S a n d E N V I R O N M E N T A L L A W C a s e s a n d S p e c i a l L a w s | 100
of Lands, with a certified copy of the decree of confirmation or judgment of the court and the plan and technical description of the land involved in the decree or judgment of the court. SECTION 57. No title or right to, or equity in, any lands of the public domain may hereafter be acquired by prescription or by adverse possession or occupancy, or under or by virtue of any law in effect prior to American occupation, except as expressly provided by laws enacted after said occupation of the Philippines by the United States. Congress: Provided, further, That any person, corporation, association or partnership disqualified from purchasing public land for agricultural purposes under the provisions of this Act, may lease land included under this title suitable for industrial or residential purposes, but the lease granted shall only-be valid while such land is used for the purposes referred to. SECTION 61. The lands comprised in classes (a), (b), and (c) of section fifty-nine shall be disposed of to private parties by lease only and not otherwise, as soon as the President, upon recommendation by the Secretary of Agriculture and Commerce. shall declare that the same are not necessary for the public service and are open to disposition under this chapter. The lands included in class (d) may be disposed of by sale or lease under the provisions of this Act. SECTION 62. The lands reclaimed by the Government by dredging, filling or otherwise shall be surveyed and may, with the approval of the Secretary of Agriculture and Commerce, be divided by the Director of Lands into lots and blocks, with the necessary streets and alley-ways between them, and said Director shall give notice to the public by publication in the Official Gazette or by other means, that the lots or blocks not needed for public purposes shall be leased for commercial or industrial or other similar purposes. SECTION 63. Whenever it is decided that lands covered by this chapter are not needed for public purposes, the Director of Lands shall ask the Secretary of Agriculture and Commerce for authority to dispose of the same. Upon receipt of such authority, the Director of Lands shall give notice by public advertisement in the same manner as in the case of leases or sales of agricultural public land, that the Government will lease or sell, as the case may be, the lots or blocks specified in the advertisement, for the purpose stated in the notice and subject to the conditions specified in this chapter. SECTION 64. The leases executed under this chapter by the Secretary of Agriculture and Commerce shall, among other conditions, contain the following: (a) The rental shall not be less than three per centum of the appraised or reappraised value of the land plus one per centum of the appraised or reappraised value of the improvements, except for lands reclaimed by the Government which shall not be less than four per centum of the appraised or reappraised value of the land plus two per centum of the appraised or reappraised value of the improvements thereon: Provided, That twentyfive per centum of the total annual rental on all lands reclaimed prior to the approval of this Act and one per centum of the appraised or reappraised value of improvements shall accrue to the construction and improvement portion of the Portworks Fund: And provided, further,That the annual rental on lands reclaimed using the Portworks Fund together with the fee due on account of the improvement thereon after the effectivity of this Act shall all accrue to the construction and improvement portion of the Portworks Fund. (b) The land rented and the improvements thereon shall be reappraised every ten years if the term of the lease is in excess of that period.

TITLE III Lands for Residential, Commercial or Industrial Purposes and Other Similar Purposes

CHAPTER VIII Classification and Concession of Public Lands Suitable for Residence, Commerce and Industry SECTION 58. Any tract of land of the public domain which, being neither timber nor mineral land, is intended to be used for residential purposes or for commercial, industrial, or other productive purposes other than agricultural, and is open to disposition or concession, shall be disposed of under the pro-visions of this chapter and not otherwise. SECTION 59. The lands disposable under this title shall be classified as follows: (a) (b) Lands reclaimed by the Government by dredging, filing, or other means; Foreshore;

(c) Marshy lands or lands covered with water bordering upon the shores or banks of navigable lakes or rivers; (d) Lands not included in any of the foregoing classes.

SECTION 60. Any tract of land comprised under this title may be leased or sold, as the case may be, to any person, corporation, or association authorized to purchase or lease public lands for agricultural purposes. The area of the land so leased or sold shall be such as shall, in the judgment of the Secretary of Agriculture and Natural Resources, be reasonably necessary for the purposes for which such sale or lease is requested, and shall in no case exceed one hundred and forty-four hectares: Provided, however, That this limitation shall not apply to grants, donations, transfers made to a province, municipality or branch or subdivision of the Government for the purposes deemed by said entities conducive to the public interest; but the land so granted donated, or transferred to a province, municipality, or branch or subdivision of the Government shall not be alienated, encumbered, or otherwise disposed of in a manner affecting its title, except when authorized by

L A W O N N A T U R A L R E S O U R C E S a n d E N V I R O N M E N T A L L A W C a s e s a n d S p e c i a l L a w s | 101
(c) The term of the lease shall be as prescribed by section thirty-eight of this Act. SECTION 66. The kind of improvements to be made by the lessee or the purchaser, and the plans thereof, shall be subject to the approval of the Secretary of Public Works and Communications, in case they are constructions or improvements which if by the Government, would properly have to be executed under the supervision of the Bureau of Public Works. SECTION 67. The lease or sale shall be made through oral bidding; and adjudication shall be made to the highest bidder. However, where an applicant has made improvements on the land by virtue of a permit issued to him by competent authority, the sale or lease shall be made by sealed bidding as prescribed in section twenty-six of this Act, the provisions of which shall be applied wherever applicable. If all or part of the lots remain unleased or unsold, the Director of Lands shall from time to time announce in the Official Gazette or in any other newspapers of general circulation, the lease or sale of those lots, if necessary SECTION 68. The Secretary of Agricultural and Commerce may grant to qualified persons temporary permission, upon payment of a reasonable charge, for the use of any portion of the lands covered by this chapter for any lawful private purpose, subject to revocation at any time when, in his judgment, the public interest shall require it.

(d) The lessee shall construct permanent improvements appropriate for the purpose for which the lease is granted, shall commence the construction thereof within six months from the date of the award of the right to lease the land, and shall complete the said construction within eighteen months from said date. (e) At the expiration of the lease or of any extension of the same, all improvements made by the lessee, his heirs, executors, administrators, successors, or assigns shall become the property of the Government. (f) The regulation of all rates and fees charged to the public; and the annual submission to the Government for approval of all tariffs of such rates and fees. (g) The continuance of the easements of the coast police and other easements reserved by existing law or by any laws hereafter enacted. (h) Subjection to all easements and other rights acquired by the owners of lands bordering upon the foreshore or marshy land. The violation of one or any of the conditions specified in the contract shall give rise to the rescission of said contract. The Secretary of Agriculture and Commerce may, however, subject to such conditions as he may prescribe, waive the rescission arising from a violation of the conditions of subsection (d), or extend the time within which the construction of the improvements shall be commenced and completed. SECTION 65. The sale of the lands comprised in classes (c) and (d) of section fifty-nine shall, among others, comprise the following conditions: (a) The purchaser shall make improvements of a permanent character appropriate for the purpose for which the land is purchased, shall commence work thereon within six months from the receipt of the order of award, and shall complete the construction of said improvements within eighteen months from the date of such award; otherwise the Secretary of Agriculture and Natural Resources may rescind the contract. (b) The purchase price shall be paid in cash or in equal annual installments, not to exceed ten. The contract of sale may contain other conditions not inconsistent with the provisions of this Act.

TITLE IV Lands for Educational, Charitable, and Other Similar Purposes

CHAPTER IX Concession of Lands for Educational, Charitable, and Other Similar Purposes SECTION 69. Whenever any province, municipality, or other branch or subdivision of the Government shall need any portion of the land of the public domain open to concession for educational, charitable or other similar purposes, the President, upon recommendation by the Secretary of Agriculture and Commerce, may execute contracts in favor of the same. in the form of donation, sale, lease, exchange, or any other form, under terms and conditions to be inserted in the contract; but land so granted shall in no case be encumbered or alienated, except when the public service requires their being leased or exchanged, with the approval of the President, for other lands belonging to private parties, or if the National Assembly disposes otherwise. SECTION 70. Any tract of public land of the class covered by this title maybe sold or leased for the purpose of founding a cemetery, church, college, school, university, or other institutions for educational, charitable or philanthropical purposes or scientific research, the area to be such as may actually and reasonably be necessary to carry out such purpose, but not to exceed ninety-six hectares in any case. The sale or lease shall be made subject to the same conditions as required for the sale and lease of agricultural public land, but the Secretary of Agriculture and Commerce may waive the conditions requiring cultivation. The Secretary of Agriculture and Commerce, if conveyance he sees fit, may order the sale to be made without public auction, at a price to be fixed by said Secretary, or

L A W O N N A T U R A L R E S O U R C E S a n d E N V I R O N M E N T A L L A W C a s e s a n d S p e c i a l L a w s | 102
the lease to be granted without auction, at a rental to be fixed by him. In either case it shall be a condition that the purchaser or lessee or their successors or assigns shall not sell transfer, encumber or lease the land for the purposes of speculation or use it for any purpose other than that contemplated in the application, and that the violation of this condition shall give rise to the immediate rescission of the sale or lease, as the case may be, and to the forfeiture to the Government of all existing improvements: Provided, That it shall in no case be sublet, encumbered or resold under the conditions above set forth except with the approval of the Secretary of Agriculture and Commerce. SECTION 75. Unless the necessary reservations are made in the proclamation of the President, the Director of Lands, with the approval of the Secretary of Agriculture and Commerce, shall reserve out of the land by him to be subdivided lots of sufficient size and convenient situation for public use, as well as the necessary avenues, streets, alleyways, parks, and squares. The avenues, streets, alleys, parks, plazas, and lots shall be laid out on the plat as though the lands owned or claimed by private persons were part of the public domain and part of the reservation, with a view to the possible subsequent purchase or condemnation thereof, if deemed necessary by the proper authorities. SECTION 76. At any time after the subdivision has been made, the President may, in case the public interest requires it, reserve for public purposes any lot or lots of the land so reserved and not disposed of. SECTION 77. If, in order to carry out the provisions of this chapter, it shall be necessary to condemn private lands within the limits of the new town, the President shall direct the SolicitorGeneral or officer acting in his stead to at once begin proceedings for condemnation, in accordance with the provisions of existing law. SECTION 78. When the plat of subdivision has been finally approved by the Secretary of Agriculture and Commerce, the Director of Lands shall record the same in the records of his office and shall forward a certified copy of such record to the register of deeds of the province in which the land lies, to be by such register recorded in the records of his office SECTION 79. All lots, except those claimed by or belonging to private parties and those reserved for parks, buildings, and other public uses, shall be sold, after due notice, at public auction to the highest bidder, after the approval and recording of the plat of subdivision as above provided, but no bid shall be accepted that does not equal at least two-thirds of the appraised value, nor shall bids be accepted from persons, corporations, associations, or partnerships not authorized to purchase public lands for commercial, residential or industrial purposes under the provisions of this Act. The provisions of sections twenty-six and sixty-five of this Act shall be observed in so far as they are applicable. Lots for which satisfactory bids have not been received shall be again offered for sale, under the same conditions as the first time, and if they then remain unsold, the Director of Lands shall be authorized to sell them at private sale for not less than two-thirds of their appraised value. SECTION 80. All funds derived from the sale of lots shall be covered into the Philippine Treasury as part of the general funds. SECTION 81. Not more than two residence lots and two lots for commercial and industrial uses in any one town site shall be sold to any one person, corporation, or association without the specific approval of the Secretary of Agriculture and Commerce. SECTION 82. The Assembly shall have the power at any time to modify, alter, rescind, repeal, annul, and cancel, with or without conditions, limitation, exceptions, or reservations, all and any dispositions made by the executive branch of the Philippine Government by virtue of this chapter, and the exercise of this power shall be understood as reserved in all cases, as an inherent condition thereof.

TITLE V Reservations

CHAPTER X Town Site Reservations SECTION 71. Whenever it shall be considered to be in the public interest to found a new town. the Secretary of Agriculture and Commerce shall direct the Director of Lands to have a survey made by his Bureau of the exterior boundaries of the site on which such town is to be established, and upon the completion of the survey he shall send the same to said Secretary, with his recommendations. SECTION 72. The Secretary of Agriculture and Commerce, if he approves the recommendations of the Director of Lands, shall submit the matter to the President to the end that the latter may issue a proclamation reserving the land surveyed, or such part thereof as he may deem proper, as a town site, and a certified copy of such proclamation shall be sent to the Director of Lands and another to the register of deeds of the province in which the surveyed land lies. SECTION 73. It shall then be the duty of the Director of Lands, after having recorded the proclamation of the President and the survey accompanying the same, and having completed the legal proceedings prescribed in chapter thirteen of this Act, to direct a subdivision in accordance with the instructions of the Secretary of Agriculture and Commerce, if there shall be such instructions, and if there shall not be any, then in the manner which may to the Director of Lands seem best adapted to the convenience and interest of the public and the residents of the future town. SECTION 74. The plat of the subdivision shall designate certain lots for commercial and industrial uses and the remainder as residence lots, and shall also reserve and note the lots owned by private individuals as evidenced by record titles, or possessed or claimed by them as private property. Such lots, whether public or private, shall be numbered upon a general plan or system. The plat prepared by the Director of Lands shall be submitted to the Secretary of Agriculture and Commerce for consideration, modification, amendment, or approval.

L A W O N N A T U R A L R E S O U R C E S a n d E N V I R O N M E N T A L L A W C a s e s a n d S p e c i a l L a w s | 103
break, and prepare for cultivation the lands of said colony and establish the necessary irrigation system and suitable roads and fences; but final disposition shall be made of the land in accordance with the provisions of this Act, subject, however, to such conditions as the National Assembly may establish for the reimbursement of the expense incurred in putting such lands in condition for cultivation: Provided, That the National Assembly may direct that such land so prepared for cultivation may be disposed of only by sale or lease.

CHAPTER XI Reservations for Public and Semi-Public Purposes SECTION 83. Upon the recommendation of the Secretary of Agriculture and Commerce, the President may designate by proclamation any tract or tracts of land of the public domain as reservations for the use of the Commonwealth of the Philippines or of any of its branches, or of the inhabitants thereof, in accordance with regulations prescribed for this purpose, or for quasi-public uses or purposes when the public interest requires it, including reservations for highways, rights of way for railroads, hydraulic power sites, irrigation systems, communal pastures or leguas comunales, public parks, public quarries, public fishponds, workingmen's village and other improvements for the public benefit. SECTION 84. Upon recommendation of the Secretary of Agriculture and Commerce, the President, may by proclamation, designate any tract or tracts of the public domain for the exclusive use of the non-Christian Filipinos, including in the reservation, in so far as practicable, the lands used or possessed by them, and granting to each member not already the owner, by title or gratuitous patent, of four or more hectares of land, the use and benefit only of a tract of land not to exceed four hectares for each male member over eighteen years of age or the head of a family. As soon as the Secretary of the Interior shall certify that the majority of the non-Christian inhabitants of any given reservation have advanced sufficiently in civilization, then the President may order that the lands of the public domain within such reservation be granted under the general provisions of this Act to the said inhabitants, and the subdivision and distribution of said lands as above provided shall be taken into consideration in the final disposition of the same. But any non-Christian inhabitant may at any time apply for the general benefits of this Act provided the Secretary of Agriculture and Commerce is satisfied that such inhabitant is qualified to take advantage of the provisions of the same: Provided, That all grants, deeds, patents and other instruments of conveyance of land or purporting to convey or transfer rights of property, privileges, or easements appertaining to or growing out of lands, granted by sultans, datus, or other chiefs of the so-called non-Christian tribes, without the authority of the Spanish Government while the Philippines were under the sovereignty of Spain, or without the consent of the United States Government or of the Philippine Government since the sovereignty over the Archipelago was transferred from Spain to the United States, and all deeds and other documents executed or issued or based upon the deeds, patents, and documents mentioned, are hereby declared to be illegal, void, and of no effect. SECTION 85. Upon recommendation by the Secretary of Agriculture and Commerce, the President may, by proclamation designate any tract or tracts of land of the public domain for the establishment of agricultural colonies; and although the disposition of the lands to the colonists shall be made under the provisions of this Act, yet, while the Government shall have the supervision and management of said colonies, the Secretary of Agriculture and Commerce may make the necessary rules and regulations for the organization and internal administration of the same. The Secretary of Agriculture and Commerce may also, under conditions to be established by the Assembly, turn over a colony so reserved to any person or corporation, in order that such person or corporation may clear,

CHAPTER XII Provisions Common to Reservations SECTION 86. A certified copy of every proclamation of the President issued under the provisions of this title shall be forwarded to the Director of Lands for record in his office, and a copy of this record shall be forwarded to the register of deeds of the province or city where the land lies. Upon receipt of such certified copy, the Director of Lands shall order the immediate survey of the proposed reservation if the land has not yet been surveyed, and as soon as the plat has been completed, he shall proceed in accordance with the next following section. SECTION 87. If all the lands included in the proclamation of the President are not registered under the Land Registration Act, the Solicitor-General, if requested to do so by the Secretary of Agriculture and Commerce, shall proceed in accordance with the provision of section fifty-three of this Act. SECTION 88. The tract or tracts of land reserved under the provisions of section eighty-three shall be non-alienable and shall not be subject to occupation, entry, sale, lease, or other disposition until again declared alienable under the provisions of this Act or by proclamation of the President.

TITLE VI General Provisions

CHAPTER XIII Applications: Procedure, Concession of Lands, and Legal Restrictions and Encumbrances SECTION 89. All applications filed under the provisions of this Act shall be addressed to the Director of Lands. SECTION 90. Every application under the provisions of this Act shall be made under oath and shall set forth: (a) The full name of applicant, his age, place of birth, citizenship, civil status, and postoffice address. In case the applicant is a corporation, association or copartnership, the

L A W O N N A T U R A L R E S O U R C E S a n d E N V I R O N M E N T A L L A W C a s e s a n d S p e c i a l L a w s | 104
application shall be accompanied with a certified copy of its articles of incorporation, association or copartnership together with an affidavit of its President, manager, or other responsible officer, giving the names of the stockholders or members, their citizenship, and the number of shares subscribed by each. (b) (c) That the applicant has all the qualifications required by this Act in the case. That he has none of the disqualifications mentioned herein. demarcated and preserved as permanent timberland to be planted exclusively to trees of known economic value, and that he shall not make any clearing thereon or utilize the same for ordinary farming purposes even after patent shall have been issued to him or a contract of lease shall have been executed in his favor. 68 SECTION 91. The statements made in the application shall be considered as essential conditions and parts of any concession, title, or permit issued on the basis of such application, and any false statements therein or omission of facts altering, changing, or modifying the consideration of the facts set forth in such statements, and any subsequent modification, alteration, or change of the material facts set forth in the application shall ipso facto produce the cancellation of the concession, title, or permit granted. It shall be the duty of the Director of Lands, from time to time and whenever he may deem it advisable, to make the necessary investigations for the purpose of ascertaining whether the material facts set out in the application are true, or whether they continue to exist and are maintained and preserved in good faith, and for the purposes of such investigation, the Director of Lands is hereby empowered to issue subpoenas and subpoenas duces tecum and, if necessary, to obtain compulsory process from the courts. In every investigation made in accordance with this section, the existence of bad faith, fraud, concealment, or fraudulent and illegal modification of essential facts shall be presumed if the grantee or possessor of the land shall refuse or fail to obey asubpoena or subpoena duces tecum lawfully issued by the Director of Lands or his authorized delegates or agents, or shall refuse or fail to give direct and specific answers to pertinent questions, and on the basis of such presumption, an order of cancellation may issue without further proceedings. SECTION 92. Although the maximum area of public land that may be acquired is fixed, yet the spirit of this Act is that the rule which must deter-mine the real area to be granted is the beneficial use of the land. The concession or disposition shall be for less than the maximum area authorized if, at the time of the issuance of the patent or of the concession or disposition, it shall appear that the applicant is utilizing and is only able to utilize a smaller area, even though the application is for a greater area. For the purposes of this section, the Director of Lands is authorized to determine the area that may be granted to the applicant, and to deny or cancel or limit any application for concession, purchase, or lease if convinced of the lack of means of the applicant for using the land for the purpose for which he has requested it. SECTION 93. Lands applied for under this Act shall conform to the legal subdivisions and shall be contiguous if comprising more than one subdivision. If subdivisions have not been made on the date of the application, the lands shall be rectangular in form so far as practicable, but it shall be endeavored to make them conform to the legal subdivision as soon as the same has been made, provided the interests of the applicant or grantee are protected; and the subdivision assigned to the applicant or grantee shall, so far as practicable, include the land improved or cultivated. The regulations to be issued for the execution of the provisions of this section shall take into account the legal subdivision to be made by the Government and the inadvisability of granting the best land at a given place to only one person. cd i

(d) That the application is made in good faith, for the actual purpose of using the land for the object specified in the application and for no other purpose, and that the land is suitable for the purpose to which it is to be devoted. (e) That the application is made for the exclusive benefit of the application and not, either directly or indirectly, for the benefit of any other person or persons, corporation, association, or partnership. (f) As accurate a description of the land as may be given, stating its nature the province, municipality, barrio, and sitio where it is located, and its limits and boundaries, specifying those having reference to accidents of the ground or permanent monuments, if any. (g) Whether all or part of the land is occupied or cultivated or improved, and by whom, giving his post-office address, and whether the land has been occupied or cultivated or improved by the applicant or his ascendant, the name of the ascendant, the relationship with him, the date and place of the death of the ascendant, the date when the possession and cultivation began, and description of the improvements made, accompanying satisfactory evidence of the relationship of the applicant with the ascendant, and of the death of the latter and the descendants left by him, in case it is alleged that he occupied and cultivated the land first; or whether there are indications of its having been occupied, cultivated, or improved entirely or partially, and if so, in what such indications consist, whether he has made investigations as to when and by whom such improvements were made, and if so, how such investigations were made and what was the result thereof; or whether the land is not occupied, improved, or cultivated either entirely or partially, and there are no indications of it having ever been occupied, improved, or cultivated, and in this case, what is the condition of the land. (h) That the land applied for is neither timber nor mineral land and does not contain guano or deposits of salts or coal. (i) That the applicant agrees that a strip forty meters wide starting from the bank on each side of any river or stream that may be found on the land applied for, shall be

L A W O N N A T U R A L R E S O U R C E S a n d E N V I R O N M E N T A L L A W C a s e s a n d S p e c i a l L a w s | 105
SECTION 94. In case the legal subdivisions have already been made at the time of the filing of the application, no charge shall be made for the survey; but if the legal subdivisions have not yet been made, the cost of the survey shall be charged to the Government, except in the following cases: (a) In purchases under chapters five and ten of this Act, the cost of the survey shall be charged to the purchaser if the same is a corporation, association, or partnership; in other purchases the purchases, whoever it be, shall pay the total cost of the survey. (b) In leases, the cost of the survey shall be paid by the lessee; but at any time after the first five years from the approval of the lease, and during Cost of the life of the same, the lessee shall be entitled to the reimbursement of one-half of the cost of the survey, if he shows to the satisfaction of the Director of Lands that he has occupied and improved a sufficient area of the land or incurred sufficient expenses in connection therewith to warrant such reimbursement. SECTION 95. If before the delimitation and survey of a tract of public land the President shall declare the same disposable or alienable and such land shall be actually occupied by a person other than the applicant, the Director of Lands shall inform the occupant of his prior right to apply for the land and shall give him one hundred and twenty days time in which to file the application or apply for the concession by any of the forms of disposition authorized by this Act, if such occupant is qualified to acquire a concession under this Act. SECTION 96. As soon as any land of the public domain has been surveyed, delimited, and classified, the President may, in the order issued by him declaring it open for disposition, designate a term within which occupants with improvements but not entitled to free patents may apply for the land occupied by them, if they have the qualifications required by this Act. SECTION 97. If in the case of the two last preceding sections, the occupant or occupants have not made application under any of the provisions of this Act at the expiration of the time limit fixed, they shall lose any prior right to the land recognized by this Act, and the improvements on the land, if any, shall be forfeited to the Government. SECTION 98. All rights in and interest to, and the improvements and crops upon, land for which an application has been denied or canceled or a patent or grant refused, or a contract or concession rescinded or annulled, shall also be forfeited to the Government. SECTION 99. The Secretary of Agriculture and Commerce may order such improvements and crops to be appraised separately, for sale to the new applicant or grantee, or may declare such land open only to sale or lease. SECTION 100. In case the cancellation is due to delinquency on the part of the applicant or grantee, the same shall be entitled to the reimbursement of the proceeds of the sale of the improvements and crops, after deducting the total amount of his indebtedness to the Government and the expense incurred by it in the sale of the improvements or crops and in the new concession of the land. SECTION 101. All actions for the reversion to the Government of lands of the public domain or improvements thereon shall be instituted by the Solicitor-General or the officer acting in his stead, in the proper courts, in the name of the Commonwealth of the Philippines. SECTION 102. Any person, corporation, or association may file an objection under oath to any application or concession under this Act, grounded on any reason sufficient under this Act for the denial or cancellation of the application or the denial of the patent or grant. If, after the applicant or grantee has been given suitable opportunity to be duly heard, the objection is found to be well founded, the Director of Lands shall deny or cancel the application or deny patent or grant, and the person objecting shall, if qualified, be granted a prior right of entry for a term of sixty days from the date of the notice. SECTION 103. All the proofs, affidavits, and oaths of any kind required or necessary under this Act may be made before the justice of the peace 71 of the municipality in which the land lies, or before the judge or clerk of the Court of First Instance of the province in which the land lies, or before any justice of the peace or chargeable notary public of the province in which the land lies, or before any officer or employee of the Bureau of Lands authorized by law to administer oaths. The fees for the taking of final evidence before any of the officials herein-before mentioned shall be as follows: For each affidavit, fifty centavos. For each deposition of the applicant or the witness, fifty centavos. SECTION 104. Any owner of uncultivated agricultural land who knowingly permits application for the same to be made to the Government and the land to be tilled and improved by a bona fide grantee without protesting to the Bureau of Lands within one year after cultivation has begun, shall lose all to the part of the land so cultivated and improved, unless he shall bring action in the proper court before such action for recovery prescribes and obtains favorable judgment therein, in which case the court shall, upon its decision becoming final, order the payment to the grantee, within a reasonable period, of the indemnity fixed by said court for the cultivation and improvement. SECTION 105. If at any time the applicant or grantee shall die before the issuance of the patent or the final grant of the land, or during the life of the lease, or while the applicant or grantee still has obligations pending towards the Government, in accordance with this Act, he shall be succeeded in his rights and obligations with respect to the land applied for or granted or leased under this Act by his heirs in law, who shall be entitled to have issued to them the patent or final concession if they show that they have complied with the requirements therefor, and who shall be subrogated in all his rights and obligations for the purposes of this Act. SECTION 106. If at any time after the approval of the application and before the issuance of a patent or the final concession of the land, or during the life of the lease, or at any time when the applicant or grantee still has obligations pending with the Government, in accordance with this Act, it

L A W O N N A T U R A L R E S O U R C E S a n d E N V I R O N M E N T A L L A W C a s e s a n d S p e c i a l L a w s | 106
appears that the land applied for is necessary, in the public interest, for the protection of any source of water or for any work for the public benefit that the Government wishes to undertake, the Secretary of Agriculture and Commerce may order the cancellation of the application or the non issuance of the patent or concession or the exclusion from the land applied for of such portion as may be required, upon payment of the value of the improvements, if any. SECTION 107. All patents or certificates for land granted under this Act shall be prepared in the Bureau of Lands and shall be issued in the name of the Government of the Republic of the Philippines under the signature of the President of the Philippines: Provided, however, That the President of the Philippines may delegate to the Secretary of Agriculture and Natural Resources74 and/or the Under secretary for Natural Resources 74 the power to sign patents or certificates covering lands not exceeding one hundred forty-four hectares in area, and to the Secretary of Agriculture and Natural Resources 75 the power to sign patents or certificates covering lands exceeding one hundred forty-four hectares in area: Provided, further, That District Land Officers in every province are hereby empowered to sign patents or certificates covering lands not exceeding five hectares in area when the office of the District Land Officer is properly equipped to carry out the purposes of this Act: Provided, That no applicant shall be permitted to split the area applied for by him in excess of the area fixed in this section among his relatives within the sixth degree of consanguinity or affinity excepting the applicant's married children who are actually occupying the land: Provided, finally, That copies of said patents issued shall be furnished to the Bureau of Lands for record purposes. No patent or certificate shall be issued by the District Land Officer unless the survey of the land covered by such patent or certificate, whether made by the Bureau of Lands or by a private surveyor, has been approved by the Director of Lands. The Director of Lands shall promptly act upon all surveys submitted to him for approval and return the same to the District Land Officer within ninety days after receipt of such surveys by his office. In case of disapproval, the Director of Lands shall state the reasons therefor. Any person aggrieved by the decision or action of the District Land Officer may, within thirty days from receipt of the copy of the said decision, appeal to the Director of Lands. Such patents or certificates shall be effective only for the purposes defined in Section one hundred and twenty-two of the land Registration Act, and actual conveyance of the land shall be effected only as provided in said section. All surveys pending approval by the Director of Lands at the time this Act takes effect shall be acted upon by him within ninety days from the effectivity of this Act. SECTION 108. No patent shall issue nor shall any concession or contract be finally approved unless the land has been surveyed and an accurate plat made thereof by the Bureau of Lands. SECTION 109. In no case shall any land be granted under the provisions of this Act when this affects injuriously the use of any adjacent land or of the waters, rivers, creeks, foreshore, roads, or roadsteads, or vest the grantee with other valuable rights that may be detrimental to the public interest. SECTION 110. Patents or certificates issued under the provisions of this Act shall not include nor convey the title to any gold, silver, copper, iron, or other metals or minerals, or other substances containing minerals, guano, gums, precious stones, coal, or coal oil contained in lands granted thereunder. These shall remain to be property of the State. SECTION 111. All persons receiving title to lands under the provisions of this Act shall hold such lands subject to the provisions hereof and to the same public servitudes as exist upon lands owned by private persons, including those with reference to the littoral of the sea and the banks of navigable rivers or rivers upon which rafting may be done. SECTION 112. Said land shall further be subject to a right-of-way not exceeding sixty (60) meters in width for public highways, railroads, irrigation ditches, aqueducts, telegraph and telephone lines and similar works as the Government or any public or quasi-public service or enterprise, including mining or forest concessionaires, may reasonably require for carrying on their business, with damages for the improvements only. 77 SECTION 113. The beneficial use of water shall be the basis, the measure, and the limit of all rights thereto, and the patents herein granted shall be subject to the right of the Government to make such rules and regulations for the use of water and the protection of the water supply, and for other public purposes, as it may deem best for the public good. Whenever, by priority of possession, rights to the use of water for mining, agricultural, manufacturing, or other purposes have vested and accrued, and the same are recognized and acknowledged by the local customs, or by the laws and decisions of the courts, the possessors and owners of such vested rights shall be maintained and protected in the same, and all patents granted under this Act shall be subject to any vested and accrued rights to ditches and reservoirs used in connection with such water rights as may have been acquired in the manner above described prior to April eleven, eighteen hundred and ninety-nine. SECTION 114. There is hereby reserved from the operation of all patents, certificates, entries, and grants by the Government authorized under this Act the right to use for the purposes of power any flow of water in any stream running through or by the land granted, the convertible power from which at ordinary low water exceeds fifty horse power. Where the convertible power in any stream running through or by land granted under the authority of this Act thus exceeds fifty horsepower, and there is no means of using such power except by the occupation of a part of the land granted under authority of this Act, then so much land as is reasonably necessary for the mill site or site for the power house, and for a suitable dam and site for massing the water, is hereby excepted from such grants, not exceeding four hectares, and a right of way to the nearest public highway from the land thus excepted, and also a right of way for the construction and maintenance of such flumes, aqueducts, wires, poles, o r order conduits as may be needed in conveying the water to the point where its fall will yield the greatest power, or the power from the point of conversion to the point of use, is reserved as a servitude or easement upon the land granted by authority of this Act: Provided,however, That when the Government or any concessionaire of the Government shall take possession of the land under this section which a grantee under this Act shall have paid for, supposing it to be subject to grant under this Act, said grantee shall be entitled to indemnity from the Government or the concessionaire, as the case may be, in the amount, if any, paid by him to the Government for the land taken from him by virtue of this section: And provided, further, That with respect to the flow of water, except for converting the same into power exceeding fifty horse power,

L A W O N N A T U R A L R E S O U R C E S a n d E N V I R O N M E N T A L L A W C a s e s a n d S p e c i a l L a w s | 107
said grantee shall be entitled to the same use of the water flowing through or along his land that other private owners enjoy under the law, subject to the governmental regulation provided in the previous section. Water power privileges in which the convertible power at ordinary low water shall exceed fifty horse power shall be disposed of only upon terms established by an Act of the Assembly concerning the use, lease or acquisition of such water privilege. SECTION 115. All lands granted by virtue of this Act, including homesteads upon which final proof has not been made or approved, shall, even though and while the title remains in the State, be subject to the ordinary taxes, which shall be paid by the grantee or the applicant, beginning with the year next following the one in which the homestead application has been filed, or the concession has been approved, or the contract has been signed, as the case may be, on the basis of the value fixed in such filing, approval or signing of the application, concession or contract. SECTION 116. The appraisal or reappraisal of the lands or improvements subject to concession or disposition under this Act shall be made by the Director of Lands, with the approval of the Secretary of Agriculture and Commerce. The Director of Lands may request the assistance of the provincial treasurer of the province in which the land lies or may appoint a committee for such purpose in the province or in the municipality in which the land lies. In no case shall the appraisal or reappraisal be less than the expense incurred or which may be incurred by the Government in connection with the application or concession, nor shall any reappraisal be made with an increase of more than one hundred per centum upon the appraisal or reappraisal next preceding. SECTION 117. All sums due and payable to the Government under this Act, except homestead fees, shall draw simple interest at the rate of four per centum per annum from and after the date in which the debtor shall become delinquent. SECTION 118. Except in favor of the Government or any of its branches, units, or institutions, lands acquired under free patent or homestead provisions shall not be subject to encumbrance or alienation from the date of the approval of the application and for a term of five years from and after the date of issuance of the patent or grant, nor shall they become liable to the satisfaction of any debt contracted prior to the expiration of said period, but the improvements or crops on the land may be mortgaged or pledged to qualified persons, associations, or corporations. No alienation, transfer, or conveyance of any homestead after five years and before twentyfive years after issuance of title shall be valid without the approval of the Secretary of Agriculture and Commerce, which approval shall not be denied except on constitutional and legal grounds. SECTION 119. Every conveyance of land acquired under the free patent or homestead provisions, when proper, shall be subject to repurchase by the applicant, his widow, or legal heirs, within a period of five years from the date of the conveyance. SECTION 120. Conveyance and encumbrance made by persons belonging to the so-called "non-Christian Filipinos" or national cultural minorities, when proper, shall be valid if the person making the conveyance or encumbrance is able to read and can understand the language in which the instrument or conveyance or encumbrances is written. Conveyances and encumbrances made by illiterate non-Christian or literate non-Christians where the instrument of conveyance or encumbrance is in a language not understood by the said literate non-Christians shall not be valid unless duly approved by the Chairman of the Commission on National Integration. SECTION 121. Except with the consent of the grantee and the approval of the Secretary of Natural Resources, and solely for commercial, industrial, educational, religious or charitable purposes or for a right of way, no corporation, association, or partnership may acquire or have any right, title, interest, or property right whatsoever to any land granted under the free patent, homestead, or individual sale provisions of this Act or to any permanent improvement on such land. The provisions of Section 124 of this Act to the contrary notwithstanding, any acquisition of such land, rights thereto or improvements thereon by a corporation, association, or partnership prior to the promulgation of this Decree for the purposes herein stated is deemed valid and binding; Provided, That no final decision of reversion of such land to the State has been rendered by a court; And Provided, further, That such acquisition is approved by the Secretary of Natural Resources within six (6) months from the effectivity of this Decree. SECTION 122. No land originally acquired in any manner under the provisions of this Act, nor any permanent improvement on such land, shall encumbered, alienated, or transferred, except to persons, corporations, associations, or partnerships who may acquire lands of the public domain under this Act or to corporations organized in the Philippines authorized therefor by their charters. Except in cases of hereditary succession, no land or any portion thereof originally acquired under the free patent, homestead, or individual sale provisions of this Act, or any permanent improvement on such land, shall be transferred or assigned to any individual, nor shall such land or any permanent improvement thereon be leased to such individual, when the area of said land, added to that of his own, shall exceed one hundred and forty-four hectares. Any transfer, assignment, or lease made in violation hereof, shall be null and void. SECTION 123. No land originally acquired in any manner under the pro-visions of any previous Act, ordinance, royal order, royal decree, or any other provision of law formerly in force in the Philippines with regard to public lands, terrenos baldios y realengos, or lands of any other denomination that were actually or presumptively of the public domain, or by royal grant or in any other form, nor any permanent improvement on such land, shall be encumbered, alienated, or conveyed, except to persons, corporations or associations who may acquire land of the public domain under this Act or to corporate bodies organized in the Philippines whose charters authorize them to do so: Provided, however, That this prohibition shall not be applicable to the conveyance or acquisition by reason of hereditary succession duly acknowledged and legalized by competent courts; Provided,further, That in the event of the ownership of the lands and improvements mentioned in this section and in the last preceding section being transferred by judicial decree to persons, corporations or associations not legally capacitated to acquire the same under the provisions of this Act, such persons, corporations, or associations shall be obliged to alienate said lands or improvements to others so capacitated within the precise period of five years; otherwise, such property shall revert to the Government.

L A W O N N A T U R A L R E S O U R C E S a n d E N V I R O N M E N T A L L A W C a s e s a n d S p e c i a l L a w s | 108
SECTION 124. Any acquisition, conveyance, alienation, transfer, or other contract made or executed in violation of any of the provisions of sections one hundred and eighteen, one hundred and twenty, one-one hundred and twenty-one, one hundred and twenty two, and one hundred and twenty-three of this Act shall be unlawful and null and void from its execution and shall produce the effect of annulling and cancelling the grant, title, patent, or permit originally issued, recognized or confirmed, actually or presumptively, and cause the reversion of the property and its improvements to the State. SECTION 125. The provisions of sections twenty-two, twenty-three, thirty-three, one hundred and twenty-two, and one hundred and twenty-three of this Act, and any other provision or provisions restricting or tending to restrict the right of persons, corporations, or associations to acquire, hold, lease, encumber, dispose of, or alienate land in the Philippines, or permanent improvements thereon, or any interest therein, shall not be applied in cases in which the right to acquire, hold or dispose of such land, permanent improvements thereon or interests therein in the Philippines is recognized by existing treaties in favor of citizens or subjects of foreign nations and corporations or associations organized and constituted by the same, which right, in so far as it exists under such treaties, shall continue and subsist in the manner and to the extent stipulated in said treaties, and only while these are in force, but not thereafter. SECTION 126. All public auctions provided for in the foregoing chapters in the disposition of public lands shall be held, wherever possible, in the province where the land is located, or, in the office of the Bureau of Lands in Manila SECTION 130. Any person who voluntarily and maliciously prevents or hinders or attempts to prevent or hinder the presentation of any application for public land under this Act, or who in any manner attempts to execute or executes acts intended to dissuade or discourage, or aid to dissuade or discourage, the acquisition of public lands, shall be deemed guilty of coercion and be punished accordingly. SECTION 131. Any person who sells forms issued and distributed gratuitously under this Act or who, being an officer charged with distributing them, refuses or fails, without sufficient reason, to furnish the same, shall be punished for each offense by a fine of not more than one hundred pesos or by imprisonment for not more than three months, or both, in the discretion of the court. SECTION 132. Any person, corporation, association or partnership which, not being qualified or no longer authorized to apply for public land under the provisions of this Act, files or induces or knowingly permits another person, corporation, association or partnership to file an application in his or its behalf or for his or its interest, benefit or advantage, shall be punished by a fine of not less than two hundred nor more than five thousand pesos or by imprisonment for not less than two months nor more than five years, or both, in the discretion of the court; and the application shall be cancelled. SECTION 133. Any person who, without having the qualifications required by this Act, shall by deceit or fraud acquire or attempt to acquire lands of the public domain or other real property or any right, title or interest, or property right of any class to the same, and any person aiding and abetting him therein or serving as a means or tool therefor, shall, upon conviction, be punished by a fine of not more than five thousands pesos, or by the imprisonment for not more than five years, or both, in the discretion of the court.

CHAPTER XIV Transitory Provisions SECTION 127. During the existence and continuance of the Commonwealth and before the Republic of the Philippines is finally established, citizens and corporations of the United States shall enjoy the same rights granted to citizens and corporations of the Philippines under this Act. SECTION 128. During the period specified in the next preceding section, the President of the Philippines, upon receipt of the order of the President of the United States, shall, by proclamation, designate such land as the latter may set aside for military, naval or other reservations for use of the Government of the United States.

TITLE VII Final Provisions

CHAPTER XVI Effectiveness of this Act SECTION 134. If, for any reason, any section or provision of this Act is challenged in a competent court and is held to be unconstitutional, none of the other sections or provisions thereof shall be affected thereby and such other sections and provisions shall continue to govern as if the section or provisions so annulled, disapproved, or repealed had never been incorporated in this Act, and in lieu of the section or provision so annulled, disapproved, or repealed, the provisions of law on the subject thereof in force prior to the approval of this Act shall govern until the Assembly shall otherwise provide in the premises. SECTION 135. All laws and regulations, or parts thereof, inconsistent with the provisions of this Act, are hereby repealed.

CHAPTER XV Penal Provisions SECTION 129. Any person who presents or causes to be presented, or cooperates in the presentation of, any false application, declaration, or evidence, or makes or causes to be made or cooperates in the making of a false affidavit in support of any petition, claim, or objection respecting lands of the public domain, shall be deemed guilty of perjury and punished accordingly.

L A W O N N A T U R A L R E S O U R C E S a n d E N V I R O N M E N T A L L A W C a s e s a n d S p e c i a l L a w s | 109
SECTION 136. This Act shall take effect on December first, nineteen hundred and thirty-six unless the President shall, in the proclamation announcing its effectiveness, designate a prior date, in which case this Act shall take effect on the date so designated. Approved: November 7, 1936

L A W O N N A T U R A L R E S O U R C E S a n d E N V I R O N M E N T A L L A W C a s e s a n d S p e c i a l L a w s | 110
Republic Act No. 8371 October 29, 1997 as well as other services of ICCs/IPs, in order to render such services more responsive to the needs and desires of these communities. Towards these ends, the State shall institute and establish the necessary mechanisms to enforce and guarantee the realization of these rights, taking into consideration their customs, traditions, values, beliefs, their rights to their ancestral domains. CHAPTER DEFINITION OF TERMS I Section 3. Definition of Terms. - For purposes of this Act, the following terms shall mean: a) Ancestral Domains - Subject to Section 56 hereof, refer to all areas generally belonging to ICCs/IPs comprising lands,inland waters, coastal areas, and natural resources therein, held under a claim of ownership, occupied or possessed by ICCs/IPs, themselves or through their ancestors, communally or individually since time immemorial, continuously to the present except when interrupted by war, force majeure or displacement by force, deceit, stealth or as a consequence of government projects or any other voluntary dealings entered into by government and private individuals, corporations, and which are necessary to ensure their economic, social and cultural welfare. It shall include ancestral land, forests, pasture, residential, agricultural, and other lands individually owned whether alienable and disposable or otherwise, hunting grounds, burial grounds, worship areas, bodies of water, mineral and other natural resources, and lands which may no longer be exclusively occupied by ICCs/IPs but from which their traditionally had access to for their subsistence and traditional activities, particularly the home ranges of ICCs/IPs who are still nomadic and/or shifting cultivators; b) Ancestral Lands - Subject to Section 56 hereof, refers to land occupied, possessed and utilized by individuals, families and clans who are members of the ICCs/IPs since time immemorial, by themselves or through their predecessors-in-interest, under claims of individual or traditional group ownership,continuously, to the present except when interrupted by war, force majeure or displacement by force, deceit, stealth, or as a consequence of government projects and other voluntary dealings entered into by government and private individuals/corporations, including, but not limited to, residential lots, rice terraces or paddies, private forests, swidden farms and tree lots; c) Certificate of Ancestral Domain Title - refers to a title formally recognizing the rights of possession and ownership of ICCs/IPs over their ancestral domains identified and delineated in accordance with this law; d) Certificate of Ancestral Lands Title - refers to a title formally recognizing the rights of ICCs/IPs over their ancestral lands; II

AN ACT TO RECOGNIZE, PROTECT AND PROMOTE THE RIGHTS OF INDIGENOUS CULTURAL COMMUNITIES/INDIGENOUS PEOPLE, CREATING A NATIONAL COMMISSION OF INDIGENOUS PEOPLE, ESTABLISHING IMPLEMENTING MECHANISMS, APPROPRIATING FUNDS THEREFOR, AND FOR OTHER PURPOSES Be it enacted by the Senate and House of Representatives of the Philippines in Congress assembled:: CHAPTER GENERAL PROVISIONS Section 1. Short Title. - This Act shall be known as "The Indigenous Peoples Rights Act of 1997." Section 2. Declaration of State Policies. - The State shall recognize and promote all the rights of Indigenous Cultural Communities/Indigenous Peoples (ICCs/IPs) hereunder enumerated within the framework of the Constitution: a) The State shall recognize and promote the rights of ICCs/IPs within the framework of national unity and development; b)The State shall protect the rights of ICCs/IPs to their ancestral domains to ensure their economic, social and cultural well being and shall recognize the applicability of customary laws governing property rights or relations in determining the ownership and extent of ancestral domain; c) The State shall recognize, respect and protect the rights of ICCs/IPs to preserve and develop their cultures, traditions and institutions. It shall consider these rights in the formulation of national laws and policies; d) The State shall guarantee that members of the ICCs/IPs regardless of sex, shall equally enjoy the full measure of human rights and freedoms without distinctions or discriminations; e) The State shall take measures, with the participation of the ICCs/IPs concerned, to protect their rights and guarantee respect for their cultural integrity, and to ensure that members of the ICCs/IPs benefit on an equal footing from the rights and opportunities which national laws and regulations grant to other members of the population and f) The State recognizes its obligations to respond to the strong expression of the ICCs/IPs for cultural integrity by assuring maximum ICC/IP participation in the direction of education, health,

L A W O N N A T U R A L R E S O U R C E S a n d E N V I R O N M E N T A L L A W C a s e s a n d S p e c i a l L a w s | 111
e) Communal Claims - refer to claims on land, resources and rights thereon, belonging to the whole community within a defined territory f) Customary Laws - refer to a body of written and/or unwritten rules, usages, customs and practices traditionally and continually recognized, accepted and observed by respective ICCs/IPs; g) Free and Prior Informed Consent - as used in this Act shall mean the consensus of all members of the ICCs/IPs to; be determined in accordance with their respective customary laws and practices, free from any external manipulation, interference and coercion, and obtained after fully disclosing the intent and scope of the activity, in a language an process understandable to the community; h) Indigenous Cultural Communities/Indigenous Peoples - refer to a group of people or homogenous societies identified by self-ascription and ascription by other, who have continuously lived as organized community on communally bounded and defined territory, and who have, under claims of ownership since time immemorial, occupied, possessed customs, tradition and other distinctive cultural traits, or who have, through resistance to political, social and cultural inroads of colonization, non-indigenous religions and culture, became historically differentiated from the majority of Filipinos. ICCs/IPs shall likewise include peoples who are regarded as indigenous on account of their descent from the populations which inhabited the country, at the time of conquest or colonization, or at the time of inroads of non-indigenous religions and cultures, or the establishment of present state boundaries, who retain some or all of their own social, economic, cultural and political institutions, but who may have been displaced from their traditional domains or who may have resettled outside their ancestral domains; i) Indigenous Political Structure - refer to organizational and cultural leadership systems, institutions, relationships, patterns and processed for decision-making and participation, identified by ICCs/IPs such as, but not limited to, Council of Elders, Council of Timuays, Bodong Holder, or any other tribunal or body of similar nature; j) Individual Claims - refer to claims on land and rights thereon which have been devolved to individuals, families and clans including, but not limited to, residential lots, rice terraces or paddies and tree lots; k) National Commission on Indigenous Peoples (NCIP) - refers to the office created under this Act, which shall be under the Office of the President, and which shall be the primary government agency responsible for the formulation and implementation of policies, plans and programs to recognize, protect and promote the rights of ICCs/IPs; l) Native Title - refers to pre-conquest rights to lands and domains which, as far back as memory reaches, have been held under a claim of private ownership by ICCs/IPs, have never been public lands and are thus indisputably presumed to have been held that way since before the Spanish Conquest; m) Nongovernment Organization - refers to a private, nonprofit voluntary organization that has been organized primarily for the delivery of various services to the ICCs/IPs and has an established track record for effectiveness and acceptability in the community where it serves; n) People's Organization - refers to a private, nonprofit voluntary organization of members of an ICC/IP which is accepted as representative of such ICCs/IPs; o) Sustainable Traditional Resource Rights - refer to the rights of ICCs/IPs to sustainably use,manage, protect and conserve a) land, air, water, and minerals; b) plants, animals and other organisms; c) collecting, fishing and hunting grounds; d) sacred sites; and e) other areas of economic, ceremonial and aesthetic value in accordance with their indigenous knowledge, beliefs, systems and practices; and p) Time Immemorial - refers to a period of time when as far back as memory can go, certain ICCs/IPs are known to have occupied, possessed in the concept of owner, and utilized a defined territory devolved to them, by operation of customary law or inherited from their ancestors, in accordance with their customs and traditions. CHAPTER RIGHTS TO ANCESTRAL DOMAINS III

Section 4. Concept of Ancestral Lands/Domains. - Ancestral lands/domains shall include such concepts of territories which cover not only the physical environment but the total environment including the spiritual and cultural bonds to the area which the ICCs/IPs possess, occupy and use and to which they have claims of ownership. Section 5. Indigenous Concept of Ownership. - Indigenous concept of ownership sustains the view that ancestral domains and all resources found therein shall serve as the material bases of their cultural integrity. The indigenous concept of ownership generally holds that ancestral domains are the ICC's/IP's private but community property which belongs to all generations and therefore cannot be sold, disposed or destroyed. It likewise covers sustainable traditional resource rights. Section 6. Composition of Ancestral Lands/Domains. - Ancestral lands and domains shall consist of all areas generally belonging to ICCs/IPs as referred under Sec. 3, items (a) and (b) of this Act.

L A W O N N A T U R A L R E S O U R C E S a n d E N V I R O N M E N T A L L A W C a s e s a n d S p e c i a l L a w s | 112
Section 7. Rights to Ancestral Domains. - The rights of ownership and possession of ICCs/IPs t their ancestral domains shall be recognized and protected. Such rights shall include: a. Rights of Ownership.- The right to claim ownership over lands, bodies of water traditionally and actually occupied by ICCs/IPs, sacred places, traditional hunting and fishing grounds, and all improvements made by them at any time within the domains; b. Right to Develop Lands and Natural Resources. - Subject to Section 56 hereof, right to develop, control and use lands and territories traditionally occupied, owned, or used; to manage and conserve natural resources within the territories and uphold the responsibilities for future generations; to benefit and share the profits from allocation and utilization of the natural resources found therein; the right to negotiate the terms and conditions for the exploration of natural resources in the areas for the purpose of ensuring ecological, environmental protection and the conservation measures, pursuant to national and customary laws; the right to an informed and intelligent participation in the formulation and implementation of any project, government or private, that will affect or impact upon the ancestral domains and to receive just and fair compensation for any damages which they sustain as a result of the project; and the right to effective measures by the government to prevent any interfere with, alienation and encroachment upon these rights; c. Right to Stay in the Territories- The right to stay in the territory and not be removed therefrom. No ICCs/IPs will be relocated without their free and prior informed consent, nor through any means other than eminent domain. Where relocation is considered necessary as an exceptional measure, such relocation shall take place only with the free and prior informed consent of the ICCs/IPs concerned and whenever possible, they shall be guaranteed the right to return to their ancestral domains, as soon as the grounds for relocation cease to exist. When such return is not possible, as determined by agreement or through appropriate procedures, ICCs/IPs shall be provided in all possible cases with lands of quality and legal status at least equal to that of the land previously occupied by them, suitable to provide for their present needs and future development. Persons thus relocated shall likewise be fully compensated for any resulting loss or injury; d. Right in Case of Displacement. - In case displacement occurs as a result of natural catastrophes, the State shall endeavor to resettle the displaced ICCs/IPs in suitable areas where they can have temporary life support system: Provided, That the displaced ICCs/IPs shall have the right to return to their abandoned lands until such time that the normalcy and safety of such lands shall be determined: Provided, further, That should their ancestral domain cease to exist and normalcy and safety of the previous settlements are not possible, displaced ICCs/IPs shall enjoy security of tenure over lands to which they have been resettled: Provided, furthermore, That basic services and livelihood shall be provided to them to ensure that their needs are adequately addressed: e. Right to Regulate Entry of Migrants. - Right to regulate the entry of migrant settlers and organizations into the domains; f. Right to Safe and Clean Air and Water. - For this purpose, the ICCs/IPs shall have access to integrated systems for the management of their inland waters and air space; g. Right to Claim Parts of Reservations. - The right to claim parts of the ancestral domains which have been reserved for various purposes, except those reserved and intended for common and public welfare and service; and h. Right to Resolve Conflict. - Right to resolve land conflicts in accordance with customary laws of the area where the land is located, and only in default thereof shall the complaints be submitted to amicable settlement and to the Courts of Justice whenever necessary. Section 8. Rights to Ancestral Lands. - The right of ownership and possession of the ICCs/IPs, to their ancestral lands shall be recognized and protected. a. Right to transfer land/property. - Such right shall include the right to transfer land or property rights to/among members of the same ICCs/IPs, subject to customary laws and traditions of the community concerned. b. Right to Redemption. - In cases where it is shown that the transfer of land/property rights by virtue of any agreement or devise, to a non-member of the concerned ICCs/IPs is tainted by the vitiated consent of the ICCs/IPs,or is transferred for an unconscionable consideration or price, the transferor ICC/IP shall have the right to redeem the same within a period not exceeding fifteen (15) years from the date of transfer. Section 9. Responsibilities of ICCs/IPs to their Ancestral Domains. - ICCs/IPs occupying a duly certified ancestral domain shall have the following responsibilities: a. Maintain Ecological Balance- To preserve, restore, and maintain a balanced ecology in the ancestral domain by protecting the flora and fauna, watershed areas, and other reserves; b. Restore Denuded Areas- To actively initiate, undertake and participate in the reforestation of denuded areas and other development programs and projects subject to just and reasonable remuneration; and c. Observe Laws- To observe and comply with the provisions of this Act and the rules and regulations for its effective implementation.

L A W O N N A T U R A L R E S O U R C E S a n d E N V I R O N M E N T A L L A W C a s e s a n d S p e c i a l L a w s | 113
Section 10. Unauthorized and Unlawful Intrusion. - Unauthorized and unlawful intrusion upon, or use of any portion of the ancestral domain, or any violation of the rights herein before enumerated, shall be punishable under this law. Furthermore, the Government shall take measures to prevent non-ICCs/IPs from taking advantage of the ICCs/IPs customs or lack of understanding of laws to secure ownership, possession of land belonging to said ICCs/IPs. Section 11. Recognition of Ancestral Domain Rights. - The rights of ICCs/IPs to their ancestral domains by virtue of Native Title shall be recognized and respected. Formal recognition, when solicited by ICCs/IPs concerned, shall be embodied in a Certificate of Ancestral Domain Title (CADT), which shall recognize the title of the concerned ICCs/IPs over the territories identified and delineated. Section 12. Option to Secure Certificate of Title under Commonwealth Act 141, as amended, or the Land Registration Act 496. - Individual members of cultural communities, with respect to individually-owned ancestral lands who, by themselves or through their predecessors-in -interest, have been in continuous possession and occupation of the same in the concept of owner since the immemorial or for a period of not less than thirty (30) years immediately preceding the approval of this Act and uncontested by the members of the same ICCs/IPs shall have the option to secure title to their ancestral lands under the provisions of Commonwealth Act 141, as amended, or the Land Registration Act 496. For this purpose, said individually-owned ancestral lands, which are agricultural in character and actually used for agricultural, residential, pasture, and tree farming purposes, including those with a slope of eighteen percent (18%) or more, are hereby classified as alienable and disposable agricultural lands. The option granted under this Section shall be exercised within twenty (20) years from the approval of this Act. CHAPTER RIGHT TO SELF-GOVERNANCE AND EMPOWERMENT IV Section 20. Means for Development /Empowerment of ICCs/IPs. - The Government shall establish the means for the full development/empowerment of the ICCs/IPs own institutions and initiatives and, where necessary, provide the resources needed therefor. CHAPTER SOCIAL JUSTICE AND HUMAN RIGHTS V Section 15. Justice System, Conflict Resolution Institutions and Peace Building Processes. - The ICCs/IPs shall have the right to use their own commonly accepted justice systems, conflict resolution institutions, peace building processes or mechanisms and other customary laws and practices within their respective communities and as may be compatible with the national legal system and with internationally recognized human rights. Section 16. Right to Participate in Decision -Making. - ICCs/IPs have the right to participate fully, if they so choose, at all levels of decision-making in matters which may affect their rights, lives and destinies through procedures determined by them as well as to maintain and develop their own indigenous political structures. Consequently, the State shall ensure that the ICCs/IPs shall be given mandatory representation in policy-making bodies and other local legislative councils. Section 17. Right to Determine and Decide Priorities for Development . - The ICCs/IPs shall have the right to determine and decide their own priorities for development affecting their lives, beliefs, institutions, spiritual well-being, and the lands they own, occupy or use. They shall participate in the formulation,implementation and evaluation of policies, plans and programs for national, regional and local development which may directly affect them. Section 18. Tribal Barangays. - The ICCs/IPs living in contiguous areas or communities where they form the predominant population but which are located in municipalities, provinces or cities where they do not constitute the majority of the population, may form or constitute a separate barangay in accordance with the Local Government Code on the creation of tribal barangays. Section 19. Role of Peoples Organizations. - The State shall recognize and respect the role of independent ICCs/IPs organizations to enable the ICCs/IPs to pursue and protect their legitimate and collective interests and aspirations through peaceful and lawful means.

Section 13. Self-Governance. - The State recognizes the inherent right of ICCs/IPs to self-governance and self-determination and respects the integrity of their values, practices and institutions. Consequently, the State shall guarantee the right of ICCs/IPs to freely pursue their economic, social and cultural development. Section 14. Support for Autonomous Regions. - The State shall continue to strengthen and support the autonomous regions created under the Constitution as they may require or need. The State shall likewise encourage other ICCs/IPs not included or outside Muslim Mindanao and the Cordillera to use the form and content of their ways of life as may be compatible with the fundamental rights defined in the Constitution of the Republic of the Philippines and other internationally recognized human rights.

Section 21. Equal Protection and Non-discrimination of ICCs/IPs. - Consistent with the equal protection clause of the Constitution of the Republic of the Philippines, the Charter of the United Nations, the Universal Declaration of Human Rights including the Convention on the Elimination of Discrimination Against Women and International Human Rights Law, the State shall, with due recognition of their distinct characteristics and identity, accord to the members of the ICCs/IPs the rights, protections and privileges enjoyed by the rest of the citizenry. It shall extend to them the same employment rights, opportunities, basic services, educational and other rights and privileges available to every member of the society.

L A W O N N A T U R A L R E S O U R C E S a n d E N V I R O N M E N T A L L A W C a s e s a n d S p e c i a l L a w s | 114
Accordingly, the State shall likewise ensure that the employment of any form of force of coersion against ICCs/IPs shall be dealt with by law. The State shall ensure that the fundamental human rights and freedoms as enshrined in the Constitution and relevant international instruments are guaranteed also to indigenous women. Towards this end, no provision in this Act shall be interpreted so as to result in the diminution of rights and privileges already recognized and accorded to women under existing laws of general application. Section 22. Rights during Armed Conflict. - ICCs/IPs have the right to special protection and security in periods of armed conflict. The State shall observe international standards, in particular, the Fourth Geneva Convention of 1949, for the protection of civilian populations in circumstances of emergency and armed conflict, and shall not recruit members of the ICCs/IPs against their will into armed forces, and in particular, for the use against other ICCs/IPs; not recruit children of ICCs/IPs into the armed forces under any circumstance; nor force indigenous individuals to abandon their lands, territories and means of subsistence, or relocate them in special centers for military purposes under any discriminatory condition. Section 23. Freedom from Discrimination and Right to Equal Opportunity and Treatment . - It shall be the right of the ICCs/IPs to be free from any form of discrimination, with respect to recruitment and conditions of employment, such that they may enjoy equal opportunities as other occupationally-related benefits, informed of their rights under existing labor legislation and of means available to them for redress, not subject to any coercive recruitment systems, including bonded labor and other forms of debt servitude; and equal treatment in employment for men and women, including the protection from sexual harassment. Towards this end, the State shall within the framework of national laws and regulations, and in cooperation with the ICCs/IPs concerned, adopt special measures to ensure the effective protection with regard to the recruitment and conditions of employment of persons belonging to these communities, to the extent that they are not effectively protected by the laws applicable to workers in general. ICCs/IPs shall have the right to association and freedom for all trade union activities and the right to conclude collective bargaining agreements with employers' conditions. They shall likewise have the right not to be subject to working conditions hazardous to their health, particularly through exposure to pesticides and other toxic substances. Section 24. Unlawful Acts Pertaining to Employment. - It shall be unlawful for any person: a. To discriminate against any ICC/IP with respect to the terms and conditions of employment on account of their descent. Equal remuneration shall be paid to ICC/IP and non-ICC/IP for work of equal value; and b. To deny any ICC/IP employee any right or benefit herein provided for or to discharge them for the purpose of preventing them from enjoying any of the rights or benefits provided under this Act. Section 25. Basic Services. - The ICC/IP have the right to special measures for the immediate, effective and continuing improvement of their economic and social conditions, including in the areas of employment, vocational training and retraining, housing, sanitation, health and social security. Particular attention shall be paid to the rights and special needs of indigenous women, elderly, youth, children and differently-abled persons. Accordingly, the State shall guarantee the right of ICCs/IPs to government 's basic services which shall include, but not limited to water and electrical facilities, education, health and infrastructure. Section 26. Women. - ICC/IP women shall enjoy equal rights and opportunities with men, as regards the social, economic, political and cultural spheres of life. The participation of indigenous women in the decision-making process in all levels, as well as in the development of society, shall be given due respect and recognition. The State shall provide full access to education, maternal and child care, health and nutrition, and housing services to indigenous women. Vocational, technical, professional and other forms of training shall be provided to enable these women to fully participate in all aspects of social life. As far as possible, the State shall ensure that indigenous women have access to all services in their own languages. Section 27. Children and Youth. - The State shall recognize the vital role of the children and youth of ICCs/IPs in nation-building and shall promote and protect their physical, moral, spiritual, moral, spiritual, intellectual and social well-being. Towards this end, the State shall support all government programs intended for the development and rearing of the children and youth of ICCs/IPs for civic efficiency and establish such mechanisms as may be necessary for the protection of the rights of the indigenous children and youth. Section 28. Integrated System of Education. - The State shall, through the NCIP, provide a complete, adequate and integrated system of education, relevant to the needs of the children and Young people of ICCs/IPs. CHAPTER CULTURAL INTEGRITY VI

Section 29. Protection of Indigenous Culture, traditions and institutions . - The state shall respect, recognize and protect the right of the ICCs/IPs to preserve and protect their culture, traditions and institutions. It shall consider these rights in the formulation of national plans and policies.

L A W O N N A T U R A L R E S O U R C E S a n d E N V I R O N M E N T A L L A W C a s e s a n d S p e c i a l L a w s | 115
Section 30. Educational Systems. - The State shall provide equal access to various cultural opportunities to the ICCs/IPs through the educational system, public or cultural entities, scholarships, grants and other incentives without prejudice to their right to establish and control their educational systems and institutions by providing education in their own language, in a manner appropriate to their cultural methods of teaching and learning. Indigenous children/youth shall have the right to all levels and forms of education of the State. Section 31. Recognition of Cultural Diversity. - The State shall endeavor to have the dignity and diversity of the cultures, traditions, histories and aspirations of the ICCs/IPs appropriately reflected in all forms of education, public information and cultural-educational exchange. Consequently, the State shall take effective measures, in consultation with ICCs/IPs concerned, to eliminate prejudice and discrimination and to promote tolerance, understanding and good relations among ICCs/IPs and all segments of society. Furthermore, the Government shall take effective measures to ensure that State-owned media duly reflect indigenous cultural diversity. The State shall likewise ensure the participation of appropriate indigenous leaders in schools, communities and international cooperative undertakings like festivals, conferences, seminars and workshops to promote and enhance their distinctive heritage and values. Section 32. Community Intellectual Rights. - ICCs/IPs have the right to practice and revitalize their own cultural traditions and customs. The State shall preserve, protect and develop the past, present and future manifestations of their cultures as well as the right to the restitution of cultural, intellectual, religious, and spiritual property taken without their free and prior informed consent or in violation of their laws, traditions and customs. Section 33. Rights to Religious, Cultural Sites and Ceremonies . - ICCs/IPs shall have the right to manifest, practice, develop teach their spiritual and religious traditions, customs and ceremonies; the right to maintain, protect and have access to their religious and cultural sites; the right to use and control of ceremonial object; and the right to the repatriation of human remains. Accordingly, the State shall take effective measures, in cooperation with the burial sites, be preserved, respected and protected. To achieve this purpose, it shall be unlawful to: a. Explore, excavate or make diggings on archeological sites of the ICCs/IPs for the purpose of obtaining materials of cultural values without the free and prior informed consent of the community concerned; and b. Deface, remove or otherwise destroy artifacts which are of great importance to the ICCs/IPs for the preservation of their cultural heritage. Section 34. Right to Indigenous Knowledge Systems and Practices and to Develop own Sciences and Technologies. - ICCs/IPs are entitled to the recognition of the full ownership and control and protection of their cultural and intellectual rights. They shall have the right to special measures to control, develop and protect their sciences, technologies and cultural manifestations, including human and other genetic resources, seeds, including derivatives of these resources, traditional medicines and health practices, vital medicinal plants, animals and minerals, indigenous knowledge systems and practices, knowledge of the properties of fauna and flora, oral traditions, literature, designs, and visual and performing arts. Section 35. Access to Biological and Genetic Resources . - Access to biological and genetic resources and to indigenous knowledge related to the conservation, utilization and enhancement of these resources, shall be allowed within ancestral lands and domains of the ICCs/IPs only with a free and prior informed consent of such communities, obtained in accordance with customary laws of the concerned community. Section 36. Sustainable Agro-Technical Development. - The State shall recognize the right of ICCs/IPs to a sustainable agro-technological development and shall formulate and implement programs of action for its effective implementation. The State shall likewise promote the bio-genetic and resource management systems among the ICCs/IPs and shall encourage cooperation among government agencies to ensure the successful sustainable development of ICCs/IPs. Section 37. Funds for Archeological and Historical Sites. - The ICCs/IPs shall have the right to receive from the national government all funds especially earmarked or allocated for the management and preservation of their archeological and historical sites and artifacts with the financial and technical support of the national government agencies. CHAPTER NATIONAL COMMISSION ON INDIGENOUS PEOPLES (NCIP) VII

Section 38. National Commission on Indigenous Cultural Communities /Indigenous Peoples (NCCP) . - to carry out the policies herein set forth, there shall be created the National Commission on ICCs/IPs (NCIP), which shall be the primary government agency responsible for the formulation and implementation of policies, plans and programs to promote and protect the rights and well-being of the ICCs/IPs and the recognition of their ancestral domains as well as their rights thereto. Section 39. Mandate. - The NCIP shall protect and promote the interest and well-being of the ICCs/IPs with due regard to their beliefs, customs, traditions and institutions. Section 40. Composition. - The NCIP shall be an independent agency under the Office of the President and shall be composed of seven (7) Commissioners belonging to ICCs/IPs, one (1) of whom shall be the Chairperson. The Commissioners shall be appointed by the President of the Philippines from a list of recommendees submitted by authentic ICCs/IPs: Provided, That the seven (7) Commissioners shall be appointed specifically from each of the following ethnographic areas: Region I and the Cordilleras; Region II; the rest of Luzon; Island Groups including Mindoro, Palawan, Romblon, Panay and the rest of the Visayas; Northern and Western Mindanao; Southern and Eastern Mindanao; and Central Mindanao: Provided, That at least two (2) of the seven (7) Commissioners shall be women.

L A W O N N A T U R A L R E S O U R C E S a n d E N V I R O N M E N T A L L A W C a s e s a n d S p e c i a l L a w s | 116
Section 41. Qualifications, Tenure, Compensation . - The Chairperson and the six (6) Commissioners must be natural born Filipino citizens, bonafide members of ICCs/IPs as certified by his/her tribe, experienced in ethnic affairs and who have worked for at least ten (10) years with an ICC/IP community and/or any government agency involved in ICC/IP, at least 35 years of age at the time of appointment, and must be of proven honesty and integrity: Provided, That at least two (2) of the seven (7) Commissioners shall be the members of the Philippine Bar: Provided, further, That the members of the NCIP shall hold office for a period of three (3) years, and may be subject to re-appointment for another term: Provided, furthermore, That no person shall serve for more than two (2) terms. Appointment to any vacancy shall only be for the unexpired term of the predecessor and in no case shall a member be appointed or designated in a temporary or acting capacity: Provided, finally, That the Chairperson and the Commissioners shall be entitled to compensation in accordance with the Salary Standardization Law. Section 42. Removal from Office. - Any member of the NCIP may be removed from office by the President, on his own initiative or upon recommendation by any indigenous community, before the expiration of his term for cause and after complying with due process requirement of law. Section 43. Appointment of Commissioners. - The President shall appoint the seven (7) Commissioners of the NCIP within ninety (90) days from the effectivity of this Act. Section 44. Powers and Functions. - To accomplish its mandate, the NCIP shall have the following powers, jurisdiction and function: a) To serve as the primary government agency through which ICCs/IPs can seek government assistance and as the medium, thorough which such assistance may be extended; b) To review and assess the conditions of ICCs/IPs including existing laws and policies pertinent thereto and to propose relevant laws and policies to address their role in national development; c) To formulate and implement policies, plans, programs and projects for the economic, social and cultural development of the ICCs/IPs and to monitor the implementation thereof; d) To request and engage the services and support of experts from other agencies of government or employ private experts and consultants as may be required in the pursuit of its objectives; e) To issue certificate of ancestral land/domain title; f) Subject to existing laws, to enter into contracts, agreements, or arrangement, with government or private agencies or entities as may be necessary to attain the objectives of this Act, and subject to the approval of the President, to obtain loans from government lending institutions and other lending institutions to finance its programs; q) To represent the Philippine ICCs/IPs in all international conferences and conventions dealing with indigenous peoples and other related concerns. Section 45. Accessibility and Transparency. - Subject to such limitations as may be provided by law or by rules and regulations promulgated pursuant thereto, all official records, documents and papers pertaining g) To negotiate for funds and to accept grants, donations, gifts and/or properties in whatever form and from whatever source, local and international, subject to the approval of the President of the Philippines, for the benefit of ICCs/IPs and administer the same in accordance with the terms thereof; or in the absence of any condition, in such manner consistent with the interest of ICCs/IPs as well as existing laws; h) To coordinate development programs and projects for the advancement of the ICCs/IPs and to oversee the proper implementation thereof; i) To convene periodic conventions or assemblies of IPs to review, assess as well as propose policies or plans; j) To advise the President of the Philippines on all matters relating to the ICCs/IPs and to submit within sixty (60) days after the close of each calendar year, a report of its operations and achievements; k) To submit to Congress appropriate legislative proposals intended to carry out the policies under this Act; l) To prepare and submit the appropriate budget to the Office of the President; m) To issue appropriate certification as a pre-condition to the grant of permit, lease, grant, or any other similar authority for the disposition, utilization, management and appropriation by any private individual, corporate entity or any government agency, corporation or subdivision thereof on any part or portion of the ancestral domain taking into consideration the consensus approval of the ICCs/IPs concerned; n) To decide all appeals from the decisions and acts of all the various offices within the Commission: o) To promulgate the necessary rules and regulations for the implementation of this Act; p) To exercise such other powers and functions as may be directed by the President of the Republic of the Philippines; and

L A W O N N A T U R A L R E S O U R C E S a n d E N V I R O N M E N T A L L A W C a s e s a n d S p e c i a l L a w s | 117
to official acts, transactions or decisions, as well as research data used as basis for policy development of the Commission shall be made accessible to the public. Section 46. Officers within the NCIP. - The NCIP shall have the following offices which shall be responsible for the implementation of the policies herein after provided: a. Ancestral Domains Office - The Ancestral Domain Office shall be responsible for the identification, delineation and recognition of ancestral land/domains. It shall also be responsible for the management of ancestral lands/domains in accordance with the master plans as well as the implementation of the ancestral domain rights of the ICCs/IPs as provided in Chapter III of this Act. It shall also issue, upon the free and prior informed consent of the ICCs/IPs concerned, certification prior to the grant of any license, lease or permit for the exploitation of natural resources affecting the interests of ICCs/IPs in protecting the territorial integrity of all ancestral domains. It shall likewise perform such other functions as the Commission may deem appropriate and necessary; b. Office on Policy, Planning and Research - The Office on Policy, Planning and Research shall be responsible for the formulation of appropriate policies and programs for ICCs/IPs such as, but not limited to, the development of a Five-Year Master Plan for the ICCs/IPs. Such plan shall undergo a process such that every five years, the Commission shall endeavor to assess the plan and make ramifications in accordance with the changing situations. The Office shall also undertake the documentation of customary law and shall establish and maintain a Research Center that would serve as a depository of ethnographic information for monitoring, evaluation and policy formulation. It shall assist the legislative branch of the national government in the formulation of appropriate legislation benefiting ICCs/IPs. c. Office of Education, Culture and Health - The Office on Culture, Education and Health shall be responsible for the effective implementation of the education, cultural and related rights as provided in this Act. It shall assist, promote and support community schools, both formal and non-formal, for the benefit of the local indigenous community, especially in areas where existing educational facilities are not accessible to members of the indigenous group. It shall administer all scholarship programs and other educational rights intended for ICC/IP beneficiaries in coordination with the Department of Education, Culture and Sports and the Commission on Higher Education. It shall undertake, within the limits of available appropriation, a special program which includes language and vocational training, public health and family assistance program and related subjects. It shall also identify ICCs/IPs with potential training in the health profession and encourage and assist them to enroll in schools of medicine, nursing, physical therapy and other allied courses pertaining to the health profession. Towards this end, the NCIP shall deploy a representative in each of the said offices who shall personally perform the foregoing task and who shall receive complaints from the ICCs/IPs and compel action from appropriate agency. It shall also monitor the activities of the National Museum and other similar government agencies generally intended to manage and preserve historical and archeological artifacts of the ICCs /IPs and shall be responsible for the implementation of such other functions as the NCIP may deem appropriate and necessary; d. Office on Socio-Economic Services and Special Concerns - The Office on Socio-Economic Services and Special Concerns shall serve as the Office through which the NCIP shall coordinate with pertinent government agencies specially charged with the implementation of various basic socio-economic services, policies, plans and programs affecting the ICCs/IPs to ensure that the same are properly and directly enjoyed by them. It shall also be responsible for such other functions as the NCIP may deem appropriate and necessary; e. Office of Empowerment and Human Rights - The Office of Empowerment and Human Rights shall ensure that indigenous socio- political, cultural and economic rights are respected and recognized. It shall ensure that capacity building mechanisms are instituted and ICCs/IPs are afforded every opportunity, if they so choose, to participate in all level decision-making. It shall likewise ensure that the basic human rights, and such other rights as the NCIP may determine, subject to existing laws, rules and regulations are protected and promoted; f. Administrative Office - The Administrative Office shall provide the NCIP with economical, efficient and effective services pertaining to personnel, finance, records, equipment, security, supplies, and related services. It shall also administer the Ancestral Domains Fund; and g. Legal Affairs Office - There shall be a Legal Affairs Office which shall advice the NCIP on all legal matters concerning ICCs/IPs and which shall be responsible for providing ICCs/IPs with legal assistance in litigation involving community interest. It shall conduct preliminary investigation on the basis of complaints filed by the ICCs/IPs against a natural or juridical person believed to have violated ICCs/IPs rights. On the basis of its findings, it shall initiate the filing of appropriate legal or administrative action to the NCIP. Section 47. Other Offices. - The NCIP shall have the power to create additional offices as it may deem necessary subject to existing rules and regulations. Section 48. Regional and Field Offices. - Existing regional and field offices shall remain to function under the strengthened organizational structure of the NCIP. Other field office shall be created wherever appropriate and the staffing pattern thereof shall be determined by the NCIP: Provided, That in provinces where there are ICCs/IPs but without field offices, the NCIP shall establish field offices in said provinces.

L A W O N N A T U R A L R E S O U R C E S a n d E N V I R O N M E N T A L L A W C a s e s a n d S p e c i a l L a w s | 118
Section 49. Office of the Executive Director. - The NCIP shall create the Office of the Executive Director which shall serve as its secretariat. The office shall be headed by an Executive Director who shall be appointed by the President of the Republic of the Philippines upon the recommendation of the NCIP on a permanent basis. The staffing pattern of the office shall be determined by the NCIP subject to existing rules and regulations. Section 50. Consultative Body. - A body consisting of the traditional leaders, elders and representatives from the women and youth sectors of the different ICCs/IPs shall be constituted by the NCIP from the time to time to advise it on matters relating to the problems, aspirations and interests of the ICCs/IPs. CHAPTER DELINEATION AND RECOGNITION OF ANCESTRAL DOMAINS VIII coordination with the community concerned and shall at all times include genuine involvement and participation by the members of the communities concerned; d. Proof required - Proof of Ancestral Domain Claims shall include the testimony of elders or community under oath, and other documents directly or indirectly attesting to the possession or occupation of the area since time immemorial by such ICCs/IPs in the concept of owners which shall be any one (1) of the following authentic documents: 1. Written accounts of the ICCs/IPs customs and traditions; 2. Written accounts of the ICCs/IPs political structure and institution; 3. Pictures showing long term occupation such as those of old improvements, burial grounds, sacred places and old villages; 4. Historical accounts, including pacts and agreements concerning boundaries entered into by the ICCs/IPs concerned with other ICCs/IPs; 5. Survey plans and sketch maps; 6. Anthropological data; 7. Genealogical surveys; 8. Pictures and descriptive histories of traditional communal forests and hunting grounds; 9. Pictures and descriptive histories of traditional landmarks such as mountains, rivers, creeks, ridges, hills, terraces and the like; and 10. Write-ups of names and places derived from the native dialect of the community. e. Preparation of Maps - On the basis of such investigation and the findings of fact based thereon, the Ancestral Domains Office of the NCIP shall prepare a perimeter map, complete with technical descriptions, and a description of the natural features and landmarks embraced therein; f. Report of Investigation and Other Documents - A complete copy of the preliminary census and a report of investigation, shall be prepared by the Ancestral Domains Office of the NCIP;

Section 51. Delineation and Recognition of Ancestral Domains . - Self-delineation shall be guiding principle in the identification and delineation of ancestral domains. As such, the ICCs/IPs concerned shall have a decisive role in all the activities pertinent thereto. The Sworn Statement of the Elders as to the Scope of the territories and agreements/pacts made with neighboring ICCs/IPs, if any, will be essential to the determination of these traditional territories. The Government shall take the necessary steps to identify lands which the ICCs/IPs concerned traditionally occupy and guarantee effective protection of their rights of ownership and possession thereto. Measures shall be taken in appropriate cases to safeguard the rights of the ICCs/IPs concerned to land which may no longer be exclusively occupied by them, but to which they have traditionally had access for their subsistence and traditional activities, particularly of ICCs/IPs who are still nomadic and/or shifting cultivators. Section 52. Delineation Process. - The identification and delineation of ancestral domains shall be done in accordance with the following procedures: a. Ancestral Domains Delineated Prior to this Act - The provisions hereunder shall not apply to ancestral domains/lands already delineated according to DENR Administrative Order No. 2, series of 1993, nor to ancestral lands and domains delineated under any other community/ancestral domain program prior to the enactment of his law. ICCs/IPs enactment of this law shall have the right to apply for the issuance of a Certificate of Ancestral Domain Title (CADT) over the area without going through the process outlined hereunder; b. Petition for Delineation - The process of delineating a specific perimeter may be initiated by the NCIP with the consent of the ICC/IP concerned, or through a Petition for Delineation filed with the NCIP, by a majority of the members of the ICCs/IPs; c. Delineation Paper - The official delineation of ancestral domain boundaries including census of all community members therein, shall be immediately undertaken by the Ancestral Domains Office upon filing of the application by the ICCs/IPs concerned. Delineation will be done in

L A W O N N A T U R A L R E S O U R C E S a n d E N V I R O N M E N T A L L A W C a s e s a n d S p e c i a l L a w s | 119
g. Notice and Publication - A copy of each document, including a translation in the native language of the ICCs/IPs concerned shall be posted in a prominent place therein for at least fifteen (15) days. A copy of the document shall also be posted at the local, provincial and regional offices of the NCIP, and shall be published in a newspaper of general circulation once a week for two (2) consecutive weeks to allow other claimants to file opposition thereto within fifteen (15) days from the date of such publication: Provided, That in areas where no such newspaper exists, broadcasting in a radio station will be a valid substitute: Provided, further, That mere posting shall be deemed sufficient if both newspaper and radio station are not available; h. Endorsement to NCIP - Within fifteen (15) days from publication, and of the inspection process, the Ancestral Domains Office shall prepare a report to the NCIP endorsing a favorable action upon a claim that is deemed to have sufficient proof. However, if the proof is deemed insufficient, the Ancestral Domains Office shall require the submission of additional evidence: Provided, That the Ancestral Domains Office shall reject any claim that is deemed patently false or fraudulent after inspection and verification: Provided, further, That in case of rejection, the Ancestral Domains Office shall give the applicant due notice, copy furnished all concerned, containing the grounds for denial. The denial shall be appealable to the NCIP: Provided, furthermore, That in cases where there are conflicting claims, the Ancestral Domains Office shall cause the contending parties to meet and assist them in coming up with a preliminary resolution of the conflict, without prejudice to its full adjudication according to the selection below. i. Turnover of Areas Within Ancestral Domains Managed by Other Government Agencies - The Chairperson of the NCIP shall certify that the area covered is an ancestral domain. The secretaries of the Department of Agrarian Reform, Department of Environment and Natural Resources, Department of the Interior and Local Government, and Department of Justice, the Commissioner of the National Development Corporation, and any other government agency claiming jurisdiction over the area shall be notified thereof. Such notification shall terminate any legal basis for the jurisdiction previously claimed; j. Issuance of CADT - ICCs/IPs whose ancestral domains have been officially delineated and determined by the NCIP shall be issued a CADT in the name of the community concerned, containing a list of all those identified in the census; and k. Registration of CADTs - The NCIP shall register issued certificates of ancestral domain titles and certificates of ancestral lands titles before the Register of Deeds in the place where the property is situated. Section 53. Identification, Delineation and Certification of Ancestral Lands . a. The allocation of lands within any ancestral domain to individual or indigenous corporate (family or clan) claimants shall be left to the ICCs/IPs concerned to decide in accordance with customs and traditions; b. Individual and indigenous corporate claimants of ancestral lands which are not within ancestral domains, may have their claims officially established by filing applications for the identification and delineation of their claims with the Ancestral Domains Office. An individual or recognized head of a family or clan may file such application in his behalf or in behalf of his family or clan, respectively; c. Proofs of such claims shall accompany the application form which shall include the testimony under oath of elders of the community and other documents directly or indirectly attesting to the possession or occupation of the areas since time immemorial by the individual or corporate claimants in the concept of owners which shall be any of the authentic documents enumerated under Sec. 52 (d) of this act, including tax declarations and proofs of payment of taxes; d. The Ancestral Domains Office may require from each ancestral claimant the submission of such other documents, Sworn Statements and the like, which in its opinion, may shed light on the veracity of the contents of the application/claim; e. Upon receipt of the applications for delineation and recognition of ancestral land claims, the Ancestral Domains Office shall cause the publication of the application and a copy of each document submitted including a translation in the native language of the ICCs/IPs concerned in a prominent place therein for at least fifteen (15) days. A copy of the document shall also be posted at the local, provincial, and regional offices of the NCIP and shall be published in a newspaper of general circulation once a week for two (2) consecutive weeks to allow other claimants to file opposition thereto within fifteen (15) days from the date of such publication: Provided, That in areas where no such newspaper exists, broadcasting in a radio station will be a valid substitute: Provided, further, That mere posting shall be deemed sufficient if both newspapers and radio station are not available f. Fifteen (15) days after such publication, the Ancestral Domains Office shall investigate and inspect each application, and if found to be meritorious, shall cause a parcellary survey of the area being claimed. The Ancestral Domains office shall reject any claim that is deemed patently false or fraudulent after inspection and verification. In case of rejection, the Ancestral Domains office shall give the applicant due notice, copy furnished all concerned, containing the grounds for denial. The denial shall be appealable to the NCIP. In case of conflicting claims among individual or indigenous corporate claimants, the Ancestral domains Office shall cause the contending parties to meet and assist them in coming up with a preliminary resolution of the conflict, without prejudice to its full adjudication according to Sec. 62 of this Act. In all proceedings for the identification or delineation of the ancestral domains as herein provided, the Director of Lands shall represent the interest of the Republic of the Philippines; and

L A W O N N A T U R A L R E S O U R C E S a n d E N V I R O N M E N T A L L A W C a s e s a n d S p e c i a l L a w s | 120
g. The Ancestral Domains Office shall prepare and submit a report on each and every application surveyed and delineated to the NCIP, which shall, in turn, evaluate or corporate (family or clan) claimant over ancestral lands. Section 54. Fraudulent Claims. - The Ancestral Domains Office may, upon written request from the ICCs/IPs, review existing claims which have been fraudulently acquired by any person or community. Any claim found to be fraudulently acquired by, and issued to, any person or community may be cancelled by the NCIP after due notice and hearing of all parties concerned. Section 55. Communal Rights. - Subject to Section 56 hereof, areas within the ancestral domains, whether delineated or not, shall be presumed to be communally held: Provide, That communal rights under this Act shall not be construed as co-ownership as provided in Republic Act. No. 386, otherwise known as the New Civil Code. Section 56. Existing Property Rights Regimes. - Property rights within the ancestral domains already existing and/or vested upon effectivity of this Act, shall be recognized and respected. Section 57. Natural Resources within Ancestral Domains. - The ICCs/IPs shall have the priority rights in the harvesting, extraction, development or exploitation of any natural resources within the ancestral domains. A non-member of the ICCs/IPs concerned may be allowed to take part in the development and utilization of the natural resources for a period of not exceeding twenty-five (25) years renewable for not more than twenty-five (25) years: Provided, That a formal and written agreement is entered into with the ICCs/IPs concerned or that the community, pursuant to its own decision making process, has agreed to allow such operation: Provided, finally, That the all extractions shall be used to facilitate the development and improvement of the ancestral domains. Section 58. Environmental Consideration. - Ancestral domains or portion thereof, which are found necessary for critical watersheds, mangroves wildlife sanctuaries, wilderness, protected areas, forest cover, or reforestation as determined by the appropriate agencies with the full participation of the ICCs/IPs concerned shall be maintained, managed and developed for such purposes. The ICCs/IPs concerned shall be given the responsibility to maintain, develop, protect and conserve such areas with the full and effective assistance of the government agencies. Should the ICCs/IPs decide to transfer the responsibility over the areas, said decision must be made in writing. The consent of the ICCs/IPs should be arrived at in accordance with its customary laws without prejudice to the basic requirement of the existing laws on free and prior informed consent: Provided, That the transfer shall be temporary and will ultimately revert to the ICCs/IPs in accordance with a program for technology transfer: Provided, further, That no ICCs/IPs shall be displaced or relocated for the purpose enumerated under this section without the written consent of the specific persons authorized to give consent. Section 59. Certification Precondition. - all department and other governmental agencies shall henceforth be strictly enjoined from issuing, renewing, or granting any concession, license or lease, or entering into any production-sharing agreement, without prior certification from the NCIP that the area affected does not overlap with any ancestral domain. Such certificate shall only be issued after a field-based investigation is conducted by the Ancestral Domain Office of the area concerned: Provided, That no certificate shall be issued by the NCIP without the free and prior informed and written consent of the ICCs/IPs concerned: Provided, further, That no department, government agency or government-owned or -controlled corporation may issue new concession, license, lease, or production sharing agreement while there is pending application CADT: Provided, finally, That the ICCs/IPs shall have the right to stop or suspend, in accordance with this Act, any project that has not satisfied the requirement of this consultation process. Section 60. Exemption from Taxes. - All lands certified to be ancestral domains shall be exempt from real property taxes, specially levies, and other forms of exaction except such portion of the ancestral domains as are actually used for large-scale agriculture, commercial forest plantation and residential purposes and upon titling by other by private person: Provided, that all exactions shall be used to facilitate the development and improvement of the ancestral domains. Section 61. Temporary Requisition Powers. - Prior to the establishment of an institutional surveying capacity whereby it can effectively fulfill its mandate, but in no case beyond three (3) years after its creation, the NCIP is hereby authorized to request the Department of Environment and Natural Resources (DENR) survey teams as well as other equally capable private survey teams, through a Memorandum of Agreement (MOA), to delineate ancestral domain perimeters. The DENR Secretary shall accommodate any such request within one (1) month of its issuance: Provided, That the Memorandum of Agreement shall stipulate, among others, a provision for technology transfer to the NCIP. Section 62. Resolution of Conflicts. - In cases of conflicting interest, where there are adverse claims within the ancestral domains as delineated in the survey plan, and which cannot be resolved, the NCIP shall hear and decide, after notice to the proper parties, the disputes arising from the delineation of such ancestral domains: Provided, That if the dispute is between and/or among ICCs/IPs regarding the traditional boundaries of their respective ancestral domains, customary process shall be followed. The NCIP shall promulgate the necessary rules and regulations to carry out its adjudicatory functions: Provided, further, That in any decision, order, award or ruling of the NCIP on any ancestral domain dispute or on any matter pertaining to the application, implementation, enforcement and interpretation of this Act may be brought for Petition for Review to the Court of Appeals within fifteen (15) days from receipt of a copy thereof. Section 63. Applicable Laws. - Customary laws, traditions and practices of the ICCs/IPs of the land where the conflict arises shall be applied first with respect to property rights, claims and ownerships, hereditary succession and settlement of land disputes. Any doubt or ambiguity in the application of laws shall be resolved in favor of the ICCs/IPs. Section 64. Remedial Measures. - Expropriation may be resorted to in the resolution of conflicts of interest following the principle of the "common good". The NCIP shall take appropriate legal action for the cancellation of officially documented titles which were acquired illegally: Provided, That such procedure

L A W O N N A T U R A L R E S O U R C E S a n d E N V I R O N M E N T A L L A W C a s e s a n d S p e c i a l L a w s | 121
shall ensure that the rights of possessors in good faith shall be respected: Provided, further, That the action for cancellation shall be initiated within two (2) years from the effectivity of this Act: Provided, finally, That the action for reconveyance shall be a period of ten (10) years in accordance with existing laws. CHAPTER JURISDICTION AND PROCEDURES FOR ENFORCEMENT OF RIGHTS IX d. To enjoin any or all acts involving or arising from any case pending therefore it which, if not restrained forthwith, may cause grave or irreparable damage to any of the parties to the case or seriously affect social or economic activity. Section 70. No restraining Order or Preliminary Injunction. - No inferior court of the Philippines shall have the jurisdiction to issue any restraining order or writ of preliminary injunction against the NCIP or any of its duly authorized or designated offices in any case, dispute or controversy to, or interpretation of this Act and other pertinent laws relating to ICCs/IPs and ancestral domains. CHAPTER ANCESTRAL DOMAINS FUND X

Section 65. Primary of Customary Laws and Practices. - When disputes involve ICCs/IPs, customary laws and practices shall be used to resolve the dispute. Section 66. Jurisdiction of the NCIP. - The NCIP, through its regional offices, shall have jurisdiction over all claims and disputes involving rights of ICCs/IPs; Provided, however, That no such dispute shall be brought to the NCIP unless the parties have exhausted all remedies provided under their customary laws. For this purpose, a certification shall be issued by the Council of Elders/Leaders who participated in the attempt to settle the dispute that the same has not been resolved, which certification shall be a condition precedent to the filing of a petition with the NCIP. Section 67. Appeals to the Court of Appeals. - Decisions of the NCIP shall be appealable to the Court of Appeals by way of a petition for review. Section 68. Execution of Decisions, Awards, Orders. - Upon expiration of the period here provided and no appeal is perfected by any of the contending parties, the Hearing Officer of the NCIP, on its own initiative or upon motion by the prevailing party, shall issue a writ of execution requiring the sheriff or the proper officer to execute final decisions, orders or awards of the Regional Hearing Officer of the NCIP. Section 69. Quasi-Judicial Powers of the NCIP. - The NCIP shall have the power and authority: a. To promulgate rules and regulations governing the hearing and disposition of cases filed before it as well as those pertaining to its internal functions and such rules and regulations as may be necessary to carry out the purposes of this Act; b. To administer oaths, summon the parties to a controversy, issue subpoenas requiring the attendance and testimony of witnesses or the production of such books, papers, contracts, records, agreements and other document of similar nature as may be material to a just determination of the matter under investigation or hearing conducted in pursuance of this Act; c. To hold any person in contempt, directly or indirectly, and impose appropriate penalties therefor; and

Section 71. Ancestral Domains Fund. - There is hereby created a special fund, to be known as the Ancestral Domains Fund, an initial amount of the One Hundred thirty million pesos(P130,000,000) to cover compensation for expropriated lands, delineation and development of ancestral domains. An amount of Fifty million pesos (P50,000,000) shall be sourced from the gross income of the Philippine Charity Sweepstakes Office (PCSO) from its lotto operation, Ten millions pesos (P10,000,000) from the gross receipts of the travel tax of the preceding year, the fund of the Social Reform Council intended for survey and delineation of ancestral lands/domains, and such other source as the government may be deem appropriate. Thereafter such amount shall be included in the annual General Appropriations Act. Foreign as well as local funds which are made available for the ICCs/IPs through the government of the Philippines shall be coursed through the NCIP. The NCIP may also solicit and receive donations, endowments shall be exempted from income or gift taxes and all other taxes, charges or fees imposed by the government or any political subdivision or instrumentality thereof. CHAPTER PENALTIES XI

Section 72. Punishable Acts and Applicable Penalties. - Any person who commits violation of any of the provisions of this Act, such as, but not limited to, authorized and/or unlawful intrusion upon any ancestral lands or domains as stated in Sec. 10, Chapter III, or shall commit any of the prohibited acts mentioned in Sections 21 and 24, Chapter V, Section 33, Chapter VI hereof, shall be punished in accordance with the customary laws of the ICCs/IPs concerned: Provided, That no such penalty shall be cruel, degrading or inhuman punishment: Provided, further, That neither shall the death penalty or excessive fines be imposed. This provision shall be without prejudice to the right of any ICCs/IPs to avail of the protection of existing laws. In which case, any person who violates any provision of this Act shall, upon conviction, be punished by imprisonment of not less than nine (9) months but not more than twelve (12) years or a fine not less than One hundred thousand pesos (P100,000) nor more than Five hundred thousand pesos (P500,000) or both such fine and imprisonment upon the discretion of the court. In addition, he shall be obliged to pay to the ICCs/IPs concerned whatever damage may have been suffered by the latter as a consequence of the unlawful act.

L A W O N N A T U R A L R E S O U R C E S a n d E N V I R O N M E N T A L L A W C a s e s a n d S p e c i a l L a w s | 122
Section 73. Persons Subject to Punishment. - If the offender is a juridical person, all officers such as, but not limited to, its president, manager, or head of office responsible for their unlawful act shall be criminally liable therefor, in addition to the cancellation of certificates of their registration and/or license: Provided, That if the offender is a public official, the penalty shall include perpetual disqualification to hold public office. CHAPTER XII MERGER OF THE OFFICE FOR NORTHERN CULTURAL COMMUNITIES (ONCC) AND THE OFFICE FOR SOUTHERN CULTURAL COMMUNITIES (OSCC) Section 74. Merger of ONCC/OSCC. - The Office for Northern Cultural Communities (ONCC) and the Office of Southern Cultural Communities (OSCC), created under Executive Order Nos. 122-B and 122-C respectively, are hereby merged as organic offices of the NCIP and shall continue to function under a revitalized and strengthened structures to achieve the objectives of the NCIP: Provided, That the positions of Regional Directors and below, are hereby phased-out upon the effectivity of this Act: Provided, further, That officials and employees of the phased-out offices who may be qualified may apply for reappointment with the NCIP and may be given prior rights in the filing up of the newly created positions of NCIP, subject to the qualifications set by the Placement Committee: Provided, furthermore, That in the case where an indigenous person and a non-indigenous person with similar qualifications apply for the same position, priority shall be given to the former. Officers and employees who are to be phased-out as a result of the merger of their offices shall be entitled to gratuity a rate equivalent to one and a half (1 1/2) months salary for every year of continuous and satisfactory service rendered or the equivalent nearest fraction thereof favorable to them on the basis of the highest salary received. If they are already entitled to retirement benefits or the gratuity herein provided. Officers and employees who may be reinstated shall refund such retirement benefits or gratuity received: Provided, finally That absorbed personnel must still meet the qualifications and standards set by the Civil Service and the Placement Committee herein created. Section 75. Transition Period. - The ONCC/OSCC shall have a period of six (6) months from the effectivity of this Act within which to wind up its affairs and to conduct audit of its finances. Section 76. Transfer of Assets/Properties. - All real and personal properties which are vested in, or belonging to, the merged offices as aforestated shall be transferred to the NCIP without further need of conveyance, transfer or assignment and shall be held for the same purpose as they were held by the former offices: Provided, That all contracts, records and documents shall be transferred to the NCIP. All agreements and contracts entered into by the merged offices shall remain in full force and effect unless otherwise terminated, modified or amended by the NCIP. Section 77. Placement Committee. - Subject to rules on government reorganization, a Placement Committee shall be created by the NCIP, in coordination with the Civil Service Commission, which shall assist in the judicious selection and placement of personnel in order that the best qualified and most deserving persons shall be appointed in the reorganized agency. The placement Committee shall be composed of seven (7) commissioners and an ICCs/IPs representative from each of the first and second level employees association in the Offices for Northern and Southern Cultural Communities (ONCC/OSCC), nongovernment organizations (NGOs) who have served the community for at least five (5) years and peoples organizations (POs) with at least five (5) years of existence. They shall be guided by the criteria of retention and appointment to be prepared by the consultative body and by the pertinent provisions of the civil service law. CHAPTER FINAL PROVISIONS XIII

Section 78. Special Provision. - The City of Baguio shall remain to be governed by its Chapter and all lands proclaimed as part of its townsite reservation shall remain as such until otherwise reclassified by appropriate legislation: Provided, That prior land rights and titles recognized and/or required through any judicial, administrative or other processes before the effectivity of this Act shall remain valid: Provided, further, That this provision shall not apply to any territory which becomes part of the City of Baguio after the effectivity of this Act. Section 79. Appropriations. - The amount necessary to finance the initial implementation of this Act shall be charged against the current year's appropriation of the ONCC and the OSCC. Thereafter, such sums as may be necessary for its continued implementation shall be included in the annual General Appropriations Act. Section 80. Implementing Rules and Regulations. - Within sixty (60) days immediately after appointment, the NCIP shall issue the necessary rules and regulations, in consultation with the Committees on National Cultural Communities of the House of Representatives and the Senate, for the effective implementation of this Act. Section 81. Saving Clause. - This Act will not in any manner adversely affect the rights and benefits of the ICCs/IPs under other conventions, recommendations, international treaties, national laws, awards, customs and agreements. Section 82. Separability Clause. - In case any provision of this Act or any portion thereof is declared unconstitutional by a competent court, other provisions shall not be affected thereby. Section 83. Repealing Clause. - Presidential Decree NO. 410, Executive Order Nos. 122-B and 122-C, and all other laws, decrees, orders, rules and regulations or parts thereof inconsistent with this Act are hereby repealed or modified accordingly. Section 84. Effectivity. - This Act shall take effect fifteen days (15) days upon its publication in the Official Gazette or in any two (2) newspapers of general circulation.

L A W O N N A T U R A L R E S O U R C E S a n d E N V I R O N M E N T A L L A W C a s e s a n d S p e c i a l L a w s | 123
Approved: 29 October 1997.

Das könnte Ihnen auch gefallen